You are on page 1of 567

LRT CLINICAL EXAM RECALLS

1. ENUMERATE THE 8 EXTRAPULMONARY SITES OF TUBERCULOSIS IN DECREASING ORDER

Lymph nodes, pleura, genitourinary tract, bones and joints, meninges, peritoneum, pericardium
- Harrison’s 20th ed p1243

2. WHAT IS THE KEY PATHWAY IN TYPE 1 HYPERSENSITIVITY REACTION:


A. IgG
B. IgE
C. IMMUNE COMPLEX
D. T LYMPHOCYTE

3. THE FOLLOWING IS NOT AN ADVERSE REACTION TYPE IN TYPE 1 HYPERSENSITIVITY?


A. CONTACT DERMATITIS-type4
B. URTICARIA
C. ANGIOEDEMA
D. ANAPHYLAXIS

4. THE FOLLOWING IS NOT AN ADVERSE REACTION IN TYPE 4 HYPERSENSITIVITY?

a. CONTACT DERMATITIS
b. STEVEN JOHNSON SYNDROME
c. DRUG-INDUCED HYPERSENSITIVITY
d. DRUG-INDUCED HEMOLYSIS

5. WHICH OF THE FOLLOWING IS AN INDICATION OF IMPENDING RESPIRATORY FAILURE IN


ASTHMA?

A. NORMAL Pco2
B. BILATERAL INFILTRATES ON CXR
C. EXCESSIVE DIAPHORESIS
D. TACHYCARDIA

Ratio: A normal or rising PCO2 is an indication of an impending respiratory failure and requires immediate
monitoring and therapy.
Reference: Harri 20th Ed Ch 281 p1968

6. THIS IS NOT A CONTROLLER THERAPY FOR ASTHMA:

A. CROMOLYN SODIUM
B. MONTELUKAST
C. IPRATROPIUM
D. METHYLPREDNISOLONE

Harrisons 20th Ed Ch 281 p1966-1967


Controller Therapies
1. Cromones - Cromolyn sodium and nedocromil sodium are asthma controller drugs
2. Systemic Corticosteroids - Corticosteroids are used intravenously (hydrocortisone or methylprednisolone) for
the treatment of acute severe asthma
3. Antileukotrienes, such as montelukast and zafirlukast, block cys-LT1-receptors and provide modest clinical
benefit in asthma

7. THE MOST EFFECTIVE CONTROLLER THERAPY FOR ASTHMA:

A. INHALED CORTICOSTEROIDS
B. CHROMONES
C. ANTILEUKOTRIENES
D. SYSTEMIC CORTICOSTEROIDS

Ratio: IM plat, p87

9. THIS IS PAIRED WITH INHALED CORTICOSTEROIDS FOR BETTER ASTHMA CONTROL:

A. FORMOTEROL (ICS-Formoterol reduces asthma exacerbation)- Mentioned from GINA 2021


B. ALBUTEROL
C. SALBUTAMOL
D. TERBUTALINE

10. THE FOLLOWING IS NOT A PATHOGENETIC MECHANISM INVOLVED IN ASTHMA:

A. EOSINOPHILIC INFILTRATION
B. CYTOKINE-MEDIATED AIRWAY INFLAMMATION
C. MAST CELL ACTIVATION
D. REFLEX INACTIVATION OF CHOLINERGIC PATHWAYS

Rationale:
Harrisons 20th Ed Ch 281 p1961-62
Pathophysiology
Airway inflammation
a. Mast Cells
b. Eosinophils
Inflammatory mediators
c. Cytokines

11. THIS IS THE CHARACTERISTIC PHYSIOLOGIC ABNORMALITY OF ASTHMA AND DESCRIBES THE
EXCESSIVE BRONCHOCONSTRICTOR RESPONSE TO MULTIPLE INHALED TRIGGERS:

a.MUCUS HYPERSECRETION
b.AIRWAY HYPERRESPONSIVENESS
c.MAST CELL ACTIVATION
d.ACUTE INFLAMMATORY REACTION

Rationale:
Harrisons 20th Ed Ch 281 p1963
Airway Hyperresponsiveness
AHR is the characteristic physiologic abnormality of asthma and describes the excessive
bronchoconstrictor response to multiple inhaled triggers that would have no effect on normal airways.

12. Which of the following clinical features is not consistent with bronchial asthma

a.Variable wheezing, dyspnea, and coughing


b.Symptoms worse during the day
c.PRODROMAL SYMPTOMS MAY PRECEDE AN EXACERBATION
d.INCREASED MUCUS PRODUCTION WITH DIFFICULTY EXPECTORATING

Ratio: Harri 20th Ed Ch 281 p1963


Symptoms may be worse at night and the px typically awake in the early morning hours

13. WHICH OF THE FOLLOWING DRUGS MAY NOT TRIGGER ASTHMA:

a.LOSARTAN
b.AMLODIPINE - calcium channel blocker
c.CAPTOPRIL - ace inhibitor
d.METOPROLOL- beta blocker

14. WHICH OF THE FOLLOWING IS A COMMON PATHOLOGIC FINDING IN FATAL ASTHMA?

a.PRESENCE OF EXUDATES IN THE ALVEOLI


b.OCCLUSION OF AIRWAY BY MUCUS PLUGS
c.UNIFORM THINNING OF THE AIRWAY WALL
d.VASOCONSTRICTION OF BRONCHIAL VESSELS

Harrison 20th Ed Ch 281 p 1960


The airway wall itself may be thickened and edematous, particularly in fatal asthma. Another common finding in
fatal asthma is occlusion of the airway lumen by a mucous plug, which is comprised of mucous glycoproteins
secreted from goblet cells and plasma proteins from leaky bronchial vessels (Fig. 281-1). There is also vasodilation and
increased numbers of blood vessels (angiogenesis). Direct observation Bronchoscopy indicates that the airways may be
narrowed, erythematous, and edematous.

15. THE COMMONEST TRIGGER OF ACUTE SEVERE ASTHMA EXACERBATIONS:

a.HAY FEVER
b.URTIs
c.EXERCISE
d.FOOD ALLERGIES

Ratio: Harri 19th Ed p1671


16. THE MOST COMMON ALLERGEN TO TRIGGER ASTHMA:

a.ANIMAL DANDER
b.ANIMAL FEATHERS
c.HOUSE DUST MITES
d.PLANT POLLEN

Harrisons 20th Ed Ch 281 p1958


The allergens that lead to sensitization are usually proteins that have protease activity, and the most common allergens
are derived from house dust mites, cat and dog fur, cockroaches (in inner cities), grass and tree pollens, and rodents
(in laboratory workers).

Question 17
THE MAJOR RISK FACTOR FOR THE DEVELOPMENT OF ASTHMA

a.HOUSE ALLERGENS
b.ATOPY
c.ENVIRONMENTAL POLLUTION
d.OCCUPATIONAL EXPOSURES

Harrisons 20th Ed Ch 281 p1958


Atopy is the major risk factor for asthma, and non-atopic individuals have a very low risk of developing asthma.

Question 18
WHICH OF THE FOLLOWING IS NOT A RISK FACTOR FOR ASTHMA DEATHS?

a.PREVIOUS ADMISSIONS TO HOSPITAL WITH NEAR-FATAL ASTHMA


b.FAMILY HISTORY OF ASTHMA
c.LACK OF POOR COMPLIANCE WITH ICS THERAPY
d.FREQUENT USE OF BRONCHODILATORS

Harrisons 20th Ed Ch 281 p1958


Major risk factors for asthma deaths are poorly controlled disease with frequent use of bronchodilator inhalers,
lack of or poor compliance with ICS therapy, and previous admissions to hospital with near-fatal asthma.

Question 19
WHAT IS THE MOST COMMON REASON FOR POOR CONTROL OF ASTHMA?

a.NON-COMPLIANCE WITH MEDICATION


b.GERD
c.SEVERE RHINOSINUSITIS
d.STEROID RESISTANCE
Harrisons 20th ed Ch 281 p1968 Special Considerations: Mechanism
The most common reason for poor control of asthma is poor adherence with medication, particularly ICS.

Question 20
WHAT IS THE MOST COMMON SIDE EFFECT OF ANTICHOLINERGICS?

a.GLAUCOMA
b.PALPITATIONS
c.DRY MOUTH
d.URINARY RETENTION

Harrisons 18th Ed Ch 309 Asthma p 1677


Side effects are not usually a problem because there is little or no systemic absorption. The most common side effect is
dry mouth; in elderly patients, urinary retention and glaucoma may also be observed.

Question 21
THIS IS NOT A BRONCHODILATOR THERAPY FOR ASTHMA:

a.INHALED CORTICOSTEROIDS - ANTI-INFLAMMATORY NOT BRONCHODILATOR


b.BETA 2 AGONISTS
c.THEOPHYLLINE
d.ANTICHOLINERGICS

Question 22
THIS MUSCARINIC RECEPTOR ANTAGONIST PREVENTS BRONCHOCONSTRICTION AND MUCUS
SECRETION IN ASTHMA:

a.THEOPHYLLINE
b.IPRATROPIUM
c.MONTELUKAST
d.SALMETEROL

Ratio: Ipratropium: Uses, Interactions, Mechanism of Action | DrugBank Online

Harrisons 20th ed Ch 281 p1965 Treatment: Anticholinergics


Muscarinic receptor antagonists, such as ipratropium bromide, prevent cholinergic nerve-induced bronchoconstriction
and mucus secretion.

Question 23
THIS IS NOT AN EFFECT OF BETA 2 AGONISTS IN ASTHMA:

a.DECREASE IN AND CONTROL OF CHRONIC INFLAMMATION - bronchodilator, not anti-inflammatory


b.REDUCTION OF PLASMA EXUDATION AND AIRWAY EDEMA
c.INHIBITION OF MAST CELL MEDIATOR RELEASE
d.INCREASED MUCOCILIARY CLEARANCE

Ratio: Harri 20th Ed Ch 281 p1965


Mode of Action
The primary action of β2-agonists is to relax airway smooth-muscle cells of all airways, where they act as
functional antagonists, reversing and preventing contraction of airway smooth-muscle cells by all known
bronchoconstrictors. This generalized action is likely to account for their great efficacy as bronchodilators in
asthma. There are also additional non-bronchodilator effects that may be clinically useful, including inhibition
of mast cell mediator release, reduction in plasma exudation, and inhibition of sensory nerve
activation. Inflammatory cells express small numbers of β2-receptors, but these are rapidly down-regulated
with β2-agonist activation so that, in contrast to corticosteroids, there are no effects on inflammatory cells in the
airways and there is no reduction in AHR.

Question 24
WHICH OF THE FOLLOWING MICROORGANISMS IS/ARE IMPLICATED AS A CAUSE OF
BRONCHIECTASIS:

a.PSEUDOMONAS AERUGINOSA
b.HAEMOPHILUS INFLUENZAE
c.ALL OF THE ABOVE
d.NONE OF THE ABOVE
e.MYCOBACTERIUM AVIUM-INTRACELLULARE COMPLEX

Ratio: Harri 19th Ed p1694 and 1696

Question 25
BRONCHIECTASIS IS TYPICALLY ASSOCIATED WITH THIS PATTERN ON PULMONARY FUNCTION
TESTING:
a.OBSTRUCTIVE → COPD, Bronchiectasis etc
b.RESTRICTIVE
c.NEITHER A NOR B
d.BOTH A AND B

Ratio: Harri 19th Ed p1695


Question 26
THE FOLLOWING IS/ARE CAUSES OF DIFFUSE BRONCHIECTASIS:
a.NONE OF THE ABOVE
b.ALL OF THE ABOVE
c.BACTERIAL OR NONTUBERCULOUS MYCOBACTERIAL INFECTION
d.RECURRENT INFECTION
e.CYSTIC FIBROSIS

Harrisons 20th Ed Ch 284 p1983


Diffuse bronchiectasis is characterized by widespread bronchiectatic changes throughout the lung and often arises from
an underlying systemic or infectious disease process.
PATTERN OF LUNG INVOLVEMENT - Diffuse
1. Infection (bacterial, nontuberculous mycobacterial)
2. Immunodeficiency (hypogammaglobulinemia, HIV infection, bronchiolitis obliterans after lung transplantation)
3. Genetic causes (cystic fibrosis, Kartagener’s syndrome, α1 antitrypsin deficiency)
4. Autoimmune or rheumatologic causes (rheumatoid arthritis, Sjögren’s syndrome, inflammatory bowel disease);
immune-mediated disease (allergic bronchopulmonary aspergillosis)
5. Recurrent aspiration
6. Miscellaneous (yellow nail syndrome, traction bronchiectasis from postradiation fibrosis or idiopathic pulmonary
fibrosis)
7. Idiopathic
Question 27
TRUE STATEMENTS ABOUT BRONCHIECTASIS SECONDARY TO TUBERCULOSIS:
a.DIAGNOSED BY MEASUREMENT OF CHLORIDE LEVELS IN SWEAT
b.NONE OF THE ABOVE
c.ALL OF THE ABOVE
d.MALNUTRITION IS FREQUENTLY A CONFOUNDING PROBLEM
e.OFTEN OCCURS AS A SEQUELA OF GRANULOMATOUS INFECTION

Harrisons 20th Ed Ch 284 p1984


In areas where tuberculosis is prevalent, bronchiectasis more frequently occurs as a sequela of granulomatous
infection.

Question 28
THE CURRENT STANDARD IMAGING TECHNIQUE FOR DETECTING OR CONFIRMING
BRONCHIECTASIS
a.CXR
b.BRONCHOGRAPHY
c.CHEST CT SCAN
d.MRI
Ratio: Harri 19th Ed 1695

Question 29
TYPICAL CHEST X-RAY FINDING IN BRONCHIECTASIS
a.SIGNET RING
b.LACK OF BRONCHIAL TAPERING
c.TREE IN BUD PATTERN
d.TRAM TRACKS APPEARANCE

Harrison 20th Ed Ch 284 p1984 Diagnosis


Although chest radiographs lack sensitivity, the presence of “tram tracks” indicating dilated airways is consistent with
bronchiectasis.

Question 30
CYANOSIS IS NOT USUALLY SEEN IN THESE AREAS OF THE BODY:

a.MALAR EMINENCES
b.EAR
c.EYES
d.LIPS

Harrisons 20th Ed Ch 36 p236


It is usually most marked in the lips, nail beds, ears, and malar eminences.

Question 31
WHICH OF THE FOLLOWING OCCURS IN SEVERE HYPOXIA, WHEN ATP PRODUCTION IS
INADEQUATE TO MEET THE ENERGY REQUIREMENTS OF IONIC AND OSMOTIC EQUILIBRIUM WHICH
LEADS TO CELL SWELLING AND ULTIMATELY DEATH:
a.CELL MEMBRANE DEPOLARIZATION AND UNCONTROLLED SODIUM INFLUX
b.CELL MEMBRANE DEPOLARIZATION AND UNCONTROLLED CALCIUM INFLUX
c.CELL MEMBRANE REPOLARIZATION AND UNCONTROLLED POTASSIUM INFLUX
d.CELL MEMBRANE REPLORIZATION AND UNCONTROLLED CHLORIDE INFLUX

Harrisons 20th Ed Ch 36 p234


In severe hypoxia, when ATP production is inadequate to meet the energy requirements of ionic and osmotic equilibrium,
cell membrane depolarization leads to uncontrolled Ca2+ influx and activation of Ca2+-dependent
phospholipases and proteases. These events, in turn, cause cell swelling, activation of apoptotic pathways, and,
ultimately, cell death.

Question 32
THE VOLUME OF GAS THAT IS EXHALED FROM THE LUNGS IN GOING FROM TOTAL LUNG CAPACITY
TO RESIDUAL VOLUME IS KNOWN AS:

a.FORCED EXPIRATORY VOLUME


b.FUNCTIONAL RESIDUAL CAPACITY
c.MAXIMAL MIDEXPIRATORY FLOW RATE
d.VITAL CAPACITY

Harrison’s 20th Ed Ch 279 pg. 1946


The excursion between full and minimal lung inflation is called vital capacity and is readily seen to be the difference
between volumes at two unrelated stiffness extremes—one determined by the lung (TLC) and the other by the chest
wall or airways (RV).

Question 33
THE SWITCH FROM AEROBIC TO ANAEROBIC METABOLISM, MAINTAINS SOME, ALBEIT REDUCED,
ATP PRODUCTION IS CALLED:

a.THE PASTEUR EFFECT


b.THE SEYER EFFECT
c.THE KRUKENBERG EFFECT
d.THE BOHR EFFECT

Harrisons 20th Ed Ch 36 p234


Decreased O2 availability to cells results in an inhibition of oxidative phosphorylation and increased anaerobic glycolysis.
This switch from aerobic to anaerobic metabolism, the Pasteur effect, reduces the rate of adenosine 5′-triphosphate
(ATP) production.

Question 34
CLUBBING OF DIGITS IS NOT SEEN IN:

a.CYSTIC FIBROSIS
b.IDIOPATHIC PULMONARY FIBROSIS
c.LUNG CANCER
d.COPD

Harrison 20th Ed Ch 286 p1995


Clubbing of the digits is not a sign of COPD, and its presence should alert the clinician to initiate an investigation for
causes of clubbing.
Question 35
THIS TERM IMPLIES A TENDENCY TO MANIFEST ASTHMA, RHINITIS, URTICARIA, IN ASSOCIATION
WITH THE PRESENCE OF ALLERGEN SPECIFIC IgE:

a.ATOPY
b.ANAPHYLAXIS
c.ALLERGY
d.SENSITIZATION

Harrisons 20th Ed Ch 281 p1958 Risk Factors & Triggers


ATOPY:
Patients with asthma commonly suffer from other atopic diseases, particularly allergic rhinitis, which may be found in
>80% of asthmatic patients, and atopic dermatitis (eczema). Atopy is due to the genetically determined production of
specific IgE antibody, with many patients showing a family history of allergic diseases.

Harrisons 20E Ch 364 p2498


Question 36
MASSAGE OR GENTLE WARMING OF A CYANOTIC EXTREMITY ABOLISHES WHAT TYPE OF
CYANOSIS:
a.BOTH
b.NEITHER
c.PERIPHERAL
d.CENTRAL

Harrison 20th Ed Ch 36 p237


Massage or gentle warming of a cyanotic extremity will increase peripheral blood flow and abolish peripheral, but not
central, cyanosis.

Question 37
WHAT IS/ARE AN EXAMPLE OF PERIPHERAL CYANOSIS

a.CARBOXYHEMOGLOBINEMIA
b.MULTIPLE SMALL INTRAPULMONARY SHUNTS
c.COLD EXPOSURE
d.DECREASED ATMOSPHERIC PRESSURE

Harrisons 20th Ed Ch 36 p237


Peripheral Cyanosis
Probably the most common cause of peripheral cyanosis is the normal vasoconstriction resulting from exposure to
cold air or water.

Question 38
WHICH OF THE FOLLOWING IS NOT A CAUSE OF CENTRAL CYANOSIS

a.DECREASED ARTERIAL OXYGEN SATURATION


b.COLD EXPOSURE
c.METHEMOGLOBINEMIA
d.PULMONARY ARTERIOVENOUS FISTULAS

Harrisons 20th Ed Ch 36 p236

Question 39
IN THIS TYPE OF CYANOSIS, IT IS DUE TO A SLOWING OF BLOOD FLOW AND ABNORMALLY GREAT
EXTRACTION OF 02 FROM NORMALLY SATURATED ARTERIAL BLOOD

a.PERIPHERAL CYANOSIS
b.BOTH
c.NEITHER
d.CENTRAL CYANOSIS

Harrisons 20th Ed Ch 36 p236


Peripheral cyanosis is due to a slowing of blood flow and abnormally great extraction of O2 from normally saturated
arterial blood.

Question 40
IN THIS TYPE OF CYANOSIS, THE SA02 IS REDUCED OR AN ABNORMAL HEMOGLOBIN DERIVATIVE IS
PRESENT

a.BOTH
b.NEITHER
c.PERIPHERAL CYANOSIS
d.CENTRAL CYANOSIS

Harrisons 20th Ed Ch 36 p236


In central cyanosis, the Sao2 is reduced or an abnormal hemoglobin derivative is present.

Question 41
IN GENERAL, CYANOSIS BECOMES APPARENT WHEN THE CONCENTRATION OF REDUCED
HEMOGLOBIN IN CAPILLARY BLOOD EXCEEDS:

a.40 G/L
b.30 G/L
c.20 G/L
d.50 G/L

Harrisons 20th Ed Ch 36 p236. In general, cyanosis becomes apparent when the concentration of reduced hemoglobin in
capillary blood exceeds 40 g/L (4 g/dL).
Question 42
THIS RESULTS IN AN INHIBITION OF OXIDATIVE PHOSPHORYLATION AND INCREASED ANAEROBIC
GLYCOLYSIS IN SOME CELLS:

a.DECREASED O2 AVAILABILITY
b.INCREASED CO2 AVAILABILITY
c.DECREASED CO2 AVAILABILITY
d.INCREASED 02 AVAILABILITY

Harrisons 20th Ed Ch 36 p234


Decreased O2 availability to cells results in an inhibition of oxidative phosphorylation and increased anaerobic
glycolysis.

Question 43
A TOTAL LUNG CAPACITY <80% OF THE PREDICTED VALUE FOR A PATIENT’S AGE, RACE, SEX, AND
HEIGHT DEFINES WHAT TYPE OF PATHOPHYSIOLOGY?

a.RESTRICTIVE
b.ATELECTATIC
c.OBSTRUCTIVE
d.EDEMATOUS

Harrisons 18th Ed Ch 305 p1662


A total lung capacity <80% of the predicted value for a patient’s age, race, sex, and height defines restrictive
pathophysiology.

Question 44
THE FOLLOWING IS AN EXAMPLE OF TYPE 4 HYPERSENSITIVTY REACTION
a.URTICARIA
b.TUBERCULIN SKIN TEST
c.DRUG-INDUCED LUPUS
d.SERUM SICKNESS
Ratio: PPD Skin Test - StatPearls - NCBI Bookshelf
a. Type 1
b. Type 3
c. Type 4
d. type 3
Purified protein derivative test (PPD skin test), administered through the Mantoux technique, is a
type IV hypersensitivity skin reaction to 'tuberculin. ' Therefore, also known as the tuberculin
skin test (TST skin test) and Mantoux test.Peb 4, 2021

Question 45
THE KEY IMMUNE MEDIATORS IN TYPE 3 HYPERSENSITIVITY REACTION
a.IgG + ANTIGEN
b.IgG
c.CYTOTOXIC T LYMPHOCYTES
Ratio: Robbins 10th Ed Ch 6 p205

In immune complex–mediated disorders (type III hypersensitivity), IgG and IgM antibodies bind antigens
usually in the circulation, and the antigen-antibody complexes deposit in tissues and induce inflammation

Question 46
THE KEY PATHWAY FOR TYPE 3 HYPERSENSITIVITY REACTION

a.IgG
b.T LYMPHOCYTE
c.IgE
d.IMMUNE COMPLEX
Ratio: IM plat p233
Robbins 9E pg. 201

Question 47
TYPE OF HYPERSENSITIVITY REACTION IN WHICH THE KEY IMMUNE MEDIATOR ARE NEUTROPHILS

a.TYPE 3
b.TYPE 4D
c.TYPE 2 -??
d.TYPE 4A

Question 48
THE BINDING OF IgE TO HUMAN MAST CELLS AND BASOPHILS, WHICH PREPARES THESE CELLS
FOR SUBSEQUENT ANTIGEN-SPECIFIC ACTIVATION:

a.ALLERGY
b.SENSITIZATION
c.ANAPHYLAXIS
d.ATOPY

Question 49
THE KEY PATHWAY FOR TYPE 2 HYPERSENSITIVITY REACTION:

a.T-LYMPHOCYTE-MEDIATED MACROPHAGE INFLAMMATION


b.IgG-MEDIATED CYTOTOXITY(?)
c.IgE
d.IgG

Question 50
CYANOSIS REFERS TO A BLUISH COLOR OF THE SKIN AND MUCOUS MEMBRANES RESULTING
FROM:

a.BLOOD LOSS
b.CONGENITAL PROCESSES
c.DECREASED QUANTITY OF SULFHEMOGLOBIN
d.INCREASED QUANTITY OF DEOXYGENATED HEMOGLOBIN

Harrisons 20th Ed Ch 36 p236


Cyanosis refers to a bluish color of the skin and mucous membranes resulting from an increased quantity of reduced
hemoglobin (i.e., deoxygenated hemoglobin) or of hemoglobin derivatives (e.g., methemoglobin or sulfhemoglobin) in the
small blood vessels of those tissues.

Question 51
THIS IS NOT A RECOMMENDED TREATMENT MODALITY FOR COPD:

a.TIOTROPIUM
b.THEOPHYLLINE
c.CHRONIC ORAL GLUCOCORTICOIDS
d.BETA AGONSITS

Harrisons 20th Ed Ch 286 p 1997 Oral Glucocorticoids


The chronic use of oral glucocorticoids for treatment of COPD is not recommended because of an unfavorable
benefit/risk ratio.

Question 52
IN ADVANCED COPD, THE PARADOXICAL INWARD MOVEMENT OF THE RIB CAGE WITH INSPIRATION
IS KNOWN AS:

a.MITCHELL’S SIGN
b.THOMPSON’S SIGN
c.MASTER’S SIGN
d.HOOVER’S SIGN
Harrions 20th Ed Ch 286 p1995
Some patients with advanced disease have paradoxical inward movement of the rib cage with inspiration (Hoover’s
sign), the result of alteration of the vector of diaphragmatic contraction on the rib cage as a result of chronic
hyperinflation.

Question 53
THE FOLLOWING IS NOT AN EFFECTIVE MANAGEMENT OF ACUTE EXACERBATIONS OF COPD:

a.GLUCOCORTICOIDS
b.ANTIBIOTICS
c.LEUKOTRIENE INHIBITORS
d.INHALED BRONCHODILATORS

Harrisons 20th Ed Ch 286 p 1998


Treatment of Acute Bronchodilator
1. Bronchodilator (inhaled β agonists and muscarinic antagonists)
2. Antibiotics
a. Systemic glucocorticoids - reduces the length of stay, hastens recovery, and reduces the chance of
subsequent exacerbation or relapse
3. Oxygen
4. Mechanical Ventilation Support

Question 54
THE ONLY THERAPY DEMONSTRATED TO DECREASE MORTALITY IN COPD:

a.INHALED GLUCOCORTICOIDS
b.SUPPLEMENTAL OXYGEN
c.PULMONARY REHABILITATION
d.BRONCHODILATORS

Harrison 20th Ed Ch 286 p1997


Supplemental O2 is the only pharmacologic therapy demonstrated to unequivocally decrease mortality rates in patients
with COPD.

THE FOLLOWING IS NOT AN INTERVENTION DEMONSTRATED TO INLFUENCE THE NATURAL


HISTORY OF COPD PATIENTS:

a.LUNG VOLUME SURGERY


b.INHALED CORTICOSTEROIDS
c.SMOKING CESSATION
d.OXYGEN THERAPY

Harrisons 20th Ed Ch 286 p1996


Only three interventions—smoking cessation, oxygen therapy in chronically hypoxemic patients, and lung volume
reduction surgery (LVRS) in selected patients with emphysema—have been demonstrated to improve survival of
patients with COPD.

56.WHICH OF THE FOLLOWING IS NOT A PHYSICAL SIGN ASSOCIATED WITH COPD?


a.BARREL CHEST
B.TRIPOD POSTURE
c.CLUBBING OF DIGITS
d.WEIGHT LOSS
Harrison 20th Ed Ch 286 p1995
Clubbing of the digits is not a sign of COPD, and its presence should alert the clinician to initiate an investigation for
causes of clubbing.

57.THE MOST FREQUENT TYPE OF EMPHYSEMA ASSOCIATED WITH CIGARETTE SMOKING:

a.MUTILOBULAR
b.FOCAL
c.CENTRIACINAR
d.PANACINAR

Harrisons 20th Ed Ch 286 p1992


Centrilobular emphysema, the type most frequently associated with cigarette smoking, is characterized by enlarged
air spaces found (initially) in association with respiratory bronchioles.

58. HYPOXEMIA IN COPD IS ALMOST ALWAYS A RESULT OF:

a.VENTILATION PERFUSION MISMATCHING


b.DECREASED RESPIRATORY DRIVE
c.ATTENDANT HYPERCARBIA
d.SHUNTING

Harrisons 20th Ed Ch 286 p1992


Non-uniform ventilation and ventilation-perfusion mismatching are characteristic of COPD, reflecting the
heterogeneous nature of the disease process within the airways and lung parenchyma.

59. THE MOST HIGHLY SIGNIFICANT PREDICTOR OF FEV1 IN COPD:

a.PREVIOUS HISTORY OF ASTHMA


b.CHRONIC BRONCHITIS
c.PACK YEARS OF CIGARETTE SMOKING
d.LACK OF ELASTIC RECOIL

Harrisons 20th ed Ch 286 p 1993 Risk Factors


Subsequent longitudinal studies have shown accelerated decline in FEV1 in a dose response relationship to the intensity
of cigarette smoking, which is typically expressed as pack-years (average number of packs of cigarettes smoked per
day multiplied by the total number of years of smoking).

60. WHAT IS THE INITIAL TEST OBTAINED TO ASSESS FOR OBSTRUCTIVE PATHOPHYSIOLOGY AS
SEEN IN ASTHMA, COPD AND BRONCHIECTASIS?

a.CHEST X-RAY
b.SPIROMETRY
c.BRONCHOSCOPY
d.ARTERIAL BLOOD GAS

Harrisons 20th Ed Ch 278 p 1944


The initial pulmonary function test obtained is spirometry. This study is an effort-dependent test used to assess for
obstructive pathophysiology as seen in asthma, COPD, and bronchiectasis.

61. WHICH OF THE FOLLOWING PULMONARY FUNCTION TEST ABNORMALITIES IS DIAGNOSTIC OF


OBSTRUCTION?
a.DECREASED TOTAL LUNG CAPACITY
b.DECREASED RESIDUAL VOLUME
c.DECREASED FEV1/FORCED VITAL CAPACITY
d.INCREASED DIFFUSION CAPACITY

Harrison 20th Ed Ch 278 p1944


A diminished-forced expiratory volume in 1 s (FEV1)/forced vital capacity (FVC) (often defined as <70%) is diagnostic
of obstruction.

62. IN ADDITION TO MEASURING FEV1/FVC, THE CLINICIAN SHOULD EXAMINE THE FLOW-VOLUME
LOOP WHICH IS EFFORT DEPENDENT. A PLATEAU OF THE INSPIRATORY AND EXPIRATORY CURVES
SUGGESTS WHAT TYPE OF OBSTRUCTION IN INTRATHORACIC AND EXTRATHORACIC LOCATIONS
RESPECTIVELY?

a.SMALL-AIRWAY OBSTRUCTION
b.LARGE-AIRWAY OBSTRUCTION
c.NONE OF THE ABOVE
d.BOTH A AND B

Harrisons 20th Ed Ch 279 p1944


A diminished-forced expiratory volume in 1 s (FEV 1)/forced vital capacity (FVC) (often defined as <70%) is diagnostic of
obstruction. In addition to measuring FEV1 and FVC, the clinician should examine the flow-volume loop (which is less
effort-dependent). A plateau of the inspiratory and expiratory curves suggests large-airway obstruction in extrathoracic
and intrathoracic locations, respectively.

63. Infectious agents that cause pediatric community-acquired pneumonia _________.

a.are all viruses


b.vary by age
c.are likely bacteria
d.are parasites

64. THE FOLLOWING IS NOT AN ATYPICAL MICROORGANISM CAUSING COMMUNITY-ACQUIRED


PNEUMONIA:

a.CHLAMYDIA
b.MYCOPLASMA
c.LEGIONELLA
d.KLEBSIELLA

Harrisons 20th Ed Ch121 p910


The “atypical” organisms include Mycoplasma pneumoniae, Chlamydia pneumoniae, and Legionella species as well
as respiratory viruses such as influenza viruses, adenoviruses, human metapneumovirus, and respiratory syncytial
viruses.

65. TO BE ADEQUATE FOR CULTURE, A SPUTUM SAMPLE IN PATIENTS WITH PNEUMONIA SHOULD
HAVE:

a.NUMEROUS GOBLET CELLS SEEN


b.ALL OF THE ABOVE
c.A AND B ONLY (?)
d.<10 SQUAMOUS EPITHELIAL CELLS/LPF
e.>25 POLYMORPHONUCLEARS/LPF
Harrisons 20th Ed Ch 121 p911
To be adequate for culture, a sputum sample must have >25 neutrophils and <10 squamous epithelial cells per
low-power field.

66. The following statements is/are not true of pneumonia:

a.CBC is routinely recommended


b.Chest x-ray is confirmatory
c.Blood cultures are routinely recommended to all patients
d.Gram Staining is also a recommendation

Harrison 20th Ed Ch 121 p911


Because of the low yield and the lack of significant impact on outcome, blood cultures are no longer considered de
rigueur for all hospitalized CAP patients.

67. Which of the following statement/s is/are true of pneumonia:

a.Atypical pneumonia in young children will present as cough, tachypnea, crackles on auscultation and
concomitant chlamydial conjunctivitis.
b.Bacterial pneumonia will present as high fever, chills, cough, dyspnea and auscultatory findings of lung
consolidation
c.All of the above
d.Viral pneumonia usually presents as cough, wheezing, stridor and less prominent fever

68. Which of the following risk classification of PCAP need to be admitted?

a.PCAP A & PCAP B


b.PCAP C & PCAP D
c.PCAP D & PCAP A
d.PCAP B & PCAP C

PCAP A & B - Outpatient (site of care)


PCAP C & D - Admit (on ward for C; to a critical care facility for D)

69. The following IS NOT A complication of pneumonia:

a.Meningitis
b.Pericarditis
c.Pleural effusion
d.Otitis Media

Nelsons 20th Ed Ch 428 p2273


Complications of pneumonia are usually the result of direct spread of bacterial infection within the thoracic cavity (pleural
effusion, empyema, and pericarditis) or bacteremia and hematologic spread. Meningitis, endocarditis, suppurative
arthritis, and osteomyelitis are rare complications of hematologic spread of pneumococcal or H. influenzae type b
infection.

70. WHICH OF THE FOLLOWING IS SLOWEST TO RESOLVE IN PNEUMONIA?

a.FEVER
b.LEUKOCYTOSIS
c.CRACKLES
d.CHEST RADIOGRAPHIC ABNORMALITIES

Harrisons 20th Ed Ch 121 p914


Fever and leukocytosis usually resolve within 2–4 days in otherwise healthy patients with CAP, but physical findings
may persist longer. Chest radiographic abnormalities are slowest to resolve (4–12 weeks), with the speed of
clearance depending on the patient’s age and
underlying lung disease.

71. which of The following is not a factor suggesting need for hospitalization of children with pneumonia:

a.Age <6 mo
b.Vomiting
c.Single lobe involvement
d.Social factors (e.g. caregiver)

Nelsons 20th Ed Ch 428 CAP p2271

72. THE MOST COMMON CAUSE OF COMMUNITY ACQUIRED PNEUMONIA:

a.H. INFLUENZAE
b.S. PNEUMONIAE
c.S. AUREUS
d.M. PNEUMONIAE

Harrisons 20th Ed Ch 121 p910


Although Streptococcus pneumoniae is most common, other organisms must also be considered in light of the
patient’s risk factors and severity of illness.

73. THE MOST COMMON PATHOGENETIC MECHANISM FOR THE PRODUCTION OF PNEUMONIA?

a.INHALATION OF INFECTIOUS AEROSOLS


b.ASPIRATION OF PULMONARY PATHOGENS FROM THE OROPHARYNX
c.NONE OF THE ABOVE
d.HEMATOGENOUS DISSEMINATION

Harrison 20th Ed Ch 121 p909


The most common is by aspiration from the oropharynx.

74. IT IS CONSIDERED THE MAJOR RISK FACTOR FOR PNEUMONIA CAUSED BY ANAEROBES ASIDE
FROM ASPIRATION:

a.OLD AGE dili old age?


b.SIGNIFICANT GINGIVITIS
c.PRESENCE OF EMPYEMA
d.BACTERIAL ENDOCARDITIS

Harrisons 20th Ed Ch 121 p910


Anaerobes play a significant role only when an episode of aspiration has occurred days to weeks before presentation for
pneumonia. The combination of an unprotected airway and significant gingivitis constitutes the major risk factor.

Question 75
OUTPATIENT TREATMENT OF CHOICE FOR CAP WITH NO CO-MORBIDITIES AND NO ANTIBIOTICS IN
THE PAST 3 MONTHS:

a.COAMOXICLAV
b.CEFUROXIME
c.RESPIRATORY FLUOROQUINOLONES
d.MACROLIDE

2016 CAP Guidelines p5


For low-risk CAP without comorbid illness, AMOXICILLIN remains the standard drug of choice. Use of extended
macrolides may also be considered.

76. THE FOLLOWING DISEASES OR STATES SUGGESTS RESTRICTION

a.PARENCHYMAL DISEASES
b.NEUROMUSCULAR WEAKNESS
c.ALL OF THE ABOVE
d.PLEURAL DISEASES

Harrisons 20th ed Ch 278 p1944


Restriction can result from parenchymal disease, neuromuscular weakness, or chest wall or pleural diseases
(Table 278-1 p1943).
77. RECOMMENDED TREATMENT REGIMEN FOR TB DURING PREGNANCY:

a.3HRS
b.2HRE 7HR - HARRISON’S 20TH ED p1251 TABLE 173-3
c.2HRZE 4HR
d.2HRZE 2 HR

78. LEAST HEPATOTOXIC ANTI-TB DRUG

a.RIFAMPICIN
b.ISONIAZID
c.ETHAMBUTOL
d.PYRAZINAMIDE ??
79. THE FOLLOWING STATEMENTS IS NOT TRUE REGARDING TUBERCULOSIS:

a.TRANSMISSION OCCURS THRU AIRBORNE SPREAD OF DROPLET NUCLEI FROM AN INFECTED


PERSON
b.MONITORING OF THE RESPONSE TO TREATMENT BY SERIAL CXRs IS NOT RECOMMENDED
c.PATIENTS WITH EXTRAPULMONARY TB ARE ESSENTIALLY INFECTIOUS
d.PRIMARY TB MOST COMMONLY INVOLVES THE MIDDLE AND LOWER LUNG ZONES

Harrisons 20th Ed Ch 173 p1238


Those with culture-negative pulmonary TB and extrapulmonary TB are essentially noninfectious.

80. THE FOLLOWING IS A TRUE STATEMENT REGARDING MYCOBACTERIUM TUBERCULOSIS


BACTERIA:

a.IS A ROD-SHAPED, NON-SPORE FORMING THIN AEROBIC BACTERIUM - Harrisons


b.IS A ROD-SHAPED, SPORE-FORMING, THIN ANAEROBIC BACTERIUM
c.IS A ROD-SHAPED, NON-SPORE FORMING, THICK ANAEROBIC BACTERIUM
d.IS A ROD-SHAPED, SPORE-FORMING, THIN AEROBIC BACTERIUM

Harrisons 20th Ed Ch 173 p1236


M. tuberculosis is a rod-shaped, non-spore-forming, thin aerobic bacterium measuring 0.5 μm by 3 μm.

81. THE FOLLOWING STATEMENTS IS/ARE NOT TRUE REGARDING THE ACID-FASTNESS OF
MYCOBACTERIUM:
a.IT IS DUE MAINLY TO THE ORGANISMS’ LOW CONTENT OF MYCOLIC ACIDS
b.THE BACILLI CANNOT BE DECOLORIZED BY ACID ALCOHOL
c.NONE OF THE ABOVE
d.IT IS NEUTRAL ON GRAM’S STAINING

Harrisons 20th Ed Ch 173 p1236


Acid fastness is due mainly to the organisms’ high content of mycolic acids, long-chain cross-linked fatty acids, and
other cell-wall lipids.

82. THE FOLLOWING IS NOT A COMPONENT OF THE MYCOBACTERIAL CELL WALL:


a.LIPIDS
b.NONE OF THE ABOVE
c.ARABINOGALACTAN
d.PEPTIDOGLYCAN

Harrisons 20th Ed Ch 173 p1236


In the mycobacterial cell wall, lipids (e.g., mycolic acids) are linked to underlying arabinogalactan and peptidoglycan.
This structure results in very low permeability of the cell wall, thus reducing the effectiveness of most antibiotics.

83. THIS MOLECULE IN THE MYCOBACTERIAL CELL WALL IS INVOLVED IN THE PATHOGEN-HOST
INTERACTION AND FACILITATES SURVIVAL WITHIN MACROPHAGES:
a.ARABINOGALACTAMANNAN
b.PEPTIDOGLYCAN
c.ARABINOGALACTAN
d.LIPOARABINOMANNAN

Harrisons 20th Ed Ch 173 p 1236


Another molecule in the mycobacterial cell wall, lipoarabinomannan, is involved in the pathogen–host interaction and
facilitates the survival of M. tuberculosis within macrophages.

84. THERE MAY BE AS MANY AS _______ INFLECTIOUS NUCLEI PER COUGH OF AN INFECTED TB
INDIVIDUAL:

a.5000
b.4000
C.3000 - Harrisons

Harrisons 20th Ed Ch 173 p1238


There may be as many as 3000 infectious nuclei per cough.

85. PRIMARY TB IS COMMON AMONG:


a.NEITHER A NOR B
b.CHILDREN IN THE FIRST FEW YEARS OF LIFE
c.IMMUNOCOMPROMISED PERSONS
d.BOTH A AND B

Harrisons 20th Ed Ch 173 p1242


Small granulomatous lesions develop in multiple organs and may cause locally progressive disease or result in
tuberculous meningitis; this is a particular concern in very young children and immunocompromised persons (e.g.,
patients with HIV infection).

86. THE FOLLOWING STATEMENT/S IS/ARE NOT TRUE REGARDING PRIMARY TB DISEASE:
a.MAY PRESENT WITH FEVER
b.IT MAY BE ASYMPTOMATIC
c.MAY PRESENT OCCASIONAL PLEURITIC CHEST PAIN
d.IT OCCURS RIGHT AFTER THE INITIAL INFECTION WITH TUBERCLE BACILLI

Harrisons 20th Ed Ch 173 p1241


Primary pulmonary TB occurs soon after the initial infection with tubercle bacilli. It may be asymptomatic or may
present with fever and occasionally pleuritic chest pain.

87. WHAT COMPROMISES THE GHON COMPLEX?


a.PLEURAL THINNING
b.THE LESION FORMED AFTER INITIAL INFECTION
c.EITHER A OR B
d.PRESENCE OF REGIONAL LYMPHADENOPATHY
e.BOTH A AND C

Ratio: Ghon complex


a. Ghon focus - lesion formed after initial infection
b. Pleuritic chest pain
c. Thickening
d. Regional LAD

Harrisons 20th Ed Ch 173 p1241


The Ghon focus, with or without overlying pleural reaction, thickening, and regional lymphadenopathy, is referred
to as the Ghon complex.

88. IN POSTPRIMARY (ADULT-TYPE) DISEASE, WHAT SEGMENTS OF THE LUNGS ARE USUALLY
INVOLVED:
a.APICAL AND POSTERIOR SEGMENTS OF THE UPPER LOBES ONLY
b.APICAL AND ANTERIOR SEGMENTS OF THE UPPER LOBES AND THE SUPERIOR SEGMENTS OF
THE LOWER LOBES
c.SUPERIOR SEGMENTS OF THE LOWER LOBES ONLY
d.APICAL AND POSTERIOR SEGMENTS OF THE UPPER LOBES AND THE SUPERIOR SEGMENTS OF
THE LOWER LOBES

Harrisons 20th Ed Chapter 173 p1242


It is usually localized to the apical and posterior segments of the upper lobes, where the substantially higher mean
oxygen tension (compared with that in the lower zones) favors mycobacterial growth. The superior segments of the
lower lobes are also more frequently involved.
UPPER LOBE - Apical & Posterior
LOWER LOBE - Superior

89. HEMOPTYSIS, HOWEVER, MAY ALSO RESULT FROM RUPTURE OF A DILATED VESSEL IN A
CAVITY CALLED:
a.KIKUCHI’S ANEURYSM
b.RASMUSSEN’S ANEURYSM
c.KOCH’S ANEURYSM
d.GHON’S ANEURYSM

Harrisons 20th Ed Ch 173 p 1243


Hemoptysis develops in 20–30% of cases, and massive hemoptysis may ensue as a consequence of the erosion of a
blood vessel in the wall of a cavity. Hemoptysis, however, may also result from rupture of a dilated vessel in a cavity
(Rasmussen’s aneurysm) or from aspergilloma formation in an old cavity.

90.WHICH OF THE FOLLOWING IS NOT A FIRST LINE DRUG FOR TB:


a.ISONIAZID
b.ETHAMBUTOL
c.RIFAMPICIN
d.STREPTOMYCIN
Harrisons 20th Ed. Ch 173, p 1250
Four major drugs are considered first-line agents for the treatment of
TB: isoniazid, rifampin, pyrazinamide, and ethambutol.

91.POTT’S DISEASE USUALLY AFFECTS WHICH PART OF THE SPINE IN ADULTS?

a.UPPER THORACIC AND UPPER LUMBAR


b.LOWER THORACIC AND LOWER LUMBAR
c.LOWER THORACIC AND UPPER LUMBAR
d.UPPER THORACIC AND LOWER LUMBAR

Harrisons 20th Ed Ch173 p1244


Spinal TB (Pott’s disease or tuberculous spondylitis) often involves two or more adjacent vertebral bodies. Whereas the
upper thoracic spine is the most common site of spinal TB in children, the lower thoracic and upper lumbar vertebrae
are usually affected in adults.

92. TREATMENT REGIMEN OF CHOICE FOR VIRTUALLY ALL FORMS OF TB IN ADULTS AND
CHILDREN:

a.2HRZE 4 HR
b.3HRZE 6 HR
c.2 HRZE 4 HRE
d.NONE OF THE ABOVE

93.THE MOST COMMON SITE OF EXTRAPULMONARY TB:


a.GI TRACT
b.PLEURA
c.LYMPH NODES
d.BONES

Harrisons 20th Ed Ch 173 p1243


In descending order of frequency, the extrapulmonary sites most commonly involved in TB are the lymph nodes,
pleura, genitourinary tract, bones and joints, meninges, peritoneum, and pericardium.
LOWER RESPI OSCE EXAM
7 June - 13 June

Fortunato Tan 54 yr old male consulted for shortness of breath.


1 month PTA he noticed exertional dyspnea on climbing 2 flights of stairs.
3 weeks PTA he experienced shortness of breath while walking on level ground which is relieved by rest.
1 week PTA he had easy faguability and shortness of breath after walking short distances.
4days PTA he developed productive cough with yellowish sputum, not relieved by Carbocysteine associated
with sensation of difficulty breathing on lying flat in bed.
On the day of consult, patient suddenly woke up gasoing for breath prompting patient to seek admission.

1. What additional data regarding the patient would help you identify the cause of his complaint?
(may have more than 1 answer)

a.Patient works as a security guard in a cement factory


b.He smokes 1 pack of cigarette per day
c.He is a regular alcoholic drinker
d.He has family history of hypertension

Question 2
Match the differential diagnosis that you would consider given the following data in the patient’s history from
the choices above?

Patient works as a security guard in a cement factory:

a.Bronchial Asthma
b.Chronic Obstructive Pulmonary Disease - ???
c.Congestive Heart Failure
d.Community Acquired Pneumonia

Question 3
Match the differential diagnosis that you would consider given the following data in the patient’s history from
the choices above?

Patient smokes 1 pack of cigarettes per day and is hypertensive without intake of medications

a.Bronchial Aasthma
b.Pulmonary Tuberculosis
c.Congestive Heart Failure??
d.Community Acquired Pneumonia
Question 4

Which of the following physical examination findings would help you determine whether the patient is in
respiratory distress?

a.Flaring of alae nasi??


b.Dusky lips and yellow staining of teeth
c.Supraclavicular retractions
d.Increase in anteroposterior diameter of chest wall

Question 5
Which of the following physical examination findings is suggestive of impediment of flow in the airways

a.Flaring of alae nasi


b.Dusky lips and yellow staining of teeth
c.Prolongation of inspiration
d.Bulging of intercostal spaces during expiration??

Question 6
Which of the following are chronic signs of obstruction of the airways, EXCEPT?

a.Dusky lips and yellow staining of teeth-??? - FINAL ANSWER


b.Increase anteroposterior diameter of the chest wall
c.Clubbing of fingers - This is ClinMan of Bronchiectasis and Bronchiectasis is considered to be an obstructive
lung disease din.
d.Expiratory wheeze

Question 7
Which of the following are findings secondary to constriction of the airway during the respiratory cycle?

a.Dusky lips and yellow staining of teeth


b.Prolongation of expiration
c.Increase in anteroposterior diameter of the chest wall
d.Clubbing of fingers

Question 8
Which of the following initial interventions at the ER would help you relieve the patients airway obstruction?

a.Salbutamol
b.Oxygen therapy (?)
c.Antibiotics
d.Anti-tussive

Question 9

Which of the following would help you assess the effect of your intervention to relieve patient’s airway
obstruction?

a.Peak flow meter


b.Chest X-ray
c.Arterial blood gas
d.All of the above

Question 10

The Complete Blood Count of the patient reveals the following:

Hemoglobin = 16 g/dl, Hematocrit = 0.50, RBC = 6.8 x 10 12 /cu. mm.

WBC = 12,500 cells/cu mm.

Neutrophils = 68%, Lymphocytes = 20%, Eosinophils = 5%, Basophils = 1%, Monocytes = 6%

Which of the following describes the results noted above?

a.Polycythemia - ??? Increased RBC = COPD


b.Leukocytopenia -
c.Leukocytosis
d.Neutropenia

Question 11

Interpret the following ABG findings:

HCO3 = 25 meq/L, PaO2 = 60 mmHg, PaCO2 = 50 mmHg, pH = 7.35

a.Metabolic acidosis
b.Metabolic alkalosis
c.Respiratory acidosis
d.Respiratory alkalosis

Question 12

A ph of 7.35 is interpreted as:

a.Acidosis
b.Alkalosis
c.Normal
d.None of the above
Question 13
Which of the following ABG parameter will cause the acid base equilibrium to shift towards the same direction
of change as the pH?

a.HCO3 = 25 meq/L
b.PaO2 = 60 mmHg
c.PaCO2 = 50 mmHg
d.None of the above

Question 14

Chest Xray (PA) findings of the patient are as follows:


- Fibronodular densities noted over the left upper lobe
- Haziness of the right lower lung fields
- Increased radiolucency of the other lung
- Suspicious hilar densities noted in the right lung
- Cardiac silhouette appears normal

Which of the following x-ray findings is suggestive of a suspected tuberculous process?


a.Fibronodular densities noted over the left upper lobe
b.Haziness of the right lower lung fields
c.Increased radiolucency of the other lung
d.Suspicious hilar densities noted in the right lung (?)

Question 15

Chest Xray (PA) findings of the patient are as follows:


- Fibronodular densities noted over the left upper lobe
- Haziness of the right lower lung fields
- Increased radiolucency of the other lung
- Suspicious hilar densities noted in the right lung
- Cardiac silhouette appears normal

Which of the following xray findings is suggestive of a possible malignancy?

a.Fibronodular densities noted over the left upper lobe


b.Haziness of the right lower lung fields
c.Increased radiolucency of the other lung
d.Suspicious densities noted in the right lung

Question 16

Chest Xray (PA) findings of the patient are as follows:


- Fibronodular densities noted over the left upper lobe
- Haziness of the right lower lung fields
- Increased radiolucency of the other lung
- Suspicious hilar densities noted in the right lung
- Cardiac silhouette appears normal

Which of the following x-ray findings is suggestive of a pleural effusion?


a.Fibronodular densities noted over the left upper lobe
b.Haziness of the right lower lung fields
c.Increase radiolucency of the other lung
d.Suspicious hilar densities noted in the right lung

Question 17

Chest Xray (PA) findings of the patient are as follows:


- Fibronodular densities noted over the left upper lobe
- Haziness of the right lower lung fields
- Increased radiolucency of the other lung
- Suspicious hilar densities noted in the right lung
- Cardiac silhouette appears normal

Which of the following xray findings is suggestive of hyperaeration of the lung?

a.Fibronodular densities noted over the left upper lobe


b.Haziness of the right lower lung fields
c.Increase radiolucency of the other lung
d.Suspicious hilar densities noted in the right lung

Question 18

You ordered a gram stain of the patient’s sputum to work up for possible active tuberculosis, what
characteristic of the etiologic agent should you look for?

(may have more than 1 answer)

a.Gram negative
b.Gram positive
c.Cocci
d.Rod shaped

Question 19

The following are the spirometry findings of the patient:


FVC 1.88 (55% of predicted value) reduced
FEV 1 1.23 (45% of predicted value) reduced
FEV 1 /FVC 0.65 reduced

Which of the following measures the total volume of air that the patient is able to blow forcefully following a full
inhalation?

a.FVC
b.FEV1
c.FEV1/FVC RATIO
d.FVC/FEV1 RATIO

Question 20

The following are the spirometry findings of the patient:


FVC 1.88 (55% of predicted value) reduced
FEV 1 1.23 (45% of predicted value) reduced
FEV 1 /FVC 0.65 reduced

Which parameter measures the volume exhaled in the 1st second after a full inhalation?

a.FVC
b.FEV1
c.FEV1/FVC RATIO
d.FVC/FEV1 RATIO

Question 21

The following are the spirometry findings of the patient:


FVC 1.88 (55% of predicted value) reduced
FEV 1 1.23 (45% of predicted value) reduced
FEV 1 /FVC 0.65 reduced

In interpreting the FEV1/FVC ratio, what percentage is considered normal?

a.Less than 60%


b.Less than 70%
c.Between 70-85%
d.Between 40-60%

Question 22

The following are the spirometry findings of the patient:


FVC 1.88 (55% of predicted value) reduced
FEV 1 1.23 (45% of predicted value) reduced
FEV 1 /FVC 0.65 reduced

Based on the FEV1/FVC ratio of 0.65 (65%) of the patient, what is the grading of the severity of the patient’s
disease?

a.Mild COPD
b.Moderate COPD
c.Severe COPD (?)
d.Very severe COPD
QUESTIONS RATIONALE
1. Excessive salivation resulting from a vagal reflex HPIM 20TH page 2210
triggered by acidification of the esophageal mucosa
a. Dysphagia
b. Odynophagia
c. Globus sensation
d. Water brash
2. The key advantages barium radiography over HPIM 20TH page 2210
endoscopy (page 1901)
a. Increases sensitivity for detection of mucosal
lesions and esophageal strictures
b. Increased sensitivity for the detection of
abnormalities mainly identifiable by color
c. Visualization of hypopharyngeal pathology and
disorders of cricopharyngeus muscle
d. All of the above
3. What is/are the major esophageal applications for HPIM 20TH page 2210
endoscopic ultrasound (page 1901)
a. Stage esophageal cancer
b. Evaluate dysplasia in barrett's esophagus
c. Assess submucosal lesions
d. All of the above
4. True statements on esophageal cancer (page 1903) HPIM 20TH page 2212
a. Strongly linked to reflux disease and barrett's
metaplasia
b. Predilection of adenocarcinoma to affect
proximal esophagus in white males
c. Predilection of squamous cell ca to affect distal
esophagus in black males with the added risk
factors of smoking and alcohol consumption
d. All of the above
5. Achalasia is diagnosed by (page 1904) HPoIM 20e p. 2213
a. Barium swallow x ray
b. Esophageal manometry
c. Endoscopy
d. Both a and b
6. True for the management of achalasia (page1904) HPoIM 20e p. 2213
a. The only durable therapies are pneumatic
Dilatation and Heller Myotomy
b. Nitrates and calcium channel blockers may be
administered after eating and has no effect on
blood pressure
c. Slidenafil is approved in the treatment of
achalasia due to its effect on les pressure
d. All of the above
7. Dominant mechanisms of esophagogastric junction HPoIM 20e p. 2216
incompetence (page 1906)
a. Transient LES relaxation
b. LES hypertension
c. Anatomic distortion exclusive of hiatal hernia
d. All of the above
8. Pathophysiologic mechanism of GERD (page 1907) HPoIM 20e p. 2216
a. Gastric acid hypersecretion is a dominant factor
in the development of esophagitis
b. Chronic H. pylori gastritis does not have
protective effect
c. Transient les constriction account for about 90%
reflux in normal patients
d. Bile is a cofactor in the pathogenesis of barrett's
metaplasia and adenocarcinoma
9. Which of the etiologic factors below is not associated HPIM 20th Ed Table 76-1 Page 566
with squamous cell cancer of the esophagus
a. Smoking
b. Excessive alcohol consumption
i. Mucosal damage from hot tea
c. Barrett's esophagus
10. Etiologic factor associated with adenocarcinoma of the HPIM 20th Table 76-2 Page 567
esophagus (page 533, table 109-2)
a. Obesity
b. Dietary deficiency of selenium, molybdenum,
zinc and vitamin A
c. Female sex
d. Ingested carcinogens such as opiates and
nitrates
11. Most of the esophageal cancers occur in the (page 533) HPIM 20th Page 567
a. Cervical esophagus
b. Middle third Upper - 5%
c. Lower third Middle - 20%
d. None of the above
Lower - 75%
12. Initial symptoms in majority of patients with HPIM 20th Page 567
esophageal cancers (page533)
a. Progressive dysphagia
b. Odynophagia
c. Vomiting
d. Chest pain
13. Management of esophageal cancer HPIM 20th Page 567
a. Surgical resection of all gross tumor is feasible in
almost all cases a. only 45% cases
b. Transhiatal resections are favored over thoracic b.
resection
c. Fundoplicative surgery as a means of cancer
prevention in patients with barrett's esophagus
d. Combination chemotherapy and radiation
therapy as the initial approach is beneficial
14. For the incurable, surgically unresectable patient with
esophageal cancer, what are the major issues
encountered?
a. Malnutrition
b. Tracheoesophageal fistulas
c. Dysphagia
d. All of the above
15. Occult GI bleeding may be identified in the absence of
overt bleeding when patients present with: hpim 19.
ed ch57 p276
a. cbc showing hb of 100 mg/dl
b. Positive fecal occult blood test
c. Angina
d. All of the above
16. Most common cause of upper gi bleeding? Hpim 19th
ed ch57 p276
a. Vascular ectasias
b. Ulcers
c. Mallory weiss tears
d. Gastroduodenal erosions
17. Which of the following statements is FALSE regarding HPIM 20th 272
the administration of high-dose, constant-infusion
intravenous proton pump inhibitors in upper gi
bleeding: hpim 19' ed ch57 p276
a. Designed to sustain intragastric ph >6
b. Enhance clot stability
c. It is given as 40mg bolus and 4mg/hr infusion
d. Decreases further bleeding and mortality in
patients with an adherent clot
18. Which of the following is not a part of the three main HPIM 20th page 272
factors in ulcer pathogenesis in which the prevention
of recurrent bleeding focuses on:
a. Alcoholism
b. Acid
c. Nsaids
d. H. pylori
19. Stress-related gastric mucosal injury occurs in the
following:
a. Traumatic brain injury due to vehicular accident
b. Patients on mechanical ventilator
c. Burns covering more than 1/3 of bsa
d. All of the above
20. it is an aberrant vessel in the mucosa bleeds from a pin- HPIM 20TH page 273
point mucosal defect
a. Oster-weber-rendu lesion
b. Hemorrhagic telangiectasia
c. Dieulafoy' s lesion
d. Antral vascular ectasia
21. Most common cause of lower GI bleeding HPIM 20th Ed
a. Hemorrhoids
b. Bleeding diverticulum
c. Vascular ectasia
d. Neoplastic disease
22. Which is true for the differentiation from upper GI
bleeding and lower GI bleeding a. most likely
a. The more proximal the bleeding site, the less b.
likely melena will occur
b. Hematochezia usually represents an upper GI
source of bleeding, although a lower GI lesion
may bleed that blood transits the bowel before
melena develops.
c. When hematochezia is the presenting symptom
of UGIB, it is not associated with hemodynamic
instability and dropping hemoglobin.
d. Bleeding lesions of the small bowel may present
as melena or hematochezia
23. True regarding the evaluation and management of
lower GI bleeding
a. Patients with hematochezia and hemodynamic
instability should have upper endoscopy before
evaluation of the lower GI tract
b. Sigmoidoscopy is used primarily in patients <40
years old with minor bleeding
c. In active LGIB, angiography can detect the site of
bleeding and permits treatment with
embolization
d. All of the above
24. A 55 year old male was admitted for melena. He
suffered from st elevation myocardial infarction last
year and has been on aspirin since then. Which of the
following is appropriate for the patient? (c57,p277)
a. Resume aspirin within 1-7 days of bleed
b. Wait 2 weeks before resuming aspirin
c. Wait 6 months before resuming aspirin
d. Avoid aspirin indefinitely
25. A 56 year old male presented with recurrent page 273
melena. Upper and lower endoscopies revealed
no identifiable source of bleeding. Which is the
most likely cause for the small intestinal bleeding
for this patient? Ch57 page 277
a. Meckel's diverticulum
b. Lymphomatous tumors
c. Hemorrhoids
d. Vascular ectasia
26. An 80 year old male presented with the first
episode of massive hematochezia. Colonoscopy
showed multiple mucosal outpouchings. Which of
the following describes diverticular bleeding?
(c57, p277)
a. Bleeding is often chronic and slow
b. Diverticular bleeding stops spontaneously in
only about 20% and would most often
require endoscopic therapy
c. Diverticular bleeding often occurs from the
left side of the colon
d. In patients with recurrent bleeding,
segmental resection is indicated
27. Which is true of the following statements regarding
rebleeding in upper gi tract (page 277)
a. Approximately 3/4 of patients with bleeding
ulcers will rebleed within the next 5years if no
preventive strategies are employed.
b. Prevention of recurrent bleeding focuses on the
three main factors in ulcer pathogenesis, such as
Helicobacter pylori , nsaids, and anticoagulants
c. Eradication of H. pylori in patients with bleeding
ulcers decreases rates of rebleeding to <5%.
d. All of the above
28. Which of the following statement/s is/are FALSE
regarding the initial assessment in a patient with GI
bleeding
a. Check blood pressure and heart rate
b. Clinically significant bleeding would result to
recumbent hypotension
c. Hemoglobin level falls immediately
d. All of the above
29. In GI bleeding, blood transfusion is recommended
when hemoglobin drops below:
a. 9 g/dl
b. 8 g/dl
c. 7 g/dl
d. None of the above
30. Melena indicates blood has been present in the GI
tract for at least:
a. 10 hours
b. 12 hours
c. 14 hours
d. 16 hours
31. What is the most likely type of ulcer if it needs IV PPI
therapy and ward admission for 3 days?
a. Active bleeding
b. Adherent clot
c. Flat, pigmented spot
d. Clean base
32. It is the most useful test for the evaluation of the
proximal GI tract
a. Roentgenogram
b. MRI
c. Endoscopy
d. Ultrasonography
33. Which of the following statement is/are FALSE
regarding achalasia?
a. It is caused by a loss of ganglion cells with the
esophageal submucosal plexus
b. The disease involves both excitatory and
inhibitory ganglionic neurons
c. There is no known way of preventing or
reversing achalasia
d. Impaired les relaxation and absent peristalsis
seen on manometry is diagnostic
34. A 66-year-old man living in Tehran, Iran has been
bothered by difficulty swallowing for the past year. He People who lives in Iran likes to eat spicy foods
is now consuming liquid food. Yesterday he
regurgitated food stained with blood. On
esophagoscopy, there is an ulcerated obstructing
lesion 20cm from the lips. Biopsies are taken and on
microscopy showing nests infiltrating cells with distinct
cell borders and hyperchromatic, angulated nuclei.
Which of the following is the most likely risk factor for
his disease?
a. Genetic susceptibility
b. Autoimmunity
c. Diet
d. Reflux
35. The following is/are NOT included in the
extraesophageal syndromes with an established b. Chronic Cough
association to GERD
a. Dental erosions
b. Acute cough
c. Asthma
d. Laryngitis
36. What is the most sensitive test in the diagnosis of HPIM 20th Ed page 2202
GERD? (p1894)
a. 24-hour ambulatory ph monitoring
b. Esophagogastroduodenoscopy
c. Esophageal ultrasound
d. Manometry
37. Factors tending to exacerbate reflux regardless of HPIM 20th ed, p2215
mechanism are the following EXCEPT:
a. Delayed gastric emptying
b. Obesity
c. Starvation
d. Pregnancy
38. The following are cause/s of prolonged acid clearance: HPIM 20th ed, p2215
a. impaired peristalsis
b. Reduced salivation
c. Both a and b
d. None of the above
39. The typical symptom/s of GERD: HPIM 20th ed, p2216
a. Pyrosis
b. Regurgitation
c. Chest pain
d. Both a and b
40. The complications of GERD are the following: HPIM 20th ed, p2216
a. Bleeding
b. Stricture
c. Carcinoma
d. All of the above
41. The most severe endoscopic and histologic
consequence of GERD
a. Tongues of reddish mucosa extending
proximally from the gastroesophageal junction
and finding of specialized columnar metaplasia
b. Tongues of reddish mucosa extending distally
from the gastroesophageal junction and finding
of specialized columnar metaplasia
c. Tongues of reddish submucosa extending
proximally from the gastroesophageal junction
and finding of specialized cuboidal metaplasia
d. Tongues of reddish submucosa extending
distally from the gastroesophageal junction and
finding of specialized columnar metaplasia
42. Metaplasia (barrett esophagus) at the
gastroesophageal junction is most closely associated
with which at the following esophageal lesions?
a. Adenocarcinoma
b. Eosinophilic esophagitis
c. Esophageal varices
d. Squamous cell carcinoma

43. What is the most broadly applicable recommendation


for lifestyle modifications as GERD therapy
a. Avoidance of eating before retiring
b. Weight reduction
c. Avoidance to acidic foods
d. Elevation of head of the bed
44. What enteric etiologic agent in particular increases its
susceptibility with indefinite PPl therapy?
a. Giardia lamblia
b. Clostridium difficile
c. Candida Albicans
d. E. coli
45. Which of the following patients would be at risk for HPIM 20th pp 2221
peptic ulcer disease? C48 p1914
a. A 5O year old cirrhotic
b. A 45 year old heavy smoker
c. A 65 year old filipino who migrated to usa
d. A 30 year old female with a BMI of 18

46. Which of the following statement/s is/are true HPIM 20th pp 2223
regarding duodenal ulcers
a. Occurs most often in the first portion of the
duodenum
b. They are bluntly (sharply) demarcated with
depth at times reaching the muscularis propria
c. They are usually at least to 2 cm (≤ 1cm) in
diameter
d. Malignant duodenal ulcers are extremely rare

47. The bacterial factor/s important in determining H. HPIM 20th pp 2224


pylori induced gastrointestinal disease
a. Porins
b. Cag a
c. Adhesins
d. All of the above

48. They play a critical role in the gi tract that its HPIM 20th pp 2226
interruption of its synthesis facilitates mucosal injury
a. Sodium bicarbonate
b. Mucin
c. Prostaglandin
d. Phospholipids
49. Which of the following has a strong association with
PUD? (c348, p1917)
a. Chronic pancreatitis
b. Hyperparathyroidism
c. Nephrolithiasis
d. Obesity
50. The following is/are drug/toxin not included as
causes of ulcers not caused by H. pylori and
nsaids:
a. Potassium chloride
b. Clopidogrel
c. Chemotherapy
d. Amoxicillin

51. These drug class are now rarely, if ever, used as the
primary therapeutic agent but instead are often used
by patients for symptomatic relief of dyspepsia:
a. H2 receptor antagonists
b. Proton pump inhibitors
c. Antacids
d. Bismuth-containing preparations
52. Presently, this class of drug is often used for treatment
of active ulcers in combination with antibiotics for
eradicating H. pylori
a. H2 receptor antagonists
b. Proton pump inhibitors
c. Antacids
d. Bismuth-containing preparations

53. This drug may have weak antiandrogenic side effects HPIM 20th Ed PAGE 2229
resulting in reversible gynecomastia and impotence
a. Cimetidine
b. Ranitidine
c. Nizatidine
d. Famotidine
54. These agents potently inhibit all phases of gastric acid
secretion
a. Proton pump inhibitors
b. Cytoprotective agents
c. Antacids
d. Antibiotics
55. Long-term acid suppression, especially with proton
pump inhibitors, has been associated with a higher a. HPIM 20th Ed page 2229
incidence of: b. HPIM 20th Ed PAGE 2242
a. Impotence c.
b. Adenocarcinoma of the stomach
d. HPIM 20th Ed PAGE 2229 associated with H2RAs
c. Community-acquired pneumonia
d. Febrile neutropenia
56. Which of the following medications included in H.
pylori eradication regimens are INCORRECTLY paired HPIM 20th ed ch. 317 pg. 2229 -31
with its possible side effects? Hpim c348 p 1920 Aluminum hydroxide can produce constipation and phosphate
a. Magnesium hydroxide - constipation depletion
b. Cimetidine — gynecomastia
c. Omeprazole — hip fractures Magnesium hydroxide may cause loose stools
d. Bismuth subsalicylate — darkening of the
Long-term use of PPIs was associated with the development of
tongue hip fractures in older women

Toxicity from this drug (Sucralfate) is rare, with constipation


being most common

Adverse effects with short-term use (Bismuth-containing


preparation) include black stools, constipation, and darkening
of the tongue
Cimetidine may have weak antiandrogenic side effects
resulting in reversible gynecomastia and impotence, primarily
in patients receiving high doses for prolonged periods of time

57. What is the most feared complication with the use of


amoxicillin in eradication regimen? HPIM 20th ed ch. 317 pg. 2232
a. Anaphylactic reaction The most feared complication with amoxicillin is
b. Pseudomembranous colitis pseudomembranous colitis
c. Antibiotic resistance of the organism
d. Hepatotoxicity
58. The test of choice for documenting eradication of H.
pylori? HPIM 20th ed ch. 317 pg. 2235
a. Culture H. pylori eradication should be documented 4 weeks after
b. Histology completing antibiotics. The test of choice for documenting
c. Urea breath test eradication is the laboratory-based validated monoclonal stool
d. Rapid urease test antigen test or a urea breath test (UBT).

59. Peptic ulcer can also penetrate into adjacent organs,


especially with posterior duodenal ulcer, which can
penetrate into the following organ/s except:
a. Biliary tree
b. Pancreas
c. Colon
d. Small intestine

20th Ed., page 2227


60. Surgical treatment for duodenal ulcers was originally
designed to decrease gastric acid secretion.
Operations most commonly performed include:
a. Vagotomy and drainage
b. Highly selective vagotomy
c. Vagotomy with antrectomy
d. All of the above

20th Ed., page 2236


61. Which ulcer surgery has the highest complication rate
and has the lowest recurrence rate
a. Vagotomy and a drainage procedure
b. Highly selective vagotomy
c. Vagotomy with antrectomy
d. Subtotal gastrectomy
20th Ed., page 2236
62. Vagotomy is a component of each of these procedures
and is aimed at decreasing acid secretion through
a. Dopaminergic input to the stomach
b. Nicotinic input to the stomach
c. Muscarinic input to the stomach
d. Cholinergic input to the stomach Harrison’s 20th ed, pp2236 (PUD Chapter)
63. Which of the following is NOT a surgery-related Surgery related complications are: (1)Recurrent
complication of pud patients? ulceration (2)Afferent loop syndrome (3)Dumping
a. Efferent loop syndromes syndrome (4)Postvagotomy Diarrhea (5)Bile reflux
b. Recurrent ulceration Gastropathy (6) Maldigestion and malabsorption
c. Dumping syndrome (7)Gastric CA (8)Others:Gallstones, Cholecystitis
d. Gallstones
Harrison’s 20th ed, pp2237-2238(PUD Chap)
64. In zollinger-ellison syndrome, initial presentation and
ulcer location may be distinguishable from common
PUD. Where is the location of the ulcer?
a. Duodenal bulb
b. Antrum
c. Fundus
d. Cardia Harrison’s 20th ed, pp2238 (PUD Chapter, rel cond)
65. When to obtain a fasting serum gastrin level
a. Multiple ulcers
b. Hypercalcemia
c. Unexplained diarrhea
d. All of the above

66. Which is most sensitive imaging studies used in


zollinger-ellison syndrome in a primary gastrinoma?
a. Endoscopic ultrasonography
b. MRI
c. CT scan
d. Selective angiography

67. Which is most sensitive imaging studies used in


zollinger-ellison syndrome in a metastatic gastrinoma?
a. Endoscopic ultrasonography
b. MRI
c. CT scan
d. Selective angiography
68. Which of the following is/are NOT part of the
etiologies considered for acute gastritis?
a. Vibrio cholerae
b. Helicobacter pylori
c. Mycobacterium
d. Phlegmonous

69. More than how many serotypes of salmonella are


pathogenic to humans?
a. 100
b. 200
c. 300
d. 400

70. The infectious dose of salmonella infections ranges


starting from how many colony forming units (cfu):
a. 100
b. 200
c. 300
d. 400

71. The following does NOT increase susceptibility to


salmonella infection?
a. Antacid ingestion
b. Achlorhydric disease
c. Inflammatory bowel disease
d. Adolescent male
72. Which of the following statements describe
salmonellae species?
a. They are gram-negative, non-spore forming,
facultatively anaerobic bacilli, measuring 2-3 x
0.4-0.6 micrometers, produces H2S gas on
glucose fermentation, do not produce
cytochrome oxidase and mostly motile by means
of peritrichous flagella
b. They are gram-negative, non-spore forming,
aerobic bacilli, measuring 2-3 x 0.4-0.6
micrometers, produces no gas on glucose
fermentation, do not produce cytochrome
oxidase and mostly motile by means of
peritrichous flagella
c. They are gram-negative, non-spore forming,
facultatively anaerobic bacilli, measuring 2-3 x
0.4-0.6 micrometers, produces H2S gas on
glucose fermentation, produces cytochrome
oxidase and mostly motile by means of
peritrichous flagella
d. They are gram-negative, non-spore forming,
aerobic bacilli, measuring 2-3 x 0.4-0.6
micrometers, produces no gas on glucose
fermentation, produces cytochrome oxidase and
mostly motile by means of peritrichous flagella
73. Typhoidal salmonellae disseminate throughout the
body in the macrophages via the lymphatics and
colonize the following tissues except:
a. Liver
b. Large intestine
c. Spleen
d. Bone marrow
74. The definitive diagnosis of Enteric Fever requires the HPIM 19th Ed. P. 1051
isolation of salmonellae. Which of the following is 55-
90% sensitivity?
a. Urine
b. Bone Marrow
c. Stool
d. Blood
75. These drug class are most effective for the treatment HPIM 20th page 1176
of drug-susceptible typhoid fever with cure rates of
around 98%
a. Macrolides
b. 3rd generation cephalosporins
c. Fluoroquinolones
d. Penicillin
76. Most malabsorption syndromes are associated with HPIM 20th page 2244
a. Peptic ulcer disease
b. Steatorrhea
c. Lactose intolerance
d. Malignancy
77. The following is NOT TRUE for the macroscopic feature
of ulcerative colitis
a. Usually involves the rectum and extends
proximally to involve all or part of a colon
b. Terminal ileum is involved in 90% of cases
c. There is noted backwash ileitis but are of little
significance
d. Proximal spread occurs in continuity
78. The following is/are TRUE of macroscopic features of
crohn's disease.
a. It can affect any part of the GI tract
b. Small intestine is affected in 75% of cases
c. It is segmental with skip areas in the midst of
diseased intestine
d. All of the above

79. The following is NOT a severe presentation of


ulcerative colitis
a. More than 6 bowel movements per day
b. Bloody stools
c. <90 cardiac bpm upon auscultation
d. Spontaneous bleeding and ulcerations on
endoscopy

80. The following statements are true regarding clinical


presentation of ulcerative colitis:
a. Fecal lactoferrin is a highly sensitive and specific
marker for detecting intestinal inflammation
b. Sigmoidoscopy is used to assess disease activity
and is performed before treatment
c. The earliest radiologic change of ulcerative
colitis seen on single-contrast barium enema is a
fine mucosal granularity
d. All of the above
81. Which of the following statements are true regarding
dermatologic manifestations of inflammatory bowel
disease?
a. P-anca positivity is found in about 60-70% CD
patients
b. Erythema nodosum occurs in up to 90% of UC
patients
c. Pyoderma gangrenosum is seen in 1-12% of UC
patients and less commonly in crohn’s colitis
d. All of the above
82. Mainstay of therapy for mild to moderate ulcerative
colitis
a. Sulfasalazine
b. Glucocorticoids
c. Azathioprine
d. Methotrexate
83. The first biologic therapy approved for crohn’s
disease
a. Methotrexate
b. Infliximab
c. Tacrolimus
d. Cyclosporine

84. True regarding nutritional therapies for inflammatory


bowel diseases
a. Patients with active CD does not respond to
bowel rest along with TPN
b. Bowel rest and TPN are as effective as
glucocorticoids at inducing remission of active
CD
c. Enteral nutrition in the form of elemental or
peptide-based preparations is not as effective as
glucocorticoids or TPN
d. Enteral diets may provide the small intestine
with nutrients vital to cell growth and have the
same complications of TPN
85. Which of the following drugs are contraindicated for Methotrexate can terminate pregnancy (GG 13th Ed, p.
pregnant patients with IBD 821, Termination of Pregnancy)
a. Methotrexate
b. Sulfasalazine
c. Glucocorticoids
d. Azathioprine
86. Patients with crohn’s disease have an increased
risk of developing malignancy such as
a. Lung cancer
b. Non-hodgkin's lymphoma
c. Small bowel adenocarcinoma
d. All of the above
87. What is the key symptom for diagnosing irritable
bowel syndrome?
a. Abdominal discomfort
b. Constipation alternating with diarrhea
c. Flatulence
d. Change in appearance of stool
88. Pathophysiology of IBS includes:
a. Decreased rectosigmoid motor activity for
up to 3 hours after
b. Heightened sensitivity elsewhere in the
body
c. Association of emotional disorders and
stress with symptom exacerbation
d. All of the above

89. Risk factors for developing post infectious IBS


include
a. Male gender
b. Prolonged duration of illness
c. Age older than 60 years
d. Adverse life events in the preceding 1 year

90. A 24yo woman has had multiple episodes of


aspiration of food associated with difficulty
swallowing during the past year. On auscultation
of her chest, crackles are heard at the base of the
right lung. A barium swallow shows marked
esophageal dilation above the level of the lower
esophageal sphincter. A biopsy specimen from
the lower esophagus shows an absence of the
myenteric ganglia. What is the most likely
diagnosis?
a. Achalasia
b. Barrett’s esophagus
c. Plummer-Vinson Syndrome
d. Sliding Hiatal Hernia
For the remaining questions, please answer with the
corresponding letters:

A. If it is predominant in UC
B. If it is under CD
C. If it is both in UC and CD
D. If it is none both UC and CD

91. The age of onset is at second to fourth decades


and seventh to ninth decades. C

92. Smoking may prevent the disease. A

93. Patients taking oral contraceptives are at


increased risk. B

94. Appendectomy is deemed to be protective. A 98 HPIM page 2267 (Table 319-5)

95. There is 38-58% concordance with monozygotic 99. HPIM page 2262 (lower right column)
twins. B
100. HPIM page 2262 and 2263
96. Antibiotic use in the first year of life has 2.9x risk
of developing IBD. C

97. It is usually associated with Turner’s syndrome.

98. The rectum is spared. B

99. It is a transmural process. B

100. It has pseudopolyps. C


Questions Rationale

1. Excessive salivation resulting from a vagal reflex


triggered by acidification of the esophageal
mucosa
a. Dysphagia
b. Odynophagia
c. Globus sensation Harrison’s 19th ed chap 347 pp. 1901
d. Water brash
2. The key advantages barium radiography over
endoscopy (page 1901)
a. Increases sensitivity for detection of
mucosal lesions and esophageal strictures
b. Increased sensitivity for the detection of
abnormalities mainly identifiable by color
c. Visualization of hypopharyngeal
pathology and disorders of
cricopharyngeus muscle
d. All of the above
3. What is/are the major esophageal applications
for endoscopic ultrasound (page 1901)
a. Stage esophageal cancer
b. Evaluate dysplasia in barrett's esophagus
c. Assess submucosal lesions
d. All of the above
4. True statements on esophageal cancer (page
1903)
a. Strongly linked to reflux disease and
barrett's metaplasia
b. Predilection of adenocarcinoma to affect
proximal esophagus in white males
c. Predilection of squamous cell ca to affect
distal esophagus in black males with the
added risk factors of smoking and alcohol
consumption
d. All of the above
5. Achalasia is diagnosed by (page 1904)
a. Barium swallow xray
b. Esophageal manometry
c. Endoscopy
d. Both a and b
6. True for the management of achalasia
(page1904)
a. The only durable therapies are pneumatic
Dilatation and Heller Myotomy
b. Nitrates and calcium channel blockers
may be administered after eating and has
no effect on blood pressure
c. Slidenafil is approved in the treatment of
achalasia due to its effect on les pressure
d. All of the above
7. Dominant mechanisms of esophagogastric
junction incompetence (page 1906)
a. Transient les relaxation
b. LES hypertension
c. Anatomic distortion exclusive of hiatal
hernia
d. All of the above
8. Pathophysiologic mechanism of gerd (page
1907)
a. Gastric acid hypersecretion is a dominant
factor in the development of esophagitis
b. Chronic H. pylori gastritis does not have
protective effect
c. Transient les constriction account for
about 90% reflux in normal patients
d. Bile is a cofactor in the pathogenesis of
barrett's metaplasia and adenocarcinoma
9. Which of the etiologic factors below is not
associated with squamous cell cancer of the
esophagus
a. Smoking
b. Excessive alcohol consumption
c. Mucosal damage from hot tea
d. Barrett's esophagus

10. Etiologic factor associated with


adenocarcinoma of the esophagus (page 533,
table 109-2)
a. Obesity
b. Dietary deficiency of selenium,
molybdenum, zinc and vitamin A
c. Female sex
d. Ingested carcinogens such as opiates and
nitrates
11. Most of the esophageal cancers occur in the
(page 533)
a. Cervical esophagus
b. Middle third
c. Lower third
d. None of the above
12. Initial symptoms in majority of patients with
esophageal cancers (page533)
a. Progressive dysphagia
b. Odynophagia
c. Vomiting
d. Chest pain
13. Management of esophageal cancer
a. Surgical resection of all gross tumor is
feasible in almost all cases
b. Transhiatal resections are favored over
thoracic resection
c. Fundoplicative surgery as a means of
cancer prevention in patients with
barrett's esophagus Harrison’s 19th ed pp. 533
d. Combination chemotherapy and radiation
therapy as the initial approach is
beneficial
14. For the incurable, surgically unresectable
patient with esophageal cancer, what are the
major issues encountered?
a. Malnutrition
b. Tracheoesophageal fistulas
c. Dysphagia Harrison’s 19th ed pp. 533
d. All of the above
15. Occult GI bleeding may be identified in the
absence of overt bleeding when patients
present with: hpim 19. ed ch57 p276
a. cbc showing hb of 100 mg/dl
b. Positive fecal occult blood test
c. Angina
d. All of the above
16. Most common cause of upper gi bleeding?
Hpim 19th ed ch57 p276
a. Vascular ectasias
b. Ulcers
c. Mallory weiss tears
d. Gastroduodenal erosions
17. Which of the following statements is false
regarding the administration of high-dose,
constant-infusion intravenous proton pump
inhibitors in upper gi bleeding: hpim 19' ed
ch57 p276
a. Designed to sustain intragastric ph >6
b. Enhance clot stability
c. It is given as 40mg bolus and 4mg/hr
infusion
d. Decreases further bleeding and mortality
in patients with an adherent clot
18. Which of the following is not a part of the
three main factors in ulcer pathogenesis in
which the prevention of recurrent bleeding
focuses on:
a. Alcoholism
b. Acid
c. Nsaids Harrison’s 19th ed pp. 277
d. H. Pylori
19. Stress-related gastric mucosal injury occurs in
the following:
a. Traumatic brain injury due to vehicular
accident
b. Patients on mechanical ventilator
c. Burns covering more than 1/3 of bsa
d. All of the above Harrison’s 19th ed pp. 1929
20. it is an aberrant vessel in the mucosa bleeds
from a pin-point mucosal defect
a. Oster-weber-rendu lesion
b. Hemorrhagic telangiectasia
c. Dieulafoy' s lesion
d. Antral vascular ectasia
Harrison’s 19th ed pp. 277
21. Most common cause of lower GI bleeding
a. Hemorrhoids
b. Bleeding diverticulum
c. Vascular ectasia
d. Neoplastic disease
Harrison’s 19th ed pp. 277
22. Which is true for the differentiation from
upper GI bleeding and lower GI bleeding
a. The more proximal the bleeding site, the
less likely melena will occur
b. Hematochezia usually represents an
upper GI source of bleeding, although a
lower GI lesion may bleed that blood
transits the bowel before melena
develops.
c. When hematochezia is the presenting Harrison’s 19th ed pp. 278
symptom of ugib, it is not associated with
hemodynamic instability and dropping
hemoglobin.
d. Bleeding lesions of the small bowel may
present as melena or hematochezia
23. True regarding the evaluation and
management of lower GI bleeding See Harrison’s 19th ed. pp. 279
a. Patients with hematochezia and Evaluation and Management of LGIB
hemodynamic instability should have
upper endoscopy before evaluation of the
lower GI tract
b. Sigmoidoscopy is used primarily in
patients <40 years old with minor
bleeding
c. In active LGIB, angiography can detect the
site of bleeding and permits treatment
with embolization
d. All of the above
24. A 55 year old male was admitted for melena.
He suffered from st elevation myocardial
infarction last year and has been on aspirin
since then. Which of the following is
appropriate for the patient? (c57,p277)
a. Resume aspirin within 1-7 days of bleed
b. Wait 2 weeks before resuming aspirin
c. Wait 6 months before resuming aspirin
d. Avoid aspirin indefinitely
25. A 56 year old male presented with recurrent
melena. Upper and lower endoscopies
revealed no identifiable source of bleeding.
Which is the most likely cause for the small
intestinal bleeding for this patient? Ch57 page
277
a. Meckel's diverticulum
b. Lymphomatous tumors
c. Haemorrhoids
d. Vascular ecstasia
26. An 80 year old male presented with the first
episode of massive hematochezia.
Colonoscopy showed multiple mucosal
outpouchings. Which of the following
describes diverticular bleeding? (c57, p277)
a. Bleeding is often chronic and slow
b. Diverticular bleeding stops spontaneously
in only about 20% and would most often
require endoscopic therapy
c. Diverticular bleeding often occurs from
the left side of the colon
d. In patients with recurrent bleeding,
segmental resection is indicated
27. Which is true of the following statements
regarding rebleeding in upper gi tract (page
277) B (since anticoagulants are acids daw) or C
a. Approximately 3/4 of patients with
bleeding ulcers will rebleed within the
next 5years if no preventive strategies are
employed.
b. Prevention of recurrent bleeding focuses
on the three main factors in ulcer
pathogenesis, such as Helicobacter pylori ,
nsaids, and anticoagulants
c. Eradication of H. pylori in patients with Harrison’s 19th ed. pp. 277
bleeding ulcers decreases rates of
rebleeding to <5%.
d. All of the above
28. Which of the following statement/s is/are
FALSE regarding the initial assessment in a
patient with GI bleeding
a. Check blood pressure and heart rate
b. Clinically significant bleeding would result
to recumbent hypotension
c. Hemoglobin level falls immediately
d. All of the above Harrison’s 19th ed. pp 278
29. In GI bleeding, blood transfusion is
recommended when hemoglobin drops below:
a. 9 g/dl
b. 8 g/dl
c. 7 g/dl
d. None of the above
30. Melena indicates blood has been present in
the GI tract for at least:
a. 10 hours
b. 12 hours
c. 14 hours Harrison’s 19th ed. pp 278
d. 16 hours
31. What is the most likely type of ulcer if it needs
IV PPI therapy and ward admission for 3 days? See Harrisons 19th ed. pp 278
a. Active bleeding Figure 57-1
b. Adherent clot
c. Flat, pigmented spot
d. Clean base
32. It is the most useful test for the evaluation of
the proximal GI tract
a. Roentgenogram
b. MRI
c. Endoscopy
Harrisons 19th ed. pp 1901
d. Ultrasonography
33. Which of the following statement is/are false
regarding achalasia? Myenteric plexus
a. It is caused by a loss of ganglion cells with
the esophageal submucosal plexus
b. The disease involves both excitatory and
inhibitory ganglionic neurons
c. There is no known way of preventing or
reversing achalasia
d. Impaired les relaxation and absent Harrisons 19th ed. pp 1904
peristalsis seen on manomtery is diagnostic
34. A 66-year-old man living in Tehran, Iran has
been bothered by difficulty swallowing for the
past year. He is now consuming liquid food. Most Iranian dishes/cuisine are spicy foods.
Yesterday he regurgitated food stained with
blood. On esophagoscopy, there is an
ulcerated obstructing lesion 20cm from the
lips. Biopsies are taken and on microscopy
showing nests infiltrating cells with distinct cell
borders and hyperchromatic, angulated nuclei.
Which of the following is most likely risk factor
for his disease?
a. Genetic susceptibility
b. Autoimmunity
c. Diet
d. reflux
35. The following is/are not included in the Chronic cough
extraesophageal syndromes with an
established association to GERD
a. Dental erosions
b. Acute cough
c. Asthma
d. Laryngitis Harrisons 19th ed. pp 1907
36. What is the most sensitive test in the diagnosis
of gerd? (p1894)
a. 24-hour ambulatory ph monitoring
b. Esophagogastroduodenoscopy
c. Esophageal ultrasound
d. Manometry
37. Factors tending to exacerbate reflux regardless
of mechanism are the following except:
a. Delayed gastric emptying
b. Obesity
c. Starvation
d. Pregnancy Harrisons 19th ed. pp 1906
38. The following are cause/s of prolonged acid
clearance:
a. impaired peristalsis
b. Reduced salivation
c. Both a and b
d. None of the above Harrisons 19th ed. pp 1906
39. The typical symptom/s of GERD:
a. Pyrosis Dysphagia and chest pain is less common symptom of GERD.
b. Regurgitation
c. Chest pain Harrisons 19th ed. pp 1907
d. Both a and b
40. The complications of GERD are the following:
a. Bleeding
b. Stricture
c. Carcinoma
d. All of the above
41. The most severe endoscopic and histologic
consequence of GERD
a. Tongues of reddish mucosa extending
proximally from the gastroesophageal
junction and finding of specialized
columnar metaplasia
b. Tongues of reddish mucosa extending
distally from the gastroesophageal
junction and finding of specialized
columnar metaplasia
c. Tongues of reddish submucosa extending
proximally from the gastroesophageal
junction and finding of specialized cuboidal
metaplasia
d. Tongues of reddish submucosa extending Harrisons 19th ed. pp 1907
distally from the gastroesophageal
junction and finding of specialized
columnar metaplasia
42. Metaplasia (barrett esophagus) at the
gastroesophageal junction is most closely
associated with which at the following
esophageal lesions?

a. Adenocarcinoma
b. Eosinophilic esophagitis
c. Esophageal varices
d. Squamous cell carcinoma
43. What is the most broadly applicable
recommendation for lifestyle modifications as
GERD therapy
a. Avoidance of eating before retiring
b. Weight reduction Harrisons 19th ed. pp 1908
c. Avoidance to acidic foods
d. Elevation of head of the bed
44. What enteric etiologic agent in particular
increases its susceptibility with indefinite PPl Harrison’s 19th ed pp 1908
therapy?
a. Giardia lamblia
b. Clostridium difficile
c. Candida Albicans
d. E. coli
45. Which of the following patients would be at
risk for peptic ulcer disease? C48 p1914
a. A 5O year old cirrhotic
b. A 45 year old heavy smoker
c. A 65 year old filipino who migrated to usa
d. A 30 year old female with a bmi of 18
46. Which of the following statement/s is/are true
regarding duodenal ulcers

a. Occurs most often in the first portion of


the duodenum
b. They are bluntly demarcated with depth at
times reaching the muscularis propria
c. They are usually at least to 2 cm in
diameter Harrison’s 19th ed pp 1914
d. Malignant duodenal ulcers are extremely
rare
47. The bacterial factor/s important in determining
H. pylori induced gastrointestinal disease
a. Porins
b. Cag a
c. Adhesins
d. All of the above Harrison’s 19th ed pp 1916
48. They play a critical role in the gi tract that its
interruption of its synthesis facilitates mucosal
injury
a. Sodium bicarbonate
b. Mucin
c. Prostaglandin
d. Phospholipids Harrison’s 19th ed. pp. 1917

49. Which of the following has a strong association


with PUD? (c348, p1917)
a. Chronic pancreatitis
b. Hyperparathyroidism
c. Nephrolithiasis
d. Obesity
50. The following is/are drug/toxin not included as
causes of ulcers not caused by H. pylori and
nsaids:
a. Potassium chloride
b. Clopidogrel
c. Chemotherapy
d. Amoxicillin
51. These drug class are now rarely, if ever, used
as the primary therapeutic agent but instead
are often used by patients for symptomatic
relief of dyspepsia:
a. H2 receptor antagonists
b. Proton pump inhibitors Harrison’s 19th ed. pp. 1920
c. Antacids
d. Bismuth-containing preparations

52. Presently, this class of drug is often used for H2RA accdg to Harrisons 19th ed pp. 1920:
treatment of active ulcers in combination with
antibiotics for eradicating H. pylori

a. H2 receptor antagonists
b. Proton pump inhibitors
c. Antacids
d. Bismuth-containing preparations
53. This drug may have weak antiandrogenic side
effects resulting in reversible gynecomastia
and impotence
a. Cimetidine
b. Ranitidine
c. Nizatidine
d. Famotidine Harrisons 19th ed pp. 1920

54. These agents potently inhibit all phases of


gastric acid secretion

a. Proton pump inhibitors


Harrison’s 19th ed. pp. 1920
b. Cytoprotective agents
c. Antacids
d. Antibiotics
55. Long-term acid suppression, especially with
proton pump inhibitors, has been associated
with a higher incidence of:
a. Impotence
b. Adenocarcinoma of the stomach
c. Community-acquired pneumonia
Harrison’s 19th ed pp 1921
d. Febrile neutropenia
56. Which of the following medications included in
H. pylori eradication regimens are incorrectly Magnesium hydroxide may cause loose stools.
paired with its possible side effects? Hpim c348 Aluminum hydroxide can produce constipation and phosphate
p 1920 depletion.
a. Magnesium hydroxide - constipation
b. Cimetidine — gynecomastia
c. Omeprazole — hip fractures
d. Bismuth subsalicylate — darkening of the
tongue
57. What is the most feared complication with the
use of amoxicillin in eradication regimen?
a. Anaphylactic reaction
b. Pseudomembranous colitis
c. Antibiotic resistance of the organism Harrison’s 19th ed pp 1922
d. Hepatotoxicity

58. The test of choice for documenting eradication


of h. Pylori?

a. Culture
b. Histology
c. Urea breath test
d. Rapid urease test

59. Peptic ulcer can also penetrate into adjacent


organs, especially with posterior duodenal
ulcer, which can penetrate into the following
organ/s except:

a. Biliary tree
b. Pancreas
c. Colon
d. Small intestine
60. Surgical treatment for duodenal ulcers was
originally designed to decrease gastric acid
secretion. Operations most commonly
performed include:
a. Vagotomy and drainage
b. Highly selective vagotomy
c. Vagotomy with antrectomy
d. All of the above Harrison’s 19th ed pp 1925
61. Which ulcer surgery has the highest
complication rate and has the lowest
recurrence rate

a. Vagotomy and a drainage procedure


b. Highly selective vagotomy
c. Vagotomy with antrectomy
Harrison’s 19th ed pp 1925
d. Subtotal gastrectomy
62. Vagotomy is a component of each of these
procedures and is aimed at decreasing acid
secretion through

a. Dopaminergic input to the stomach


b. Nicotinic input to the stomach
c. Muscarinic input to the stomach Harrison’s 19th ed pp 1925
d. Cholinergic input to the stomach
63. Which of the following is not a surgery-related
complication of pud patients? Recurrent ulceration, afferent loop syndromes, dumping syndromes,
postvagotomy diarrhea, maldigestion, malabsorption, gastric
a. Efferent loop syndromes adenocarcinoma, gallstones, cholecystitis
b. Recurrent ulceration
c. Dumping syndrome
d. Gallstones
64. In zollinger-ellison syndrome, initial
presentation and ulcer location may be
distinguishable from common PUD. Where is
the location of the ulcer?

a. Duodenal bulb
b. Antrum
c. Fundus Harrison’s 19th ed pp. 1927
d. Cardia
65. When to obtain a fasting serum gastrin level

a. Multiple ulcers
b. Hypercalcemia
c. Unexplained diarrhea
d. All of the above

66. Which is most sensitive imaging studies used in


zollinger-ellison syndrome in a primary
gastrinoma?

a. Endoscopic ultrasonography
b. MRI
c. CT scan
d. Selective angiography
67. Which is most sensitive imaging studies used in
zollinger-ellison syndrome in a metastatic
gastrinoma?

a. Endoscopic ultrasonography
b. MRI
c. CT scan
d. Selective angiography
68. Which of the following is/are not part of the
etiologies considered for acute gastritis?

a. Vibrio cholerae
b. Helicobacter pylori
c. Mycobacterium
d. Phlegmonous

69. More than how many serotypes of salmonella


are pathogenic to humans?

a. 100
b. 200
c. 300
d. 400
Harrison’s 19th ed chap 190 pp 1049
70. The infectious dose of salmonella infections
ranges starting from how many colony forming
units (cfu):

a. 100
b. 200
c. 300
d. 400
71. The following does not increase susceptibility
to salmonella infection?

a. Antacid ingestion Harrison’s 19th ed chap 190 pp 1049


b. Achlorhydric disease
c. Inflammatory bowel disease
d. Adolescent male

72. Which of the following statements describe


salmonellae species?

a. They are gram-negative, non-spore


forming, facultatively anaerobic bacilli,
measuring 2-3 x 0.4-0.6 micrometers,
produces H2S gas on glucose
fermentation, do not produce cytochrome
oxidase and mostly motile by means of
peritrichous flagella
b. They are gram-negative, non-spore
forming, aerobic bacilli, measuring 2-3 x
0.4-0.6 micrometers, produces no gas on
glucose fermentation, do not produce
cytochrome oxidase and mostly motile by
means of peritrichous flagella Harrison’s 19th ed chap 190 pp. 1049
c. They are gram-negative, non-spore
forming, facultatively anaerobic bacilli,
measuring 2-3 x 0.4-0.6 micrometers,
produces H2S gas on glucose
fermentation, produces cytochrome
oxidase and mostly motile by means of
peritrichous flagella
d. They are gram-negative, non-spore
forming, aerobic bacilli, measuring 2-3 x
0.4-0.6 micrometers, produces no gas on
glucose fermentation, produces
cytochrome oxidase and mostly motile by
means of peritrichous flagella

73. Typhoidal salmonellae disseminate throughout


the body in the macrophages via the
lymphatics and colonize the following tissues
except:

a. Liver Harrison’s 19th ed chap 190 pp. 1049


b. Large intestine
c. Spleen
d. Bone marrow
74. The definitive diagnosis of Enteric Fever
requires the isolation of salmonellae. Which of
the following is 55-90% sensitivity?

a. Urine
b. Bone Marrow
c. Stool
Harrison’s 19th ed chap 190 pp. 1051
d. Blood

75. These drug class are most effective for the


treatment of drug-susceptible typhoid fever
with cure rates of around 98%

a. Macrolides
b. 3rd generation cephalosporins
c. Fluoroquinolones
Harrison’s 19th ed chap 190 pp. 1052
d. Penicillin
76. Most malabsorption syndromes are associated
with
a. Peptic ulcer disease
b. Steatorrhea
c. Lactose intolerance
d. Malignancy Harrison’s 19th ed chap 349 pp. 1932
77. The following is not true for the macroscopic
feature of ulcerative colitis
a. Usually involves the rectum and extends
proximally to involve all or part of a colon
b. Terminal ileum is involved in 90% of cases
c. There is noted backwash ileitis but are of
little significance
d. Proximal spread occurs in continuity Harrison’s 19th ed chap 351 pp. 1951

78. The following is/are true of macroscopic


features of crohn's disease.
a. It can affect any part of the GI tract
b. Small intestine is affected in 75% of cases
c. It is segmental with skip areas in the
midst of diseased intestine
d. All of the above Harrison’s 19th ed chap 351 pp. 1951
79. The following is not a severe presentation of
ulcerative colitis
a. More than 6 bowel movements per day
b. Bloody stools
c. <90 cardiac bpm upon auscultation
d. Spontaneous bleeding and ulcerations on
endoscopy

80. The following statements are true regarding


clinical presentation of ulcerative colitis:
a. Fecal lactoferrin is a highly sensitive and
specific marker for detecting intestinal
inflammation
b. Sigmoidoscopy is used to assess disease
activity and is performed before
treatment
c. The earliest radiologic change of
ulcerative colitis seen on single-contrast
barium enema is a fine mucosal
granularity
d. All of the above Harrison’s 19th ed chap 351 pp. 1952
81. Which of the following statements are true
regarding dermatologic manifestations of
inflammatory bowel disease?
a. P-anca positivity is found in about 60-70%
CD patients
b. Erythema nodosum occurs in up to 90%
of UC patients
c. Pyoderma gangrenosum is seen in 1-12%
of UC patients and less commonly in
crohn’s colitis Harrison’s 19th ed chap 351 pp. 1957
d. All of the above
82. Mainstay of therapy for mild to moderate
ulcerative colitis
a. Sulfasalazine
b. Glucocorticoids
c. Azathioprine
d. Methotrexate Harrison’s 19th ed chap 351 pp. 1959
83. The first biologic therapy approved for crohn’s
disease
a. Methotrexate
b. Infliximab
c. Tacrolimus Harrison’s 19th ed chap 351 pp. 1961
d. Cyclosporine
84. True regarding nutritional therapies for
inflammatory bowel diseases
a. Patients with active CD does not respond
to bowel rest along with TPN
b. Bowel rest and TPN are as effective as
glucocorticoids at inducing remission of
active CD
c. Enteral nutrition in the form of elemental
or peptide-based preparations is not as Harrison’s 19th ed chap 351 pp. 1962
effective as glucocorticoids or TPN
d. Enteral diets may provide the small
intestine with nutrients vital to cell
growth and have the same complications
of TPN
85. Which of the following drugs are
contraindicated for pregnant patients with IBD
a. Methotrexate
b. Sulfasalazine
c. Glucocorticoids
Harrison’s 19th ed chap 351 pp. 1960
d. Azathioprine
86. Patients with crohn’s disease have an
increased risk of developing malignancy such
as
a. Lung cancer
b. Non-hodgkins lymphoma
c. Small bowel adenocarcinoma
d. All of the above
87. What is the key symptom for diagnosing
irritable bowel syndrome?
a. Abdominal discomfort
b. Constipation alternating with diarrhea
c. Flatulence
d. Change in appearance of stool

88. Pathophysiology of IBS includes:


a. Decreased rectosigmoid motor activity for
up to 3 hours after
b. Heightened sensitivity elsewhere in the
body
c. Association of emotional disorders and
stress with symptom exacerbation
d. All of the above

89. Risk factors for developing post infectious IBS


include
a. Male gender
b. Prolonged duration of illness
c. Age older than 60 years
d. Adverse life events in the preceding 1
Harrison’s 19th ed chap 352 pp. 1966
year
90. A 24yo woman has had multiple episodes of
aspiration of food associated with difficulty
swallowing during the past year. On
auscultation of her chest, crackles are heard at
the base of the right lung. A barium swallow
shows marked esophageal dilation above the
level of the lower esophageal sphincter. A
biopsy specimen from the lower esophagus
shows an absence of the myenteric ganglia.
What is the most likely diagnosis?
a. Achalasia
b. Barrett’s esophagus
c. Plummer-Vinson Syndrome
d. Sliding Hiatal Hernia
For the remaining questions, please answer with
the corresponding letters:

A. If it is predominant in UC
B. If it is under CD
C. If it is both in UC and CD
D. If it is none both UC and CD

91. The age of onset is at second to fourth


decades and seventh to ninth decades. C

92. Smoking may prevent the disease. A

93. Patients taking oral contraceptives are at


increased risk. B

94. Appendectomy is deemed to be protective. A

95. There is 38-58% concordance with


monozygotic twins. B

96. Antibiotic use in the first year of life has 2.9x


risk of developing IBD. A

97. It is usually associated with Turner’s


syndrome. C

98. The rectum is spared. B

99. It is a transmural process. B

100. It has pseudopolyps. C


GI II CLINICAL (Retake)

QUESTION RATIONALE
1. Which of the following statement is TRUE of Hepatitis A
and E?
a.May progress to chronic infection
b.Transmitted via percutaneous route
c. No antiviral therapies for treatment
d.Risk factor for liver cirrhosis

2. Among the notable differences between hepatitis B and C,


which of the following statement is TRUE?
a.HCV is a vaccine preventable disease
b.Current treatments for HCV are not completely curable
c. Hepatitis B and C may cause chronic infection
d.Hepatitis D requires the presence of HCV to propagate

3. The urgency for liver transplantation in individuals with HPIM 20th ed. Chap. 329 pp. 2337
decompensated cirrhosis determined using the MELD
score, which is calculated based on a patient’s laboratory
parameters, EXCEPT?
a. ALT
b. Bilirubin
c. Creatinine
d. INR

4. What are the first indicators of HBV infection to appear in Lippincott’s Illustrated Review Microbiology 2​nd​ Ed Page 27
the blood?
a. Anti-HBcAg and Anti-HBeAg
b. HBsAg and Anti-HBcAg
c. HBsAg and HBcAg
d. HBeAg and HBsAg

5. Presence of what titer confers immunity to HBV reinfection?


a. HBeAg and HBsAg
b. Anti-HBcAg and Anti-HBeAg
c. Anti-HBcAg and Anti-HBsAg
d. HBsAg and Anti-HBsAg

6. Which of the following are the characteristics of Hepatitis E


Virus?
a. DNA Virus, double stranded, enveloped
b. RNA Virus, single stranded, non-enveloped,
icosahedral
c. RNA Virus, single stranded, enveloped, icosahedral
d. RNA virus, double stranded, enveloped, icosahedral

7. Functional cure from HBV infection is defined as? GG p. 1120


a. >2log10 decrease in HBV DNA Functional cure is defined as undetectable HBV DNA with
b. Seroconversion from HBeAg to Anti-HBe seroconversion from HBsAg to the anti-HBs.
c. Undetectable HBV cccDNA
d. Seroconversion from HBsAg to Anti-HBs

8. It has nonspecific antiviral and immunomodulatory effects? GG p. 1122


a. IFN Interferons are potent cytokines that possess antiviral,
b. ETV immunomodulatory, and antiproliferative effects.
c. TNF
d. ADV GG p. 1120
...pegIFN-a has nonspecific antiviral and immunomodulatory
effects...
9. Which of the following is NOT one of the treatment goals for GG p. 1122
HBV infection? … treatment goals with either pegIFN-a or nucleoside/-tide
a. Suppress HBV DNA replication analogues are to suppress HBV DNA replication and promote
b. Promote seroconversion from HBsAg to Anti-HBs seroconversion from HBeAg to anti-HBe, which in turn will slow
c. Slow progression of liver disease the progression of liver disease; reduce the risk for
d. Reduce the risk of complications such as HCCa complications, including hepatocellular carcinoma; and prolong
survival.
10. What is the advantage of pegIFN alpha over the GG p. 1123
nucleoside/-tide analogues besides finite treatment of The advantage of a-IFN (pegIFN-a) compared with
infection? nucleoside/-tide analogues, beside finite treatment duration, is
a. Higher rate of HBeAg loss/HBsAg loss the higher rate of HBeAg loss and HBsAg loss with pegIFN-a.
b. Higher rate of HBV DNA reduction
c. Higher rate of Anti-HBs seroconversion
d. Higher rate of Anti-HBc Total reduction

11. A 37 YO Female, asymptomatic, referred to you due to


HBsAg reactive test. On further work-up, she was noted
to have ALT 134 U/L, HBeAg reactive, Anti-HBs
nonreactive; HBV DNA 22,000 IU/mL. Ultrasound showed
fatty liver infiltration of the liver. What will be your
recommendation?
a. No treatment
b. Treat with oral antivirals
c. Monitor LFTs every 3 months
d. Refer for liver transplantation

12. A 23 YO Male, came in due to 1-week jaundice, body


malaise, low grade fever. On work ups, noted with ALT
899 U/L, elevated bilirubins, PT normal, Albumin normal,
Ultrasound revealed mild hepatomegaly. HBsAg reactive;
Anti-HBc IgM Reactive; Anti-HBs non-reactive. What will
be your recommendations?
a. Treat with oral antivirals
b. Monitor and observe
c. Treat with pegIFN and ribavirin
d. Refer for liver transplantation

13. A 35 YO male, came in for clearance prior to work. He


was noted to have HBsAg REACTIVE result. Further
work-ups done showed ALT 40 U/L, HBeAg negative,
Anti HBc Total reactive; Anti HBs non-reactive; HBV DNA
200 IU/mL; Ultrasound showed normal liver, gall bladder,
HBT, spleen. What will be your recommendation?
a. No treatment
b. Treat with oral antivirals
c. Treat with pegIFN
d. Refer for liver transplantation

14. A 60 YO male, known alcoholic, came in due to jaundice


and abdominal enlargement. On work ups, noted with
ALT 27, HBsAg Reactive; HBeAg Reactive; Anti-HBS
non-reactive; Ultrasound revealed nodular liver with
splenomegaly. HBV DNA is 10 IU/mL. What will be your
best recommendation?
a. No treatment
b. Treat with pegIFN
c. Consider liver biopsy
d. Refer for liver transplantation

15. What is the most common cause of acute liver failure? HPIM 20th ed. Chap. 333 pp. 2366
a. Cirrhosis
b. Drug-induced
c. Hepatorenal syndrome
d. Heart failure

16. Which is true for ​idiosyncratic drug hepatotoxicity​? HPIM 20th ed. Chap. 333 pp. 2367
a. Dose dependent a, b, c - direct toxic hepatitis
b. Delayed manifestations for 24-48 hours
c. Produce a characteristic centrilobular zonal necrosis
d. Recognized drugs include isoniazid, phenytoin and
ciprofloxacin

17. Potential mechanisms of drug-induced liver injury HPIM 20th ed. Chap. 333 pp. 2368 Figure 333-1
includes __. 6 mechanisms of liver injury:
a. Influx of intracellular calcium homeostasis resulting in (1) rupture of cell membrane
cell formation (2) injury of bile canaliculus (disruption of transport pumps)
b. Disruption of actin filaments next to canaliculus (3) P-450-drug covalent binding (drug adducts)
leading to ​addition​ of vilous processes (4) drug adducts targeted by CTLs/cytokines
c. Covalent binding to cytochrome P450 enzyme to the (5) activation of apoptic pathway by TNF-alpha/Fas
(6) inhibition of mitochondrial function
drug thus creating ​functioning​ adducts
d. Activation of apoptotic pathways by TNF alpha
a - disruption, b. loss, c. nonfunctioning
18. Which of the following produces cholestasis pattern of HPIM 20th ed. Pp. 2370 Table 333-2
injury?
a. Phenytoin
b. Acetaminophen
c. Sertraline
d. Clopidogrel

19. Which of the following is NOT a histopath finding of the HPIM 20​th​ Page 1291-1292
liver in patients with leptospirosis? Histopathology of the liver shows:
a. Widespread hepatocellular necrosis - Focal Necrosis (widespread hepatocellular necrosis usually
b. Focal necrosis not found)
c. Infiltration of ​Leptospira​ in the Disse’s space - Foci of Inflammation
d. Plugging of bile canaliculi - Plugging of bile canaliculi

20. Which of the following suggests a consumptive


coagulopathy in patients with severe leptospirosis?
a. Diminished thrombin-antithrombin complexes
b. Elevated antithrombin
c. Deregulated fibrinolytic activity
d. Elevated protein C

21. Which of the following syndromes is involved in severe


leptospirosis?
a. Fulminant hepatic necrosis
b. Necrotizing pancreatitis
c. Cholestasis
d. Splenomegaly

22. Which of the following is predictive of higher mortality


rates from severe leptospirosis?
a. Acute hepatic failure
b. GI Bleeding
c. Profound jaundice
d. Altered mental status

23. Which of the following statements is TRUE of severe


leptospirosis?
a. Altered mental status may reflect hepatic
encephalopathy
b. Profound jaundice from hepatic failure is predictive of
higher mortality rate
c. Patients may die of septic shock and/or severe GI
Bleeding
d. Elevated serum amylase levels suggesting acute
pancreatitis is also common

24. Which of the following is NOT included in the triad of


Weil’s Syndrome?
a. Hemorrhage
b. Jaundice
c. Acute liver failure
d. Acute kidney injury

25. Which of the following can be used as treatment for mild


leptospirosis?
a. Doxycycline 100mg PO bid
b. Ampicillin 1g IV q8h
c. Penicillin 1.5 million units IV or IM q6h
d. Ceftriaxone (2g/d IV)
26. Which of the following is true of Amoebic Hepatitis? Schwartz 10th ed. Chap. 31 pp. 1285
a. The abscess formed are ​multiple​ and variable in size di ko sure ani if A or B pero B akong ianswer hehe
b. Amoebas contain proteolytic enzymes that destroys
the liver parenchyma
c. The most common biochemical abnormality is
moderate to severe elevation of ​transaminases
d. The presence of preceding diarrhea is a ​common
finding

27. Which part of the liver is the common location of amoebic Schwartz 10th ed. Chap. 31 pp. 1285
abscess?
a. Superior-anterior aspect of the right lobe
b. Superior-anterior aspect of the left lobe
c. Posterior aspect of the right lobe
d. Inferior aspect of the left lobe

28. Which of the following laboratory finding is present in Schwartz 10th ed. Chap. 31 pp. 1285
amoebic hepatitis?
a. Leukocytosis
b. Elevated transaminases
c. Jaundice
d. Elevated bilirubin levels

29. Which statement about hepatic abscess is TRUE? Schwartz 10th ed. Chap. 31 pp. 1285
a. Pyogenic abscess is the most common type a. amoebic abscess
worldwide b. trophozoite
b. Amoebic cysts will pass through hepatic sinusoids
and into the systemic circulation
c. A positive fluorescent antibody test for ​E. histolytica
has a high sensitivity
c.
d. Ultrasound and CT scan of the abdomen is ​specific
d. very sensitive but nonspecific
for the detection of amoebic abscess

30. What is the most common type of hepatic abscess Schwartz 10th ed. Chap. 31 pp. 1285
worldwide?
a. Pyogenic abscess
b. Amoebic abscess
c. Granulomatous abscess
d. Bacterial abscess

31. Which is an indication for aspiration of hepatic abscess? Schwartz 10th ed. Chap 31 pp. 1285
a. Patients with ​small abscesses
b. Persistent fever
c. Abscess of the left lobe of the liver
d. Persistent abdominal pain

32. Which is true of Epstein-Barr Virus complications? a. Other ​rare complications associated with acute EBV
a. Fulminant hepatitis is a ​common​ complication infection include hepatitis (which can be fulminant),
b. Death is rare and most often due to ​hepatic failure myocarditis or pericarditis, pneumonia with pleural
c. Most neurologic complications resolve with sequelae effusion, interstitial nephritis, genital ulcerations, and
d. Splenic rupture is more common in male than in vasculitis.
females b. Deaths are very rare and are most often due to ​CNS
complications, splenic rupture, upper-airway
obstruction, or bacterial superinfection​.
c. Most cases resolve ​without​ neurologic sequelae.
d. Splenic rupture is more common among male than
female patients ​and may manifest abdominal pain,
referred shoulder pain, or hemodnamic compromise.
HPoIM 20e p. 1359
33. Which of the following is ​NOT a laboratory finding in a. The ​WBC count is usually elevated and peaks at
EBV? 10,000-20,000/uL ​during the ​2nd or 3rd wk​ of illness​.
a. Elevated WBC count in the ​first week​ of illness b. Liver fxn is abN in >90% of cases.
b. Liver function is abnormal in >90% of cases c. serum concn of bilirubin is elevated in approx 40% of
c. Serum bilirubin is elevated in ~40% of cases cases.
d. Serum aminotransferase level are mildly elevated d. Serum levels of aminotransferases and alkphos are
usually mildly elevated.
HPoIM 20e p. 1359
34. What is the most common viral pathogen complicating HPIM p. 1363
organ transplantation? CMV is the most common viral pathogen complicating organ
a. CMV transplantation.
b. EBV
c. Hepatitis B
d. Hepatitis C

35. Which of the following is present in chronic active EBV?


a. Low titers of EBV antibody persistent for >3 months
b. Persistent EBV infection causes chronic fatigue
syndrome
c. Evidence of organ involvement such as
hepatosplenomegaly
d. Chronic active EBV is common

36. Which of the following ​does not present as GI CMV HPIM p. 1363
involvement?
a. Colitis
b. Calculous cholecystitis
c. Hepatitis
d. Stomach ulcers

37. What is the most common clinical manifestation in organ


transplant recipient infected with CMV?
a. Colitis
b. Hepatitis
c. Esophageal ulcers
d. Cholecystitis

38. Which of the following are the most important risk factors HPIM 20th ed. Chap. 335 pp. 2399
for the development of alcoholic liver disease?
a. Sex and duration of alcohol consumption
b. Type of alcoholic beverage and amount consumed
c. Quantity and duration of alcohol consumption
d. Comorbidities such as fatty liver and chronic Hepatitis
C

39. Which is TRUE regarding the pathology of alcoholic liver HPIM 20th ed. Chap. 335 pp. 2399-2400
disease?
a. Hallmark findings of alcoholic hepatitis include
ballooning degeneration and spotty necrosis
b. Fatty liver is irreversible in alcoholic patients
c. Mallory-denk bodies are pathognomonic in alcoholic
hepatitis
d. The liver has extensive response to injurious stimuli,
including excessive alcohol ingestion

40. A 30 YO chronic alcoholic was brought to the ER for HPIM 20th ed. Chap. 335 pp. 2400
intoxication. Which of the following heralds severe
alcoholic hepatitis?
a. Poor nutritional status
b. Elevated ALT
c. Hypoalbuminemia
d. Seizures

41. What is the size of hepatic nodules in patients with HPIM 20th ed. Chap. 337 pp. 2405
alcoholic micronodular cirrhosis?
a. <3mm
b. <5mm
c. <3cm
d. <7mm

42. Which of the following is NOT a direct result of loss of HPIM 20th ed. Chap. 337 pp. 2405
hepatocellular function?
a. Jaundice
b. Coagulation disorders
c. Hypoalbuminemia
d. Hepatic encephalopathy

43. Which of the following is NOT a clinical manifestation of HPIM 20th ed. Chap. 336 pp. 2406
alcoholic liver cirrhosis?
a. Parotid gland enlargement
b. Menorrhagia
c. Digital clubbing
d. Gynecomastia

44. Abby, a 28 YO Female and known case of rheumatic HPIM 20th ed. Chap. 337 pp. 2409
heart disease was brought in the ER due to jaundice.
Pertinent PE findings include icteresia of sclerae, Di ko sure ani. Ang A, B, and C kay all true about ​CARDIAC
distended neck veins, irregular heart rate, bibasal rales, CIRRHOSIS​. Pakibasa na lang sa book.
nontender palpable mass below the right costal margin,
presence of ascites and bipedal edema.
Which of the following is NOT TRUE of Abby’s condition?
a. There is an elevated venous pressure transmitted via
the inferior vena cava and hepatic veins to the
sinusoids of the liver
b. She is unlikely to develop hepatic encephalopathy
and variceal bleeding
c. ALP levels are characteristically elevated, and
aminotransferases may be normal or slightly increase
d. She is a candidate for liver transplant

45. Same Case sa #45. HPIM 20th ed. Chap. 337 pp. 2410
What category of portal hypertension does the abovementioned Di pud ko sure ani. Sorry haha. But relating the definition of
patient has? cardiac cirrhosis:
a. Intrahepatic
b. Posthepatic
c. Prehepatic sa type of portal HTN:
d. Presinusoidal

46. What is the treatment of choice for decompensated HPIM 20th ed. Chap. 337 pp. 2408
cirrhosis due to primary biliary cholangitis (PBC)?
a. Pentoxifylline PBC dosage: ​UDCA 13-15 mg/kg per day (dangerous for PSC if
b. Ursodeoxycholic acid 20 mg/kg per day)
c. Silymarin
d. Liver transplant

47. What is the only approved treatment that slows the rate of
progression of PBC?
a. UDCA
b. Pentoxifylline
c. Silymarin
d. Infliximab

48. What is the dreaded complication of primary sclerosing HPIM 20th ed. Chap. 337 pp. 2409
cholangitis?
a. Cystic fibrosis
b. Cholangiocarcinoma
c. Overt hepatic encephalopathy
d. Massive variceal bleeding

49. Which is true for Idiosyncratic drug hepatotoxicity? HPIM 20th ed. Chap. 333 pp. 2367
a. Dose dependent a, b, c - direct toxic hepatitis
b. Delayed manifestations for 24-48 hours
c. Produce a characteristic centrilobular zonal necrosis
d. Recognized drugs include isoniazid, phenytoin and
ciprofloxacin

50. Which is NOT a cause of abdominal swelling? HPIM 20th ed. Chap. 46 pp. 282
a. Aerophagia
b. Severe constipation
c. Abdominal aortic aneurysm
d. Parapneumonic effusion
d. accumulation of fluid sa ​pleural​ cavity dili sa abdominal cavity
51. Which is a ​LESS COMMON CAUSE​ of ascites? HPIM 20th ed. Chap. 46 pp. 283
a. Infection
b. Familial Mediterranean fever
c. Cirrhosis
d. Hypothyroidism

52. Patient came in at the emergency room due to abdominal HPIM 20th ed. Chap. 46 pp. 284 Figure 46-3
distention. Paracentesis was done which showed a
serum-ascites albumin gradient of ​<1.1g/dL​. Which of the
following is the likely etiology for his ascites?
a. Nephrotic syndrome
b. Cirrhosis
c. Heart failure
d. Liver metastasis

53. Sittie, 67 YO teacher, nonalcoholic drinker, complains of


edema of both lower extremities for 6 months now
associated with nocturnal dyspnea, 2 pillow orthopnea
and is sometimes short of breath. She noticed an
increase in her abdominal girth. What is the most likely
cause of her ascites?
a. Liver cirrhosis
b. Budd-Chiari syndrome Paki-look up na lang sa characteristics/signs/symptoms sa BCS
guys. Di ko sure ani. I think late BCS na ni since nag-increase
c. Heart failure
na ang abdominal girth sa px so it can be inferred na probably
d. Aortic aneurysm
nadamage na ang hepatic sinusoids (late vs. early BCS)
54. What is the most likely laboratory result that best explains
Sittie’s condition?
a. SAAG >1.1g/dL, ascitic protein <2.5g/dL
b. SAAG >1.1g/dL, ascitic protein >2.5g/dL
c. SAAG <1.1g/dL, ascitic protein <2.5g/dL
d. SAAG <1.1g/dL, ascitic protein >2.5g/dL

55. Aldrin is diagnosed with liver cirrhosis and was advised HPIM 20th ed. Chap. 46 pp. 284
sodium restriction to 2g/d and was given Spironolactone
as maintenance medication, however, he complained of
painful gynecomastia with this medication. What
medication may be substituted for spironolactone for this
scenario?
a. Furosemide
b. Amiloride
c. Midodrine
d. B-Blockers

56. What is the mainstay of treatment for hepatic HPIM 20th ed. Chap. 337 pp. 2413
encephalopathy?
a. Zinc supplementation
b. Dietary protein restriction
c. Aminoleban goal of lactulose - to produce 2-3 soft stools per day
d. Lactulose zinc supplementation​ - relatively harmless
dietary protein restriction​ - discouraged
57. Which of the following statement is TRUE of hepatorenal HPIM 20th ed. Chap. 337 pp. 2413
syndrome (HRS)?
a. The best therapy is ​kidney transplant
b. The best therapy for HRS is liver transplant
c. Type I HRS has better outcome than Type II HRS
d. Dopamine​ is currently used for treatment Type II HRS has better outcome than Type I HRS

58. What is the common organism responsible for HPIM 20th ed. Chap. 46 pp. 285
spontaneous bacterial peritonitis (SBP)?
a. Enterococcus
b. Staphylococcus
c. Streptococcus
d. E. coli

59. Which is NOT TRUE of ​primary​ SBP? HPIM 20th ed. Chap. 46 pp. 285
A. Cirrhotic patients with GI Bleeding should receive oral
daily norfloxacin
B. Diuretics may increase the risk of SBP
C. Culture of ascitic fluid typically reveals ​2 or more
organisms
D. SBP is defined by PMN count of ​>150u/L

Hi. Basin 250 ang sa D. Or sayop lang jud ang nabutang sa


exam. If ever the same question ang ibalik kay letter C akong
ianswer hehe. Laban!

See also ​HPIM 20th ed. Chap. 337 pp. 2413


60. Which of the following is an effective treatment against HPIM 20th ed. Chap. 46 pp. 285
Gram-negative and Gram-Positive anaerobes in SBP?
a. IV Cefotaxime
b. IV Metronidazole
c. IV Ciprofloxacin
d. Rifaximin

61. Which of the following is an inherited disorder of iron HPIM 20th ed. Chap. 337 pp. 2409
metabolism that results in progressive increase in hepatic a. inherited disorder of copper metabolism
iron deposition? b.
a. Wilson’s disease
b. Hemochromatosis
c. α1 AT Deficiency
d. NOTA
c. abnormal folding of α1 AT protein
62. Treatment for Wilson’s Disease involves _______. HPIM 20th ed. Chap. 337 pp. 2409
a. UDCA
b. Copper chelators
c. Iron chelators
d. Phospholipids for hemochromatosis - ​regular therapeutic phlebotomy

63. Which of the following is diagnostic of hemochromatosis? HPIM 20th ed. Chap. 337 pp. 2409
a. Elevated transferrin saturation
b. Decreased​ ferritin levels
c. Increased ceruloplasmin levels
d. Increased 24hr urine copper levels b. increased
c & d - wilson's disease
64. Which of the following drugs is associated with hepatic HPIM 20th ed. Chap. 336 pp. 2402
steatosis?
a. Lanoxin Refer to ​Figure 336-2
b. Spironolactone
c. Estrogens
d. Losartan

65. What is the most common cause of acute liver failure?


a. Cirrhosis
b. Drug-induced
c. Hepatorenal syndrome
d. Heart failure

66. Which of the following suggests a diagnosis of advanced HPIM 20th ed. Chap. 336 pp. 2403
NAFLD?
a. Concomitant upper GI Bleeding
b. Prothrombin time prolongation
c. Pedal edema
d. Elevated transaminases to more than 2x the upper
limit

67. What is the gold standard for establishing the severity of HPIM 20th ed. Chap. 336 pp. 2403
liver injury and fibrosis in NAFLD?
a. CT Scan of the liver
b. History and PE of the patient
c. Collation of albumin, protime, bilirubin findings
d. Liver biopsy

68. What is the foundation for NAFLD treatment? HPIM 20th ed. Chap. 336 pp. 2403
a. Antioxidants
b. Metformin
c. Lifestyle changes and dietary modification
d. Liver transplant

69. Which of the following is NOT a mechanism of gallstone HPIM 20th ed. Chap. 339 pp. 2423
formation?
a. Increased biliary cholesterol secretion a. the most important mechanism
b. Gallbladder hypomotility Two other conditions are associated with cholesterol-stone or
c. Nucleation of cholesterol monohydrate crystals biliary sludge formation: ​pregnancy and ​rapid weight reduction
d. Increased peripheral fat mobilization through a very-low-calorie diet

70. Which of the following is a predisposition to develop HPIM 20th ed. Chap. 339 pp. 2425
pigment stone?
a. Cystic fibrosis
b. Fasting
c. Pregnancy
d. Prolonged parenteral nutrition

71. Which of the following is a predisposition to develop


cholesterol stone?
a. Estrogen
b. Pernicious anemia
c. Ileal resection
d. Alcoholic liver cirrhosis

72. What is the gold standard in treating symptomatic HPIM 20th ed. Chap. 339 pp. 2427
cholelithiasis?
a. Meperidine IV
b. Ursodeoxycholic acid
c. Laparoscopic cholecystectomy
d. ERCP
73. During deep inspiration or cough during subcostal HPIM 20th ed. Chap. 339 pp. 2427
palpation of RUQ usually produces increased pain and
inspiratory arrest called ______.
a. Murphy’s sign
b. Rovsing’s Sign
c. Levine’s sign
d. Mirizzi sign

74. What is the complication in which a gallstone becomes HPIM 20th ed. Chap. 339 pp. 2427
impacted in cystic duct or gallbladder neck causing CBD
compression, resulting to CBD obstruction and jaundice?
a. Murphy’s sign
b. Zieve’s syndrome
c. Mirizzi’s syndrome
d. Levine’s syndrome

75. What is the most frequently isolates in emphysematous HPIM 20th ed. Chap. 339 pp. 2428
cholecystitis?
a. Enterococci
b. Bacteroides
c. Streptococci
d. Clostridium perfringens

76. Tita was ​coughing after a trip to ​China with a special Belizario Chap. 5 pp. 245
friend. Sputum results showed a ​coffee bean shaped egg, "The adult lung fluke is reddish brown and measures ​7-12 mm in
reddish-brown and measures ​7 to 12 m in length, this is length, 4-6 mm in width, 3.5-5 mm in thickness, and ​resembles a
most probably which of the following? coffee bean​."
a. Fasciolopsis buski
b. Paragonimus westermani HPIM 20th ed. Chap. 230 pp. 1641
c. Schistosoma japonicum "The symptoms and signs of paragonimiasis are fever, cough,
hemoptysis, and peripheral eosinophilia."
d. Clonorchis sinensis

77. The reason why Tita was having the disease is because HPIM 20th Ed. Chap. 230 pp. 1641
of which of the following
a. Her special friend is poisoning her because of her "Paragonimiasis is a parasitic lung infection caused by lung
bad mole and bad attitude and bad dancing flukes of the genus Paragonimus. It is a food-borne parasitic
b. She had ingested some raw salad and raw or zoonosis, with most cases reported from Asia and attributable
insufficiently cooked pork to consumption of raw or undercooked freshwater crustaceans.
c. She was able to ingest raw or insufficiently cooked
"Anamnestic information about the consumption of raw or
crabs
undercooked freshwater crabs by immigrants, expatriates, and
d. She was able to eat food contaminated with cat fecal
returning travelers."
matter

78. Consumption of the infective larval stage encysted on For #s 78-84:


aquatic plants that have not been cooked results in
infections with:
a. Clonorchis sinensis
b. Heterophyes heterophyes
c. Fasciola hepatica
d. Paragonimus westermani

79. John went to a dinner conference at a 5-star hotel in USA


for “Physicians of the 3​rd World Country.” He was elected
as the President of the society. Seven days after coming
from the conference, Troy’s face was swollen, as well as
his abdomen and lower limbs and his right testicle. His
stool was that of ​Fasciolopsis buski​. In the party menu,
aquatic plants were served. Which of the following plant
was served?
a. Trapabicornis
b. Ipomea obscura
c. Kangkong
d. AOTA

80. A Schistosoma egg with a terminal lateral spine would be


most likely found in which of the following?
a. Feces
b. Bile
c. Sputum
d. Urine

81. The infective form of ​Schistosoma mansoni is which of


the following?
a. Cercaria swimming in water
b. Metacercaria encysted on water chestnuts or
vegetables
c. Metacercaria on or in fish
d. Cercaria on water chestnuts, bamboo shoots, or
vegetables

82. In the life cycle of hermaphroditic flukes, the infective


stage for the snail is which of the following?
a. Cercaria
b. Miracidium
c. Metacercaria
d. Egg

83. The adult form of the sheep liver fluke produces large,
ovoid, unembryonated eggs with a yellowish-brown shell
and inconspicuous operculum. These eggs are
morphologically most similar to which of the following?
a. Fasciola hepatica
b. Fasciolopsis buski
c. Clonorchis sinensis
d. Paragonimus westermani

84. The incidence of small, yellowish, oval, operculated eggs


with a short, comma-like extension of the shell opposite
the operculum increased markedly in stools examined in
the US with the influx of refugees from Vietnam and
nearby countries. Although not transmitted in the US,
human infection in endemic acquired by which of the
following?
a. Consumption of infected water vegetation
b. Direct larval penetration of human skin
c. Eating infected crabs or crayfish
d. Ingestion of infected freshwater fish
85. What is the major symptom of acute pancreatitis? HPIM 20e p. 2439
a. Jaundice
b. Fever Abdominal pain is the major sx of AP. PAin may vary from a
c. Abdominal pain mild discomfort to severe, constant, and incapacitating distress.
d. Vomiting

86. The faint blue discoloration around the umbilicus that HPIM 20e p. 2439
occur as a result of hemoperitoneum due to pancreatitis
is called? Cullen’s sign​: faint blue discoloration around the umbilicus
a. Turner’s sign (may occur as the result of hemoperitoneum)
b. Cullen’s sign
c. Splading’s sign Turner’s sign​: blue-red-purple or green-brown discoloration of
d. Rovsing’s sign the flanks (ref;ects tissue catabolism of hemoglobin from severe
necrotizing pancreatitis w/ hemorrhage
87. John came in to the ER due to 1 week history of severe
epigastric discomfort associated with fever and vomiting.
CT Scan was ordered which revealed an acute
inflammation of the pancreatic parenchyma and
peripancreatic tissues, what is the most likely diagnosis
according to revised Atlanta criteria?
a. Pancreatic pseudocyst
b. Interstitial pancreatitis
c. Necrotizing pancreatitis
d. Walled-off necrosis

88. What laboratory test is more specific of pancreatitis? Lipase is the single best enzyme to measure for the dx of AP.
a. Amylase
b. Lipase HPIM 20e p. 2434
c. Hyperbilirubinemia
d. Hemoconcentration

89. Which among the following parameters are NOT included


as a marker of severity at admission or within 24 hours in
Acute Pancreatitis?
a. Hemoconcentration
b. BUN>25 mg/dL
c. Pleural effusion
d. Hypocalcemia
90. What is the severity classification of pancreatitis in which
there is a transient organ failure that resolves in less than
48 hours?
a. Mild acute pancreatitis
b. Mild to moderate acute pancreatitis
c. Moderately severe acute pancreatitis
d. Severe acute pancreatitis

91. What is the management of acute pancreatitis?


a. Aggressive fluid resuscitation
b. Oral analgesics
c. Hematocrit and BUN every 4 hours
d. Prophylactic antibiotics

92. Marker of severity during hospitalization includes: refer to table 341-3 on #89
a. Persistent hemoconcentration
b. Elevated BUN
c. Persistent organ failure
d. Elevated serum amylase

93. TRUE regarding initiation of feeding in patients with Acute


Pancreatitis
a. Low fat solid diet can be given only after abdominal
pain is resolved
b. Parenteral nutrition should be initiated 2-3 days
admission in subjects with more sever pancreatitis
c. Parenteral feeding maintains gut barrier integrity and
limits bacterial translocation
d. AOTA
94. What is the pathophysiology of Chronic Pancreatitis?
a. Stellate cell inactivation resulting in cytokine
overexpression
b. Production of ECM proteins causing collagen
deposition
c. Progressive destruction in paracrine and exocrine
tissues
d. AOTA

95. What is the MOST common environmental risk factor for


pancreatic cancer?
a. Alcohol
b. Recurrent viral infection
c. Smoking
d. Chronic pancreatitis

96. What is the MOST common clinical presentation if CA is


in the head of the pancreas?
a. Abdominal pain
b. Jaundice
c. Vomiting
d. Weight loss

97. Which of the following statements are true of hypovolemic


shock?
a. Infusion of 2-3 L of salt solution over 20-30 mins
should restore hemodynamic parameters
b. Continuing acute blood loss with hemoglobin
concentration declining to <100g/L should initiate
blood transfusion
c. Successful resuscitation also requires O​2 support of
respiratory function, hence, O​2 therapy
​ should be
provided
d. All are true

Answers on Qs 97-100 are found in HPoIM 19e Ch.324.


98. What is the frequent adverse consequence of massive
volume resuscitation?
a. Pulmonary congestion
b. Hypothermia
c. Acute kidney injury
d. ARDS

99. What is the major determinant of tissue perfusion?


a. Cardiac output
b. Cardiac preload
c. Myocardial contractility
d. Respiratory Rate

100. Which of the following sympathomimetic amines that acts Refer to screenshot in #98 :)
as inotropic and chronotropic agent that also supports
vascular resistance in those whose BP will not tolerate
peripheral vascular dilation?
a. Norepinephrine
b. Dobutamine
c. Dopamine
d. Epinephrine

PART II.
1. Which of the following hepatitis virus replicates like a retrovirus?
a. Hep A
b. Hep B
c. Hep C
d. Hep E
2. The ff is NOT true for hep A virus?
a. It has an incubation period of ~4 weeks
b. Its replication is limited to liver
c. The diagnosis of hep A is made during the acute illness by demonstrating anti-HAV
IgMclass
d. Anti-HAV IgM class will persist for up to 8-12 mos

3. Which of the following statements is NOT true about hepatitis B?


a. disappearance of HBeAg is a harbinger of clinical improvement
b. persistence of HBeAg in serum beyond 3 mos. of acute infection may be predictive of
development of chronic infection
c. the presence of HBeAg during chronic hepatitis is associated with ongoing viral replication,
infectivity, and inflammatory liver injury
d. the 3rd and largest of the HBV gene is the C gene

4. Which of the following is the protective antibody?


a. Anti-HBs
b. Anti-HBc IgG
c. anti-HBe
d. anti-HBcIgM
5. Which of the following hepatitis virus is a defective RNA virus that coinfects and
requires the helper function of HBV?
a. Hepatitis A
b. Hepatitis B
c. Hepatitis C
d. Hepatitis D
6. Which of the following is a linear, single strand, positive-sense, 9600 nucleotide RNA
virus and is also called non-A, non-B hepatitits?
a. Hepatitits D
b. Hepatitits C
c. Hepatitis B
d. Hepatitits A
7. What timing of infection can culminate later in cirrhosis and hepatocellular CA?
a. neonatally acquired HBV infection
b. adolescence
c. adulthood
d. elderly acquired

8. Which of the following lab findings do not correlate with severe hepatocellular
disease?
a. low serum albumin
b. hypoglycemia
c. very high serum bilirubin
d. very high aminotransferases
9. Fulminant Hepatitis is rarely seen in which of the following?
a. Hepa A ??
b. Hepa B
c. Hepa C ??
d. Hepa D

10. You are consulted by a nurse after she had a needle stick injury while inserting IV
line on a live liver cirrhosis patient who happened to be hepatitis reactive. How will you
manage?
a. give a single dose HBIG at 0.66 ml/kg soon after exposure
b. give hepatitis B vaccine
c. request for HBsAg and give HBIg if reactive, otherwise give hepa B vaccine
d. give reassurance that transmission of the disease is unlikely
Harrison p.2365 20th ed
11. What Hepatitis profile do you observe in Acute Hepatitis C?
a. (+) HBsAg, (+) anti-HBc IgM, (-) anti HAV IgM
b. (+)HBsAg, (-) anti-HBc IgM, (-) anti HAV IgM
c. (-) HbsAg, (-) anti-HBc IgM, (+) anti HCV
d. (+) HBsAg, (+) anti-HBc IgM, (+) anti-HAV IgM
12. What Hepatitis profile do you observe in Acute Hepatitis A and B?
a. (+) HBsAg, (+) anti-HBc IgM, (-) anti HAV IgM
b. (+)HBsAg, (-) anti-HBc IgM, (-) anti HAV IgM
c. (-) HbsAg, (-) anti-HBc IgM, (+) anti HCV
d. (+) HBsAg, (+) anti-HBc IgM, (+) anti-HAV IgM
13. What Hepatitis profile do you observe in Chronic Hepatitis B?
a. (+) HBsAg, (+) anti-HBc IgM, (-) anti HAV IgM
b. (+)HBsAg, (-) anti-HBc IgM, (-) anti HAV IgM
c. (-) HbsAg, (-) anti-HBc IgM, (+) anti HCV
d. (+) HBsAg, (+) anti-HBc IgM, (+) anti-HAV IgM Harrison p.2361 20th ed
14. What Hepatitis profile do you observe in Acute Hepatitis B?
a. (+) HBsAg, (+) anti-HBc IgM, (-) anti HAV IgM
b. (+)HBsAg, (-) anti-HBc IgM, (-) anti HAV IgM
c. (-) HbsAg, (-) anti-HBc IgM, (+) anti HCV
d. (+) HBsAg, (+) anti-HBc IgM, (+) anti-HAV IgM
15. What Hepatitis profile do you observe in Acute Hepatitis A?
a. (+) HBsAg, (+) anti-HBc IgM, (-) anti HAV IgM
b. (+)HBsAg, (-) anti-HBc IgM, (-) anti HAV IgM
c. (+) HbsAg, (+) anti-HBc IgM, (+) anti HCV
d. (-) HBsAg, (-) anti-HBc IgM, (+) anti-HAV IgM
16. Which of the following is not among the goal therapy for patients with fulminant hepatitis?
a. maintenance of fluid balance
b. correction of hypoglycemia
c. circulatory and respiratory support
d. glucocorticoid therapy
Harrison p.2364 20th
17. What is the proper Hepatitis A prophylaxis for less than 4-week exposure to Hepa A
endemic places?
a. give IG prophylaxis at 0.02 ml/kg
b. give IG prophylaxis at 0.06 ml/kg
c. give advice only to just be cautious with the food she eats and do frequent handwashing
d. give IG prophylaxis at 0.3 ml/kg
18. Which of the following drugs is pregnancy category Band may be given during the
third trimester to prevent mother to child transmission as well as maternal treatment for
Hepatitis B?
a. adefovir
b. tenofovir
c. PEG-INF
d. lamivudine

19. Which of the following antivirals for Hepatitis B is contraindicated in cirrhotics due
to risk of decompensation?
a. lamivudine
d. entecavir
c. tenofovir
d. pegylated interferon
Case Scenario for 20-23
Shiela, a 20 year old male, was brought to the emergency room due to loss of consciousness.
Patient allegedly ingested large amount of acetaminophen 5 hours prior to consult
20. At what dose will Acetaminophen toxicity lead to a fulminant disease?
a. 8 grams
b. 10 grams
c. 25 grams
d. 80 grams

Harrison p.2371 20th


21. what laboratory tests for this patient revealed hyper acute injury of the liver showing
____
a. elevated AST, low bilirubin
b. elevated AST, high bilirubin
c. elevated alkaline Phosphatase (ALT), high bilirubin
d. elevated ESR, elevated BUN Harrison p.2371 20th
22. At this time, acetaminophen blood level >200ug/mL, what is the best treatment for
this patient?
A. IV glutathione
b. oral activated charcoal per NGT
c. oral administration of cholestyramine
d. IV administration of N-acetylcysteine
23. However, on the 5th hospital day, patient still had progressive jaundice and
coagulopathy despite medical treatment. What is the next step?
A. initiate STAT hemodialysis
b. suggest liver transplantation
c. continuous IV N-acetylcysteine administration
d. admit to ICU and attach to mechanical ventilator
24. Which of the following is a potential mechanism of drug-induced liver injury?
a. influx of intracellular calcium homeostasis resulting in cell formation
b. disruption of actin filaments next to canaliculus leading to addition of vilous processes
c. covalent binding to cytochrome P450 enzyme to the drug, creating functioning adducts
d. activation of apoptotic pathways by TNF alpha

25. A 25 year old male complaining of RUQ pain radiating to the shoulder associated
with fever for 12 days, Upon examination, there is point tenderness over the liver. Which
of the following tests would NOT help you in your diagnosis?
a. ultrasound of the abdomen
b. fluorescent antibody test for E. histolytica
c. stool exam
d. liver transaminases
26. Which complication carries the worst prognosis for hepatic abscess?
a. hepatobronchial fistula
b. abscess rupture into the peritoneum
c. abscess rupture into the pericardium
d. abscess rupture into the pleural space

27. Which of the following DOES NOT indicate/warrant surgical drainage of hepatic
abscess?
a. nonresponders to medical treatment
b. those who appear superinfected
c. Abscess on the left lobe
d. Multiple abscess

28. Which of the following is NOT true about leptospirosis?


a. severe leptospirosis is characterized by jaundiced, renal dysfunction, and hemorrhagic
diathesis
b. rodents are the most important reservoir
c. human-human transmission is rare
d. it is associated with fulminant hepatic necrosis

29. What is the drug of choice for severe leptospirosis?


a. Ceftriaxone (2nd if )
b. Penicillin (1st)
c. Meropenem
d. Piperacillin-tacobactam

30. What is the recommended oral treatment for mild leptospirosis?


a. Cefixime
b. Cefalixen
c. Ciprofloxacin
d. Doxycycline
1. Hypertension is thought to be mostly a polygenic B. Beta-2 adrenoreceptor dapat
disorder with significant contribution from Harrison's, pg 1891
environmental exposure. Genome-wide
association studies have identified the
hypertension-related genes EXCEPT
a. Renin-angiotensin-aldosterone
components
b. Alpha 2 adrenoreceptor
c. Atrial natriuretic peptide
d. Alpha adducin

2. The ff environmental factors contribute to hypertension D. Low dietary intake of calcium and potassium also may
EXCEPT contribute to risk of HTN
A. High dietary NaCl intake Harrison's, pg 1891
B. High-intensity aerobic exercise
C. Degree of alcohol consumption
D. High dietary calcium and potassium intake

3. You have a newly-diagnosed, 54 y.o. female C.


hypertensive patient in your clinic, for better compliance Harrison’s pg 1894
with lifestyle modification and medications, you can
encourage her with which of the ff benefits of
well-controlled BP
a. Reduction of hemorrhagic but not ischemic strokes
b. Improvement of increased dementia scores
c. Reversal of existing left ventricular hypertrophy
d. Reversal of advanced glomerulosclerosis

4. Which of the ff statements on the different methods of A.


measuring BP is TRUE? Harrison's pg 1895
A. Nighttime BP are 10-20% lower than daytime
pressures
B. Home BP are generally higher than office/clinic
pressures
C. Attenuated nighttime BP “dips” are associated with
lower CV diseases risk
D. Office/clinic BP more accurately predict end-organ
damage than home BP

5. What is the most common cause of secondary B.


hypertension? Harrison’s pg 1895
a. Thyrotoxicosis
b. Renal disease
c. Obstructive sleep apnea
d. Fibromuscular dysplasia

For #s 6-8
A 76 y.o male with a 5-y history of coronary artery disease
consults at your clinic for ​new onset stage II hypertension
detected only a month prior. He claims that he has been
regularly monitoring his BP​ at home for the past 5 y with
acceptable results​. He is also a​ chronic heavy smoker​.
The patient consulted another physician 2 wks ago and
was started on ​Enalapri​l but follow up with labs reveals a
large increase in serum creatinine after starting the drug​.
Baseline s​erum potassium is norma​l. PE shows ​no
significant dermatologic or joint abnormalities

6. What possible secondary cause of hypertension is most B?


likely in this patient? Harrison’s pg 1897
a. Lupus nephritis x
b. Real artery stenosis
c. Primary aldosteronism x
d. Adrenal cushing's disease
7. Which important PE finding will greatly support your C.
diagnosis?
a. Retinal hemorrhages
b. Decreased ankle-brachial index
c. Lateralizing flank bruit
d. Systolic pressure gradient between right arm and
legs

8. What is the “gold standard” diagnostic test for D.


confirming your impression?
a. Antinuclear antibody test
b. Serum aldosterone-renin ratio
c. Adrenal venous sampling for cortisol
d. Renal artery angiography

9. What is the most common congenital cardiovascular C.


cause of hypertension? Harrison’s pg 1899
a. Patent ductus arteriosus
b. Situs inversus
c. Coarctation of the aorta
d. Mitral valve prolapse

10. C.L. is a 60 y.o. Patient with hypertension and


advanced chronic kidney disease (not yet on dialysis);
currently with bipedal edema. Latest labs show creatinine
of 255 umol/L and ​serum potassium of 6.4 mEq/L​. Which
of the ff anti-hypertensive drugs can be safely given for
this case?
a. Furosemide
b. Enalapril
c. Irbesartan
d. Spironolactone -x: K+ sparing

11. Which of the ff dietary recommendations for 2 possible answers: A and B


hypertension are FALSE? Pero murag naa to NOT more than 2 drinks sa exam, so
a. Males should consume more than 2 drinks of basin B lang ang answer if naay NOT sa A
alcohol per day Harrison’s pg 1901
b. Diabetic or heart failure patients on diuretics
should aim for very rigorous dietary salt reduction
c. Potassium supplementation may result in
decreased stroke mortality
d. Regular intake of low fat dairy products should be
encouraged

12. H.L. is a 55 y.o. Hypertensive female with concomitant Not sure


bronchial asthma who had a recent exacerbation 2 wks
prior. The ff. antihypertensive agents may be given for her
BP EXCEPT? Dili letter C? Kay ang beta blockers diba kay one of their
A. Perindopril side effects kay bronchospasm?
B. Candesartan
C. Metoprolol
D. Felodipine

13. An obese patient seeks medical consult for distinctive HPIM 20th ed. Chap. 400 pp. 2896
papule clusters on the knees and elbows growing to the
size of grapes, associaetd with orange-yellow
discolorations of the creases of both palms. He probably
has what type of primary hyperlipoproteinemia?
a. Familial dysbetalipoproteinemia
b. Familial hepatic lipase deficiency
c. Sitosterolemia
d. Lysosomal acid lipase deficiency

14. You have a patient who was diagnosed with familial HPIM 20th ed. Chap. 400 pp. 2894
hypertriglyceridemia several months ago after presenting
with acute pancreatitis. Which of the following lipid-profile
findings do you expect to see in this patient?
a. High​ HDL levels
b. High​ apoB levels
c. Low VLDL levels
d. Low to normal LDL levels

15. Among patients with familial hypercholesterolemia HPIM 20th ed. Chap. 400 pp. 2894
syndrome (FH) what is the most commonly mutated
gene?
a. ABCG5 gene
b. ApoE2 variant
c. LDLR gene
d. PCSK9

16. H.T. is a 58 y.o. Female who underwent total HPIM 20th ed. Chap. 400 pp. 2894
thyroidectomy 5 y ago, with poor compliance to
levothyroxine replacement. The ff. Lipid profile Thyroid replacement therapy usually ameliorates the
derangements are expected in her lipid profile EXCEPT? hypercholesterolemia; if not, the px probably has a primary
a. High LDL levels lipoprotein disorder and may require lipid-lowering drug
b. Low HDL levels therapy with a statin.
c. High IDL levels
d. High triglycerides levels Di ko sure ani. Tabang haha

17. One of your patient asks you regarding the potential


benefits of regular alcohol intake after reading an internet
blog. What specific benefit in the lipid profile does regular
alcohol intake provide?
a. Lowers LDL levels
b. ​Increases HDL levels
c. Lowers​ VLDL levels
d. Lowers lipoprotein A levels

18. Which of the ff genetic defects DECREASES the risk HPIM 20th ed. Chap. 400 pp. 2898
of coronary heart disease?
a. Familial hypobetalipoproteinemia
b. Abetalipoproteinemia
c. Sitosterolemia
d. PCSK9 deficiency

19. The ff. Are conditions highly associated with obesity HPIM 20th ed. Chap. 394 pp. 2841
EXCEPT? Syndromes of obesity include: Prader-Willi,
A. Prader-Willi syndrome Laurence-Moon Biedl, Ahlstrom's, Cohen's, and
B. Conn’s syndrome Carpenter's.
C. Carpenter’s syndrome
D. Cushing syndrome Refer to ​Table 394-2.

20. Which of the following statements best describes the HPIM 20th ed. Chap. 394 pp. 2842
status of leptin in patients with common obesity?
a. They have functional leptin resistance
b. They have leptin deficiency
c. They have mutated leptin receptors
d. They have increased levels of enzymes that
metabolize

21. You have an obese but otherwise healthy female HPIM 20th ed. Chap. 394 pp. 2843
patient in the clinic seeking consult for her wight. The ff.
are common consequences of her obesity that you should Obesity in females is associated with higher mortality
educate her about EXCEPT? from cancer of the ​gallbladder, bile ducts, breasts,
a. Osteoarthritis endometrium, cervix, and ovaries.
b. Diabetes mellitus Obesity in males is associated with higher mortality from
c. Congestive heart failure cancer of the esophagus, colon, rectum, pancreas, liver,
d. Liver​ cancer and prostate.

22. M.F. is a 28 y.o. female seeking medical assistance for HPIM 20th ed. Chap. 397 pp. 2865
her progressive increase in weight. Upon reviewing her
concomitant medications which of the ff. is NOT likely to Metformin reduces fasting plasma glucose (FPG) and
contribute to her problem? insulin levels, improves the lipid profile, and ​promotes
a. Clozapine modest weight loss.​ An extended-release form is
b. Pioglitazone available and may have fewer GI side effects (diarrhea,
c. Metformin anorexia, nausea, metallic taste).
d. hydrocortisone

23. What is the typical caloric content of a very low calorie HPIM 20th ed. Chap. 395 pp. 2845
diets (VLCS)?
a. ≤800 kcal/day
b. 1000-1200 kcal/day
c. 1200-1500 kcal/day
d. 1500-1800 kcal/day

24. Which of the ff. Intervention for wt. Loss have been HPIM 20th ed. Chap. 395 pp. 2850
found to cause complete remission of type 2 DM in many
cases?
a. Liraglutide
b. Stress management
c. Use of meal replacements
d. roux-en-Y gastric bypass

Schwartz 10th ed. Chap. 27 pp. 1100, 1116


"The ​Roux-en-Y gastric bypass​ is the most commonly
performed bariatric procedure.
"Resolution of comorbidities varies, but is over 90% for
GERD and venous stasis ulcers and ​over 80% for patients
with type 2 diabetes of less than 5 years in duration​."

25. Which of the ff lab results is consistent with a HPIM 20th ed. Chap. 396 pp. 2850
diagnosis of pre-diabetes?
a. Glycosylated hemoglobin of ​7.0%
b. Fasting plasma glucose of ​130 mg/dL
c. 2 hr OGTT result of 160 mg/dL
d. Trace of glycosuria on urine dipstick

26. You have a patient with gestational diabetes with HPIM 20th ed. Chap. 396 pp. 2851
adequate glycemic control on proper diet alone. She
recently gave birth via spontaneous vaginal delivery and
the ff. Statement regarding her long-term prognosis are
true EXCEPT?
a. She has a large chance of reverting back to normal
glucose tolerance postpartum
b. She has a 35-60% risk of developing DM in the
next 10-20 y
c. Her child has increased risk of developing type 1
DM during adolescence
d. She should undergo lifelong screening for the
development of diabetes

27. The ff. Epidemiologic trends regarding DM are true HPIM 20th ed. Chap. 396 pp. 2852
EXCEPT
a. Asians have increasing diabetes prevalence at
lower BMI compared to caucasioans
b. Prevalence of type 1 DM is highest in
Scandinavian countries
c. Prevalence of type 2 DM is highest in the Pacific
Islands and Middle East
d. Women have higher prevalence of DM compared
to men

28. Which of the ff. Statements regarding the HPIM 20th ed. Chap. 396 pp. 2855
pathophysiology of type 1 DM is FALSE?
A. Autoimmune destruction is mainly caused by
T-lymphocyte cytotoxicity and release of cytokines
B. Autoimmune destruction in type 1 DM also involves
alpha, delta and PP-producing islet cells
C. Testing for GAD-65 and insulin antibodies helps in
identifying type 1 diabetics
D. Specific environmental triggers have been
conclusively linked to type 1 DM

29. You have an elderly type 2 diabetic patient who is HPIM 20th ed. Chap. 396 pp. 2858
scheduled for his annual foot examination. This will include
the ff. Assessments EXCEPT?
a. 10-g monofilament test
b. Palpation of pedal pulses
c. Ankle reflexes
d. Foot extension moto strength

30. X.M. is a 25 y.o. Female with type 1 DM who received Di ko sure asa ani pero murag letter C ang answer. RBS
blood transfusion 1 mo. prior due to abnormalvaginial kay same lang sa capillary blood glucose di ba? Please
bleeding. The ff. Tests can be used to determine her pakicheck! Thanks
glycemic control EXCEPT?
a. Fasting plasma glucose
b. 2 h OGTT Hindi ba Hba1c yung answer? nasa previous ratio to ng
c. Capillary blood glucose 1st exam before nung retake. HPIM20th ed pg. 2862
d. Hba1c

31. Which of the ff. Dietary recommendations in DM is HPIM 20th ed. Chap. 397 pp. 2860
INCORRECT?
a. Fructose is preferred over sucrose
b. Routine use of vitamins and antioxidants is
recommended
c. Trans fat consumption should be minimized
d. Diet should be rich in monounsaturated fatty acids

32. When prescribing an exercise regimen to a type 1 DM HPIM 20th ed. Chap. 397 pp. 2861
patient which of the ff. Recommendations should be
advised?
a. Glucose monitoring should be done before, during
and after prolonged exercise
b. Delay exercise if glucose >250 mg/dL and ​negative
for ketones
c. Ingest carbohydrates prior to exercise if glucose <
200 mg /dL
d. Adjust insulin injection site to a ​highly exercising
area

33. Insulin should be considered in the ff. Conditions as HPIM 20th ed. Chap. 397 pp. 2868
the initial therapy in type 2 DM EXCEPT (H 3196)
a. In all ​obese​ individuals
b. In patients with underlying renal or hepatic disease
c. In hospitalized or acutely ill patients
d. Patients intolerant to first line oral medications

34. The ff. Theories on the mechanisms of chronic HPIM 20th ed. Chap. 398 pp. 2876-2877
diabetes complications are true EXCEPT?
a. Increased intracellular glucose leads to the
degradation​ of advanced glycosylation end
products
b. Hyperglycemia increases glucose metabolism via
the sorbitol pathway
c. Hyperglycemia increases the formation of
diacylglycerol leading to activation of protein kinase
C
d. Hyperglycemia increases the flux through the
hexosamine pathway generating fructose-6-PO4

35. Which of the ff statements is true of diabetic HPIM 20th ed. Chap. 398 pp. 2879
neuropathy? (H 3208) a. Chronic, painful diabetic neuropathy is difficult to
a. Chronic, painful diabetic neuropathy is ​easy to treat treat with only symptomatic treatment being
with pregabalin, duloxetine & opioids available
b. Carpal tunnel entrapment is a form of diabetic b. Mononeuropathy
polyradiculopathy
c. Development of neuropathy correlates with the
duration of diabetes & glycemic control c.
d. Orthostatic hypotension i secondary to ​peripheral d. Autonomic neuropathy
diabetic neuropathy

36. Which of the ff. Insulin types is injected usually once a Di ko sure pero pag long-acting insulin kay once or twice a
day for fasting glucose control? day lang (@ bedtime)? Then ang short-acting kay during
a. Glargine meals (2-3x a day)?
b. Aspart
c. Glulisine
d. Human regular insulin

37. The ff clinical features are consistent with metabolic


syndrome EXCEPT?
a. Triglycerides ​≥150 mg/dL
b. HDL >50 mg/dL in men
c. Diabetes controlled on insulin therapy
d. BP ≥ 130/85 mmHg

38. The ff statements are true regarding risk factors for


metabolic syndrome EXCEPT?
a. Central adiposity is a key feature of the syndrome
b. Monogenic mutations underlying the syndrome
have not been identified
c. Prevalence decreases with increasing age
d. Both genetic and acquired causes of lipodystrophy
are associated with increased risk
39. Which of the most accepted and unifying hypothesis
underlying the pathophysiology of metabolic syndrome?
a. Endothelial dysfunction
b. Abnormal intestinal flora
c. Increased angiotensinogen gene expression
d. Insulin resistance

40. Metabolic syndrome is associated with the ff conditions


EXCEPT?
a. Thyrotoxicosis
b. Alzheimer's disease
c. Hyperuricemia
d. Polycystic ovary syndrome

41. Which of the ff is the most important component of


lifestyle modification for metabolic syndrome?
a. smoking cessation
b. Behavioral modification
c. Caloric restriction
d. Physical activity

42. Which of the ff pathophysiologic mechanisms link


hypertension with insulin resistance? HPIM 20th p 2906
a. Insulin decreases endothelin secretion
b. Insulin increases sympathetic nervous system
activity
c. Insulin acts as a direct vasoconstrictor
d. Insulin increases nitric oxide production

43. Which of the ff. Is a cause of primary hyperthyroidism? HPIM 19th p 2293
a. Gestational thyrotoxicosis
b. Subacute thyroiditis
c. TSH-secreting pituitary adenoma
d. Toxic adenoma

44. Which of the ff antibodies is NOT usually expected to


be increased in patients with Graves’ disease?
a. Thyroid stimulating immunoglobulin
b. Anti-thyroid peroxidase
c. Anti-pendrin
d. anti-thyroglobulin

45. A 22 y.o., female college student comes to your clinic


with unexplained weight loss. The ff. Clinical Features will
make you suspect hyperthyroidism EXCEPT?
a. Irritability
b. Menorrhagia
c. Palpitations
d. Impaired concentrations

46. Which of the ff statements describing Graves’


ophthalmopathy is FALSE?
a. The main underlying mechanism is lid retraction
from sympathetic overactivity
b. May occur in clinically eutyroid patients
c. Smoking is recognized as a major risk factor
d. May lead to optic nerve compression in severe
cases

47. The ff conditions may present with reduced or absent


thyroid radionuclide uptake EXCEPT
a. Thyrotoxicosis factitia
b. Toxic multinodular goiter
c. Subacute thyroiditis
d. Struma ovarii

48. What is the most common cause of acute thyroiditis in HPIM 19th p 2298
children and young adults?
a. Thyroid malignancy
b. Amiodarone intake
c. Mycobacterial infection
d. Presence of piriform sinus

1-3 What Is the minimum BP level which corresponds to a diagnosis of


hypertension using different methods?
1. office /clinical BP
140/90 mmHg

2. Average awake blood pressure (24 h monitoring)


>135/85 mmHg

3. Average asleep blood pressure (24 h monitoring)


>120/75 mmHg

4-13 List down the TEN basic laboratory test for the initial evaluation of all
Harrison’s pg 1901 hypertensive patients

Drug Class Compelling Contraindications ​(aside


Indication​ (aside from hypotension)
from BP lowering)

14-15 Aldosterone
antagonists

16-17 beta-blockers

18-19 ACE inhibitors

20-21 Non-dihydropyri
dine calcium
channel
blockers

22-23 Loop diuretics

24-28 Aside from age, give FIVE risk factors that warrant screening for DM
in overweight persons

29-32 Provide the numerical cut offs for the diagnostic criteria for DM
(specify units used)
- Random blood glucose
- Fasting plasma glucose
- Hemoglobin A1c
- 2 h plasma glucose

33-36 For patients already diagnosed with DM, provide the general
treatment goals below (specify units used)
- hemoglobin a1c
- Preprandial capillary blood glucose
- Postprandial capillary blood glucose

Drug Class Example Specific Advantage Specific Side Effect

37-39 Biguanides

40-42 SGLT2 inhibitors


43-45 Sulfonylureas

46-48 Thiazolidinedione
s

49-54 Supply the correct targets on the general goals of obesity treatment
- initial wt loss goal = (8) to (10)% within the time period of the first (___) month/s
- Computation for daily caloric requirement
- Deficit of (500) to (750) kcal/d less from the patient’s habitual diet OR
- Diet of (1500) to (1800) kcal/d for men
- Diet of (1200) to (1500) kcal/d for women
- Rate of wt loss = (1) to (2) lbs/wk
QUESTION RATIONALE
1. True about genetic considerations in hypertension 30-40% heritability (cant remember the other choices)
(Set A)

2. Environmental factor affecting hypertension (set A) (i cant remember the choices or the complete question so
kani na lang:)
obesity and weight gain- independent factors
dietary nacl intake and age-related inc in BP may be
augmented by high nacl intake
low dietary intake of Ca and K - risk of HPN
urine Na-K ratio: stronger correlate of BP than either na
or k alone
alcohol consumption, psychosocial stress, low levels of
physical activity - contribute to HPN
epigentic modifications of DNA contribute to heritability
of HPN
epigenome relatively susceptible to modification by envi
exposures

3.

4.

5. Which of the following DO NOT have HPN? (set A)

6. The following conditions are associated with HPN,


except: ( set A)

a. Central obesity
b. Insulin restriction
c. Glomerulonephritis
d. Patent Ductus Arteriosus

7.

8.

9.

10.

11.

12.

13.

14. Safe anti-HPN drug for elevated crea, history of heart


failure, tremors and palpitations

15. which patient does not have HPN? ​(Same ata ni sa 2018 guidelines for ​HPN
#5)
office BP: >/= 140/90
a. Office BP of ambulatory:
b. Average Awake BP of daytime >/=135/85
c. Ave Sleeping BP of night >/=120/75
24h >/=110/80
home BP: >/=115/85

16.

17. environmental factors of HTN, except

18. genetic something, ana bout HTN. (​Can’t remember,


huhu. Sorry​)

19.

20.

21.

22.

23.

24.

25.

26.

27.

28.

29.

30.

31.

32.
33.

34.

35.

36.

37.

38.

39.

40.

41.

42.

43.

44.

45.

46.

a.

47.

48.

49.

50.

51.

52.

53.

54.

55.

56.
57.

58.

59.

60.

61. (Set A)

a. Hduieie
b. Toxic multinodular goiter
c. Subacute Thyroiditis
d.

62. Which of the ff.drugs is not effective on the


thryrotoxic state of subacute thyroiditis? (set A)

A. PTU
B. Aspirin
C. Methimazole
D. Beta Adrenergic Antagonist

NEW SET

QUESTION RATIONALE
1. Which of the ff lab results is consistent with
non-pregnant hyperglycemic…
a. Glycosylated plasma glucose of 6.2%
b. Fasting plasma glucose of 119 mg/dL
c. 2-hr OGTT result of 119 mg/dL
d. Glucosuria of 3+ on urine dipstick
2. The ff epidemiologic trends regarding DM is
true EXCEPT
a. Asians have increasing diabetes
prevalence at lower BMI…
b. ​Prevalence of Type 1 DM is highest in
Scandinavian countries
c. Prevalence of type 2 DM is highest in
pacific islands
d. Women have higher prevalence of
DM, compared to men

3. The ff. endocrine conditions are associated with W/ increased glucose except:
increased (insulin except, ba yun?)
a. Pheochromocytoma A. Pheochromocytoma (Increase gluc coz of increased
b. Insulinoma catecholamines epineph & norepineph)
c. Acromegaly
d. Hyperthyroidism B. Insulinoma (DECREASE gluc coz of excessive insulin)

C. Acromegaly (Increase gluc coz of increased Growth


Hormone)
D. Hyperthyroidism (Increase gluc coz of increased
thyroid hormones

4. You have a px with gestational diabetes with


adequate glycemic control … gave birth via
spontaneous vaginal delivery and the ff statement is
… true EXCEPT?
a.​ ​She has a large chance of reverting
back to normal glucose tolerance
b.​ ​She has a 35-60% risk of developing
DM in the next 10-20 years
c.​ ​Her child has increased risk of
developing type 1 DM​ during
adolescence
d. She should undergo lifelong s​creening
of development of diabetes or pre
diabetes at least every 3 years

5. JCM is a 25 yr old female with type 1 DM who


received blood transfusion 8 weeks ago… vaginal
bleeding the following tests can be used to determine
…​EXCEPT
a.​ ​Fasting plasma glucose
b.​ ​2-hour OGTT
c.​ ​Capillary blood glucose
d.​ ​Hba1C

6. Which of the ff statements is NOT true


regarding pathophysiology of type 1 DM?
a.​ ​Autoimmune destruction is mainly
caused by T-lymphocytes
b.​ ​ autoimmune destruction in type 1
DM also involve alpha, delta and PP
c.​ ​Testing of GAD-65 and insulin
antibodies helps in identifying type …
d.​ ​Specific environmental triggers have
been conclusively linked …
7. You have an elderly Type 2 diabetic px who
is scheduled for foot examinations. The following
assessments should be done EXCEPT?
a.​ ​10-g monofilament test
b.​ ​Palpation of pedal pulses
c.​ ​Ankle reflexes
d.​ ​Foot extension motor strength

8. Which of the ff dietary recommendations is


NOT true in DM?
a.​ ​Fructose is preferred over sucrose
b.​ ​Routine use of vitamins and
anti-oxidants is recommended
c.​ ​Trans fat consumption should be
minimized
d.​ ​Diet should be rich in monosaturated
fatty acids

9. When prescribing an exercise regimen to


type 1DM patient, which of the ff is advised?
a.​ ​Glucose monitoring should be done
before, during, and after exercise
b.​ ​Delay exercise if glucose > 250
mg/dL and negative for ketone
c.​ ​Ingest carbohydrates prior to exercise
if glucose < 200 mg/dL
d.​ ​Adjust insulin injection site to a highly
exercising area
10.​ ​The following theories on the mechanism of
chronic diabetes chronic complications are true,
EXCEPT
a.​ ​Increased intracellular glucose leads
to the formation of advanced glycosylation
end products
b.​ ​Hyperglycemia increases glucose
metabolism via a sorbitol pathway
c.​ ​Hyperglycemia increases formation of
diacylglycerol, leading to activation of
protein kinase C
d.​ ​Hyperglycemia ​decreases​ the flux
through the hexosamine

11.​ ​Which of the ff statements is​ true​ of diabektic


neuropathy
a.​ ​A chronic painful neuropathy is easy
to treat with …
b.​ ​Carpal tunnel entrapment is form of
diabetic polyradiculopathy …
c.​ ​Development of neuropathy
correlates with the duration of diabetes
and glycemic control
d.​ ​Orthostatic hypotension is secondary
to peripheral diabetes

12.

Aspart because it is rapid acting insulin and can be given


13.​ ​Which of the ff can be given multiple times a multiple times.
day? (Not sure with the question)
a.​ ​Degludec
b.​ ​Glargine
c.​ ​NPH
d.​ ​Aspart

14.​ ​ Di klaro ang question. About sa patient with:


congestive heart failure, elevated serum creatinine,
tremors around midnight. Which of the following oral
antidiabetic agents can be safely given to her?

A. Biguanides

B. DPP-4 inhibitors
C. Thiazolidinediones

D. Sulfonylureas??

15.​ ​Insulin therapy is considered in the ff conditions


as the initial therapy in type 2 DM, EXCEPT?
a.​ ​Those with severe weight loss
b.​ ​In patients with underlying renal or
hepatic disease
c.​ ​In hospitalized or acutely ill patients
d.​ ​Patients with high hypoglycemic risk

16.​ ​Which of the ff patients DO NOT have


hypertension?
a.​ ​Office BP of 125/85 mmHg
b.​ ​Average awake BP of 135/90 mmHg
c.​ ​Average asleep BP of 120/80 mmHg
d.​ ​Home BP of 150/95 mmHg and office
BP of 120/85 mmHg

17.​ ​These ff environmental factors contribute to


hypertension, EXCEPT?
a.​ ​High dietary NaCl intake
b.​ ​Degree of alcohol consumption
c.​ ​ ​High​ dietary calcium and potassium
intake
d.​ ​High​ intensity ​aerobic​ exercise

18.​ ​Which of the following statements regarding


genetic component of hypertension is true?
a.​ ​Majority of cases follow mendelian
form of inheritance
b.​ ​heritability is estimated at 30-40%
c.​ ​Epigenetic dysregulation plays a
minor role in hypertension development
d.​ ​… of end organ damage is
determined solely by BP elevation, with
little genetic …
19.​ ​Pathophysiologic mechanism contribute to
hypertension EXCEPT?
a.​ ​Blunted​ sympathetic outflow
b.​ ​Decreased renal perfusion pressure
leading to renin hypersecretion
c.​ ​Increased arterial stiffness
d.​ ​Inflammation and generation of
reactive oxygen species

20.​ ​The ff are basic laboratory examinations should


be ordered for all newly diagnosed HTN, EXCEPT?
a.​ ​fasting blood glucose
b.​ ​Serum SGPT/ALT
c.​ ​Electrocardiogram
d.​ ​Serum BUN/ Creatinine

21.​ ​You have a newly diagnosed 48 y.o male patient


in your clinic. You should warn him about the ff
complications hypertension EXCEPT?
a.​ ​Atrial fibrillation
b.​ ​Heart failure
c.​ ​Ischemic stroke
d.​ ​Portal hypertension

22.​ ​Following conditions are associated with


hypertension, EXCEPT?
a.​ ​Central obesity
b.​ ​Insulin resistance
c.​ ​Glomerulonephritis
d.​ ​Patent ductus arteriosus
23-25 … is a known hypertensive for 2 years,
admitted several times due to CHF? and has an
elevated serum creatinine with associated muscle
weakness. Her BMI is normal and cardiac PE is
normal. Her BP is uncontrolled with good compliance
with ACE-I and a calcium channel blocker.
Subsequent serum potassium of 2.5 mEq/L; serum
creatinine and urinalysis are unremarkable.

23.​ ​Primary … cause of hypertension is likely in this


patient?
a.​ ​Cushing syndrome
b.​ ​Primary aldosteronism
c.​ ​..
d.​ ​..

24. What lab diagnostics would you order to support


your impression for #23?

PA:PRA RATIO

25.​ ​What oral anti-hypertensive agent is the most


effective for her BP control pending …
a.​ ​Terasozin
b.​ ​Spironolactone
c.​ ​Enalapril
d.​ ​Clonidine

26.​ ​… is a 58 y.o male diagnosed with hypertension


for 5 years. He claimed good compliance… asks you
specific dietary recommendatins to help control his
BP, which of the ff is ​TRUE​ … (table 271-1 20​th​ ed)
a.​ ​Maintain BMI < 3o kg.m2
b.​ ​… dietary salt intake < 10 mg/dL
c.​ ​… <3 drinks alcohol/day
d.​ ​… fruits, vegetables, low fat dietary
products
27.​ ​… patient with hypertension and Chronic kidney
disease … 345 mmol/L and serum potassium of 5.5
mEq/L, which of the ff… for this case?
a.​ ​Amlodipine​???
b.​ ​Enalapril???.?
c.​ ​losartan????
d.​ ​spirinolactone????

28.​ ​… hypertensive female with concomitant


bronchial asthma who had a … anti-hypertesive
agents may be given for her blood pressure …

29-30 … female patient with hypertension for > 10


years, admitted at the … speech with an initial BP
220/140 mmHg. Initial STAT …

(Maybe this is a hypertensive emergency?)

29.​ P
​ referred agents for lowering her BP. EXCEPT?

a.​ ​Nitroprusside
b.​ ​phentolamine
c​.​ ​amlodipine
d.​ ​labetalol

30.​ ​.. … should be lowered with the following goals,


EXCEPT
a.​ ​… arterial pressure lowered not more
than 25%
b.​ ​… pressure lowered to a range of
140/80 mmHg
c.​ ​… BP attained in 12 hours
d.​ ​Rapid BP lowering is needed to avoid
cerebral infarction

31.​ ​Which is the most accepted and unifying The most accepted and unifying hypothesis to describe
hypothesis underlying the pathophysiology for the pathophysiology of the metabolic syndrome is
metabolic syndrome INSULIN RESISTANCE
a.​ ​Endothelial dysfunction
b.​ ​Abnormal intestinaù Page 2904 of harrisons
l flora
c.​ ​Increased angiotensinogen gene
expression
d.​ ​Insulin resistance
32.​ ​.. The following clinical features are consistent
with metabolic syndrome EXCEPT

1. LDL > 190 mg/dL


2. HDL < 40 mg/dL in men
3. Pre-diabetes
4. BP controlled while on an ACE-inhibitor

33.​ ​The following statements are true regarding risk


factors for metabolic syndrome, except
a.​ ​Central adiposity is a key feature of
the syndrome
b.​ ​Monogenic mutations underlying the
syndrome have been identified
c.​ ​Prevalence increases with increasing
age
d.​ ​Both genetic and acquired causes of
lipodystrophy are associated …

34.​ ​Metabolic syndrome is associated with the


following conditions, EXCEPT?
a.​ ​Hypohomocysteinemia
b.​ ​Hyperuricemia
c.​ ​Non-alcoholic steatohepatitis
d.​ ​Polycystic ovary syndrome
35.​ ​Which of the following is the most important Harri. 20th ed 2907
component of lifestyle …
a.​ ​Smoking cessation
b.​ ​Behavioral modification
c.​ ​Caloric restriction
d.​ ​Physical activity

36.​ ​ p. 2908 LDL risk something

37.​ .​ . EXCEPT

a. Triglycerides 200-600 mg/dL

b. HDL < 50 mg/dL in female patient

c. Coronary heart disease in elderly 1​st​ degree


relatives

d. d. Total cholesterol 200-400 mg/dL

38.​ ​you have a patient who was recently diagnosed


with familial hypertriglyceridemia presenting with
acute pancreatitis. Which of the following lipid profile
findings do… patient?

a. ​Low to normal LDL levels

b. High HDL levels

c. Low apoB levels

d. Low VLDL levels


39.​ ​among patients with hyperchylomicronemia
syndrome, what is the …

a. LPL

b. ​ApoC-II

c. Apo-E

d. PC5K9

40.​ ​H. T is a 58-year old female who underwent total


thyroidectomy 5 years ago, … levothyroxine
replacement. The ff lipid profile derangement are
expected…

a. High LDL levels

b. Low HDL levels

c. High IDL levels

d. ​High Triglyceride levels

41.​ ​one of your patients asks you regarding the Increase HDL-c p2892 harrisons (2° causes of of vldl
potential benefits of regular alcohol intake …what overproduction)
specific benefits in the lipid profile does regular
alcohol intake promotes …

a. Lowers LDL levels

b. Increases HDL levels

c. Lowers VLDL levels

d. Lowers lipoproteins A levels


42.​ ​which of the ff. genetic defects DECREASES the
risk of coronary heart disease…

a. Familial hypobetaliproteinemia

b. Abetalipoproteinemia

c. Sitosterotemia

d. ​PCSK9 deficiency

43-44 K. P is a 55 yr old obese woman with no


known comorbidities who come to your clinic … her
results are surprisingly normal except for Triglyceride
levels of 600mg/dL … proper diet and exercise

43.​ ​What complication will compel you to start


pharmacologic therapy?

a. Sudden cardiac death

b. Pancreatitis

c. Hemorrhagic stroke

d. Acute pericarditis

44.​ ​What is the drug class to target her form of


dyslipidemia?
a.​ ​Fibrates
b.​ ​Cholesterol absorption inhibitors
c.​ ​Bile acid sequestrants
d.​ ​PCSK9 inhibitors

…..year old male with a family history of


coronary heart disease

45. the following patients should be started on statins


as 1​st​ line therapy …?
1. …..year old male with a family history of
coronary heart disease
2. 55-y.o female( premenopausal​Menopause?)
3. 40-y.o male with personal history of ischemic
stroke
4. 37-y.o female with 10 year ASCVD risk of 6%

46.​ ​You have an obese but otherwise healthy Esophageal cancer is associated with males not in
(?fe)male patient in the clinic … common females
consequences of his obesity that you should educate
her … Harrison page 2842-2843, ​PATHOLOGIC CONSEQUENCES
a.​ ​Impotency OF OBESITY
b.​ ​DM
c.​ ​CHF
d.​ ​Esophageal cancer

47.​ ​Which of the following are conditions highly Harri 20th ed p. 2842 TABLE 394-2
associated with obesity, EXCEPT?
a.​ ​Prader-willi syndrome page 2841
b.​ ​Bardet-biedl syndrome
c.​ ​Conn’s syndrome
d.​ ​Hypothyroidism

OTHER SPECIFIC SYNDROME ASSOCIATED WITH OBESITY

48.​ ​Which of the following statements best describes Harrison page 2842
the status of leptin in genetic …
a.​ ​They have functional leptin resistance
b.​ ​They have leptin deficiency
c.​ ​They have mutated leptin receptors
d.​ ​They have increased levels of
enzymes that metabolize leptin
49.​ ​… year old female seeking medical assistance
for her progressive … among the medications, which
of the following is NOT likely to contribute …
a.​ ​..
b.​ ​Glibenclamide
c.​ ​Amlodipine
d.​ ​Prednisone

50.​ ​G. G is your 25 year old cousin with BMI of


29kg/m2 and no other co-morbidities who wants to
lose weight. The following recommendations can be
given, EXCEPT?

a. Reduce caloric intake by 500-750 kcal/day


compared to habitual diet

b. Rate of weight loss at 1-2 lbs/week

c. Total caloric intake is more important than


macronutrient composition

d. Adding water or fiber to food to increase


energy density

51.​ ​Which of the ff interventions for weight loss have


been found to cause complete … DM in many
cases?
a.​ ​Liraglutide
b.​ ​Stress management
c.​ ​Use of meal replacements
d.​ ​Roux en y gastric by pass

52-54 K. L is a 54 year old businessman consulting


at your clinic for persistent … snoring at night. He is
also known diabetic and was diagnosed with …
anthropometric measurements are as follows: Height
= 150 cm, weight

52.​ W
​ hat is his present BMI (using global cut-offs)? Harri 20th ed P. 2845
a.​ ​Overweight
b.​ ​Obese class I
c.​ ​Obese class II
d.​ ​Obese class III (extreme)

​37.something
53.​ ​What is the appropriate management for his
case?
a.​ ​Diet and exercise
b.​ ​Diet and exercise + behavioral
therapy
c.​ ​Diet and exercise + behavioral
therapy + pharmacotherapy
d.​ ​Diet and exercise + behavioral
therapy + pharmacotherapy + surgery

(With surgery kasi 37 pataas tapos with


comorbidities)

54.​ ​Based on the ideal management for this case,


he might require lifelong supplement of
micronutrients, EXCEPT?
a.​ ​Vitamin B12
b.​ ​Iron
c.​ ​Calcium
d.​ ​Magnesium

Harrison page 2845


55.​ ​You have decided to place your morbidly obese
patient under a very-low caloric diet for the next 6
months, which of the following is a complication you
should warn the patient about?
a.​ ​Osteoarthritis
b.​ ​Cholelithiasis
c.​ ​Diarrhea
d.​ ​Suicidal ideation
56.​ ​… patient with a goiter brings the following Harri 20th ed. P. 2705, Figure 377-2
thyroid function test results to your clinic. Free T4 =
elevated, TSH = elevated also … what is the most
likely diagnosis?
a.​ ​Grave’s disease
b.​ ​Toxic adenoma
c.​ ​Silent thyroiditis
d.​ ​TSH secreting pituitary adenoma

57.​ ​The patient #56, what is the next diagnostic test


to confirm your diagnosis
a.​ ​Thyroid stimulating immunoglobulin
assay
b.​ ​Thyroid radionuclide uptake scan
c.​ ​Thyroid ultrasound with Doppler
studies
d.​ ​Pituitary CT/MRI

58.​ ​Which of the following is a cause of secondary


hyperthyroidism?

a. Thyrotoxicosis factitia

b. Grave’s disease

c. Chronic gonadotropin secreting tumor

d. struma ovarii
59.​ ​Which condition might precipitate Grave’s
Disease, EXCEPT?
a.​ ​ Immediate postpartum period
b.​ ​Increase iodine intake
c.​ ​Immune reconstitution after HAART
d.​ ​..

60.​ ​ …..not usually present in patients with graves Page 2703 20th ed harrisons
disease

A. Thyroid-stimulating immunogllobulin
B. Anti-thyroid peroxidase
C. Anti-thyroglobulin
D. Anti-pendrin

61.​ ​ Student comes to ur clinic with unexplained weight


loss. The following….. hyperthyroidism, EXCEPT?

A. Amenorrhea??

B. Diarrhea

C. Oligomenorrhea

D. Impaired concentration

62. Which of the ​following statements describing graves’ Harri 20th p. 2709
ophthalmopathy is ​false​?

A. …..main underlying mechanism is lid retraction from


sympathetic overactivity….

B. …..occur in clinically euthyroid patients

C. ….smoking is recognized as ​major​ risk factor

D. …..may lead to optic nerve compression in severe


cases
63.​ ​…..y.o school teacher diagnosed with multinodular
toxic goiter 1 year prior… (8?) weeks pregnant confirmed
by pelvic ultrasound, and asks you what is the
safe…..time?

A. Radioactive iodine treatment

B. Propylthiouracyl

C. Methimazole

D. (Super?)saturated potassium iodide

HPoIM p. 2707
64.​ ​…..following conditions may present with reduced or
absent thyroid radionuclide uptake….

A. Thyrotoxicosis factitia

B. Toxic multinodular goiter

C. Subacute thyroiditis

D. Struma ovarii

Harrison 20th p. 2709

65.​ ​ The following options have no role during the


thyrotoxic phase of subacute thyroiditis

A. Methimazole

B. aspirin

C. Prednisone

D. B-adrenergic blockers
Part II

1. Pathophysiology of thyroid associated ophthalmopathy

● Signs and Symptoms of Hyperthyroidism

● Interpret the following lab results: FT4 = 58pmol/L and TSH = <0.005 uIU/L

Normal Values from Navarro:

Free T4 = 12-30 pm ol/L TSH = 0.5-5.0 uIU/L

RESULT DIFFERENTIALS

TSH FT4

Low High ● Primary thyrotoxicosis: Graves’ disease, multinodular goiter, toxic


adenoma
● Destructive thyroiditis, excess iodine intake, excess thyroid
hormone

Low Normal ● Subclinical hyperthyroidism (if normal FT3)


● T3 toxicosis (if high FT3)

Normal/High High ● Secondary thyrotoxicosis: TSH secreting pituitary adenoma or


thyroid hormone resistance syndrome
Rationale of the following drugs

Drug Drug Class Mechanism/ Rationale of use in hyperthyroidism

Methimazole Antithyroid Agents Inhibits the Iodination of Thyroglobulin thus preventing the formation of
MIT and DIT, and/or
Inhibits the coupling of MIT and DIT → not sure bitaw

Propranolol Non-selective reduces the symptoms of b-adrenergic stimulation secondary to


β-adrenergic receptor increased thyroid hormone by competitively blocking the sympathetic
antagonist stimulation to b1 and b2 adrenergic receptors

● What are the indications for the diff. treatment modalities for hyperthyroidism

Antithyroid Medications used to reduce thyroid hormone formation/production in cases of thyrotoxicosis

Radioactive Iodine used as initial treatment or for relapses after a trial of antihyperthyroid drugs

Thyroidectomy used in thyroid neoplasms and/or cases of thyrotoxicosis refractory to more conservative
treatments such as radiotherapy and pharmacologic therapy

● Pathologic consequences of obesity:

Insulin/Glucose Metabolism decreased insulin sensitivity

Reproductive male hypogonadism, gynecomastia, PCOS

Cardiovascular increased risk of cvd secondary to vascular changes brought upon by hyperglycemia
due to insulin resistance and increased risk of atherosclerosis due to impaired lipid
metabolism secondary to insulin resistance

Pulmonary decreased lung capacity due to increased abdominal pressure, obesity hypoventilation
syndrome, obstructive sleep apnea

Hepatobiliary increased risk of nafld, nash, and gallstone formation due to impaired lipid metabolism
secondary to insulin resistance

Bone and Joints increased risk for osteoarthritis due to increased stress on weight bearing joints
secondary to increased weight
● A 44-Year-Old Female came in for consult due to unintentional weight loss associated with frequent urination,
increased thirst and appetite. RBS at health center was 273 mg/dL .

Vital Signs: BP = 160/100 ; RR = 18 ; PR = 76 ; Weight = 78 kgs ; Height= 5’4”

PE findings: yellowish plaques on eyelids,normal chest and lungs, flabby abdomen with silvery striae

Lab Exams revealed: FBS = 228 mg/dL ; HbA1c= 8.4% ; Creatinine = 81 umol/L ; K = 3.9 meqs/L; Urinalysis = +2
glucose and protein; Total Cholesterol = 258 mg/dL ; LDL= 250 mg/dL ; Triglycerides = 400 mg/dL; HDL = 31 mg/dL

● What is your complete Diagnosis?


● Formula for BMI= kg/m2
● What medication/s will you giver her lipid levels?
● Write the criteria for metabolic syndrome in the space below:

● Pathophysiologic mechanism of microvascular and macrovascular complications of diabetes

● Pathophysiologic mechanism of HPN in patients with excessive weight gain, obesity and sedentary lifestyle

● Pathophysiologic mechanism of HPN due to increased salt intake:


Additional questions na I can remember (Sorry I forgot the question and the choices):

Insulin resistance Most accepted and unifying hypothesis to describe the (Source: Harrison’s Principles of
pathophysiology of hypertension??? Something like that Internal Medicine, 20th Ed, Chapter
401, p. 2904)

PCSK9 What form of dyslipidemia daw that provides protection Heterozygosity for nonsense
from CHD?/ What gene is involved?? mutations in PCSK9 that interfere
with the synthesis of the protein are
associated with increased hepatic LDL
receptor activity and reduced plasma
levels of LDL-C. Such mutations are
most frequent in individuals of
African descent. ​Individuals who are
heterozygous for a loss- of- function
mutation in PCSK9 have a 30- 40%
reduction in the plasma levels of
LDL-C and have a substantial
protection from CHD relative to
those without a PCSk9 mutation,
presumably due to having lower
plasma cholesterol levels at birth.
(Source: Harrison’s Principles of
Internal Medicine, 20th Ed, Chapter
400, p. 2988)

Gallstones/ Patients having Very- Low- Calorie Diets (VLCDs) are more The risk for gallstones increases
Cholelithiasis likely to to have??? exponentially at rates of weight loss >
1.5 kg/ week (3.3 lb/ week).
prophylaxis against gallstone
formation with ursodeoxycholic acid
(600 mg/ day) is effective in reducing
this risk.
(Source: Harrison’s Principles of
Internal Medicine, 20th Ed, Chapter
395, p. 2847)

Relative INSULIN Underlying causes of Hyperglycemic Hyperosmolar State Insulin Deficiency​→ increased
DEFICIENCY and Hepatic glucose production +
INADEQUATE impaired glucose utilization (muscles)
FLUID INTAKE Hyperglycemia​→ Osmotic Diuresis
→ Intravascular Volume Depletion
(exacerbated by Inadequate Fluid
replacement)
(Source: Harrison’s Principles of
Internal Medicine, 20th Ed, Chapter
397, p. 2872)

AST/ ALT Following tests are to be ordered for HTN? EXCEPT See Table 271- 6 Basic Laboratory
SGPT/ SGOT Tests for Initial Evaluation
(Hypertension)
(Source: Harrison’s Principles of
Internal Medicine, 20th Ed, Chapter
271, p. 1901)

ADDITIONAL QUESTIONS: ABOUT DM PREVALENCE

TYPE 1 ; HIGHEST IN SCANDINAVIAN

LOWEST: PACIFIC RIM

INTERMEDIATE: NORTHERN EUROPE AND US

TYPE 2: HIGHEST: PACIFIC ISLANDS AND MIDDLE EAST

INTERMEDIATE: INDIA AND US

SCREENING/ DIAGNOSIS SA PATIENT (IDK UNSA IYA SAKIT BUT ABOUT SECONDARY CAUSE OF HPN TO)

PRIMARY ALDOSTERONISM- 30:1 ratio of PA:PRA and plasma aldosterone >555pmol/L(screening)

or failure to suppress plasma aldosterone to <277 after IV 2l saline for 4h (dx)


CUSHING SYNDROME: SCREENING - 24 h excretion rates of urine-free cortisol or overnight dexamethasone-suppression
test

PHEOCHROMOCYTOMA - screening: 24h urine metanephrine secretion or fractionated plasma-free metanephrines


HYPERTENSION

1.Determinant of BP, aside from PVR


A. ​Cardiac output
B. Heart rate
C. Stroke volume

2.Obesity is a risk factor of the following Ref. Harrison 19th ed. page 2392
disease, EXCEPT?
A. ​Type1 DM Obesity is associated with an increased risk of
B. N? Nephrotic Syndrome? multiple health problems, including
C. Obstructive Sleep Apnea hypertension, TYPE 2 DIABETES,
D. Malignancies dyslipidemia, OBSTRUCTIVE SLEEP APNEA,
non-alcoholic fatty liver disease, degenerative
joint disease, and SOME MALIGNANCIES.

3. Which is an example of hypertension related


to increase catecholamine production?
A. Polycystic Kidney Disease
B. Pheochromocytoma
C. Cushing’s Syndrome
D. Unilateral Renal Agenesis

4. The following are ​basic​ laboratory tests are


recommended for the initial evaluation of
hypertension, EXCEPT
A. Urinalysis
B. Total cholesterol and LDL
C. serum potassium
D. Serum aldosterone/renin ratio

5. This is one of the strong independent


predictor of CVD and all-cause mortality.
A. Aortic augmentation index
6. Which of the following is TRUE about Angiotensin II - active ​octapeptide
angiotensin 2?
a. A potent mineralocorticoid a. ALDOSTERONE is a potent
b. Excess angiotensin 2 protects against mineralocorticoid
LVH and atherosclerosis b. Independent of its hemodynamic
c. Synthesized ​exclusively​ in kidneys and effects, ​angiotensin II may play a role
adrenal in the PATHOGENESIS of
d. - D ang answer. huehue atherosclerosis​​…
c. Angiotensinogen, renin, and
ANGIOTENSIN II are also synthesized
locally in many tissues,​​ including the
brain, pituitary, aorta, arteries, heart,
adrenal glands, kidneys, adipocytes,
leukocytes, ovaries, testes, uterus,
spleen, and skin.

Ref. Harrison 19th ed. page 1614

7. Which of the following is FALSE involved in Administer antihpn/antithrombo only if you


the treatment of malignant hypertension have the following BP target.
a. Degree of target organ damage rather
than the level of BP alone determine the Cerebral infarction
ability with which BP should be Antihtn - >220/130
lowered….. - true Antithrombolytic - <185/110
b. For patients with cerebral infarction and
non candidates for thrombolytic Hemorrhagic stroke
therapy…… - false Antihtn - >180/130

Harrison’s 19th ed page 1627

MATCHING TYPE: (8-12)

8. Stroke - ​nicardipine
9. Myocardial infarction - ​nitroglycerine
10. Acute Left ventricular failure - ​loop diuretics
11. Adrenergic crisis - ​phentolamine
12. Eclampsia/Pre-eclampsia - ​hydralazine

Case scenario: (13-15)


CJ, 45 yo, male, known hypertensive for 10 yrs
with his BP maintained 130-150?????. Current
meds: amlodipine and losartan.. Rushed to
ER… generalized headache and projectile
vomiting, BP 240/120 mmhg

13. What is the preferred antihypertensive drug


for CJs condition?
A. Nitroglycerine
B. nitroprusside​ - or nicardipine
C. captopril
D. Hydralazine

14. What physical examination is used to Urgency​​ - bp of >220 but asymptomatic ang
assess end organ involvement in relation to patient
Cjs condition? (increased ICP) Emergency​​ - bp >220 but with presenting
A. Deep tendon reflexes symptoms of organ damage (headaches,
B. Chest auscultation confusion, blurred vision,
C. Peripheral pulses nausea and vomiting, seizures, pulmonary
D. Fundoscopy edema, oliguria, and grade 3 or grade 4
hypertensive retinopathy)

15​. Early symptoms of increased intracranial Scotoma - ​a partial loss of vision or a blind spot in
pressure, EXCEPT: an otherwise normal visual field.
A. Headache
B. Vomiting Plethora - ​an excess of a bodily fluid, particularly
C. Scotomas blood.
D. Plethora

16. 60 yr old ​hypertensive​ patient with ​left LVH, LV dysfunction - ACEI/ARB


ventricular systolic dys​function, ejection
fraction of 40% via 2d echo. What to
prescribe?
A. amlodipine
B. Metoprolol
C.​ Valsartan​ - ARBs
D. Captopril
17. TRUE about isolated systolic hypertension
B. systolic blood pressure >140 and diastolic
blood presssure <80
C. have effect ...
D. more common in​ <​60 years old - False

18. The following are true EXCEPT


a. Increasing age does not correlate with
wide pulse pressure….
b. Cardiovascular disease risk doubles for
every 20-mmHg increase in systolic and
10-mmHg increase in diastolic pressure.
c.
d. There is an increased risk for
cardiovascular disease in increased
systolic and diastolic blood pressure

19. The following are advantages of


ambulatory BP home monitoring, EXCEPT
A. Home bp is usually ​highe​r than clinic,
therefore, this is the best method​ -
LOWER - white coat htn
B. It provides nocturnal nondippers, who
have ​attenuated​ nighttime BP……
C. It provides comprehensive assessment
for CVD burden of hypertension
D. It prevents target organ damage

20. Possible mechanism/s of kidney related Decreased capacity to excrete-> retain


hypertension: sodium -> inc ECF -> inc volume -> inc bp
A. Loss of autoregulation of blood flow
causes ​vasodilatation​ and hyperperfusion of
kidneys - false
B. ​Decreased​ renin secretion in relation to
fluid volume status - INCREASED
C. ​Decreased capacity of the kidneys to
excrete sodium
D. ​Parasympathetic​ nervous system
activation - SYMPATHETIC

21. Correct BP TECHNIQUE measurement,


EXCEPT:
A. SITTING….WITH THE FEET ​DANGLED -
flat on the ground
B. CUFF SHOULD COVER AT LEAST 80%
ARM CIRCUMFERENCE
C. RATE DEFLATION 5 MMHG ​(SHOULD
BE 2 MMHG)
D. AT LEAST 2 BP READINGS
22. Initial work up of a hypertensive patient
should include the following, EXCEPT
A.2D echocardiography with doppler studies
B. Lipid profile and FBG
C. Urinalysis with measure of albuminuria
D. serum electrolytes of Na, K

23. Fundoscopic exam for severe


hypertension EXCEPT
A. Arteriovenous crossing defects
B. Absent​ Arteriolar Light Reflex​ ​- Should
be increased
C. Hemorrhages and exudates
D. Papilledema

24. Lifestyle modifications for hypertension


should include the following, EXCEPT
A. Limit alcohol intake 3 drinks for men​ ​(2
drinks for men and 1 drink for women)
B. Nacl intake less 6 grams per day
D. DASH diet
25. Better antihypertensive drug of choice to
give to patient ​with tachycardia
a. Amlopidine
b. Hydralazine
c. ​Metoprolol​ - Beta blocker -> lowers heart
rate
d. Losartan

26. Common side effect of long-term use of


spironolactone:
A. Gynecomastia and impotence​ -
another common SE is on the
potassium, Beta blocker can also
cause impotence
B. Peripheral edema
C. Tachycardia
D. Cough

27. TRUE of alpha adrenergic blocking agent


(clonidine example) (​dili man alpha blocker
ang clonidine wooiii, a1 agonist man sya​)
A. Clinical used has been confined to
symptomatic treatment of prostatic
hypertrophy and pheochromocytoma
B. Ideal for post myocardial infarction px
and px with coronary artery disease -
BB, ACEI for post MI
C. Protective against CHF - FALSE -
clonidine has no effect in heart end
points (death, stroke, MI, heart failure)
BB, ACEI, ARB, mineralocorticoid
(spironolactone) ​have cardiac
protection/mortality/morbidity
D. lowering bp by decreasing cardiac
output
28. The following are direct vasodilator,
EXCEPT
A. Hydralazine
B. Clonidine​ - alpha adrenergic blocking
agent
C. Minoxidil - SE is hair growth
D. Nitroprusside

29. ​Good​ antihypertensive agent for diabetes causes increase in blood sugar, but it
patient, EXCEPT does not mean that you can no longer
A. ACEI - captopril give BB in patients with DM, esp with
B. ARB - losartan those who are post MI and with
C. Spironolactone tachycardia
D. BB - Metoprolol​ -

30. Which of the following is FALSE about 140/90 mmHg


initiating antihypertensive medication
A. Pharmacological treatment for
asymptomatic non-diabetic patients with BP
130/85 mmHg​ is ​necessary​ -
D. Calcium channel blocker is the best choice
for protecting someone from stroke-true

Harrisons p. 1625

31. TRUE about resistant hypertension.


EXCEPT.
A. For the elderly, suggest that if the radial
pulse remains palpable despite
occlusion of the brachial artery - osler
maneuver
B. Remains >140/90 despite taking three
or more antihypertensive agents,
including diuretic
C. Pseudoresistance can be rule out
before labeling the disease.
D. Common to those patient ​younger​ than
60 years old​ - older
32. Treatment in hypertension in patients with
acute left ventricular failure​ should include the
following, EXCEPT:
a. Nitroglycerin - NTG is for myocardial
infarction/Angina pectoris
b. enalapril
c. furosemide
d. ​Atenolol​ - contraindication for giving beta
blocker - a pulmonary congestion more than
half of your lung fields and bradycardia.

(In acute LVH, there is pulmonary congestion


→ you want to lower the pressure WITHIN the
heart → How? decrease venous return and
increase the stroke volume

33. TRUE about Renovascular hypertension


EXCEPT:
a. ​Fibromuscular dysplasia is ​UNILATERAL
and is most commonly affect ​PROXIMAL
portion of renal artery​ - BILATERAL​, DISTAL
b. The arteriosclerosis is the most common ….
c. Fibromuscular dysplasia is common among
white women
d. It involves the activation of RAS

TRUE OR FALSE (34-39)

34. A diet high in salt intake is a predictor of


increased blood pressure. ​TRUE

35. End stage of renal disease is a good


example of ​vasoconstrictor mediated
hypertension​ that is correctable by dialysis.
FALSE
36. An aggressive BP control can reduce or Aggressive control of hypertension can regress
reverse LVH - ​TRUE or reverse left ventricular hypertrophy​ and
reduce the risk of cardiovascular disease. It is
not clear whether different classes of
antihypertensive agents have an added impact
on reducing left ventricular mass, independent
of their blood pressure–lowering effect. -
Harrison’s

37. Increase in BP is the strongest risk factor


for stroke. ​TRUE

38. Patients with high plasma renin have the


vasoconstrictor form of hypertension with BP
elevation brought about by the vasoconstrictor.
- ​TRUE

39. Patients 60 yo are more responsive to beta


blocker than ACEI. - ​FALSE

DYSLIPIDEMIA

40. A patient follows up in your clinic with


normal lipid profile results after taking lipid
lowering agents for 3 months and asks why he
should be taking the medication.
B. It should be continued to reduce the risk of
heart attack and stroke

41. A patient with controlled diabetes and


hypertension consults regarding LDL of 200 in
your clinic. The ​primary target​ is
A. ….Lowering LDL cholesterol
Total cholesterol
Triglyceride
HDL
42. You prescribed a diabetic patient with
history of stroke 5 yrs ago with lipid lowering
drug. Which guideline would you consider?
I. Use of maximum penultimate dose of
potent statin
II. Use of Atorvastatin
III. Use of Rosuvastatin

A. I
B. I,II
C. II,III
D. I,II,III

43. A patient was noted to have low HDL, high


triglyceride and LDL, abdominal obesity and
hypertension. This patient has METABOLIC
SYNDROME but no diabetes. What should be
your basis in giving your statins?
B. 10 years of CV risk, >7.5

44. Statin is the primary treatment for


dyslipidemia for lowering LDL cholesterol by :
A. 5-10%
B. 10-30%
C. 10-40%
D. ​15-60%

45. 2nd line of agent that is well tolerated that


reduces LDL by 15-20%
fibrates
A. omega 3
B. gemfi
C. ​ezetimibe

46. The degree of triglyceride reduction by


fenofibrates:
A. 10-25
B. 20-30
C. ​30-45
D. 40-60
47. In general, bile acid sequestrants should
not be administered to patients with serum
triglycerides level of
a. >150 mg/dL
b. >200 mg/dL
c. >250 mg/dL
d. >300 mg/dL

48. NCEP: ATP III recommend a fasting


triglyceride level of
a) <150mg/dL
b) <200mg/dL
c) <300mg/dL
d) <350mg/dL

DIABETES MELLITUS

49. For diabetes mellitus, ADA recommends


screening:
a. >45 years old every 2 years, and for
younger who are overweight
b. >45 years old every 3 years, “ “ “ “ “
c. All individuals more than 45 years old,
every 3 years, and earlier if there is
additional risk factors
d. >45 years old, every 2 years, “ “ “ “ “

50. Considered to be part of the classic


symptoms of DM:
a. Polyphagia - a symptom but not classic,
not seen in many patients
b. Weight loss
c. Frequency
d. Pruritus
51. Risk factor for type 2 DM include:
a. Race/Ethnicity
b. Previously diagnosed with Hb1ac 6.5 or
more - 5.7 - 6.4 dapat… 6.5 kay DM na.
Tapos ang question kay risk factor man
kaya wrong to.
c. History of GDM or delivery baby >10 lbs
- 9 lbs ang cut off
d. HDL cholesterol level <30 mg/dl - cut off
kay 35

Note: Criteria, FBS, Random, 2 hours,


memorize din kung <, >, greater than or equal
etc….

52. Criteria for DM (memorize!! Esp the < >)


A. Symptoms of diabetes plus random
blood glucose concentration of
≥180mg/dL
B. Fasting plasma glucose ​≥100mg/dL
C. HbA1C glucose of ​≥7.0%
D. 2h plasma glucose ​≥​200mg/dL during an
oral glucose tolerance test

53. Impaired fasting glucose is defined as? Abnormal glucose homeostasis (Fig. 417-1)
A) <99mg/dL is defined as
B) 101-125 mg/dL (1) FPG = 5.6–6.9 mmol/L (100–125 mg/dL),
C) 100-125 mg/dL which is defined as i​mpaired fasting
D) 126 mg/dL glucose (IFG);
(2) plasma glucose levels between 7.8 and
11 mmol/L (140 and 199 mg/dL) following an
oral glucose challenge, which is termed
impaired glucose tolerance (IGT); ​or
(3) HbA1c of 5.7–6.4%.
54. Most reliable and convenient test for
Diabetes mellitus in asymptomatic individuals
A. 2 hr glucose conc
B. Identification of symptoms
C. FPG
D. Random blood glucose conc

55. True of the pathophysiology of DM:


A. In initial onset of the disorder, glucose
tolerance is still at near normalLet
B. as insulin resistant and compensatory
hyperinsulinemia occurs the pancreatic
islets ​ALL​ individual… - not ALL, Some
lang
C. … insulin secretion and an increase
hepatic glucose production … diabetes
WITHOUT ​fasting hyperglycemia -
WITH dapat
D. Beta cell failure to cease…. - its the
heart? ​[Ultimately, beta cell ensues]

56. TRUE of metabolic abnormality in Type 2 Ref. Harrison’s 19th ed. page 2404.
DM
A. Insulin resistance is a prominent feature of Insulin resistance impairs glucose utilization
Type 2 by insulin-sensitive tissues and INCREASES
B. Insulin resistance impairs glucose utilization hepatic glucose output; both effects
by insulin sensitivity and ​DECREASE​ hepatic contribute to the hyperglycemia. Increased
glucose output - (INCREASE dapat) hepatic glucose output predominantly
C.Increase hepatic glucose output increases accounts for increased FPG levels, whereas
POSTPRANDIAL​ glucose - (FASTING BLOOD
GLUCOSE dapat) decreased peripheral glucose usage results
D. Decrease peripheral glucose increases in postprandial hyperglycemia.
FPG​ - (POSTPRANDIAL dapat)

~> something like this pls correct if sayop,


sorry kaayo ng shutdown jd ako brain after
exam. Took me a couple of mins to memorize
this huhu

57. Considered central to the development of


Diabetes Mellitus 2 is:
A. Insulin Resistance
B. Abnormal Insulin Secretion
C. Both A and B
D. None of the above

58. The following are diabetic macrovascular


complications except:
a. Diabetic nephropathy
b. Coronary artery disease
c. Peripheral vascular disease
d. cerebrovascular
59. The ff is/are goal/s of DM therapy:
A. Eliminate signs of hypoglycemia
B. Reduce or eliminate risk of long term
microvascular and macrovascular
complications of DM
C. Allow patient to have a normal lifestyle
as possible
D. AOTA

60. Which of the following is true about


Medical Nutrition Therapy?
a. Primary prevention measures of MNT
are directed at ​managing
diabetes-related complications​. -
TERTIARY
b. Secondary prevention measures of
MNT are directed at ​preventing or
delaying the onset of Type-2 DM​ in high
risk individuals - PRIMARY
c. Tertiary prevention measures of MNT
are directed at ​prevention or delaying
the onset of diabetes-related
complications​ - SECONDARY
d. MNT is an important component of
comprehensive management of
Diabetes

61. The following is true in monitoring the level


of glycemic control
A. Self monitoring blood glucose (SMBG)
is the standard method in glycemic
control - pinprick
B. …..regular and adopted in …. - YOU
MAY CHECK ANYTIME
C. HBA1C measurement is the standard
method for assessing SHORT term …..
- LONG TERM
D. Frutamine? Assay for short term….. 2-4
weeks - 2 weeks not 2-4 weeks
62. The following is true about the general
guidelines of establishing the target level of
glycemic control
a. It improve glycemic control
b. Reduces complications in high risk
individuals or elderly or with CVD
c. In early type 2 diabetes, it likely leads to
improve cardiovascular outcome
d. All of the above

63. The care for Type 2 DM patients must also


include
A. Detection and management of
complications
B. Treat DM-related conditions: obesity,
hypertension, dyslipidemia, CVD
C. The goal for Type 2 DM is ​NOT​ similar
to Type 1 DM.
D. A and B are correct.

64. The following is/are True of Type 1 and


Type 2 DM treatment:
a. Medical Treatment therapy and Physical
Activity decrease insulin resistance and
increase insulin secretion.
b. Amylin agonists, GLP-receptor agonist,
insulin are injectables ​and tablets​ that
decrease hyperglycemia. - these are
ALL INJECTABLES
c. Biguanides…….. do not cause
hypoglycemia … ​increase​ hepatic
glucose production - DECREASE
d. Both A and C

65. Insulin should be considered in the


following conditions as the initial treatment.
A. Lean body or with severe weight loss - SE
of insulin: weight gain
B. with underlying Renal and hepatic disease
C. Hospitalized patient - control blood sugar
ASAP esp with cardiac problem
D. All of the above
66.Which is/are true in the management of DM
A. Insulin cannot be initiated with dm patients
with severe hyperglycemia
B. Metformin is a good choice for initial therapy
because of it's known side effect, cost and
efficacy​ - the initial therapy if patient has no
contraindications
C. Insulin secretagogues are best when insulin
therapy is already initiated
D. B and C is both correct

67. DKA Note: HHS, Familiarize blood sugar level


a. DKA is a result of relative or absolute…. HHS - usually elderly, T2DM
- it can occur anytime
b. Ketosis result from a marked increase
free fatty acid release from
adipocytes….
c. Nausea, vomiting, hypotension,
kussmauls are manifestations of DKA -
kussmaul-rapid shallow breathing
d. all of the above
68. Management of DKA includes:
A. Replace fluid with ​0.9 Lactated Ringers​ at
10-20ml/kg 1-3hr - ​PNSS dapat
B. Give short-acting insulin by ​subcutaneous
at 0.1 iu/kg - ​intravenous dapat
C. ​Assess serum electrolytes, acid-base
status, renal function
D. Replace potassium if >5.2 meq/L -​should
be “if <5.0-5.2 meq/L”, NEVER replace!!!

69. Which of the following is true of


Hyperglycemic Hyperosmolar State (HHS)?
A. It is often precipitated by serious,
concurrent illnesses like myocardial
infarction, stroke, sepsis.
B. Vomiting, nausea, hypotension,
abdominal pain, Kaussmaul respiration
are present. ​[Clinical signs of DKA]
C. Absolute​ insulin deficiency and
inadequate fluid intake are usually the
underlying cause.
D. Marked Hyperglycemia (plasma glucose
200-300 mg/dL)​ and ​hyperosmolality
(200-300 mosmol/L) ​and prerenal
azotemia are notably seen.
70. HHS management
a. Fluid replacement of. 1 to 3 L of 0.9
normal saline in first 2-3 hours
b. Potassium repletion is usually
necessary
c. Bicarbonate replacement is usually
necessary
d. BOTH A and C
A and B answer sa review

71. The following is/are the theory/ies on


mechanisms of diabetic chronic complications:
a. Increased intracellular glucose leads to
the formation of advanced glycosylation
end products.
b. Hyperglycemia increases glucose
metabolism via the sorbitol pathway
c. Hyperglycemia increases the formation
of diacylglycerol, leading to activation of
protein kinase C
d. All of the above

72. The following are TRUE of diabetic Nonproliferative diabetic retinopathy


-appears late in the first decade or early in the second decade
retinopathy (DR) -retinal vascular microaneurysms, blot hemorrhages, and
a. Neovascularization is a feature of cotton-wool spots
nonproliferative DR​- PROLIFERATIVE
Mild nonproliferative retinopathy
b. Proliferative DR​ appears in the first and -changes in venous vessel caliber, intraretinal microvascular
second generation of diabetes with abnormalities, and more numerous microaneurysms and
retinal Microvascular aneurysm, blot hemorrhages
hemorrhages and cotton wool spots - Proliferative diabetic retinopathy
NONPROLIFERATIVE -appearance of neovascularization in response to retinal
c. Dyslipidemia, nephropathy, hypoxemia is the hallmark sign

hypertension are risk factors for DR -


NOT A RISK FACTOR
d. The most effective therapy is
prevention; Diabetic retinopathy is
treated by intensive glycemic and blood
pressure control

73. The follgowing are true of Diabetic


Nephropathy
a. Diabetic Nephropathy is the leading microalbuminuria as defined as 30–299 mg/d
cause of CKD, ESRD and CKD in a 24-h collection or 30–299 μg/mg creatinine
requiring renal transplant in a spot collection
b. Microalbuminuria in urinary protein
30-399 mg/d associated risk of CVD
c. Screen for microalbuminuria in patient
with type 1 DM of <5 yrs, type 2 DM
during pregnancy
d. ACTH/ARBs effective in disappearance
of albuminuria

74. The following are true of diabetic As with other complications of DM, the
neuropathy development of neuropathy correlates with the
C. The onset of neuropathy correlates duration of diabetes and glycemic control. -
with the duration of diabetes and Harrison
glycemic control

75.Cardiovascular diseases risk doubles in


every what level of mmhg in Systolic and
Diastolic BP?
A.30-20
B.20-10
C.20-15
D.10-5

OBESITY

76. The following are definition of BMI


except….
A) It is used to classify weight status and
CV risk
B) Excess abdominal fat is independently
assoc. w/ a higher risk of diabetes
mellitus and CV disease
C) The BMI is calculated as weight (lbs)/
height (inches)2 x 703
D) It should be performed in the horizontal
plane below the iliac crest ​<---ABOVE
the iliac crest dapat
77. The following are true about the goals of
treatment for obesity, EXCEPT:
A. To improve obesity-related comorbid
conditions and to reduce the risk of developing
future comorbidities
B. The goal of reducing weight by 8-10% over
6 months is a realistic target
C. Lifestyle management is done first before
initiating pharmacological therapy or surgery
regardless of BMI risk category
D. Not all patients who are deemed obese by
BMI alone need to be treated.

78. A selective 5-HT2C receptor agonist with a


functional selectivity ~15 times that of 5-HT2A
receptors and 100 times that of 5-HT2B
receptors - ​Lorcaserin

79. A glucagon-like peptide 1 receptor agonist


currently approved for the treatment of type 2
diabetes, has independent weight loss effects
via hypothalamic neural activation causing
appetite suppression - ​Liraglutide

80. A synthetic hydrogenated derivative of a


naturally occurring lipase inhibitor, lipostatin -
Orlistat

81. Which of the following is/are indication/s for


bariatric surgery?
A. Pt with a BMI above or equal 40kg/m2
B. Pt with a BMI above or equal 35kg/m2 with
co-morbidities
C. Pt with a BMI ...
D. A and B

82. A ….. malabsorptive bypass procedure … Roux-en-Y is the most commonly undertaken
most commonly undertaken and most and most accepted bypass procedure. It may
accepted siya… and can be done by be performed with an open incision or by
laparoscopy laparoscopy. -Harrison
B.Roux-en Y

METABOLIC SYNDROME Familiarize with the criteria

83.Visceral fat mass is a culprit in metabolic


syndrome and it is associated with
Low testosterone, high …… level, increase
levels of proinflammatory markers

84. The most challenging feature of


Metabolic Syndrome
answer- ​waist circumference

85. The following in metabolic syndrome is


associated with sedentary lifestyle
a. Increase adipose tissue
b. Reduced HDL
c. High bad cholesterol
d. AOTA

86. According to NCEP:ATPIII a low HDL


level is defined as
A level <50 for women, <40 for men
87. What is the waist circumference based
on the Harmonizing definition
A. >80 cm for men, >102 cm for women
B. >90 cm for women >80 cm for men for
South Asian, Chinese, and ethnic
South and Central American
C. >90 cm for men, >80 cm for women for
South Asian, Chinese, and ethnic
South and Central American
D. >90 cm for women, >85 cm for men for
South Asian, Chinese, and ethnic
South and Central American

88. Under normal physiologic condition,


insulin is a vasodilator with secondary effects
on sodium reabsorption in the kidneys…
insulin resistant is variant in your… The
signaling pathway that is impaired in insulin
resistance
A. Endothelin signaling
B. Phosphatidylinositol-3-kinase pathway
C. Nitric oxide pathway

89. The most important component for weight


loss in metabolic syndrome?
A. Physical activity
B. Behavioral mgt
C. Caloric restriction
D. Medical therapy

90. Bariatric surgery is an option for patients


with ​except
A. BMI of >35 kg/m2
B. BMI of >35 kg/m2 with comorbidities
C. BMI of (not sure if < or > ba) 40 kg/m2
D. None of the above

Case: Patient: MS, 45, male, known


hypertensive and diabetic. Non smoker, but
alcoholic beverage drinker.
Cc: snoring, daytime sleepiness, occasional
dyspnea
Med: taking losartan before but stopped bc
okay na daw iya BP. Still taking metformin
500mg BID.
PE: BP= 160/90, HR=80, RR=20, afebrile,
BMI 35kg/m^2. Waist circumference=105.
Cardiopulmonary findings are normal. (Basin
naa koy namali paki-check huhuh sa
number 91-94ish man guro ni na items)

91. What is the diagnosis?


a. Obstructive sleep apnea
b. Narcolepsy
c. Obstruction of Upper respiratory tract
d. Asthma

92. What diagnostic test would you order to


confirm diagnosis?
A. Overnight polysomnography
B. Polysomnography with multiple sleep
latency test
C. Fiberoptic laryngoscopy
D. Pulmonary fxn test

93. In patients with above-mentioned


conditions, which treatment improves insulin
sensitivity?
A. Inhaled corticosteroids
B. Methylphenidate
C. Oral steroids
D. ​Continue positive airway pressure (CPAP)

94. The patient came back with the following


results: TC= 200 LDL=160 TG=150
HBAC1=7% FBG=126. Will you treat the
patient with statin?
A. YES BECAUSE PT HAS MS AND DM
B. YES BECAUSE THE PT HAS
METABOLIC SYNDROME AND
OTHER RISK FACTORS
C. YES BECAUSE THE PT HAS A HIGH
10 YEAR ASVD RISK SCORE
D. AOTA
95. High intensity statin can be given, except
A. Atorvastatin 40-80 mg
B. Rosuvastatin 20-40mg
C. Simvastatin 20-40mg
D. None of the above

96. The following are therapeutic… …


target….. in metabolic syndrome, except
A.tryglecerol <...
B. Fasting blood glucose of <110 - normal ang
value diri
C. when the blood pressure is less than 130/85
..

THYROID

97.Diagnostic picture of hyperthyroidism


A. Decrease tsh decrease free t3 - secondary
hypothyroidism
B. Decrease tsh increase free t4
C. Increase tsh increase free t4 - secondary
hyperthyroidism
D. Increase tsh decrease free t4 -
98. Serum thyroglobulin levels are increased
except
answer: ​thyrotoxicosis factitia​ - no Tg levels

99. The following are clinical SYMPTOM


observed in hypothyroidism EXCEPT:
A. Tiredness
B. Constipation
C. Bradycardia
D. Feeling cold

Take note sa SIGNS VS SYMPTOMS

100. Levothyroxine is best taken


A. During breakfast
B. 30 mins before breakfast
C. 1 hr after breakfast
D. 2 hrs after breakfast

101. In the nuclear imaging, there is an


increase marker uptake in the following
conditions EXCEPT:
a) ​subacute thyroiditis​ - no increase secretion
but you have leak out
b) Graves disease - increase thyroid secretion
from thyroid gland…. Do uptake hyper --- black
image
c) toxic adenoma - nodule that is
hyperfunctioning --- dark image
d) toxic multinodular goiter - nodules that are
hyperfunctioning
102. The ​primary​ causes of primary
hypothyroidism include the following, EXCEPT:
a. Hashimoto’s thyroiditis
b. RAI
c. Sheehan syndrome​ - pregnant --->
during delivery there is massive blood
loss ---> hypoperfusion in the pituitary
gland where TSH is from (secondary
cause)
d. Total thyroidectomy

103. The common SYMPTOMS of p


thyrotoxicosis include the following, EXCEPT
C. atrial fibrillation​ - not a symptom
D. Fatigue and weakness

104. A rare and most important side effect of


anti-thyroid drugs
agranulocytosis
105. Grace is given methimazole 20mg once a
day for Graves' Disease. Her next follow up
test for unbound ...?
a. 3 months from initial treatment
​b. ​4-6 weeks from initial tx
​c. 6 months from initial tx
​d. 1 year from initial tx

106. The most common pattern for Sick


euthyroid syndrome (nonthyroidal illness)
Low T3, normal TSH, T4

107. Absolute contraindication for RAI


C. Pregnancy​ - and breastfeeding

108. The ​definitive​ management for toxic


multinodular​ goiter
A. Total thyroidectomy
B. Radioactive Iodine

MATCHING TYPE (109-114)


a. Acute thyroiditis
b. De quervain’s thyroiditis (or Subacute
thyroiditis)
c. Hashimoto
d. Silent thyroiditis

109. With ​underlying​ autoimmune thyroid


disease - ​D - Silent thyroiditis ​ (page 2299
Harrison)
110. Also known as painless thyroiditis - ​D -
silent thyroiditis
111. The thyroid usually shows a characteristic
patchy infiltrates with disruption of the thyroid
follicles - ​B - De Quervain’s thyroiditis
112. Present a firm or hard goiter of variable “
sizes and is most common cause of chronic
thyroiditis - ​C. - Hashimoto
113. Suppurative infection of the thyroid, with
biopsy findings of polymorphonuclear
leukocytes - ​A.- Acute thyroiditis
114. Clinical course is characterized by
thyrotoxic phase, hypothyroid phase, recovery
phase - ​B - De Quervain’s thyroiditis

115. type of thyroid carcinoma exist in 3


familial forms, and whose management is
primarily surgical.
A. Papillary thyroid cancer
B. Anaplastic thyroid cancer
C. Follicular adenoma
D. Medullary thyroid cancer
116. This type of Cancer is the most common
type of thyroid cancer 90%…..
a. Papillary thyroid cancer
b. Anaplastic thyroid cancer
c. Follicular adenoma
d. Medullary thyroid cancer

117. The bethesda system provides uniform


terminology about thyroid nodule cytology
result, except:
A. ​Benign, hyperfunctioning
B. Non diagnostic cytology
C. Atypia of unknown significance
D. Suspicious for Malignancy

118.The following are risk factor in patients


with thyroid nodule, EXCEPT
a. Female gender
b. New enlarging neck mass
C. Increase nodule size >4cm
D. <20 >65 y.o

119. 20 y.o Evaluated with thyroid nodule that


she noticed for sometime, what other imaging
modality should be requested next?
a. Neck CT scan
b. Thyroid scan
c. ​Thyroid ultrasound
d. Neck MRI

120. After further workup, she decided for


surgery for 3.5cm tumor and her finding was
…. thyroid cancer stage 1. What additional
treatment should be done?
A. Chemotherapy
B. Radiotherapy
C.
​D. Radioactive Iodine ablation
CV2 CLINICAL
Question Rationale

1. True about the diameter of the aorta in adults Harrison’s 20th ed (p 1917)
a. 1.8cm at the origin
b. 2 cm in the ascending portion
c. 2.5 cm in the descending potion in the thorax
d. ~3 cm in abdomen

2. True aneurysm Harrison’s 20th ed (p


Ans: involves all three layers of the vessel wall

3. results from conditions that cause degradation or Harrison's 20th ed, pg 1917
abnormal production of structural components of the
aortic wall:collagen and elastin.
Ans:aortic aneurysm

4.The following makes the aorta prone to injury and P 1917 20th
disease except; Laplace’s law: Tension = pressure x radius
A. Exposure to high pulsatile pressure amd
shear stress
B. Continuous mechanical trauma
C. Development of aneurysmal dilation
D. Increase in wall tension which is proportional
to ​sum​ of pressure and radius

5. Which of the following is incorrectly defined DEFINITIONS OF ANEURYSM CLASSIFICATIONS


(Stated below are the CORRECT definitions)

Answer: ● True Aneurysm - involves ​all three layers of the


vessel wall
● Pseudoaneurysm - ​intimal and medial layers ​are
B. ​Pseudoaneurysm - intimal and medial layers are disrupted and the ​dilated segment ​of the aorta is
disrupted and the dilated segment of the aorta is lined by ​adventitia only
lined by adventitia only ● Fusiform Aneurysm - ​affects the ​entire
circumference ​of a segment of a vessel resulting in
diffusely dilated artery
● Saccular Aneurysm - involves ​only a portion ​of the
circumference resulting in an ​outpouching of the
vessel wall

Harrisons 20th ed, page 1917

6.The following portions of the aorta can be affected Harrison 20th ed, page 1917-1918
by medial degeneration except
A. Proximal aorta Medial degeneration (cystic medial necrosis?) -
B. Ascending aorta degeneration of collagen and elastic fibers in media
C. Sinuses of Valsalva and loss of medial cells, replaced by multiple clefts of
D. Transverse aorta mucoid material (eg proteoglycans)
- Affects proximal aorta (results to
circumferential weakness and dilation, and
causes fusiform type
- Also affects ascending aorta and sinuses of
valsava

7. Which of the following is true regarding infectious The infectious causes of aortic aneurysms include ​syphilis,
causes of aneurysm? tuber​culosis, and other bacterial infections​.
a. Syphilic periaortitis and mesoaortitis are Syphilis i​ s a rela​tively uncommon cause of aortic aneurysm.
located mostly in the aortic arch and Syphilitic periaortitis and mesoaortitis damage elastic
descending aorta fibers, resulting in thickening and weak​ening of the aortic
b. Tuberculous aneurysms typically affect the wall. ​Approximately ​90% of syphilitic aneurysms are located in
the ascending aorta or aortic arch​.
abdominal aorta
c. Granulomatous destruction of the medial Tuberculous aneurysms ​typically affect the thoracic aorta and
layer causes loss of aortic wall elasticity in result from ​direct extension of infection from hilar lymph
mycotic aneurysm nodes or contiguous abscesses as well as from bacterial
seeding. Loss of aortic wall elasticity results from
d. A saccular tuberculous aneurysm develops granulomatous destruction of the medial layer.
as a result of staphylococcal, streptococcal,
salmonella​ infections A ​mycotic aneurysm ​is a rare condition that develops as a result
of staphylococcal, strepto​coccal, ​Salmonella,​ or other bacterial
or fungal infections of the aorta, usually at an atherosclerotic
plaque​. These aneurysms are usually ​saccular​. Blood cultures
are often positive and reveal the nature of the infective agent.

Harrisons 20th ed, page 1918

8. Which of the following condition is most frequently Harrison 20th, THORACIC AORTIC ANEURYSM
associated with aneurysms of the descending part
thoracic aorta?
a. Cystic medial necrosis - Atherosclerosis: most frequently associated
b. Syphilis condition with descending thoracic aorta
c. Atherosclerosis - Medial generation: most common pathology
d. Tuberculosis associated with ascending aortic aneurysm
9. The risk of rupture is related to the size of the
aneurysm and the presence of symptoms, which of
the following is false?
a. Average growth rate of thoracic aneurysm is
0.1-0.2cm per year
b. Abdominal aortic aneurysm​s associated with
Marfan’s syndromeor aortic dissection may
expand at a greater rate
c. Risk of rupture is approximately 2-3% per
year for thoracic aortic aneurysm <4.0cmin
diameter
d. Risk of rupture is approximately 7% per year
for those >6cm in diameter

Thoracic dapat

10.Which of the following is TRUE regarding the HPIM 20th Page 1918
clinical presentation of aortic aneurysms?
a. Thoracic aortic aneurysms are mostly
symptomatic
b. Compression or erosion of adjacent tissue by
aneurysms may cause symptoms such as
chest pain, shortness of breath, cough,
hoarseness and dysphagia
c. Aneurysmal dilation of the ​transverse aorta
may cause CHF as a consequence of aortic
regurgitation
d. Compression of the ​Inferior Vena Cava may
produce congestion of the head, neck and
upper extremities

11. In asymptomatic patients whose aneurysms are


too small to justify surgery, how often should non
invasive with either contrast-enhanced ct or mri be
performed to monitor expansion?
A. At least every 6-12 months
B. At least every 3-6 months
C. At least every 24 months
D. At least every 18 months

12. Which of the following are acceptable treatment HPIM 20th Page 1918
modalities in managing thoracic aneurysms to
reduce the further expansion? B blockers - esp with Marfan or aortic root dilation,
A. B-adrenergic blockers decrease rate of expansion
B. ACE inhibitors
C. Operative repair ARB - reduce TGF B signaling, decreasing rate of
D. Placement of prosthetic graft aortic dilation

Operative repair with placement of prosthetic graft


- For symptomatic ascending thoracic aortic
aneurysm
- For most asymptomatic
- Associated with aortic valves
- Aortic root or ascending aortic >= 5.5
cm
- Growth rate: >0.5 cm/yr
- With degenerative descending
thoracic >6cm

- Endovascular repair of descending thoracic if


feasible: >5.5 cm
- Growth rate >1cm/yr
Surgery - Marfan - ascending thoracic of 4.5 cm
Replacement of ascending aorta >4.5cm with
bicuspid aortic valves undergoing aortic valve
replacement due to severe aortic stenosis or aortic
regurgitation

13.Operative repair with placement of a prosthetic


graft is indicated in patient with for most
asymptomatic aneurysm including those associated
with bicuspid aortic valves, in the following situations,
except:
a. When the aortic root or ascending aortic
diameter is greater than or equal to 5.5cm
b. When the growth rate for ascending aortic
aneurysm is >0.5cm per year
c. Replacement of the ascending aorta >4.5
with bicuspid aortic valve undergoing valve
replacement.
d. Asymptomatic ascending thoracic aneurysm
is 5cm

14. Which of the following is FALSE regarding for Answer is D kay dpat >1cm (see no 12)
operative repair to thoracic aneurysms?
a. In px with Marfan syndrome, ascending
thoracic aneurysm of 4-5 cm should be
considered for surgery
b. Operative repair is indicated for patients with
degenerative thoracic aortic aneurysm when
the diameter is >6cm
c. Endovascular repair should be considered if
feasible when the diameter is>5.5cmwith
degenerative descending thoracic aneurysm
d. Repair has recommended when the diameter
of descending thoracic aortic aneurysm has
increased ​>0.5​cm per year

15. What is Most potent modifiable risk factor for Harrison 20th,Page 1919
development of abdominal aortic aneurysm?
a. Male gender
b. Increases in age
c. Cigarette smoking
d. hypertension

16. The risk of rupture increases with size of the H 20th Prognosis of abdominal aortic aneurysm
aneurysm. Which of the ff is true regarding 5 year - Depends on size of aneu and severity of
riak? coexisting coronary artery and
a. 10% for aneurysm <5cm cerebrovascular disease
b. 20-40% for aneurysm >5cm in diameter - Risk of rupture inc with size
c. 1-2% for aneurysms <6cm - 5yr risk: <5cm = 1-2%
d. 20-40% for aneurysms >6cm in diameter - >5cm = 20-40%

17. Which of the following is useful for serial Harrison 20th, ABDOMINAL AORTIC ANUERYSM
documentation of aneurysm size and can be used to part
screen patients at risk for developing an aortic
aneurysm and is recommended for men aged 65-75 Abdominal ultrasound - delineate transverse and
years who have ever smoked? longitudinal dimension
a. X-ray - May detect mural thrombus
b. Ultasound - Useful for serial documentation of aneurysm
c. CT Scan size
d. MRI - For screening patients at risk for developing
aortic aneurysm
- Men 65-75 yo
- With Siblings or offsprings with
abdominal aortic aneurysm
- Ppl with thoracic aortic aneu/
peripheral arterial aneu

18. Accurate non invasive tests to determine the


location and size of abdominal aortic aneurysms and
to plan endovascular or open surgical repair

CT with contrast and MRI


19. Which of the following are accurate indication for Harrison 20th, ABDOMINAL AORTIC ANEURYSM
drugs given in abdominal aortic aneurysm? part
A. Statins-to reduce the risk of CV events Statins -reduce cv risk
related to atherosclerosis B blockers and raas inhibitors - not effective
B. B-adrenergic blockers-reduce the rate of
aneurysmal growth
C. Renin-angiotensin inhibitor-reduce the rate of
aneurysmal growth
D. None of the above

20.Operative repair of the aneurysm with insertion of Harrison 20th, AAA part
a prosthetic graft or endovascular placement of aortic
stent graft is indicated for AAA, except? Op repair for AAA
A. Aneurysm of any size that are expanding - Any size, expanding rapidly or with sxs
rapidly - Asymptomatic: >= 5.5 cm
B. Aneurysm assoc with symptoms
C. Asymptomatic AAA with diameter of ​less or Serial noninvasive follow-up of smaller aneurysm
equal to5.5cm <5.5cm
D. Symptomatic AAA is less than 5.5cm

21. Which of the following is not part of the four


major acute aortic syndromes?
a. Aortic rupture and dissection
b. Aortic occlusion
c. Intramural hematoma
d. Penetrating atherosclerotic ulcer

22. W/c of the ff is false regarding acute intramural Harrison 20th, AAS part
hematoma; Pathology and radiologic variants:
A. Result from rupture of vasa vasorum Intramural hematoma without intimal flap
B. Hemorrhage into the wall of the aorta - Mostly at descending thoracic
C. Most common location occur in descending - From rupture of vasa vasorum with
abdominal​ aorta hemorrhage into wall
D. May progress to dissection and rupture - May progress to dissection and rupture
Penetrating atherosclerotic ulcer
- By erosion of plaque into aortic media,
usually localized, not assoc with extensive
propagation
- Middle and distal desc thoracic, assoc with
extensive athero diseases
- Can erode beyond internal elastic lamina,
cause medial hematoma and lead to false
aneurysm formation or rupture
23. The classification of dissection which involves the
ascending aorta
A. Debakey I
B. Debakey II
C. Debakey III
D. Stanford A

24.(same choices as number 23) It is the intimal tear DeBakey type III
located in descending aorta with distal propagation of
the dissection

25. Dissection is limited to the ascending aorta DeBakey type II

26. Proximal dissection Stanford type A

27. Intimal tear occurs in the ascending aorta but DeBakey type I
involves descending aorta as well

28. Limited to the aortic arch and descending aorta Debakey 1 is entire aorta
(distal dissection) Debakey 2 does not involve descending aorta
A. DeBakey 1 Stanford A w/ ascending aorta
B. DeBakey 2 Stanford B w/o ascending aorta
C. Stanford A
D. Stanford B
29. Question is about the leading cause of PAD in
patient 40 years old above: ​ATHEROSCLEROSIS

30. Primary site of peripheral arterial sites, except: Harrison’s 20th ed (p 1923)
A. Abdominal aorta and iliac artery (30%,
symptomatic)
B. Femoral and popliteal artery ( 80-90 %)
C. Tibial and peroneal artery (40-50%)
D. Saphenous veins (20%)

31. Atherosclerotic lesions occur preferentially at Harrison’s 20th ed (p 1923)


arterial branch points, which are sites with the
following charactersistics:
a. Increased turbulence
b. Increased diameter
c. Altered shear stress
d. Intimal injury

32. Which of the ff occurs in px with severe arterial Harrison p 1923 20th
occlusive disease in whom resting blood flow cannot
accommodate basal nutritional needs of the tissues?
A. Muscle fatigue
B. Claudication
C. Critical limb Ischemia
D. All of the above
33. The following may occur in patients with critical
limb ischemia:
a. Development of ulcer or gangrene
b. Pallor of the soles of the feet upon elevation
of legs and repeated flexing of the calf
muscles
c. Rubor from reactive hyperemia develop when
legs are dependent
d. Numbness and ​hypereflexia from ischemic
neuropathy

34. False about ABI Harrison’s 20th ed (p 1923)


Answer: A.) normal is 1.00-1.20 Normal 1-1.40
Border- 0.91-0.99
PAD - <0.90
Noncompressible arteries 2ndry to vascular
calcification - >1.40

35. False regarding prognosis of px with PAD? Harrison PAD prognosis part
A. ⅓ to ½ of px have evidence of on CAD on > ½ dapat
clinical presentation and ECG
B. Less than ½ have significant CAD by
coronary angiography
C. 15-25% 5- year mortality rate among px with
PAD
D. 2-6 fold increased risk of death from coronary
heart disease and are highest in those with
severe PAD

36. Hyperplasia disorder which involves small and Harrison’s 20th ed (p 1925)
medium sized arteries
A. Raynaud's phenomenon
B. Buerger's Disease
C. Fibromuscular Dysplasia
D. Thomboangitis Obliterans

37. Inflammatory occlusive vascular disorder inv Harrison p 1925 20th


small and medium-size arteries and veins in the Buerger’s or thromboangiitis obliterans
distal upper and lower extremities
A. Raynaud’s phenomenon
B. Buerger's Disease
C. Fibromuscular dysplasia
D. Vasculitis

38. Vascular disorder characterized thickened Harrison p 1925 20th


fibromuscular contiguous with thin, less inv portions Please memorize key words each disease sa other
of arterial wall, identified angiography by “​string of arterial diseases
beads”
A. Thromboangitiis obliteran
B. Atheroembolism
C. Fibromuscular dysplasia
D. Raynaud’s phenomenon

39. Thromboangiits Obliterans


a. Men >40
b. Asian
c. African
d. Female

40. The triad for the manifestations of Buerger’s P 1928 20th


disease includes:
a. Claudication of the affected extremity
b. Raynaud’s phenomenon
c. Migratory superficial vein thrombophlebitis
d. All of the above

41. Clinical features of thromboangitiis obliterans, P 1925 20th


except​:
A. Proximal atherosclerotic disease
B. Claudication of affected extremity
C. Raynaud’s phenomenon
D. Migratory superficial vein thrombosis

42. True about the management of thromboangitiis P 1926 20th


obliterans Thromboangiitis - no known cause, but strong link to
a. No specific treatment except abstention from cig smoking
tobacco Antibiotics may be helpful
b. Antibiotics are ​necessary Glucocorticoids and anticoagulants are not helpful
c. Glucocorticoids are the ​mainstay​ of treatment
d. Anticoagulants are ​contraindicated

43. A hyperplastic disorder that affects medium size P 1925 20th


and small arteries. It occurs predominantly in
females and usually involves the renal and carotid
arteries but can affect extremity vessels such as the
iliac and subclavian arteries.
A. Fibromuscular dysplasia
B. Thromboangiitis obliterans
C. Vasculitis
D. Giant cell arteritis

44. In this condition, multiple small deposits of fibrin, P 1926 20th


platelets, and cholesterol debris embolize from
proximal atherosclerotic lesions
A. Acute limb ischemia
B. Atheroembolism
C. Popliteal artery aneurysm
D. Popliteal artery entrapment

45. Raised erythematous lesions develop on the P 1930 20th


lowert part of legs and feet in COLD weather pernio/ chilblains
Pernio
Erythromelalgia - burning pain, feet, ​warm
environment
Frostbite - pain, paresthesia, white waxy skin, ​cold
exposure

46. It is characterized by episodic digital ischemia Harrisons 20th p. 1928


manifested clinically by the sequential development
of blanching, cyanosis, and rubor of the fingers or
toes after cold exposure.
A. Reynaud’s Phenomenon
B. Acrocyanosis
C. Popliteal Artery Entrapment
D. Acute Limb Ischemia

47. Vasoconstriction and secondary dilation of the P 1929 20th


capillaries and venules with resulting persistent
cyanosis of the hands and, less frequently, the feet
and intensifies by exposure to cold.

A. ​Acrocyanosis
B. Raynaud’s phenomenon
C. Atheroembolism
D. Pernio

48. This happens during ischemic phase of RP P 1928 20th


a. Capillaries and venules constrict Capillaries and venules DILATE
b. Cyanosis result from deoxygenated blood CONSTRICTION of arteries
present in the vessel Skin is COLD
c. Vasodilation of arteries
d. Skin is warm to touch

49. It is the thickening and tightening of the digital


subcutaneous tissue that develops in 10% of patients
with Raynaud's Phenomenon.
A. Scleroderma
B. ​Sclerodactyly
C. Vasospasm
D. Vasculitis

50. RP occurs often in px exposed to this activity


a. Use of vibrating hand tools
b. Playing piano
c. Use of jackhammers
d. All of the above
51. A common cause of Raynaud’s Phenomenon in 2ndry RP p 1929 20th
men > 50 years M >50 : athero
a. Atherosclerosis of the extremities Young male smokers: thromboangiitis obliterans,
b. Hypertension uncommon
c. Diabetes mellitus - These two cause cold pallor in 1-2 digits
d. trauma

52. Treatment used for Raynaud’s phenomenon


a. CCB - nondihydropyridine
b. CCB - dihydropyridine
c. ACEI
d. B-blocker

53. Reduction in CV morbidity and mortality in All are mentioned sa harrison tx of pad
patients with PAD have been documented with the
use of:
a. Aspirin
b. Ticlopidine
c. ACE inhibitor
d. All of the above

54. True about the management of claudication PAD TX HARRISON 20th


a. Drugs used specifically in the treatment of A - no definitive medical therapy for cli
lower extremity claudicationinclude B- tx of dm and control of hpn - improve outcome,
pentoxifylline and cilostazol reduce cv risk, anti hpn meds- do not worsen
b. Niether intensive treatment of DM nor claudication
anti-hypertensive appears to alter the
progression of symptoms of claudication
c. Physical exercise, rehabilitation and smoking
cessation have proven efficacy
d. ALL of the above

55. originate in the superficial system and result from Harrison p 1931 20th
defective structure and function of the valves of the Primary - anatomy/ structure defect
saphenous veins, intrinsic weakness of the vein wall, Secondary - venous hypertension
and high intraluminal pressure

A. Primary varicose veins

56. Results from venous hypertension assoc. w/ Harrison’s, 20th (p. 1931, 2nd paragraph)
deep venous insufficiency or deep-venous
obstruction
A. Secondary varicose veins
57. This occurs following deep vein thrombosis, as
the delicate valve leaflets become thickened and
contracted and can no longer prevent retrograde flow
of blood, and the vein itself become rigid and thick
walled.
a. Chronic Venous Insufficiency
b. Deep Venous Insufficiency
c. Primary Varicose Veins
d. Secondary Varicose Veins

58. True about symptoms of varicose veins P 1931 20th


A. Stabbing pain A- dull ache
B. Throbbing or heavinesress C- after prolonged standing
C. Pressure sensation in the legs while on rest D- elevating legs
D. Relieved while hanging the legs

59. The combination of induration, hemosiderin Page 1931, Harrisons PIM 20th ed
deposition, and inflammation, and typically occurs in Chapter 276 Chronic Venous Disease and
the lower part of the leg just above the ankle. Lymphedema
A. Lipodermatosclerosis (Clinical Manifestations of CVD)
B. Atrophie Blanche
C. Lymphedema
D. Varicosities

60. Bedside maneuver to distinguish whether P 1931 20th


varicose veins are secondary to deep-venous Brodie - elevate - tourniquet- stand- 30s filling veins :
insufficiency. Px is lying supine, leg is elevated and varicose 2ndary to deep venous insuff
veins are allowed to empty. Tourniquet is placed on - If prompt filling after toruniquet removal:
proximal part of the thigh then Px is asked to stand. primary varicose with superficial VI
Perthes - stand - tourniqeut - 5min walk
Answer: ​Brodie-Trendelenburg Test - Patent: collapse distal vein
- Obstructed: dilated superficial vein

61. A test where the midthigh is applied with See no 60


tourniquet, then the patient is asked to walk for 5
minutes
A.?
B.Brodie-Trendelenburg test
C.Perthes Test
D.?

62. The principal diagnostic test to evaluate px with P 1932 20th


chronic venous disease
A. Magnetic resonance
B. Computed tomographic
C. Conventional venography
D. Venous duplex ultrasonography
63. Which of the ff is true about CVD treat treatment? P 1932 harrissons 20th
A. Diuretics may reduce edema, but at risk of vol Rationale:
depletion Topical steroids can only be used for a SHORT
B. Topical steroids may be used for a long period
period Herbal meds has VASOCONSTRICTIVE effects
C. Herbal extract have vasodilating and anti
Inflammatory effects
D. All of the above

64. Mgt of venous insufficiency that involves injection Endovenous thermal ablation procedures of the
of chemical in the vein to cause fibrosis and saphenous veins include ​endovenous laser
obstruction. therapy​ and ​radiofrequency ablation​.
A. Endovenous laser therapy
B. Radiofrequency ablation Sclerotherapy involves the injection of a chemical
C. Endovenous thermal ablation into a vein to cause fibrosis and obstruction.
D. Sclerotherapy Sclerosing agents approved by the U.S. Food and
Drug Administration include sodium tetradecyl
sulfate, polidocanol, sodium morrhuate, and glycerin.

HPoIM, 20e p. 1933

65. Chronic condition caused by impaired transport P 1933 20th


of lymph and characterIzed by swelling of one or
more limbs and occasionally the trunk and genitalia
A. Lymphedema
B. Chronic venous insuff
C. Deep venous ins
D. PAD

66. Which of the ff statements is true about Read table 276-2 p 1934 20th
lymphedema Read also the text kay didto gikan hehe
A. Congenital something medyo taas to
B. Taas pud wala nako namemorize
C. The most common cause of secondary
lymphedema is filariasis
D. AOTA

67. Causes of lymphedema include/s Read table 276-2 p 1934 20th


a. Milroy’s disease
b. Meige’s disease
c. Lymphedema-distichiasis syndrome
d. AOTA

68. Which of the following is not a part of Virchow's (page 1631 HPIM 19th ed)
triad? Dili ba pud apil ang immobilization bc nag cause ug
A. Immobilization stasis? hehe
B. Hypercoagulability
C. Endothelial damage
D. Inflammation

69. What is the most common cause of acquired


thrombophilia?
● Factor V Leiden
● APAS
● SLE
● Idiopathic

Ans. APAS (antiphospholipid antibody syndrome) 😢


70.in massive pulmonary embolism, the cause of P 1911 20th
circulatory collapse is
A. Decreased cardiac contractility
B. Decreased alveolar o2
C. Decreased cardiac output from rv tension
D. All of the above

71. Utilizing the well’s criteria for pulmonary


embolism. Heart rate 105 and immobilization > 3
days gives a score of
A. 3
B. 1
C. 2
D. 4

72. Plasma D-dimer True except Harrison’s 20th page 1912


a. increased due to breakdown of fibrin by D- dimer assay is NOT SPECIFIC
plasmin
b. useful rule out test
c. 80% sensitive for DVT
d. Highly specific

73. In pulmonary embolism, the most frequently cited P 1911 20th


abnormality in ecg in addition to sinus tachycardia is
A. Atrial fibrillation
B. S1Q3T3
C. 2nd degree AV block
D. Tall peak T wave

74. Diagnostic test for DVT: ​Venous Duplex


ultrasonography

75. Which is true on the treatment of DVT PRIMARY THERAPY:


a. Consists of clot dissolution with ● Clot dissolution with pharmacomechanical
pharmacomechanical therapy including ​high therapy including ​low dose catheter-directed
dose catheter-directed thrombolysis thrombolysis
b. Reserved for patients with extensive femoral, ● Reserved for patients with extensive femoral,
iliofemoral, or upper extremity DVT iliofemoral, or upper extremity DVT
c. Anticoagulation and placement of an IVC filter
is ​primary​ prevention SECONDARY​ PREVENTION:
d. All of the above ● Anticoagulation
● Placement of an Inferior Vena Caval (IVC)
Filter

76. Treatment modalities for patient with


hemodynamic instability such as hypotension and
RV hypokinesis in echocardiogram
A. Anticoagulant alone
B. IVC filter
C. Anticoagulant + thrombolysis
D. All of the above

77. An oral anti-Xa pentasaccharide that does not


cause heparin-induced thrombocytopenia and does
not require frequent monitoring:
ANS: Fondaparinux

GG 13th Ed. Chapter 32 pp. 592

78. Warfarin a vit K antagonist that prevents GG anticoag chapter, 2nd page, or H 20TH warfarin
carboxylation activation coagulation factors therapy (vte?)
A. 1,2,5,10
B. 2,6,10,12
C. 2,7,9,10
D. 2,5,7,10
79. Target INR for warfarin
A. 1.0-2.0
B. 2.0-3.0
C. 2.5-3.5
D. 3.0-4.0

GG 13th Ed. Chapter 32 pp. 593

80. The duration of anticoagulation for px w/ DVT in


upper extremity or calf that is provoked by surgery,
trauma, estrogens or indwelling central venous
catheter or pacemaker
a. 3 mos of anticoagulation usually suffice

Harrison 20th p.1915

81. IVC filter is indicated in


a. Prophylaxis for extremely high risk

82. First line inotropic age of PE related shock 20th P 1915 mgt of massive PE part
A. Dobutamine Dobutamine and dopamine - 1st line
B. Norepinephrine
C. Vasopressin
D. Phenylephrine

83. Successful fibrinolytic therapy rapidly reverses P 1915 20th fibrinolysis part
right heart failure and may result in a lower rate of
death and recurrent PE by
A. Dissolving much or the anatomically
obstructing pulmonary arterial thrombus
B. Preventing the release of serotonin and other
neurohormonal factors that exacerbate
pulmonary hypertension
C. Lysing much of the source of the thrombus in
the pelvic or deep leg veins, thereby
decreasing likelihood of recurrent pe
D. All of the above

84. Chronic thromboembolic pulmonary hypertension P 1916 20th pulmo thromboendarterectomy part
develops in 2-4% of acute PE px, therefore PE px
who have initial pulmonary hypertension should be
followed up at about how many weeks with a repeat
ecg
A. 4 weeks
B. 6 weeks
C. 8 weeks
D. 12 weeks

85. DVT prophylaxis for px undergoing major 20th See table 273-4 p 1916
orthopedic surgery?

86. Prevention of VTE for cancer surgery HPIM 20th ed, p 1916 table 273-4
a. Enoxaparin 40mg daily for 1 month
b. UFH 5000U SC BID
c. Fondaparinaux 2.5mg daily for 1 month
d. AOTA

PART II.
A 25 year old male rushed to the ER due to 1-week history of discoloration right foot. Whole foot numb
while the rest of the right lower extremity is painful. Digits of the right foot are cyanotic, paralyzed
while there is minimal movement on the ankle and knee. ____right foot feels cold to touch with absent
dorsalis pedis and posterior tibial pulses.

87. What is your diagnosis ​(Acute Limb Ischemia? Hehe)

88. Define your diagnosis. ​ALI occurs when an arterial occlusion results in sudden cessation of blood flow to
an extremity. ALI is suggestive since the patient’s symptoms started 1 week ago and these symptoms include:
painful lower extremity, cyanosis, paralysis, coldness, absent distal pulses, numbness and discoloration which
are the common manifestations of ALI (not sure hehe)

89. Acute or Chronic? ​ACUTE

90. (define) the duration ​ACUTE <2 weeks, CHRONIC >2 weeks

91-93. Risk factors to ask in history of the px (3)


(Di ko sure guys if kanang naa sa first na bullets or kanang under sa comorbidities basta naa sa gihatag ni doc na ppt on Acute Limb
Ischemia hehe)
(Ang first bullets kay what pertinent questions should you ask dayon and second kay purely risk factors)
● SYMPTOM DURATION
● MOTOR DEFICIT
● SENSORY DEFICIT
● COMORBIDITIES
○ DIABETES
○ DYSLIPIDEMIA
○ ATHEROSCLEROSIS
○ Include (​SMOKING & FAMILY HISTORY​)

94-96. DDx and rationale other arterial occlusive diseases (eg fribomuscular dysplasia, thrombangiitis,
atheroembolism etc)
97-99. 6 Ps (½ pt each)

100-103. Table

104. Treatment​ medical? Surgery?

105. Drug and dose ​IV Heparin bolus 80 infusion 18 (check dosage)
A 38 year old female, no comorbidities with right leg swelling and in pain. Patient is taking oral
contraceptive and is on a 18-hour flight.
106. Primary impression: dvt

107. Risk factor ​long haul air travel, oral contraceptive (check risk factors dvt p 1910)

108. Define phlegmasia alba dolens?? Check nlng: but sa notes from doc lec: compressed arteries
causing white and swollen leg?

109. Define phlegmasia cerulea dolens? check also: cyanosis and bluish

110. Diagnostic test : ​d dimer? Venous ultrasonography?

Developed dyspnea with HR of 120 BP 70/60 and RR 36


111. Diagnosis: PE

112-115. What happened to the Px. (Draw)

116-118. wells category something p 1911

119. Treatment p 1914

120. Drug and dose p 1914


1. True about aneurysms
a. True aneurysm involves all three layers of the
vessel wall.
b. In a pseudoaneurysm, the intimal and medial
layers are disrupted and the constricted
segment of the aorta is lined by adventitia only.
c. A saccular aneurysm affects the entire
circumference of a segment of the vessell.
d. Fusiform aneurysm involves only a portion of
the circumference, resulting in an outpouching
of the vessel wall.

2. This result from conditions that cause degradation or


abnormal production of the structural components of
the aortic wall: elastin and collagen

a.) ​Aortic Aneurysm


b.) aortic dissection
c.) pulmonary thromboembolism
d.) aortitis

3. May occur after penetrating or nonpenetrating chest


trauma and most commonly affects the descending
aorta just beyond the site of insertion of the
ligamentum arteriosum.
a. Degenerative aneurysm
b. Traumatic aneurysm
c. Chronic aortic dissection
d. Thoracic aortic dissection

4. This is an anatomic remnant of the right aortic arch:


a. Pseudoaneurysm
b. Kommerell’s diverticulum
c. Right subclavian artery
d. Right internal carotid artery

Congenital Anomalies of the Aorta p. 1637

5. Degenerative causes for aortic aneurysm include/s:


a. Hypercholesterolemia
b. Hypertension
c. Smoking
d. All of the above
6. Infectious causes for aortitis include/s:
a. Syphilis
b. Tuberculosis
c. Mycotic
d. All of the above
Table 301-1 p.1638

7. A risk factor for acute aortic syndromes( aortic


dissection, acute intramural hematoma,penetrating
atherosclerotic ulcer).
a. Pregnancy
b. Chronic aortic dissection
c. Thromboembolism
d. All of the above

Harrison’s cardio 3rded pg538

8. True about tuberculous aneurysms


a. Typically affect the thoracic aorta
b. Loss of aortic wall elasticity results from
granulomatous destruction of the medial layer.
c. Result from direct extension of infection from
hilar lymph nodes.
d. AOTA Harrison’s cardio 3rded pg539

9. Mycotic aneurysms occur usually as this type of A mycotic aneurysm is a rare condition
aneurysm, that develops as a result o staphylococcal,
a. Fusiform streptococcal,
b. Saccular Salmonella, or other bacterial or fungal
c. True infections of the aorta, usually at an atherosclerotic
d. pseudoaneurysm plaque. These aneurysms are usually​ saccular.

Harrison’s cardio 3rded pg539

10.this is a rare condition that develops as a result of Basta ang answer diri kay mycotic
staphylococcal, streptococcal, salmonella, or other
bacterial

11. This affects the entire circumference of a segment A ​fusiform aneurysm​ affects
of the vessel, resulting in a diffusely dilated artery. the​ entire circumference of a segment of the vessel​,
a. Saccular aneurysm resulting in a diffusely dilated artery.
b. Fusiform aneurysm
c. Thoracoabdominal aortic aneurysms Harrisons cardio 3rded pg537
d. pseudoaneurysm
12. In asymptomatic patients whose aneurysms are too
small to justify surgery, noninvasive testing with either
contrast-enhanced CT or MRI should be performed
every
a. 1-2 yrs
b. 6-12 mos
c. 3-6 mos
d. If symptomatic

Thoracic Aortic Aneurysms p.1639

13. True about using chest xray as a diagnostic tool A chest x-ray may be the first test that suggests the
for thoracic aneurysms: diagnosis o a thoracic aortic aneurysm (Fig. 46-1).
a.)may be the first test that suggests the diagnosis of a Findings include widening of the mediastinal shadow
thoracic aortic aneurysm and displacement or compression of the trachea or
b.) widening of the mediatinal shadow noted left main stem bronchus.
c.)there may be displacement or compression of the
trachea or left main stem bronchus Harrison’s cardio 3rded pg539
d.) aota

14. Medications that are recomended for patients with β-Adrenergic blockers currently are recommended for
thoracic aortic aneurysms, particularly those with patients with thoracic aortic aneurysms, particularly
marfan’s syndrome, who have the evidence of aortic those
root dilatation to reduce the rate of further expansion. with Marfan’s syndrome, who have evidence of aortic
a. ACE root
b. ARBs dilatation to reduce the rate of further expansion.
c. Calcium channel Blockers
d. Beta-blockers Harrison’s cardio 3rded pg540

15.this type of aneurysm comonly produces no An ​abdominal aortic aneurysm ​commonly produces
symproms. It usually is detected on routine no symptoms. It usually is detected on routine
examination as a palpable, pulsatile, expansile, and examination
nontender mass, or it is an incedental finding observed as a palpable, pulsatile, expansile, and nontender
on an abdominal imaging study performed for other mass, or it is an incidental finding observed on an
reasons. abdominal imaging study performed or other reasons.
a. Abdominal aortic aneurysms
b. Mycotic aneurysm
c. Descending thoracic aneurysm Harrison’s cardio 3rded pg540
d. Chronic aneurysm.

16. At least 90% of all abdominal aortic aneurysms At least 90% of all
>4.0 cm are related to abdominal aortic aneurysms >4.0 cm are related to
a.) vasculitis atherosclerotic
b.) atherosclerotic disease disease​, and most of these aneurysms are
C.) thromboembolism below the level of the renal arteries.
d.) Diabetes mellitus
Harrison’s cardio 3rded pg540

17. The most common pathologic condition associated The most common pathologic condition associated
with degenerative aortic aneurysms is with degenerative aortic aneurysms is
a.) ​atherosclerosis atherosclerosis.
b.) hypertension
c.) aortic dissection Harrison’s cardio 3rded pg538
d.) aging
18. This inflammatory ds often affects the ascending TAKAYASU’S ARTERITIS/Pulseless disease
aorta and aortic arch , causing obstruction of the aorta
and its major arteries This inflammatory disease often affects the ascending
a. Pulseless disease aorta and aortic arch, causing obstruction o the
b. Giant cell arteritis aorta and its major arteries
c. Chronic atherosclerotic occlusive ds
d. Rheumatic aortitis Harrison’s cardio 3rded pg544

19. Operative repair is indicated for px w/ descending Operative repair is indicated or patients with
aortic aneurysms when the diameter is ___ descending
a. >6cm thoracic aortic aneurysms when the diameter is >6
b. 5-6cm cm, and
c. >5.5 cm endovascular repair should be considered if feasible
d. There is an increase of 0.5 cm per year when the
diameter is >5.5 cm. Repair is also recommended
when the
diameter of an aneurysm has increased >1 cm per
year.

Harrison’s cardio 3rded pg540

20. Type A Stanford classification for aortic dissections


corresponds to
a. Involvement of transverse aorta
b. Involvement of descending aorta
c. Involvement of ascending aorta
d. All of the above

21. The 4 major acute aortic syndromes refer to:


a. Aortic rupture, aortic dissection, intramural
hematoma, and penetrating athersoclerotic
ulcer

22. True about aortic dissections The peak incidence of aortic dissection is in the sixth
a. Peak incidence is in the 6th and 7th decades and seventh decades. Men are more a ected than
b. Men are more affected than women at a ration women by a ratio of 2:1.
of 2:1 The pain may be localized to the front or back o the
c. Pain may be localized to the front or back of chest,oft en the interscapular region, and typically
chestZ migrates
d. All of the above with propagation o the dissection.

Harrison’s cardio 3rded pg542

23. A sign of aortic regurgitation as a complication of


aortic dissection:
a. Bounding pulses
b. Narrow pulse pressure
c. Systolic murmur often radiating along the right
sternal border
d. Absence of congestive heart failure

Harrison’s 19th ed
24. Management of aortic dissection if nitropusside and The calcium channel antagonists verapamil and diltiazem
may be used intravenously i nitroprusside or β-adrenergic
beta adrenergic blockers cant be given: blockers cannot be employed.​ ​The addition o a parenteral
a.ACE angiotensin-converting enzyme (ACE) inhibitor such as
b.ARBs enalaprilat to a β-adrenergic blocker also may be considered.
c.BB Isolated use of a direct vasodilator such as hydralazine is
contraindicated because these agents can increase hydraulic
d.CCB shear and may propagate the dissection.

Harrison’s cardio 3rded pg543

25. Most common cause/s of perioperative mortality The major causes o perioperative
and morbidity of AoD mortality and morbidity include myocardial in arction,
paraplegia, renal failure, tamponade, hemorrhage,
a. MI and sepsis.
b. sepsis
c. paraplegia Harrison’s cardio 3rded pg543

d. aota - answer

26. This syndrome increases the risk of post-dissection


complication.
a. Ehlers-Danlos Syndrome
b. Loeys-Dietz Syndrome
c. Down Syndrome
d. Marfan’s Syndrome Source: Kasper, D., et.al (2015) - Harrison's
Principles o​ ​f Internal Medicine (19th Ed) p.1642

27.A classification scheme developed for thoracic


aortic dissections
a. Stamford
b. Well’s criteria
c. DeBakey
d. All of the above

28.This inflammatory ds often affects the ascending TAKAYASU’S ARTERITIS/Pulseless disease


aorta and aortic arch, causing obstruction of the aorta
and its major arteries This inflammatory disease often affects the ascending
a. Aortitis aorta and aortic arch, causing obstruction o the
b. Takayasu’s arteritis aorta and its major arteries
c. Giant cell arteritis
d. Rheumatic aortitis Harrison’s cardio 3rded pg544

29. True about the types of Infective aortitis Mycotic aneurysms


a. Tuberculous​ aneurysms have a predilection for have a predilection or the suprarenal abdominal
the suprarenal abdominal …. aorta.
b. Mycotic aneurysms typically affect ​adolescents Mycotic aneurysms typically
and occur in men 3x more frequently than in a ect the ​elderly​ and occur in men three times more
women frequently than in women.
c. Syphilitic aortitis is a late manifestation of luetic Syphilitic aortitis is a ​late manifestation of luetic
infection in ection​ that usually a ffects the proximal ascending
d. Syphilitic aortitis occasionally may involve the aorta, particularly the aortic root, resulting in aortic
aortic arch or the ​ascending aorta dilation and aneurysm formation. Syphilitic aortitis
occasionally may involve the ​aortic arch or the
descending
aorta.

Harrison’s cardio 3rd edpg545

30. Which gene mutation correctly matches the type of


aortic ds?
a. Ehler danlos syndrome- type IV-fibrillin-1
b. Marfan’s syndrome-Type III procollagen
c. Loeys-Dietz Syndrome-TGF-B Receptor 1
d. Turner’s Syndrome- ACTA2

p. 1638

31. Risk of PAD increase in px with


a. Cigarette smokers
b. Hypercholeterolemia
c. Renal insufficiency
d. All of the above

32. The most common symptom of the peripheral


artery ds
a. Peripheral edema on dependency
b. Rubor of affected extremity
c. Intermittent claudication
d. Decreased or absent pulse distal to obstruction

33. True about the systolic blood pressure findings in


the extremities
a. SBP is higher in the UE than in LE
b. SBP in the arms and legs is similar
c. The SBP in the legs is lower
d. The SBP in the legs is usually higher
34. The ratio of the ankle and brachial artery pressures
(termed the ankle:brachial index, or ABI) is ___ in
normal individuals
a. 1.0-1.40
b. 0.9-0.99
c. >1.0
d. 1.0-2.0

35. In PAD, symptoms are far more common in the


lower than in the upper extremities because
a. Higher incidence of obstructive lesions in the
lower extremities
b. Resting blood flow cannot accommodate
nutritional needs
c. both a and b
d. None of the above

Harrisons cardio 3rded pg546

36. This is a hyperplastic disorder of the extremities


that affects medium size and small arteries
a. Raynaud’s syndrome
b. Buerger’s ds
c. Fibromuscular dysplasia
d. Thromboangitis obliterans

37. An inflammatory occlusive vascular disorder


involving small and medium sized arteries and veins in
the distal upper and lower extremities
a. Raynaud’s syndrome
b. Buerger’s ds
c. Fibromuscular dysplasia
d. Vasculitis
Harrison’s IM 19th Ed: P.1645
38. Thromboangitis obliterans mostly affects
a. Men >40 y/o
b. Asians
c. African Americans
d. Females

Harrison’s IM 19th Ed: P.1645

39. Clinical feature of thromboangitis obliterans,


EXCEPT
a. Proximal atherosclerotic ds
b. Claudication of the affected extremity
c. Raynaud’s phenomenon
d. Migratory superficial vein thrombophlebitis

40. The triad for the manifestation of Buerger’s ds The clinical features of thromboangiitis obliterans
includes: often include a triad of claudication of the affected
a. Claudication of the affected extremity extremity, Raynaud’s phenomenon, and migratory
b. Raynaud’s phenomenon superficial vein thrombophlebitis. (as seen in first line
c. Migratory superficial vein thrombophlebitis of above pic)
d. All of the above

41.True about the management of thromboangitis


obliterans
a. No specific tx except abstention from tobacco
b. Antibiotics are necessary
c. Glucocorticoids are the mainstay of tx
d. Anticoagulants are contraindicated

42.True regarding intraarterial thrombolytic therapy in


acute limb ischemia. EXCEPT
a. Most effective when acute arterial occlusion
occurred for less than 2 weeks
b. Indicated when px overall condition
contraindicates surgical intervention
c. Indicated when small vessels are occluded
d. Preferred when restoration of blood flow must
occur w/in 24h to prevent limb loss
43. In this condition, multiple small deposits of fibrin,
platelets, and cholesterol debris embolize from
proximal atherosclerotic lesions of aneurysmal sites,
may occur after intraarterial procedures
a. Acute Limb ischemia
b. Atheroembolism
c. Popliteal artery aneurysm
d. Popliteal Artery entrapment

44. Raised erythematous lesions found on the lower


part of the legs and feet during cold weather
a. Pernio
b. Erythromelalgia
c. Frostbite
d. Livedo reticularis

45. A disease characterized by an episodic digital


ischemia and is manifested by digital blanching and
rubor of fingers or toes after cold exposure and
subsequent rewarming

A. Raynaud’s phenomenon
B. Popliteal artery entrapment
C. Atheroembolism
D. Popliteal artery aneurysm

46.In this condition, there is arterial vasoconstriction


and secondary dilation of the capillaries and venules w/
resulting persistent cyanosis of the hands, and less
frequently, the feet. Cyanosis may be intensified by
exposure to a cold environment
a. Acrocyanosis
b. Raynaud’s phenomenon
c. Atheroembolism
d. Pernio

47. This happens during the ischemic phase of


Raynaud’s phenomenon
a. Capillaries and venules​ constrict
b. Cyanosis results from the deoxygenated blood
that is present in these vessels
c. Results from the ​vasodilation​ of the digital
arteries
d. Skin is usually ​warm ​to touch

48. This is the thickening and tightening of the digital


subcutaneous tissue that develop on 10% of px w/
Raynaud’s phenomenon
a. Vasospasm
b. Sclerodactyly
c. Scleroderma
d. Vasculitis

49. A common cause of Raynaud’s phenomenon in


men >50years
a. Atherosclerosis of the extremities Harrisons cardio 3rded pg554
b. HPN
c. DM
d. Trauma

50. Raynaud’s phenomenon occurs often in px


exposed to this activity
a. Use of vibrating hand tools
b. Playing the piano
c. Use of jackhammers
d. All of the above

Harrisons cardio 3rd ed pg555

51.Medications that decrease the frequency and


severity of Raynaud’s phenomenon
a. BB
b. ACEIs
c. Non Dihydropyridine CCBs
d. Dihydropyridine CCBs

52. These originate in the superficial system and result


from defective structure and function of the valves of
the saphenous veins, intrinsic weakness of the vein
wall, and high intraluminal pressure
a. Primary varicose veins
b. Secondary varicose veins
c. Lymphedema
d. Nevi

53. This results as a consequence of incompetent


veins in which there is venous hypertension and
extravasation of fluid and blood elements in the tissue
of the limb.

A. Secondary varicose veins


B. Primary varicose veins
C. Chronic venous insufficiency
D. Lymphedema

Source: Harrison 19th ed. Pdf page 2295

54. This occurs ff. DVT, as the delicate valve leaflets


become thickened and contracted and can no longer
prevent retrograde flow of blood and the vein itself
becomes rigid and thick walled
a. Chronic venous insufficiency p. 1650 Harrison’s 19th ed
b. Deep vein insufficiency
c. Primary varicose veins
d. Secondary varicose veins

55. True about the symptoms of Varicose viens


a. Stabbing pain
b. Throbbing or heaviness
c. Pressure sensation in the legs while on rest
d. Relieved after hanging the legs

56.This is the combination of induration, hemosiderin


deposition, and inflammation and typically occurs in the
lower part of the leg just above the ankle
a. Lipodermatosclerosis
b. Atrophie blanche
c. Lymphedema
d. Varicosities

57. A bedside maneuver used to distinguish primary lisod kaayo icrop kay bulag ug column sa book. Hehe
varicose veins from secondary varicose veins due to pero B. Answer ani.
deep venous insufficiency. As the px is lying supine,
the leg is elevated and the veins allowed empty. Then, Brodie-trendelenburg.
a tourniquet is placed on the proximal part of the thigh
and the px is asked to stand.
a. Straight leg raise test
b. Brodie-Trendelenburg test
c. Perthes test
d. Drawer test
58. This test assesses the possibility of deep venous
obstruction. A tourniquet is placed on the midthigh after
the px has stood, and the varicose veins are filled. The
px is then instructed to walk for 5min.
e. Straight leg raise test
f. Brodie-Trendelenburg test
g. Perthes test
h. Drawer test

59. The principal diagnostic test to evaluate px w/


chronic venous ds
a. MRI
b. CT scan
c. Venous duplex ultrasonography
d. Conventional venography

60. True about the medical therapy for Chronic Venous


insufficiency
a. Diuretics may reduce edema but at the risk of
volume depletion and compromise in renal fxn
b. Topical steroids may be used for a longer
period of time to treat inflammation associated
with stasis dermatitis
c. Several herbal supplements are touted to have
vasodilation and anti-inflammatory properties
d. All of the above

l
61. Management for venous insufficiency w/c involves
the injection of a chemical into a vein to cause fibrosis
and obstruction
a. Endovenous laser therapy
b. Radiofrequency ablation
c. Endovenous thermal ablation
d. Sclerotherapy

62. A chronic condition caused by impaired transport of


lymph and characterized by swelling of one or more
limbs and occasionally the trunk and genitalia
a. Lymphedema
b. Chronic venous insufficiency
c. Deep venous insufficiency
d. PAD

63. True about secondary lymphedema


a. Genetic condition that results to damage to
lymphatic channels
b. Recurrent episodes of bacterial lymphangitis,
usually caused staphylococci, are a very
common cause of lymphedema
c. The most common cause of secondary
lymphedema worldwide is lymphatic filariasis
d. All of the above
64. Causes of lymphedema include/s
a. Milroy’s ds
b. Meige’s ds
c. Lymphedema-distichiasis syndrome
d. All of the above

Source: Kasper, D., et.al (2015) - Harrison's


Principles of Internal Medicine (19th Ed) p.1653

65. Virchow's Triad except


a. Immobilization
b. Hypercoagulability
c. Endothelial injury
d. Inflammation
Source: Kasper, D., et.al (2015) - Harrison's
Principles of Internal Medicine (19th Ed) p.1631
66. In massive PE, the cause of circulatory collapse is
a. Decreased cardiac contractility
b. Decreased alveolar O2
c. Decreased CO from RV tension
d. All of the above

Harrisons cardio 3rd ed pg528

67. Regarding D-dimer, w/c of the ff is not true


a. Present in other conditions: MI, sepsis, cancer,
3rd trimester of pregnancy
b. Used as rule out test
c. 80% sensitive for DVT
d. Highly specific

68. In PE, the most frequently cited abnormality in


ECG, in addition to sinus tachycardia is
a. AFib
b. S1Q3T31 sign
c. 2nd degree AV block
d. Tall peak T waves

69. Principal imaging test for the diagnosis of PE


a. Chest roentgenography
b. MR pulmonary angiography
c. Lung scan
d. Chest CT w/ contrast

70. Effective anticoagulation is the foundation for


successful Tx of DVT and PE. W/c of the
anticoagulants is an anti-Xa pentasaccharide that does
not cause heparin induced thrombocytopenia and
requires no lab monitoring?
a. UFH
b. Enoxaparin
c. Argatroban
d. Fondaparinux

71. Warfarin, a Vit K antagonist prevents carboxylation


activation of coagulation factors
a. I, II, V, X
b. II,VI,X, XII
c. II,VII,IX,X Source: Kasper, D., et.al (2015) - Harrison's
d. II,V,VII,X Principles of Internal Medicine (19th Ed) p.1635
72. Prothrombin time is standardized by calculating
INR, w/c assesses the anticoagulant effect of warfarin.
Target INR ranges
a. 1.0-2.0
b. 2.0-3.0
c. 2.5-3.5 Source: Kasper, D., et.al (2015) - Harrison's
d. 3.0-4.0 Principles of Internal Medicine (19th Ed) p.1635

73. The duration of anticoagulation for px w/ DVT


isolated to Upper Extremity or calf that has been
provoked by surgery, trauma, estrogens, or an
indwelling central venous catheter or pacemaker
a. 3mos. of anticoagulation usually suffice
b. 3-6mos of anti-coagulation are considered
sufficient
c. Prescribed LMWH as monotherapy w/o warfarin
and continue anticoagulation indefinitely Source: Kasper, D., et.al (2015) - Harrison's
d. anticoagulation w/ indefinite duration w/ target Principles of Internal Medicine (19th Ed) p.1635
INR 1.5-2

74. First line inotropic agents of PE-related shock


a. Dobutamine
b. Norepinephrine
c. Vasopressin
d. Phenylephrine

Source: Kasper, D., et.al (2015) - Harrison's


Principles of Internal Medicine (19th Ed) p.1636

75. Chronic thromboembolic pulmonary HPN develops


in 2-4% of acute PE px, therefore PE px who have
initial pulmonary HPN should be followed up about how
many weeks w/ a repeat echocardiogram
a. 4wks
b. 6wks
c. 8wks
d. 12wks

Source: Kasper, D., et.al (2015) - Harrison's


Principles of Internal Medicine (19th Ed) p.1636

76. DVT prophylaxis among hospitalized px undergoing


major orthopedic surgery includes the ff:
a. Warfarin w/ target INR
b. Aspirin 81-325mg daily
c. Intermittent pneumatic compression
d. All of the above

Source: Kasper, D., et.al (2015) - Harrison's


Principles of Internal Medicine (19th Ed) p.1637

77. Prevention of VTE for cancer surgery


a. Enoxaparin 40mg daily for 1 month
b. UFH 5000 units SC BID
c. Fondaparinux 2.5 mg daily for 1month
d. All of the above may be used as prophylaxis
strategy

NOTE:
UFH 5000 units SC BID = for High-risk nonorthopedic
surgery

Fondaparinux 2.5 mg daily for 1month = for Major


orthopedic surgery

Source: Kasper, D., et.al (2015) - Harrison's


Principles of Internal Medicine (19th Ed) p.1637

78. A mutation of ____, a transcription factor upstream


of lymphatic endothelial cell differentiation, has been
described in px w/ lymphedema, alopecia, and
teleangiectasis [hypotrichosis, lymphedema,
teleangiectasia syndrome]
a. SOX18 Source: Kasper, D., et.al (2015) - Harrison's
b. JAK2 Principles of Internal Medicine (19th Ed) p.1654
c. p18
d. p24

79. Reductions in CVD morbidity and mortality in px


with PAD have been documented with the use of
a. Aspirin
b. Ticlopidine
c. ACEIs
d. All of the above

80. True about the management of claudication


a. Drugs used specifically in the treatment of lower
extremity claudication include pentoxifylline and
cilostazol
b. Neither intensive treatment of DM nor anti-HPN
therapy appears to alter the progression of
symptoms of claudication
c. Physical exercise, rehabilitation, and smoking
cessation have proven efficacy.
d. All of the above​.
Ry
1. Patient MS, 24 year old, female came in for
consult due to fever and joint pains for 1 week. She
also mentioned that she had odynophagia 2 weeks
prior. She noted pains on her knees and elbows
which was relieved when she took in Mefenamic
acid. Physical examination findings revealed BP
100/70 mmHg, HR 72/min, RR 20/min, temp 39C,
with note of pink, macules, on the trunk, swollen
and erythematous knees, and a high-pitched,
blowing systolic murmur over the apex.

A. Acute rheumatic fever


B. Vasculitis
C. Rheumatoid arthritis
D. Septic arthritis

2 Which of the following tests would you order if


you are considering #1?

A. ASO Titer
B. ANCA (Antineutrophil cytoplasmic antibody)
C. RF assay
D. ANA (Antinuclear antibody)

3. The following are recommended tests for all


cases similar to case #1 except?

A. Blood culture
B. Joint aspirate
C. Echocardiogram
D. Erythrocyte sedimentation rate
4. Based on the 1992 Jones Criteria, supporting
evidence of a preceding streptococcus infection
within the last 45 days is confirmed by:

A. Elevated or rising anti-streptolysin 0 or


other streptococcal antibody
B. Positive throat culture
C. Rapid antigen test for group A
streptococcus Guidelines for the Diagnosis of Initial or Recurrent
D. All of the above Attack of Rheumatic Fever (Jones Criteria,
Updated 2015)1-5
MAJOR MANIFESTATIONS
Carditis
Polyarthritis
Erythema marginatum
Subcutaneous nodules
Chorea

MINOR MANIFESTATIONS
Clinical features:
Arthralgia
Fever
Laboratory features:
Elevated acute phase reactants:
Erythrocyte sedimentation rate
C-reactive protein
Prolonged P-R interval

SUPPORTING EVIDENCE OF ANTECEDENT


GROUP A STREPTOCOCCAL INFECTION

Positive throat culture or rapid streptococcal


antigen test
Elevated or increasing streptococcal antibody titer

5. The patient was ordered to have labs taken. She Penicillin is the drug of choice and can be given
was given Mefenamic acid 500 mg capsule TID. orally as:
Paracetamol 500 mg tab q 4 h. Cefuroxime 500 mg 1. phenoxymethylpenicillin, 500 mg [250 mg
tab BID. Upon follow-up, the pertinent lab results for children ≤27 kg] PO twice daily, or
revealed elevated leukocyte count, elevated ESR, 2. amoxicillin, 50 mg/kg [maximum, 1 g] daily,
a prolonged PR interval on ECG and an elevated for 10 days) or as a
antistreptolysin O titer. OR
as a single dose of 1.2 million units (600,000 units
What is the recommended antibiotic and dosage for children ≤27 kg) IM benzathine penicillin G.
for this case?
Harrison’s p.2152
A. Phenoxymethylpenicillin 500mg PO BID
B. Amoxicillin 50mg/kg for 7 days PO
C. Benzathine penicillin G 600, 000 units IM
D. Cefuroxime 30mg/kg for 7 days PO

6. If patient is allergic to Penicillin, which of the


following can be given?
a. Clindamycin 300 mg BID PO for 10 days
b. Clarithromycin 250 mg BID PO for 10 days
c. Azithromycin 500 mg tab day 1, 250 mg day
2-5
d. All of the above

7. According to the AHA Recommendations for


Duration of Secondary Prophylaxis, the duration of
prophylaxis for patients with RF with carditis but no
residual valvular disease is:
A. For 5 years after the last attack or 21 years
of age
B. For 10 years after the last attack or 21
years of age
C. For 10 years after the last attack or 40
years of age
D. Lifelong prophylaxis

8. In the proliferative phase, 1 to 6 months after the


onset of RF, these granulomatous lesions are
pathognomonic for rheumatic carditis, are seen
and can be found in valve tissue as well as in
endocardium, myocardium and pericardium.
a. ​ Aschoff bodies
b. Lewy bodies
c. Schiller-Duval bodies
d. Call-exner bodies

9. Which of the ff. Is the hallmark of rheumatic


carditis?
A. ​ Valvular damage
B. Pericardial involvement
C. Cardiomyopathy
Harrisons p.2150
D. Chamber enlargement

10. Which of the ff is TRUE regarding the damage


to the tricuspid and pulmonary valve in the
rheumatic heart disease?
A. Tricuspid and pulmonary valve calcification
secondary to chronic inflammation
B. Pulmonary and tricuspid regurgitation are
functional in nature resulting from left-sided
valvular disease
C. Pulmonary and tricuspid valve regurgitation
cause increase in pulmonary pressures
D. Tricuspid and pulmonary valves can be Harrisons p.2151
primarily affected by rheumatic heart
disease

11. Which of the following is the ECG finding in


myocardial inflammation?
A. Atrial fibrillation
B. ST segment prolongation
C. ​P-R interval prolongation
D. T-wave inversion
12. Which of the following is FALSE regarding the
management of RHD?
A. There is no treatment for ARF that has
been proven to alter the likelihood of
developing or the severity of RHD
B. 2D echo should be performed after 1 month
to monitor progression of carditis
C. Salicylates and NSAIDs are of no proven
value in the management of carditis
D. Oral Penicillin V is the best antibiotic for
secondary prevention of acute rheumatic
fever and rheumatic heart disease
Harrisons p.2153

13. The latent period between the initial attack of


rheumatic carditis and development of mitral
stenosis is:
A. 30 years
B. ​ 20 years
C. 5 years
D. 2 years

14. Latent period between the precipitating group


A streptococcal infection and the appearance of
the clinical features of ARF:
A. ~3 weeks
B. ~2 months
C. ~6 months
D. ~1 year

15. What is the treatment of choice for very severe


mitral stenosis (valve area <1cm2) with favorable
valve morphology, no clot and no or mild MR?
A. Percutaneous mitral balloon
commissurotomy
B. Mitral valve replacement
C. Open valvotomy
D. Addition of warfarin
16. Haydee has symptoms of easy fatigue and
shortness of breath, 2D echo revealed highly
calcified mitral valve with LA thrombus. She is 32
years old and has no other co-morbidities and is
not classified as high surgical risk in pre-operative
evaluation. What is the preferred approach for
Haydee?
A. Percutaneous mitral balloon
commissurotomy
B. ​Mitral valve replacement
C. Open valvotomy
D. Maximiza heart failure regimen

17. What is the valve most commonly affected in


rheumatic carditis?
A. Aortic valve
B. Pulmonic valve
C. Tricuspid valve
D. ​ Mitral valve

18. The P-R interval prolongation in rheumatic


carditis may be due to:
a. Pericarditis
b. Mitral valve stenosis
c. Pulmonary hypertension
d. ​ Myocardial inflammation

19. Chronic MR can result from the following


conditions
A. Mitral valve prolapse (MVP)
B. Extensive mitral annular calcification
C. Hypertrophic obstructive cardiomyopathy
(HOCM)
D. All of the above
20. MVP is a frequent finding in patients with
heritable disorder of connective tissue including the
following:
A. Marfan's syndrome
B. Osteogenesis Imperfecta
C. Ehlers-Danlos syndrome
D. AOTA

21. Which of the following is/are features of mitral


stenosis in rheumatic heart disease?
A. Rigidity, deformity, and retraction of the
valve cusps
B. Commisural fusion, as well as shortening,
contraction and fusion of the chordae tendineae
C. Narrowing at the apex of the
funnel-shaped (fish mouth) valve
D. ​ All of the above
22. Which of the following is FALSE regarding the
application of the JONES Criteria in the different
diagnostic
a. Primary RF: 2 major or 1 major and 2 minor
manifestations plus evidence of preceding
GAS infection
b. Recurrent attack of RF without established
RHD: 2 major or 1 major and 2 minor
manifestations plus evidence of preceding
GAS infection
c. Recurrent attack of RF with RHD: 2 minor
manifestations plus evidence of preceding
GAS infection
d. Chronic valve lesions of RHD (1st time with
pure MS): Other major manifestations or
evidence of group A streptococcal infection

23. Which of the following is NOT a major criteria


in diagnosing rheumatic fever using the jones
criteria
A. Carditis
B. Polyarthralgia
C. Chorea
D. Erythema marginatum

24. Which of the following is true regarding the


decision in choosing revascularization procedure in
patients with ischemic heart disease?
A. Patients undergoing coronary angiography
with anatomy suitable are candidates for
revascularization
B. PCI for single vessel
C. Patients with Left main coronary and/or
multi vessels affected can undergo PCI or
CABG
D. All of the above
25. Secondary measures to improve the mortality
and morbidity rate of STEMI
a. aspirin (nalimot ko ani na option ㅠㅠ)
b. chronic routine use of beta blockers for 2 years
c. modification of risk factors of atherosclerosis
d. all of the above

26. RV infarction causes R sided failure of which It should be​ jugular venous distention​.
includes the ff. EXCEPT:
SOURCE: Kasper, D., et.al (2015) - Harrison's
A. Jugular venous ​collapse Principles of Internal Medicine (19th Ed) p.1608
B. Kussmaul's sign
C. Hepatomegaly
D. Hypotension

27. Shock is the clinical syndrome that results from


inadequate tissue perfusion and is usually
accompanied by hypotension defined as:
a. MAP <40 mmHg in previously normotensive
patients
b. MAP <50 mmHg in previously normotensive
patients
c. MAP <60 ​ mmHg in previously
normotensive patients
d. MAP <70 mmHg in previously
normotensive patients
Harrisons p.1744

28. What is the reason for variability in applying HARRISON P. 363 CARDIO 3RD ED.
diagnostic criteria in low-risk vs high-risk
populations in the 2015- AHA revised Jones criteria
for the diagnosis of RF?
a. Clinical utility of the Jones criteria is
determined by the pre-test probability and
background disease prevalence
b. To avoid in underdiagnosis low-incidence
populations
c. To avoid overdiagnosis in high-risk
populations
d. All of the above

29. Which of the following is FALSE regarding the


2015 criteria’s recognition of the clinical entity of
“possible” rheumatic fever:
a. Clinical judgement may be applied in
situations wherein it is not possible to apply
the Jones criteria because of lack of
laboratory facilities
b. When possible RF is made in a high
incidence setting, it is reasonable to
consider offering 12 months of secondary
prophylaxis
c. Treatment is followed by reevaluation
based on history and physical examination
d. Repeat echocardiography is not included in
reevaluation
HARRISONS CARDIO 3RD ED PAGE 364

30. The possible etiology of Aortic Stenosis


includes
a. Syphilis
b. Hypertension
c. Ankylosing spondylitis
d. ​ Rheumatic fever

Harrison p. 1529
31. The ff are included in the etiology of Aortic
Regurgitation. Except:
A. Radiation
B. Aortic Root disease
C. Aortitis
D. Hypertension

Harrison p. 1529

32. In addition to valvular aortic stenosis, this


lesion may be also responsible for obstruction to
LV outflow:
A. Hypertrophic obstructive cardiomyopathy
B. Membranous subaortic stenosis
C. Supravalvular AS
D. All of the above
Harrison p.1530

33
The cardinal symptoms of aortic stenosis include:
a. Edema
b. Angina pectoris
c. PND
d. Orthopnea

p.1530
34. Which of the following is FALSE regarding
acute severe AR:
A.​ ​May occur in infective endocarditis, aortic

dissection or trauma
B.​ ​The LV cannot dilate sufficiently to maintain

stroke volume
C.​ ​ ​LV diastolic pressure rises ​gradually​ with
marked elevation of LA and PA wedge pressures
D.​ ​Pulmonary edema and/or cardiogenic shock

may develop rapidly

Harrison p. 1535

35. Which of the following is correct regarding the Rationale


findings of AR?
A. A ​gradually​ rising “water-hammer” pulse, A. Corrigan’s pulse is rapid, not gradual.
which collapses suddenly as arterial B. It’s Quincke
pressure falls rapidly during late systole and C. It’s Traube
diastole (Corrigan’s pulse)
B. capillary pulsations, an alternate flushing
and paling of the skin at the root of the nail
while pressure is applied to the tip of the
nail (​Traube’s sign​), are characteristic of
chronic severe AR.
C. A booming “pistol-shot” sound can be heard
over the femoral arteries (​Quincke’s pulse​)
D. to-and-fro murmur (Duroziez’s sign) is
audible if the femoral artery is lightly Harrison p. 1535-1536
compressed with a stethoscope.

36 The following are pathogenesis of acute It’s supposed to be​ S. pyogenes ​I think yan
rheumatic fever EXCEPT: pasabot ni Doc during the review but it was not
a. It is a hypersensitivity reaction attributed to stated sa Harrison, Nelson, Braunwald, or even
antibodies directed against ​Streptococcus Medscape.. Last resort Wikipedia
viridans​ that are cross-reactive with host
antigens
b. There is a 2-3 week delay in symptom
onset after infection to generate an immune
response
c. Streptococcus are completely absent from
the lesion https://en.wikipedia.org/wiki/Rheumatic_fever
d. Antibodies against M proteins of certain
streptococcal bind to proteins in the
myocardium and cardiac valves and cause
injury through the activation of complement
and macrophages

37 Physical findings in Mitral stenosis


a. In patients with severe MS, there may be a
malar flush with pinched and blue facies
b. The opening snap (OS) of the mitral valve is
most readily audible in expiration
c. Soft, grade I or II/VI systolic murmurs are
commonly heard at the apex or along the
left sternal border in patients with pure MS
d. All of the above

38. The size of the mitral valve opening in severe


Mitral Stenosis:
Very severe MS MVA ​≤​1 cm2
a. 4-6 cm2
T½ ​≥​220 ms
b. 1.5-2 cm2
c. <1.5 cm2 Severe MS MVA ​≤​1.5 cm2
d. <1 cm2 T½ ​≥​150 ms

Progressive MS MVA >1.5 cm2


T½ <150 ms

MVA, mitral valve area;


T ½, pressure half-time

Harrisons p.1541

39. In the management of patient with mitral


stenosis, for those with atrial fibrillation or history of
thromboembolism, the goal of Warfarin therapy is
to target INR to:
A. 2-3
B. 3-4
C. <2
D. 1.5-2
40. The following are pathogenesis of infective
endocarditis (IE) except:
● The causative organism differ on the
underlying risk factors
● Deformed and healthy valves can be
damaged by S. Aureus in 10-20% of cases
● 50-60% of IE cases occurring on damaged
or deformed valves are caused by group A
B-hemolytic Streptococcus
● “Culture negative” endocarditis may happen
in about 10% of all cases of endocarditis

41. Which of the the factors are predisposing to


endocarditis?

a. those that cause microorganism seeding


into the bloodstream or seemingly trivial
breaks in the epithelial barriers of the gut,
oral cavity, or skin.
b. a dental or surgical procedure
c. a contaminated needle shared by
intravenous drug users,
d. ALL OF THE ABOVE

42 Which is the most predominant cause of


infective endocarditis?
A. Native valve endocarditis
B. Prosthetic valve endocarditis
C. Pacemaker valve endocarditis
D. ICD infective endocarditis
43. Which is the most common symptom in
infective endocarditis?
A. Fever
B. Chills
C. Dyspnea
D. Chest pain
44 Which of the ff is a correct criteria according to
Duke Criteria of infective endocarditis?
A. Major criteria: predisposing heart condition
B. Minor criteria: temp >38
C. Major criteria: vascular phenomena
D. Minor criteria: endocardial inflammation

HRSON p. 819

45 ​INCORRECT​ diagnosis according to the


modified duke’s criteria

A. Possible IE: 1 major AND 1 minor


B. Definitive IE: 2 major OR 5 minor
C. Rejected diagnosis: no criteria met to
diagnose
D. Possible IE: 4 minor criteria
46. Demonstration of microorganism in culture of Harrisons p.819
histologic examination of vegetation is a pathologic
criteria of:
a. Definite IE
b. Possible IE
c. Probable IE
d. All of the above

Definite endocarditis:
● two major criteria,
● one major criterion and three minor criteria,
or
● five minor criteria

Rejected
● if an alternative diagnosis is established,
● if symptoms resolve and do not recur with
≤4 days of antibiotic therapy, or
● if surgery or autopsy after ≤4 days of
antimicrobial therapy yields no histologic
evidence of endocarditis.

Possible IE when
● one major and one minor criterion or
● three minor criteria

47 In heart failure, the impaired systolic emptying


of the ventricle(s) and/or the impairment of
ventricular relaxation results to the following
except:

a. Promotes an accumulation of blood in the


venous circulation at the expense of the
effective arterial volume
b. Heightened tone of the sympathetic
nervous system causing renal
vasoconstriction and reduction of
glomerular filtration
c. Decrease in ventricular diastolic volume
promotes a more forceful contraction and
may thereby maintain the cardiac output
d. Sodium and water retention causing
increase in blood volume accumulating in
the venous circulation, raising venous
pressure and causing edema

48 The ff are signs of heart failure except: Symptom ang dyspnea :(


A. ​Dyspnea with exertion
B. Elevated JVP
C. Ventricular (S3) Gallop
D. Peripheral cyanosis

49. The following statements regarding the etiology


of heart failure is true except:
a. I​n 20-30% of the cases of HF with a
depressed EF, the exact etiology basis is
known and labelled as nonischemic dilated,
or idiopathic cardiomyopathy
b. Prior viral infection or toxin exposure (e.g.
alcoholic or chemotherapeutic) also may
lead to a dilated cardiomyopathy
c. High output states can be due to
thyrotoxicosis, beri-beri and acute anemia
d. Both CAD and hypertension interact to
augment the risk of heart failure, as does
diabetes mellitus
50) Which of the following has the best prognosis

a. NYHA IV
b. NYHA III
c. NYHA II
d. NYHA​ I

51. Acute decompensated heart failure is a Inotropic therapy: dobutamine, milrinone,


syndrome often resulting in hospitalization with the levosimendan, omecamtiv, mecarbil
following principles to be considered except:
a. Identification of precipitants Harissons p.1509-1510
b. Volume management
c. Vasodilators
d. Inotropic therapy (dopamine)

52. Hypotension, end organ hypoperfusion or IV therapy in acute decompensated in heart failure:
shock states 1. Inotropic therapy - Use in hypotension,
end-organ hypoperfusion, or shock states
Ans. ​Dobutamine ● Dobutamine
● Milrinone
● Levosimendan
● Omecamtiv
● Mecarbil

53 Presence of pulmonary congestion for rapid IV therapy in acute decompensated in heart failure:
relief of dyspnea; in the presence of preserved BP? 2. Vasodilators - Use in presence of pulmonary
congestion for rapid relief of dyspnea,
Ans. ​Nitroglycerine in presence of a preserved blood pressure
● Nitroglycerine
● Nesiritide
● Nitroprusside
● Serelaxin

54 First line therapy for volume overload with IV therapy in acute decompensated in heart failure:
congestion: 3. Diuretics - First line of therapy in volume
overload with congestion; may use
Ans. ​Furosemide bolus or continuous dosing; initial low dose (1×
S home dose) or high
dose (2.5 × home dose) equally effective with
higher risk of renal
worsening with higher dose
● Furosemide
● Torsemide
● Bumetanide
● Adjuvant diuretics for augmentation

Harrison’s 19th ed table 280-1

55 Not a high risk factor for coronary


atherosclerosis?

a. Hyperuricemia
b. Low HDL
c. High LDL
d. Hypertension

56. Which of the following is most likely to be


presenting with angina pectoris?
A. 25 old male with non productive cough for 1
week with chest pain on deep inspiration
B. 40 year old female with point tenderness over
the left parasternal area following recent steering
wheel taruma
C. 52 year old male smoker with jaw pain on
climbing 2 flight of stairs
D. 55 year old female with persistent pain over the
left costal margin associated with a vesicular rash
of demarcational distribution

57. The following causes of chest discomfort need


urgent evaluation and management, EXCEPT

a. Pericarditis
b. Aortic Dissection
c. Pulmonary embolism
d. Pneumothorax
58 What is the typical pattern of chest pain in
stable angina pectoris
a. Immediately reaches peak intensity within
seconds
b. Prolonged and persistent for 1-3 hours
c. Crescendo- decrescendo pattern,
developing over 1-10mins
d. sharp , fleeting pain lasting for seconds

59. Which of the ff patients with chronic stable


angina is indicated to undergo coronary
angiography?
A. Pts with minimal and frequent chest pain
relieved by nitrates
B. Pts who survived from cardiac arrest
C. Pts with positive ischemic response on
stress testing
D. Pts with concomitant hypertrophic
obstructive cardiomyopathy

60. A 55 year old, seafarer, male,consults at your


clinic with the typical symptoms of angina but have
a normal resting electrocardiogram, thus you made
him undergo treadmill exercise. Which of the
following is an ischemic response to ECG?
a. Downsloping ST segment depression of
0.3mV from the baseline
b. Flat ST segment depression of 0.05mV
from the baseline
c. T wave formation
d. Upsloping ST segment depression of
0.2mV from the baseline

61. Which of the ff is FALSE regarding the


presence of ischemic symptoms in relation to
degree of epicardial coronary artery obstruction?
a. <50% postprandial chest pain
b. 50% - restriction in 02 supply
c. 75% - chest pain on exertion
d. 90% - chest pain at rest
62. In which subgroup of patients is ‘atypical
angina’ equivalents is more likely to occur?
A. Athletes
B. Diabetes
C. Smokers
D. Underweight males

Harrisons p 1581

63. Which of the following conditions should NOT


undergo a treadmill exercise test?
A. Acute myocarditis
B. Angina at rest which occured 1 week prior
C. History of treated infective endocarditis
D. Mitral regurgitation

Harrisons p. 1582 Stress Testing -


Electrcocardiogram

Contraindications to exercise stress testing include:


● rest angina within 48 h,
● unstable rhythm,
● severe aortic stenosis,
● acute myocarditis,
● uncontrolled heart failure,
● severe pulmonary hypertension,
● and active infective endocarditis.

64 Which of the following presentations of chest a. GERD


discomfort should undergo further testing for
ischemic heart disease. b. Pericarditis
a. Chest pain lasting 30 mins associated with
postprandial recumbency
b. Chest pain relieved by sitting forward and c. Costochondritis
leaning forward
c. Chest pain with swollen, tender, and warm
costochondral joints
d. Chest pain radiating to the right shoulder
triggered by exertion
65 Which of the ff statements regarding the
approach to a patient w/ chest discomfort is true?
A. Severity of chest discomfort has a high
diagnostic accuracy for Ischemic heart
disease
B. Angina typically presents with
highly-localized pain
C. Radiation of chest pain to the trapezius
ridge does not usually occur with angina
D. Intensity of angina usually changes with
position or upper extremity movement

66. These can initiate angina, except


A. Anemia
B. Hypothyroidism
C. Aortic valve stenosis
D. Hypertrophic myocardiopathy

67. Risk factor modification in managing ACS


a. Smoking cessation
b. Limitation of activity
c. Tx of LDL to <130mg/dl in Type 2 DM
patients
d. Dietary modification to increase saturated
fatty acids in the diet

68. True about adaptation of physical activities in B. isometric exercises are DISCOURAGED
chronic stable angina: C. Goal of the therapy is to allow px to eventually
A. Exercise testing can help the clinician create an have a relatively normal lifestyle
exercise prescription D. Physical limitation during early mornings and
B. Isometric exercises such as weight lifting, and after meals are encouraged.
planking should be encouraged
C. Tasks that evoke angina pectoris should be
mavoided or eliminated if at all possible
D. Due to variation of tolerance, relatively heavy
physical activities are best done in early mornings

69. Relative contraindication of beta blockers in


chronic stable angina except
A. Post-MI
B. Severe bradycardia
C. Raynaud’s Phenomenon
D. Reversible airway obstruction

70 True of Long acting nitrates


A. Can be given as sublingual tablet
B. As effective as sublingual nitroglycerin for
acute relief of angina
C. Transdermal patches can be applied
everyday, continuously without intervals
D. Doing free intervals of at

71. True of CCBs in the Tx of Angina pectoris


EXCEPT
a. As effective as B-Blockers
b. Negative inotropic effects may aggravate
LV failure
c. May be given when B-Blockers is
contraindicated or not tolerated
d. Verapamil is ideally given concomitantly
with BBs for additive effect
72. Drugs for symptom relief that improve survival
after MI.
a. Nitrates
b. Morphine
c. BBs
d. CCB

73. Which of the following are FALSE of Ischemic


heart disease in women.
a. Atherogenic risk factor is higher in
postmenopausal
b. The effect of Beta blocker and
anti-cholesterol is same as to male and
female
c. DM is high risk for Coronary heart disease
d. Low dose LDL lowering drug is effective to
the post-menopausal women than men

74-76 CASE: A 67-year old, male came in at the


ER due to sudden onset of palpitation followed by
severe chest pain while the patient was watching
TV. This is the first time he has ever experienced
having chest pain. He is hypertensive and diabetic
and has been smoking for 30 years. 12 lead ECG
is normal.

74. Diagnosis
A. Stable Angina
B. Unstable Angina
C. Acute Myocardial Infarction
D. Supraventricular tachycardia

75.
Which is not included as a possible mechanism in
causing the condition the case stated above?
A. Severe mechanism obstruction due to
progressive coronary atherosclerosis
B. Dynamic obstruction e.g. coronary spasm
C. Increased myocardial oxygen demand
produced by tachycardia on the presence of
fixed epicardial coronary obstruction
D. An imbalance between oxygen supply and
oxygen demand resulting from a totally
occluding thrombus forming on a disrupted
atherothrombotic coronary plaque

76. Which of the ff is TRUE regarding criteria for


the diagnosis of this patient:
a. It occurs at rest (or with minimal exertion)
lasting <10 minutes
b. It is of relatively recent onset
c. It occurs with a crescendo decrescendo
pattern
d. All of the above
77-78 CASE: A 45 year old male was admitted at
the ER due to sudden severe chest pain
associated with diaphoresis.

77. What is the possible diagnosis for the patient?


a. Stable angina Harrison’s p. 1600
b. Unstable angina
c. STEMI
d. NSTEMI

78. The following is false regarding the diagnosis


for this patient:
a. Usually occurs when coronary blood flow
decreases abruptly after a thrombus
occlusion of a coronary artery
b. Occurs in coronary arteries previously
affected by atherosclerosis
c. Occurs when coronary artery thrombus
develops rapidly at a site of vascular injury
d. Slowly developing, high-grade coronary
artery stenoses typically precipitates STEMI
due to collateralization

79. Which of the ff is false regarding aspirin in the


mgt of pts w/ suspected STEMI:
a. Aspirin is effective across the entire
spectrum of acs
b. Gradually​ inhibits cox1 in platelets followed
by a reduction of thromboxane a2 levels
c. Rapid action is achieved by buccal
absorption of a chewed 160-325mg tab in
ED
d. Aspirin should be subsequently given by
daily oral administration of aspirin in dose of
75-162mg
80. The ff is false regarding oxygen therapy in
patients with STEMI:
a. In patient whose arterial 02 saturation isi
normal, supplemental O2 is of limited if any
clinical benefit and therefore is not
cost-effective
b. O2 should be administered when
hypoxemia is present
c. O2 is given by nasal prongs or face mask
(2-4L/min)
d. O2 is given for the first 24 hrs after
infarction and should then be reasssured to
determine if there is a continue need for
such treatment

81. The typical initial dose of clopidogrel for acute


coronary syndrome:
a. 75mg
b. 150mg
c. 200mg
d. 300mg

82. The ff is true regarding adjunctive therapies


given to STEMI:
a. HMG-CoA reductase inhibitors (statins)are
given once lipid profile results show
elevated LDL levels
b. Administration of intensive dose of
atorvastatin 80mg/d (?) is done, prior to PCI
c. Statins may be discontinued once LDL level
is normal to reduce complications
d. All of the above
“Management of ACS.pdf” ppt ni Doc Jill
SIide #87

83. What is the standard antiplatelet agent for


patients with STEMI:
a. Aspirin
b. Clopidogrel
c. Prasugrel
d. Ticagrelor

84
What is the standard anticoagulant agent used in
acute MI?
A. Warfarin
B. Unfractionated heparin
C. Enoxaparin
D. Fondaparinux

85. Which of the ff analgesic is safe and effective


for the chest pain associated with STEMI?
a. Ketorolac
b. Paracetamol
c. Tramadol
d. Morphine

86 What is the initial typical dose of aspirin in the


setting of acute MI?
a. 325 mg
b. 80 mg
c. 162 mg
d. 75 mg

87. The indication for coronary revascularization in


stable angina pectoris include

A. Symptoms persisting despite medical


therapy
B. Symptoms accompanied by evidence of
ischemia during stress test
C. Both a and b
D. None of the above

88 What is a relative contraindication to


fibrinolysis?
A. HPN (systolic arterial pressure >180 mmHg
and/or a diastolic pressure >110 mmHg)
B. Active internal bleeding
C Bleeding diathesis
D. Cerebrovascular haemorrhage

89. What is false regarding cardiac biomarkers?


A. CKMB - earliest detected cardiac marker in
AMI
B. Troponin - remains elevated ep to 2 weeks
C. CKMB - used to assess for reinfarctionn
D. Troponin - does not elevate due to
non-cardiac causes

90 Relative contraindications to the use of


streptokinse:

a. Marked hypertension with SBP>180mmHg


and/or DBP>110mmHg
b. History of cerebrovascular hemorrhage at
any time
c. Non hemorrhagic stroke within the past
year
d. Know bleeding diathesis

91 Absolute contraindication for streptokinase


a. pregnancy
b. recent use of anticoagulant
c. Suspected aortic dissection
d. active peptic ulcer

92
To provide relief and prevention of recurrence of
chest pain, initial treatment should include bed rest,
nitrates, B-blockers, inhaled oxygen even without
hypoxemia
A. True
B. False
93. Nitrates should be given sublingually or buccal
spray in patient experiencing chest pain.
A. True
B. False

94
Intravenous nitrogylcerin is recommended if pain
persists after 3 doses of nitrates were given 5min.
Apart
A. True
B. False

95 A condition characterized by left ventricular


hypertrophy in the absence of causative
hemodynamic factors
a. ? cardiomyopathy
b. Restrictive cardiomyopathy
c. Dilated cardiomyopathy
d. Hypertrophic cardiomyopathy

96. Characteristic chest pain of pericarditis that


could differentiate it from pain of MI:
A. Usually relieved by nitrates
B. Radiates to the left arm, neck and left
shoulders
C. Usually relieved by sitting up and leaning
forward
D. There is no change in character in the
supine position
97
Most common toxic agent associated with chronic
dilated cardiomyopathy
A. Alcohol
B. Lead poisoning
C. Mercury overexposure
D. Anthracyclines and other chemo drugs

98. Clinical manifestation of Restrictive


Cardiomyopathy
a. positive Kussmaul's sign
b. pulmonary congestion occurs early
c. pedal edema and hepatic congestion is a less
common finding
d. A cardiac apex beat noted at the 6th ICS,
midaxillary line

99. Which of the ff. is FALSE regarding the It should be​ hypovolemia​.
possible complications of Acute Myocardial
Infarction? SOURCE: Kasper, D., et.al (2015) - Harrison's
Principles of Internal Medicine (19th Ed)
A. Ventricular Dysfunction - causes more pp.1607-1610
hemodynamic impairment, more heart
failure, poor prognosis
B. Hyper​volemia - contribute to
C. hypotension and vascular collapse
associated with STEMI
D. Arryhthmias - Autonomic Nervous System
imbalance, electrolyte disturbances,
ischemia, slo conduction in zones of
ischemic myocardium
E. Pericarditis - pericardial friction rub and/or
pericardial pain frequently associated with
patients with STEMI involving the
epicardium ----------------------------------------------------------

100. Which of the following is FALSE regarding the


treatment of arrhythmia in Acute Myocardial
Infarction?
A. Serum potassium concentration should be
adjusted to approximately 4.5 mmol/L and
magnesium to about 2.0mmol/L
B. Sustained ventricular tachycardia that is
well tolerated hemodynamically should be
treated with an intravenous amiodarone
C. Treatment of sinus bradycardia is indicated
if hemodynamic compromise results from
the slow heart rate
D. Routine prophylactic antiarrhythmic drug
therapy like lidocaine is recommended
CLINICAL

1. Renal causes of NaCl loss.


EXCEPT:
a. Gain of function of
mineralocorticoid
receptor
(Mineralocorticoid=
Aldosterone= retain
sodium, excrete
potassium)
b. High filtered load of
endogenous solute
c. Pharmacologic Diuretics
d. Tubulointerstitial injury

2. Which of the following is true


regarding the diagnostic
evaluation for hypovolemia?
a.
b.
c.
d. The reduction in both
GFR and distal Na
delivery can cause
increase in plasma K
concentration due to
defect in renal
potassium excretion

3. The following are common


mechanisms of hyponatraemia
EXCEPT:
a. Increase AVP and
sensitivity of kidney to
AVP
b. Increase free water
c. Increase solute intake
d.

4. In the treatment of
hyponatremia, which of the ff is
the predictor of rapid increase in
plasma sodium concentration
with saline infusion?
a.
CLINICAL
b. Urine sodium
concentration is low
<20mM
c.
d.

5. Which of the ff diuretics reduce


the ability to concentrate urine
by blunting the counter current
mechanism?
a. Loop diuretics

6. Which of the ff conditions


presents with hypovolemic
hyponatremia with urine sodium
concentration >20mM?
a. Mineralocorticoid
deficiency
CLINICAL
7. Which of the ff is most affected
in osmotic demyelination
syndrome?
a. midbrain
b. Medulla Oblongata
c. Pons
d. Cerebrum

8. Which of the ff is true regarding


the management of
hypernatraemia?
a. Water should be
administered by IV route
as the most direct way of
providing free water.
(​SHOULD BE ENTERIC​)
b. Plasma sodium
concentration should be
aggressively corrected
(​NOT BE
AGGRESSIVELY
CORRECTED)
c. Patients can receive free
water with IV solutions as
lactated ringers ​(LR IS
NOT A FREE WATER,
IS AN ISOTONIC
FLUID!)
d. Thiazides -MAY
REDUCE POLYURIA
DUE TO
NEPHROGENIC
DIABETES INSIPIDUS
BY INCREASING
CLINICAL
PROXIMAL TUBULAR
WATER
REABSORPTION .

9-13 Write Si ​Cushing Syndrome-​-- unsa man ni siya? Diba mag HIGH
C if the given item ​INCREASE Renal ALDOSTERONE? So unsay fxn ni aldosterone?--- diba sa
Loss​ of Potassium; SODIUM? Soooo KINSAY PARTNER ni sodium if magtransport na
siya? Diba si Potassium? Soooo diba everytime na ma reabsorb si
sodium may kelangan lumabas na potassium,----- so yeah mao ni
D if ​DECREASE Renal Loss ​of
siya
Potassium:
9. Cushing Syndrome ​( C ) Diba ang ​renal artery stenosis​ kay makacause ug
HYPERTENSION? So how? Sooo by stimulating RENIN release---
unsa may effect ni renin sa imong sodium? (Always jud kay kay
10. Unilateral renal artery stenosis ( sodium kay knowing na partner niya si K+ na ipagawas) soooo ang
C ) -INC RENIN PRODUCTION buhaton ni renin kay diba salt and water retention? --- so with that
NUNG KIDNEY NA ganahan man niya ireabsorb si sodium ----- so same fate ni K+
STENOSED, INC EXCRETION ---mas makagawas siya
IN CONTRALATERAL KIDNE​Y
NSAID​--- asa man siya mu act? Diba kay
prostaglandins(VASODILATE)? Unya ang effect ni nsaid kay prosta
11. NSAID ​( D ) kay inhibition man sooo CONSTRICTION soooo unsa nay nahitabo
sa imong Renal Blood Flow? Diba decrease ---then lead siya to
12. Distal Renal Tubular Acidosis decrease GFR---- unsa man dayun naa ani? Bali gamay lang ang
(RTA 1) ​( C ) filtered sodiumm-- so mi less si Na reabsorption---- gamay ang
napasulod--- for sure gamay sab ang mapagawas hahahaha
13. Diabetes Mellitus ​( D )
As for ​distal RTA​ hinumdumi lang si Porshia ​Acid​io---
HYPOKALEMIC SIYA bahalag acidosis… tungod kay grabe iyang
renal LOSS sa K+ naghypokalemia siya

Ang sa ​DM​ kay hahahaha hinumdumi lang ang role ni INSULIN


hahahah (si insulin kay mu bind siya with K+ para isulod siya balik
sa SULOD SA CELL)
CLINICAL
14. Hypocalcemia occurs in
conditions assiciated with severe
tissue injury such as
pancreatitis, burns, and
rhabdomiolysis due to the ff
mechanisms, EXCEPT:
a. Vitamin D production

b. Hypoalbuminemia
c. Hyperphosphatemia
d. Tissue deposition of
calcium
B-D occurs in SEVERE TISSUE
INJURY LIKE patients with
rhabdo, like RENATO!
CLINICAL
15. Which of the ff is an ECG
manifestation of Hypercalcemia?
a. SHORTENED QT
interval-
HYPOCALCEMIA??
b. flattening of T waves-
HYPOCALCEMIA
c. widened QRS-
HYPOKALEMIA
d. Bradyarrhythmia
CLINICAL

16-21 Matching Type : Match the given


condition with its expected metabolic
acid-base disturbance.
i. HAGMA
ii. NAGMA
iii. Metabolic
Alkalosis
16. Lactic Acidosis ​(HAGMA)

17. Diarrhea ​(NAGMA)

18. Cushing’s Syndrome ​(MELK)

19. RTA type 1 ​(NAGMA​)

20. Vomiting ​(MELK​)

21. Starvation ​(HAGMA)


CLINICAL
CLINICAL
22. In metabolic acidosis the fall in
pH is accompanied by acosteric
characteristic increase in
ventilation specially the tidal of
volume called
a. Kussmaul respiration

Harrisons pp 317

23. The ff causes metabolic


alkalosis, hypertension
EXCEPT:
a. Barter’s Syndrome-
RELATIVELY
HYPOTENSIVE

b. Hyperaldosteronism
c. Liddle’s Syndrome
d. Licorice == mimics
Liddle’s Syndrome
CLINICAL
24. What is the related electrolyte HypoK : Alkalosis
disorder associated with HyperK : Acidosis
metabolic alkalosis?
a. Hypokalaemia

25-27. A.L, 82y/o female was rushed to


the ER due to generalized weakness
with a history of days of fever
associated with productive cough. VS:
Drowsy BP: 70/40 mmHg; HR: 108
bpm; RR: 30 cpm; Temp: 38.7. PE:
Coarse crackles on the right mid to
lower lung field with blood ABG: Na:
145; Cl:110; K:4.8; pH:7.35; pO2:88;
CO2:28; HCO3: 19

25. What is the primary acid-base


disorder?
a. HAGMA
Ppt ni Doc Garingagirl
26. What underlying condition “ABG INTERPRETATION”
caused the primar acid-base
disorder? RESPIRATORY ALKALOSIS is due to post hypercapnia
a. Lactic Acidosis = (hyperventilation)
Hypotensive

27. What is the accompanying


acid-base disorder?
CLINICAL
a. Respiratory Alkalosis

LM, 21 y/o female, with/is a known case


of type 1 diabetes with poor compliance
to insulin. She was rushed to the ER
due to complains of nausea, vomiting
and abdominal pain which started a few
hours prior to admission. She was seen
with the ff examination findings: weak
looking, borderline BP of 90/50mmHg,
Tachycardic, Tachypnic, Temp: 37.4
with poor skin turgor. Lab results
random blood sugar: 500; serum
creatinine 1.5; BUN: 34mg/dL; Na:148;
K:5.5; Cl:112; ABG>> pH: 7.25; CO2:
22; pO2: 112; HCO3:9

28. What is the primary acid-base


disorder?
a. HAGMA

29. What is the most likely cause of Diabetic Ketoacidosis - complication of Type 1 DM
LM’s acid-base disorder? (insulin-dependent DM)
a. DKA
CLINICAL
30. What is the mainstay
management for LM?
a. Insulin therapy ⇒ pH di
pa abot ang
qualification for Bicarb
therapy

Harrisons pp. 318

B.B is a 9 y/o male who came for


consultation due to bipedal edema. 3
days ago started having fever and
cough​. Yesterday his urine was noted
by his mother to be tea-colored and
noted to have pitting edema on both
lower extremities. He had a history of
recurrent episodes​ of ​tea-colored urine
that would occur few days ​after
respiratory infection​ that would
spontaneously resolved. His PE findings
are BP: 104/90, (++)bipedal edema.

31. What is the most likely diagnosis


of B.B?
a. IgA Nephropahy
CLINICAL
32. What is the pathologic findings?
a. IgA deposition in
mesangium

33. What is the treatment for


hypertension in IgA
nephropathy?
a. ACE inhibitors

34. Which of the ff secondary


glomerular nephritis presents
with nephrotic syndrome
a. Lupus nephritis ⇒ Class
2
b. Hypertensive
nephrosclerosis
c. Diabetic nephropathy
d. Postinfectious
Glomerulonephritis

*nephritic pa ang ubang choices


CLINICAL
35. Which of the ff is an indicator of
presence of diabetic
nephropathy?
a. Thickening of
glomerular basement
membrane

36. Which of the ff is true about the


clinical manifestation of
POSTSTREPTOCOCCAL GN?
a. Acute nephritic syndrome
develops 2-3 wks after
an incident of
streptococcal pharyngitis
⇒ 1-3 WKS dapat
b. Urinary protein
excretion and
hypertension usually
normalizes by 4-6 wks
⇒ true 3 to 6 weeks sa Harrisons. pp1837
CLINICAL
c. Encephalopathy
presenting as new onset
seizures after altered
mental status and
headache usually
develops due to uremia
⇒ false due to
hypertensive
encephalopathy
d. Peripheral edema results
from concomitant
nephrotic syndrome
==>False

37. POST STREPTOCOCCAL GN


Occurs weeks after antecedent
streptococcal pyoderma?
a. C. 3-6 wks

Harrison 20ed page 2134


Murag dili harrison ang source diri 😔😪

38. Corticosteroid have a role in


treatment of the ff conditions
EXCEPT?
a. POSTSTREPTOCOCCA
L GN
b. IgA Nephropathy IgA Nephropathy
c. Good Pasteur’s
d. Rapidly Progressive
Nephritis/Nephropathy

Goodpasture

RPGN
CLINICAL
39. The ff statements are true about
AKI EXCEPT
D​. ​AKI recquiring
dialysis are not
increased risk for the
later development of
dialysis recquiring
end-stage disease​.
Harrisons pp 1799

40. The ff. Statements are NOT


TRUE about contrast
nephropathy:
C.​ ​The most common
clinical course of
nephropathy is
characterized by
increase serum
creatinine beginning
24-72 hrs following
exposure peeking
within 5-7 days and
resolving within 1 wk​.
(resolves on almost 2
wks d apat)

Harrisons pp 1803

It resolves within 1 week talaga. Nalito lang c Doc. Ang false dun is
24-72 hours. It should be 24-48 hours tsk tsk!
CLINICAL
41. AKI is defined by the following
criteria EXCEPT:
a. Increased serum
creatinine level from
baseline of ​<0.3mg/dL
within 48 hr​s
b. Increased serum
Harrisons pp. 1805
creatinine by at ​least
50% higher than baseline
within 1 wk
c. Reduction in urine output
to less that ​0.5mk/kg/hr
for longer than 6 hrs

d. None of the Above

42. According to the kidney disease


improving global outcomes
guidelines, patients with AKI in
hypercatabolic state receiving
continuous renal replacement
therapy should have a protein
intake of
a. 1.7g

44. Which of the ff is the


pathophysiology of the uremic
syndrome?

a. THOSE CONSEQUENT TO
THE A​CCUMULATION OF
TOXIN​ NORMALLY
UNDERGOING RENAL
EXCRETION INCLUDING
PRODUCTS OF PROTEIN Harrisons pp 1814
METABOLISM

b. THOSE ​CONSEQUENT TO
THE LOSS OF OTHER KIDNEY
FX​ SUCH AS FLUID,
ELECTROLYTES AND
HORMONE REGULATION

c. PROGRESSIVE SYSTEMIC
CLINICAL
INFLAMMATION AND
VASCULAR AND
NUTRITIONAL
CONSEQUENCES

d. ALL OF THE ABOVE

45. HYPERKALEMIA IS SEEN IN CKD


AND MAYBE PRECIPITATED IN SORT
OF SETTINGS INCLUDE, EXCEPT:
A. PROTEIN CATABOLISM
B. TRANSFUSION OF STORED
RBCS Harrisons pp. 1814
C. METAB ALKALOSIS
D. HEMORRHAGE OR
HEMOLYSIS

46. IN CKD PTS, CURRENT


PRACTICE IS TARGETED AT WHAT
LEVEL OF HGB CONCENTRATION?
A. 9-11.5
B. 9.5-11.5
C. 10-11.5 Harrisons pp. 1818
D. 10.5-11.5
100 - 115 g/L = 10 - 11.5 g/dL

47. FF CONDITIONS HAVE UTZ


FINDINGS FOR KIDNEY SIZE MAYBE
NORMAL IN THE CKD, EXCEPT:
A. DIABETIC NEPHROPATHY
B. ACUTE GN
C. HIV NEPRHOPATHY
D. AMYLOIDOSIS

Harrisons pp 1820

48. WHICH OF THE FF ​CALCIUM


CHANNEL BLOCKERS​ IS ​ANTI
PROTEINURIC AND
RENOPROTECTIVE​ EFFECT
A. AMLODIPINE
B. Harrisons pp. 1821
C.
D. VERAPAMIL- ​USUALLY
NONDIHYDROPYRIDINES
CLINICAL
49. IN CKD, PT WITH DM OR
PROTEINURIA MORE THAN 1G FOR
24H, BP SHOULD BE CONTROLLED
UP TO:
A.
B. Harrisons pp 1817
C. 130/80MMHG

50. THE CRITERIA FOR INITIATING


PT TO A MAINTENANCE DIALYSIS
AS FOLLOWS, EXCEPT:
A. BLEEDING DIATHESIS
B. HYPERKALEMIA
C. ESTIMATED GFR BELOW 15
D. PERSISTENT ECF VOLUME
EXPANSION

Harrisons pp. 1822

51. HEMODIALYSIS MAINLY RELY ON


PRINCIPLE OF:
A. DIFFUSION

*​OSMOSIS​ ANG SA PERITONIAL


DIALYSIS

Harrisons pp 1822

52. COMPLICATIONS OF
PERITONEAL DIALYSIS, EXCEPT:
A. CATHETER ASSOCIATED
PERITONITIS INFECTION
B. GAINING WEIGHT
C. METABOLIC DISTURBANCES Harrisons pp 1825
D. ANSWER hypotension-- Ang hypotension kay sa hemodialysis

53. URINARY TRACT OBSTRUCTION,


W/C OF THE FF IS TRUE ICS/BPS:
A. ...
B. THE CARDINAL SYMPTOMS
ARE PAIN, URGENCY, FREQ,
AND NOCTURIA
CLINICAL
Harrison 20ed page 286

54. WHAT IS THE DISTINGUISHING


CHARACTERISTICS OF ICS FROM
OTHER CAUSES OF PELVIC PAIN:
A. ...
B. BLADDER FILLING
EXACERBATE THE PAIN OR
BLADDER EMPTYING
RELIEVES IT

55. NOT TRUE IN THE TX OF ICS


A. THE GOAL OF THERAPY IS
TO CORRECT THE ANATOMIC
CAUSE ​- MORE ON
CONSERVATIVE MGT

56. ACQUIRED INTRINSIC DEFECT


OF THE URETER CAUSING UTO:
A. PREGNANCY
B. ….
C. CALCULI
CLINICAL
57. W/C IS A HEMODYNAMIC EFFECT
OF ACUTE BILATERAL URETERAL
OBSTRUCTION:
D. DECREASED GFR

*basta bilateral

Harrison 20ed page 2174

58.75YO, MALE, WITH HX OF


RECURRENT FLANK PAIN AND
GOUTY ARTHRITIS PRESENTED AT
ER DUE TO EXCRUCIATING FLANK
PAIN RADIATING TO THE LOWER
ABDOMEN, WC OF THE FF IS LIKELY
CASE OF HIS SX:
A. VESICOURETERAL REFLUX
B. ...
C. ...
D. ACUTE SUPRAVESICAL
OBSTRUCTION Harrisons pp.1872
CLINICAL
59. PT SUDDENLY DEVELOPED
ANURIA AND INSERTION OF
CATHETER NO URINE WAS NOTED.
WC OF THE FF IS TRUE REGARDING
DIAGNOSTIC APPROACH TO THIS
PT?
A.
B. UTZ SHOULD BE DONE TO
DETECT HYDRONEPHROSIS

Harrisons pp. 1873

60. PARTIAL BILATERAL UTO OFTEN


RESULT TO IN THE FF
CONSEQUENCES.
A. DISTAL RTA
B. HYPERKALEMIA
C. RENAL SALT WASTING
D. AOTA Harrisons pp 1872

61. WC OF THE FF IS FALSE


REGARDING POST OBSTRUCTIVE
DIURESIS:
A. RELIEF OF BILATERAL UTO
COMMONLY RESOLVE IN
POLYURIA
B. URINE OUTPUT MAYBE Harrisons pp 1874
MASSIVE
C. 3URINE IS USUALLY
HYPERTONIC
D. URINE MAY CONTAIN LARGE
AMT OF SODIUM AND CL, K,
PO4 AND MG
CLINICAL
62. PROGNOSIS OF UTO DEPENDS
ON WC OF THE FF FACTORS:
A. REVERSIBILITY OF RENAL
DAMAGE
B. BILATERAL VS UNILATERAL
C. PRESENCE OF UTI
D. AOTA
Harrisons pp 1874

63. WC OF STATEMENTS
REGARDING BPH IS TRUE:
A. BPH IT DEVELOPS TO
ANATOMICAL PERIPHERAL
ZONE- FALSE SHOULD BE
CENTRAL ZONE
B. BPH PREDOMINANTLY
COMPOSED OF SMOOTH
MUSCLE AND AS EXCELLENT
RESPONSE TO ALPHA
BLOCKERS Harrison 20ed, 631
C. PROSTATE SIZE ON DRE
CORRELATES WELL ON BPH
SYMPTOMS- FALSE
D. CORRELATES WELL ON
ESTROGEN ETCCCCC..

64. 63YO MALE PX CONSULTS YOU


FOR OBSTRUCTIVE SYMPTOMS
CONSISTENT WITH BPH. YOU
ADMINISTERED THE
INTERNATIONAL PROSTATE
SYMPTOMS SCORE AND GOT A
SCORE OF 5. WHAT IS THE
APPROPRIATE NEXT COURSE OF
ACTION?
A. WATCHFUL WAITING
CLINICAL
JJ., A 69 Y.O MALE DIAGNOSED
WITH BPH CONTEMPLATING
MEDICAL MANAGEMENT.
PERTINENT PAST HX INCLUDES A
HEART 3 YEARS AGO.

65. WHICH OF THE FF. ALPHA


BLOCKERS IS MOST SUITABLE TO
MINIMIZE THE RISK OF SYSTEMIC
SIDE EFFECTS?
C​. TAMSULOSIN - LESS EFFECT ON
THE CARDIOVASCULAR

66. JJ IS ALSO STARTED ON 5


ALPHA REDUCTASE INHIBITOR TX
FOR HIS CONDITION. WHEN
SHOULD HIS PROSTATE BE
REASSESSED TO DETERMINE
TREATMENT RESPONSE?

​ANSWER: 6 MONTHS

67. ON LONGER FF. UP, MR. JJ


SHOWED POOR TREATMENT
RESPONSE TO MEDICAL MGT AND
IS FINALLY SCHEDULED FOR TURP.
THE FF ARE POTENTIAL
COMPLICATIONS U SHOULD WARN
HIM ABOUT EXCEPT
A. DECREASED LIBIDO
B. BLADDER NECK
CONTRACTURE
C. RETROGRADE EJACULATION
D. HYPONATREMIA
CLINICAL
68. SHORTLY AFTER HIS PROSTATE
SURGERY WHICH LASTED FOR 2.5
HOURS, MR. JJ DEVELOPED ACUTE
CONFUSION, NAUSEA AND
VOMITING, AND BRADYCARDIA.
WHICH OF THE FF IS CONSIDERED
RISK FACTOR FOR HIS CURRENT
CONDITION?

ANS: B. THE USE OF HYPOTONIC


IRRIGATING SOLUTION​.

Smith p. 356

69. WHEN IS HIS TRANSURETHRAL


APPROACH NOT PRACTICAL IN BPH
PX UNDERGOING SURGICAL
MANAGEMENT?
A. WHEN PROSTATE IS 25
GRAMS IN SIZE
CLINICAL
70. THE FF ARE ESTABLISHED TO
INCREASE THE RISK OF PROSTATE
CANCER EXCEPT:
A. INCREASING AGE
B. OLDER AGE AND DIAGNOSIS
OF PROSTATE CANCER IN A FAMILY
MEMBER ​(increased risk if younger
mage diagnosis of family member)
C. HIGH DIETARY FAT INTAKE
D. AFRICAN AMERICAN
CLINICAL

71. WHICH ANATOMIC ZONE OF THE


PROSTATE DOES CARCINOMA
MOST OFTEN ARISE FROM?
ANSWER: PERIPHERAL ZONE -
KAYA THEY MAY NOT CAUSE Harrison 20ed, page 623
OBSTRUCTIVE SYMPTOMS
UNLIKE BPH - IN CENTRAL ZONE
CLINICAL
72. U HAVE AN ELDERLY WITH BPH
UNCLE WHO IS TREATED W
FINASTERIDE WHO ROUTINELY HAS
HIS PSA LEVEL CHECKED ONCE A
YEAR.DURING A PLANNED REUNION
HE BRINGS U HIS PSA RESULTS
FOR THE PAST 3 YEARS ALONG
WITH THE RECENT PROSTATE
ULTRASOUND RESULTS. HOW WILL
U ADVISE HIM REGARDING
INTERPRETING HIS PSA RESULTS?
A. PSA < 4 ng/mL CORRELATES
WITH ZERO RISK FOR
PROSTATE CANCER
B. BPH MAY RESULT IN FALSE
NEGATIVE PSA RESULT
C. ANNUAL INCREASE IN PSA BY 1
NANOGRAM PER ML PER YEAR
INDICATES RISK OF PROSTATE
CANCER
D.CONCOMITANT INTAKE OF
FINASTERIDE INCREASES PSA
LEVEL
CLINICAL
73. MAJORITY OF PROSTATE
CANCER SCREENING GUIDELINES
RECOMMENDED PSA SCREENING
SHOULD BE STARTED AT WHAT
AGE?
ANSWER: 50 YEARS OLD

Smilth 18th ed p366

74. THE FF CHARACTERISTICS


DISTINGUISH ACUTE VS CHRONIC
BACTERIAL PROSTATITIS EXCEPT
B. MORE PROMINENT PROSTATIC
EXAMINATION FINDINGS ON PE
WITH CHRONIC PROSTATITIS-
PAREHO LANG SILA NG ACUTE

75. STONE FORMATION REQUIRES PPT ni doc garingagirl- urinary stone disease
SUPERSATURATED URINE WHICH IS (basaha nlng didto ang uban hehe)
DEPENDENT ON THE FF FACTORS
EXCEPT:

B. MOLECULAR WEIGHT
CLINICAL

76.. WHICH OF THE FF IS TRUE


ABOUT URIC ACID?
A. ELEVATED PH LEVEL
INCREASES URATE - WHICH
IS SOLUBLE

77. WHICH OF THE FF IS THE


NATURAL INHIBITOR OF CALCIUM
CONTAINING STONES?

B. CITRATE
CLINICAL
78. THE FF ARE LOCAL
MECHANISMS THAT MAY
CONTRIBUTE TO THE PERCEPTION
OF PAIN IN PX WITH RENAL CALCULI
EXCEPT:
A. INFLAMMATION
B. EDEMA
D. HYPOPERISTALSIS ​- OR
HYPERPERISTALSIS
C. MUCOSAL IRRITATION

79. Infectious stone are associated with


urea splitting organisms which includes
the ff except:

E.coli

Match the stone in column A with


recommended treatment in column B.
80. Calcium Phosphate ⇒
Hydrochlorothiazide
81. Calcium Oxalate ⇒ ​Calcium
Supplementation
82. Uric Acid ⇒ ​Allopurinol
83. Cystic Stone ⇒ ​mercaptopropionyl
glycine
CLINICAL
83-85. True or False (A if True; B if
False)

83.High dietary calcium intake is related


to low risk of stone formation. ​(TRUE)
Supplementations such as tablets ang
high risk

84.Vitamin C supplementation
decreases​ the risk of stone formation
(FALSE)

85. High sodium and sucrose intake


decrease​ calcium secretion. (​FALSE)

Urinary tract infection Matching TYPE.


Match the ff complications of acute
pyelonephritis to their definition and
special circumstances where they are
usually encountered.

86. Seen mainly on diabetics and in


those with UTI involving one or all of the
renal pyramids and is usually bilateral
B. ​Papillary Necrosis
87. Seen in total or almost complete
obstruction located high to the urinary
tract with suppurative exudate of the
renal pelvis calyces and ureter.
C. ​Pyonephrosis
88. Extension of the suppurative
inflammation through the renal capsule
into the perinephric tissue.
A. ​Perinephric Abscess
CLINICAL
89. Which of the following is a renal
disorder affecting the tubules,
interstitium and renal pelvis which is
cause by bacterial infection in which the
renal legation is associated with UTI
A. ​ACUTE PYELONEPHRITIS

90. The following are risk factor for UTI l


among patient >65 years old EXCEPT!
C. ​Functional​ Genitourinary
Abnormalities

Incontinence, Catheterization and


Prostate Obstruction are all risk for UTI

91. Which of the ff causative pathogens


for UTI often result from ​hema​togenous
dissemination.
C. Staph aureus

92. What is the golden standard for


identification of UTI

URINE CULTURE
CLINICAL
93. The probability of UTI based on
urine culture is UNLIKELY in which
urine specimen
A. One specimen with >10^5 CFU in
female
B. >10^4 CFU in males
C. (1-5 x10^3mn ata) 10^3 in
asymptomatic females man ata ni.
WALA GI MENTION MY GERD.

mao daw ni ang naay lowest count if


icompare nimu to other choices
D. 5x10^5 CFU in females

94. Which of the ff duration of antibiotic


treatment is correct
a. Cystitis => 7 days (3 days LNG)
b. Pyelonephritis = 7-10 days
c. Chronic Prostatitis = 2wks
(FALSE)
d. Epididymitis = 4-6 wks (FALSE)

95. Which of the ff is false regarding IT DOESN’t resolve with antibiotics alone
emphysematous pyelonephritis?
B. ​patients presents with fever, flank
pain and vomiting that resolves with
initial management with parenteral
antibiotic
CLINICAL
CLINICAL
96. Which of the ff is a renal abscess
that extends beyond Gerotta’s fascia?
C.​ Paranephric abscess

97. HR 28 male went for consult for


three day burning pain, urination and
yellowish urethral discharge. Had
contact with a paid sexual worker two
days prior to the onset of symptoms. Dx
for Hr.
A. Urethritis
CLINICAL
98. Urethral swab for HR. Which of the
following findings will confirm diagnosis?
A. Gram negative intracellular
diplococci
CLINICAL
99. Tx of choice?
Cephalosporins

100. Co infection?
Chlamydia
CLINICAL
101. Tx of choice for non gonococcal
urethritis?
Macrolides

102. Following describes urethral


syndrome in women except?
b. ​Urinary Frequency

Harrison 20ed, page 980


CLINICAL
103. Fishy Odor with 10 percent KOH
Bacterial Vaginosis

104. Macroscopic findings of hyphae


Vulvovaginal candidiasis

105. Microscopic clue Cells


bacterial vaginosis

106. Profuse vaginal diischarge


Trichomonas

107. Clusters of ulcers painful papules


or vesicles
Herpes

108. Painless vascular red ulceration


Granuloma inguinale loop

109. Painful ulcers with suppurative


inguinal lymphadenopathy
Chancroid

110. Sharply demarcated nonpurulent


and superficial ulcer
Syphilis
CLINICAL

111. A 20 y/o, F came in d/t acne


vulgaris in the face w/c lesion is
described as ​hyperpigmented flat skin
lesion and measures <0.5 cm.
Macule
CLINICAL
112. A px presents w/ a ​well-defined,
solid, elevated skin lesion​ measuring
<0.5 cm. What is the primary skin
lesion?
Papule

113. A 62y/o F px came in d/t a


nonhealing wound on the right lower leg
Upon noting the lesion: extending into
the ​subcutaneous tissue
What is secondary lesion?
ULCER
CLINICAL
114. She prompts you if this lesion will
turn into a scar

Involvement up to the subcutaneous


tissue will always lead to scar
formation

115. While in ER, perform diascopy on


th epx who came in with purpuric
lesion…. And microscopic results to
NONBLANCHING lesion(what does
this mean)

Extravasation

116. Crepitant cellulitis


Clostridium

117. Ecthyma gangrenosum


Pseudomonas
CLINICAL
118.Carbuncle
S. aureus

119. Sub acute lymphangitis


NonTB myocardium

120. Toxic shock syndrome


S. aureus
CLINICAL
CLINICAL
2020 CLINICAL Questions
QUESTIONS RATIONALE

1. Polyuria is defined as a significant increase in 24- Harrison 20th, Ch 48, page 294
hour urine volume exceeding:

a. 1.5L
b. 2L
c. 3L
d. 4L

Bate 12ed, Ch11, page 462

2. Nocturia is defined as urinary frequency at night, Bate 12ed, Ch11, page 462
awakening the patient:

a. More than once


b. More than twice
c. More than thrice
d. More than 4 times

3. Which of the following is a cause of nocturia with Bates 12 ed, Ch 11, page 496
low urine volume?

a. Chronic renal insufficiency


b. Coffee intake at bedtime
c. Heart failure
d. Insomnia

4. The following disease entities presents with thirst, Bates 12th Ed, Page 496
polydipsia and nocturia, EXCEPT:

a. Primary polydipsia
b. Diuretic use
c. Uncontrolled DM
d. Diabetes insipidus
5. Match the different kinds of incontinence to its Bates Table 11-7 pg 498
cause

Poor urethral sphincter Stress incontinence


tone

Impaired cognition functional


incontinence

Uncontrolled detrusor urge incontinence


contractions

Anatomic obstruction over-flow


from pelvic organs or incontinence
prostate

6. Which of the following would present with an Bates Table 11-7 pg 498
enlarged bladder on physical examination?

a. Stress incontinence
b. Urge incontinence
c. Over-flow incontinence
d. Functional incontinence

7. Which of the following is FALSE about kidney


pain?

a. Visceral pain caused by distention of the renal


capsule
b. Dull, aching and steady
c. Originates from the costovertebral angle
d. Originates at or below the posterior costal
margin near the costovertebral angle

8. Which of the following physical examination Bates pg 481


finding on a left flank mass is highly suggestive of
an enlarged kidney?

a. Ability to probe with your fingers between


the mass and the costal margin
b. The edge of the mass extends beyond the
midline
c. Absence of the normal tympany in the left
upper quadrant
d. All of the above

9. Which of the following physical findings is Bates’ 12e p.501


consistent with renal artery stenosis?
A. epigastric bruit confined to systole
B. bruit in the right and left upper quadrant
C. bruit in the right and left periumbilical region
D. epigastric bruit with systolic and diastolic
components

10. A 20 y/o female in your satellite clinic due to acne


vulgaris on the face. Which lesion is described as
hyperpigmented, flat, and measures <0.5cm?
A. macule
B. papule
C. pustule
D. scar

11. A patient presents with a well-defined, solid,


elevated scaly lesion measuring <0.5cm. What is
the primary lesion?
A. macule
B. papule
C. patch
D. plaque

12. A patient presented with multiple fluid-filled, dome


shaped, elevated lesions measuring <0.5cm. The
primary lesions are:
A. vesicles
B. bulla
C. cysts
D. nodules

13. the following are primary lesions EXCEPT:

a. Wheal
b. Macule
c. Patch
d. Erosion

14. In cases of urticaria, what lesions are described


as evanescent, round or flat topped, pale red
papules and plaques?

a. Bullae
b. Vesicle
c. Wheals
d. Patch

15. a 62 year old female patient came to your clinic


due to a non-healing wound on the right lower
leg. Upon looking at the lesions, you noted a
defect that seems to extend into the
subcutaneous tissue. What is the secondary
lesion being described?

a. Ulcer
b. Erosion
c. Fissure
d. Abrasion

16. the following are FALSE statements about


pruritus, EXCEPT:

a. It is only associated with diseases of the


integument
b. It is associated with severe renal failure
c. It is the 2nd most common cutaneous
symptom
d. The sensation is carried by myelinated C-
fibers

17. The ff are disorders of the melanocyte, EXCEPT


a. Vitiligo
b. Ephelides
c. Albinism
d. Sunburn

Ppt lecture

18. These are horny plugs at the under surface of a


scale
a. Auspitz sign
b. Carpet tack sign Ppt lecture
c. Apple-Jelly sign
d. Hutchinson sign
19. These are small elevations of the skin containing
purulent material
a. Vesicles
b. Bullae
c. Pustules
d. Crusts

20. The ff statements are true about diascopy,


EXCEPT
a. Done by pressing a hard, flat,
transparent object on the skin
b. Result is noted as blanching or non-
blanching
c. Apple - Jelly sign is noted if lesions
are granulomatous
d. Purpuric lesions blanch during
diascopy
B. mentioned ni doc

21. The following are shapes of individual lesions,


EXCEPT:

a. Blaschkoid
b. Nummular
c. Annular
d. Reticular

Nummular/round/discoid
Annular (ring shaped)
Reticular (net-like)
22. The following are distribution of lesions, EXCEPT:

a. Dermatomal
b. Acral
c. Serpiginous
d. Lymphangitic

23. The following are common mechanisms of


hyponatremia, EXCEPT

a. Increased circulating AVP


b. Increased renal sensitivity to AVP
c. Increased solute intake
d. Increased intake of free water
HPIM 20th Ed. p. 298

24. Which of the following diuretics reduce the ability


to concentrate urine by blunting the countercurrent
mechanism?

a. Potassium sparing diuretics HPIM 20th Ed. p.299


b. Thiazide diuretics
c. Loop diuretics
d. Carbonic anhydrase inhibitors

25. Which of the following conditions present with


hypovolemic - hyponatremia with urine sodium
concentration more than 20 mM?
a. Mineralocorticoid Deficiency
b. Third spacing
c. Diarrhea
d. Vomiting

26. Which of the following conditions present with


hypervolemic - hyponatremia with urine sodium
concentration more than 20 mM?
a. Chronic Renal failure
b. Nephrotic Syndrome
c. Cardiac Failure
d. Cirrhosis

27. Which of the following is most affected in osmotic


demyelination syndrome secondary to rapid
correction of hyponatremia?
a. Midbrain
b. Medulla Oblongata
c. Pons
d. Cerebellum

28. Choose whether the given item increases or


decreases renal loss of potassium:
● Cushing Syndrome: Increases renal loss of
potassium
● Unilateral renal artery stenosis: Increases
renal loss of potassium
● NSAIDs: Decreases renal loss of potassium
● Distal RTA: Increases renal loss of potassium
● Diabetes mellitus: Decreases renal loss of
potassium

29. Hypocalcemia occurs in conditions associated Harrison’s 20th ed pg 314 Hypocalcemia


with severe tissue injury such as pancreatitis,
burns and rhabdomyolysis due to the following
mechanisms EXCEPT?

a. Hypoalbuminemia

b. Impaired 1,25(OH)2 D production

c. Hyperphosphatemia
d. Tissue deposition of calcium

30. Which of the following is an electrocardiographic Harrison’s 20th ed pg 313 hypercalcemia


manifestation of hypercalcemia?

Select one:
a. Widened QRS
b. Flattening of T waves
c. Prolongation of QT interval
d. Bradyarrhythmia
31.
32. The following causes metabolic alkalosis with Harrison’s 20th ed pg 321 table 51-6
hypertension, EXCEPT?

a. Bartter’s Syndrome
b. Hyperaldosteronism
c. Liddle Syndrome
d. Licorice intake

33. E.L., an 82 year old female, was rushed to the pH - 7.35 - Acidosis
emergency room due to generalized weakness HCO3 - 19 - Reduced - Acidosis (24 -19 = 5)
with a history of 3 days of fever associated with pCO2 - 28 - Reduced - Alkalosis (40 - 28 = 12)
productive cough. Her vital signs are as follows: Metabolic Acidosis
Drowsy but opens eyes when called, BP: 70/40, Anion gap - 145 - (110 + 19) = 16
HR: 118 bpm, RR: 30 cpm, Temp: 38.7C. HAGMA
Physical examination reveals coarse crackles on
the right mid to lower lung field. Blood chemistry
and ABG was done with the following results:
Serum Na - 145, Serum Cl - 110, Serum
potassium - 4.8, pH - 7.35, pO2 - 88, PCO2 28,
HCO3 – 19. What is the patient’s primary acid-
base disorder?

Select one:
a. Respiratory Alkalosis
b. Non-anion gap metabolic acidosis
c. High-anion gap metabolic acidosis
d. Metabolic alkalosis

34. What underlying condition could have caused the


patient’s primary acid-base disorder?

Select one:
a. Pneumonia
b. Extracellular volume contraction
c. Depressed respiratory center
d. Lactic acidosis

35. What is the accompanying acid-base disorder


of the patient?

Select one:
a. Respiratory Alkalosis
b. Respiratory Acidosis
c. Metabolic alkalosis
d. High anion-gap metabolic acidosis

36. T.G. is a 70 year old male, a known case of pH - 7.34 - Acidosis


COPD with cor pulmonale. He is maintained on HCO3 - 42 - Increased - Alkalosis (42 - 24 = 18)
bronchodilators and furosemide with good pCO2 - 78 - Increased - Acidosis (78 - 40 = 38)
compliance. He came in due to 3-day history of Respiratory Acidosis
difficulty of breathing and decreased sensorium
few hours prior. Findings in the ER are as follows:
Stuporous, BP: 90/50 HR: 100 RR: 28 Temp:
afebrile, (+) expiratory wheezing with prolonged
expiratory phase of respiration, full equal pulses.
Labs were done with the following result: Serum
sodium 152 mmol/L, Serum Potassium 3.3
mmol/L, Serum Chloride 95 mmol/L, ABG: pH
7.34, PCO2 78, PO2 85, HCO3 42 What is the
primary acid-base disorder of T.G.?

Select one:
a. Respiratory Acidosis
b. Metabolic Acidosis
c. Respiratory Alkalosis
d. Metabolic Alkalosis

37. Which of the following most likely caused T.G.’s Doc Kath’s Lecture
primary acid-base disorder?
A. Pneumonia A. Pneumonia → can actually cause Resp. Acidosis
B. Depressed Medullary center
C. Exacerbation of COPD in acute setting but in this case dili pa sure kung naa
D. Decreased Extracellular Fluid Volume siyay pneumonia and mas likely nga COPD ang
cause sa wheezing (emphysema + chronic bronchitis
= narrowed airways = wheezing)
B. Depressed Medullary Center → can also cause
Resp. Acidosis in acute setting but walay noted nga
opiate, anesthetic or sedative use. Ang pinakaduol
nga cause kay chronic Hypoxemia but above 80%
man ang O2 sat.
C. Exacerbation of COPD → cause for Resp.
Acidosis in chronic settings may cause the previous
options. Known and diagnosed si patient base sa
case scenario.
D. Decreased ECF volume → causes Metabolic
Alkalosis. Pinakalayo nga option

38. What is T.G. secondary acid-base disorder? Calculated HCO3 compensation in Chronic
A. Respiratory Acidosis Respiratory Acidosis = 37.3
B. Metabolic Acidosis Patient’s HCO3= 42
C. Respiratory Alkalosis Therefore, renal compensation is too high =
D. Metabolic Alkalosis Metabolic Alkalosis

39. Which of the following has most likely caused A. GI losses → no noted GI losses in the case
T.G.s secondary acid-base disorder?
A. GI losses (like vomiting or diarrhea)
B. Diuretic Use B. Diuretic Use → patient took Furosemide (can
C. Lactic Acidosis
D. Renal Tubular Acidosis cause Metabolic Alkalosis by excreting many
solutes → Na losses depletes blood volume
which also triggers RAAS → Aldosterone
increases reabsorption HCO3)
C. Lactic Acidosis → causes metabolic acidosis.
D. Renal Tubular Acidosis → also causes
metabolic acidosis

40. L.M., 21F, is a known case of type I diabetes with DM causes High Anion Gap Metabolic Acidosis
poor compliance to insulin regimen. She was ABG: Metabolic Acidosis ( pH:7.25, Bicarbonate is
rushed to the ER due to complaints of nausea, decreased vs pCO2 (which dapat mu saka si PCO2
vomiting and abdominal pain, which started few if siya ang primary cause sa acidosis)
hours prior to admission. She was seen with the Anion Gap: 27 (HAG)
following examination findings: Weak looking, BP:
90/50, HR: 110 bpm, RR: 26cpm, Temp: 37.4C,
poor skin turgor. Laboratory results are as
follows: Random Blood Sugar: 515mg/dl, Serum
Creatinine: 1.5mg/dl, BUN: 34mg/dl, Serum
sodium: 148, Serum Potassium: 5.5 mmol/L,
Serum Chloride: 112, ABG: pH 7.25, PCO2 22,
PO2 112, HCO3 9

What is the primary acid-base disorder of L.M.?

A. Respiratory Alkalosis
B. HAGMA
C. NAGMA
D. Metabolic Alkalosis

41. What is the most likely cause of L.M.s acid-base LM has HAGMA
disorder? a. Hyperventilation is more of respiratory cause
a. Hyperventilation b. DK - HAGMA
b. Diabetic Ketoacidosis c. RTA - NAGMA
c. Renal Tubular Acidosis d. Hypoaldosteronism - NAGMA
d. Hypoaldosterone state

42. What is the mainstay management for L.M.? Harrison’s 20th ed pg 318
a. IV insulin therapy
b. IV bicarbonate therapy
c. Rebreathing into a brown paper bag
d. Hemodialysis

43. The following patients should be closely Harrison’s 20ed pg 2102


monitored due to relatively high risk of post- COPD is not mentioned
operative acute kidney injury, EXCEPT?
a. Massive intra-operative blood loss
requiring blood transfusion
b. coronary artery bypass graft
procedures
c. patients with congestive heart failure
d. patients with chronic obstructive
pulmonary disease

44. N.L, is a 56-year old female patient admitted for Harrison’s 20th ed pg 2103
moderate risk pneumonia. Her baseline
Creatinine has doubled on the third hospital day.
Which of the following drugs/agents are
commonly associated with an acute increase in
Creatinine?
a. Macrolide antibiotics
b. Chest CT scan with intravenous
contrast
c. Opiate pain relievers (ex. Tramadol)
d. H2-receptor blockers

45. L.R. is a 68-year old male patient with diabetes


who recently underwent emergency laparotomy
for ruptured appendicitis. His baseline weight is
60 kgs and his baseline creatinine is 0.8 mg/dL.
Which of the following developments in course is
strongly indicative of acute kidney injury?
a) Urine output of 1L for the past 24
hours
b) Repeat creatinine of 0.95 mg/dL after
2 days
c) Urine output of 20 mL per hour for the
past 8 hours
d) Repeat creatinine of 1.1 mg/dL after 5
days

46. In a patient with no history of serial serum


creatinine monitoring, which of the following
findings suggests chronic kidney disease as
opposed to acute kidney injury?
a) Proteinuria
b) Hyperkalemia
c) Secondary hyperparathyroidism
d) High-anion gap metabolic acidosis

47. An 18-year old college student who was involved Harrison, 20th ed. (AKI, “Urine Findings”, page
in a hazing incident suffered multiple hematomas 2106)
secondary to blunt trauma. After two days, he
suddenly developed oliguria and elevated A)
creatinine. Which of the following urinalysis
findings helps confirm your suspicion of
rhabdomyolysis?
a) Presence of oxalate crystals
b) Hematuria with dysmorphic RBCs
c) Eosinophiluria
d) Positive urine heme with absent RBCs

B)

C)
D)

48. F.C. is a 28-year old female with progressively Harrison, 20th ed. (Approach to Patient with
decreasing urine output after three days of Azotemia, page 291)
vomiting and diarrhea. She presents in the ER
with sunken eyeballs and poor skin turgor. Initial
labs include elevated creatinine results. The
following lab findings will help support your
assessment of pre-renal azotemia, EXCEPT:
a) BUN/Creatinine ratio <10
b) Fractional excretion of sodium (FeNa)
<1%
c) Urine specific gravity of 1.030
d) Hyaline casts in urinalysis

49. Renal ultrasonography usually shows normal


renal size in most causes of AKI, except?
a. Acute interstitial nephritis
b. Hepato-renal syndrome
Harrison’s 20th ed, page 2017
c. Acute glomerulonephritis
d. Contrast-induced nephropathy

50. H.M is a 60-year old patient with hypertension


who just underwent coronary angiography
followed by angioplasty. What is the most
common expected course of his serum creatinine
if he suffers from AKI after IV contrast exposure
a.Rise in 12 hours,peak in 1-2D, resolution within Harrison’s 20th ed.Page 2106
2 weeks
b.Rise in 48 hours,peak in 3-5D, resolution within
1 week
c.Rise in 1 week,peak in 2nd-3rd week, resolution
within 3 months.
d.Rise in 5 days, peak in 2 weeks,resolution in 1
month.

51. The following mechanisms are contributors to


sepsis-associated AKI.Except?
a. Decreased renal perfusion due to septic shock
B. metastatic infection to renal parenchyma
through the bloodstream
c.Excessive efferent arteriole vasodilation
D. Renal tubular damage from reactive oxygen
species.
Harrison’s 20th ed, page 2102

52. For which of the ff etiologies of AKI is kidney


biopsy indicated?
a. Bladder neck obstruction
b. Aminoglycosides
c. Hepato-renal syndrome
d. Myeloma Harrison’s 20th ed, page 2017

53. The following are expected metabolic Harrison’s 20th Ed. page 2108
complications of AKI, EXCEPT?
a. Platelet dysfunction Complications of AKI:
b. Hypocalcemia Uremia
c. Hypernatremia Hypervolemia & Hypovolemia
d. Malnutrition Hyponatremia
Hyperkalemia
Acidosis
Hyperphosphatemia & Hypocalcemia
Bleeding
Infection
Cardiac complications: Arrhythmias, pericarditis, &
pericardial effusion
Malnutrition

54. V.S, is a 42 years old male with alcoholic Harrison’s 20th Ed. page 2414
cirrhosis complicated by hepatorenal-syndrome.
Which of the following is considered definitive
treatment of his condition?
a. Silymarin+Silybin
b. Liver transplant
c. Albumin infusion
d. Renal transplant

55. Which of the following complications in AKI is Harrison’s 20th Ed. page 2110
correctly matched to the appropriate treatment?
a. Hyperphosphatemia = Calcium carbonate
b. Anemia = Erythropoietin injection
c. Hyperkalemia = Spironolactone
d. Metabolic acidosis = Aluminum hydroxide

56. What is the recommended daily protein intake of Harrison’s 20th Ed. page 2110
a patient with AKI who is ongoing dialysis?
a. 0.3-0.5g/kg/day
b. 0.6-0.8g/kg/day
c. 0.8-1.0g/kg/day
d. 1.0-1.5g/kg/day

57. Dialysis is already indicated in a patient with AKI


with the following manifestations, EXCEPT?
a. Hyperkalemia reduced by furosemide
b. Pericardial effusion
c. Encephalopathy
d. Metabolic acidosis despite
bicarbonate drip

58. The following are risk factors for Chronic Kidney


Disease (CKD) in patients with normal GFR,
EXCEPT?
a. Hypertension
b. Asian ancestry
c. Childhood obesity
d. Prior episode of completely recovered
acute kidney injury
59. Which of the following statements regarding
potassium homeostasis in chronic kidney disease
is NOT true?
a. Hyperkalemia in CKD may be due to
increased dietary intake or metabolic
acidosis
b. Conditions associated with
hyporeninemic hypoaldosteronism,
especially diabetes, are at associated
with earlier and higher levels of
hyperkalemia
c. Increased GI potassium excretion
helps defend CKD from hyperkalemia
d. RAS inhibitors should be voided in all
cases of advanced CKD due to risk of
hyperkalemia

60. What is the most useful imaging test in the initial


work-up of patients with CKD?
a. KUB-ray
b. Voiding cystogram
c. Renal ultrasound
d. Renal angiography

61. C.V. is a 50-year old female with diabetes and The most important medications in this respect
hypertension for almost 10 years, with her routine include the RAS inhibitors and spironolactone and
tests showing an eGFR classified as CKD Stage other potassium-sparing diuretics such as amiloride,
III. Aside from strict glycemic control, the eplerenone, and triamterene. The benefits of the
following anti-hypertensive drugs are preferred to RAS inhibitors in ameliorating the progression of
help delay CKD progression, EXCEPT? CKD and its complications often favor their
a. Metoprolol cautious and judicious use with very close monitoring
b. Enalapril of plasma potassium concentration.
c. Verapamil (Harisson’s 20th ed pg. 2114)
d. Losartan
a. Metoprolol
b. Enalapril (ACEi)
c. Verapamil (CCB)
d. Losartan (ARB)

62. Which of the following statements regarding Peripheral neuropathy usually becomes clinically
peripheral neuropathy due to chronic kidney evident after the patient reaches stage 4 CKD,
disease is TRUE? although electrophysiologic and histologic evidence
a. This complication becomes clinically occurs earlier. Initially, sensory nerves are
apparent by CKD Stage III involved more than motor, lower extremities
b. Upper extremities are more involved more than upper, and distal parts of the
than the lower extremities more than proximal.
c. Sensory nerves are more affected (Harisson’s 20th ed pg. 2118)
compared to motor
d. Proximal extremities are more affected
compared to distal

63. V.L. is a 34-year old female with Lupus Other contraindications to renal biopsy include
Erythematosus for the past 6 years but with poor uncontrolled hypertension, active urinary tract
follow-up; complaining of progressive pedal infection, bleeding diathesis (including ongoing
edema, orthopnea and oliguria. Her eGFR is anticoagulation), and severe obesity.
computed at 35 mL/min/1.73m2; with no recent (Harisson’s 20th ed pg. 2120)
Creatinine determinations for comparison. In
which of the following circumstances is renal
biopsy contraindicated?
a. Normal-sized kidneys on ultrasound
b. BP of 180/110 mmHg
c. BMI of 19 kg/m2
d. Nephrotic-range proteinuria

64. The following pathophysiologic mechanisms


contribute to Acute Kidney Injury, EXCEPT? a. Obstruction by casts from cellular debris
a. Obstruction by casts from cellular b. Decreased tubular permeability ( It should
debris be “Increased”)
b. Decreased tubular permeability c. Vasoconstriction due to decreased PGI2 and
c. Vasoconstriction due to decreased NO
PGI2 and NO d. Direct glomerular damage
d. Direct glomerular damage
(Harisson’s 20th ed pg. 2101)

65. Which of the following histologic features are


NOT present in CKD
Select one:
a. Glomerular obliteration
b. Interstitial fibrosis
c. Loss of polarity in Tubular cells
d. Lymphocytic infiltrates

66. Which of the following is the histopathologic Harrison’s 20th ed, pg 2136
equivalent of RPGN
a. Mesangial proliferation
b. Tubulointerstitial nephritis
c. Crescentic glomerulonephritis
d. Glomerulosclerosis

67. Which of the following is NOT included in the Robbins 9th ed pg 914
definition of Nephrotic syndrome?
a. Hypercholesterolemia
b. Hypoalbuminemia
c. Heavy proteinuria >2g/24hr
d. Microscopic hematuria

68. Lesions affecting <50% of the glomeruli in a


biopsy is called:
a. Focal
b. Diffuse
c. Segmental
d. Global

69. Lesions affecting involving most of the glomerular


tuft is called:
a. Focal
b. Diffuse
c. Segmental
d. Global

70. Which of the following serologic markers for SLE


correlates best with the presence of renal
disease?
a. ANA
b. C3
c. Anti-Sm
d. Anti-dsDNA

71. Which of the following serologic markers Page 2522- HPIM 20th- Laboratory tests
correlates with SLE flares?
a. ANA
b. C3
c. Anti-Sm
d. Anti-dsDNA

72. What is the reliable method of identifying Page 2519- HPIM 20th (Renal Manifestations)
morphologic variants of lupus nephritis?
a. Urinalysis
b. Kidney Biopsy
c. Serologic Markers
d. Clinical features of the patient

73. Prolonged exposure to which treatment modality


for lupus nephritis is avoided for patients of child-
bearing age without first banking sperms or
eggs?
a. Mycophenolate mofetil
b. Prednisone
c. Cyclophosphamide
d. Azathioprine

74. B.G. is a 6 year old male who came in for consult Harrison’s 20th Ed. p.2142
due to progressive bipedal edema and periorbital
edema which started 1 week ago. There is also
note of frothy urine output. Otherwise, there are
no other associated symptoms or illnesses.
Examination findings show: BP 90/60, HR 72 reg,
RR 16, (+) periorbital edema, clear breath
sounds, no ascites, grade 3 pitting bipedal
edema. Laboratories were done with the following
results: Urine protein (dipstick) ++++, Urine RBC
0-2/hpf Total cholesterol 270mg/dl, Triglycerides (+) proteinuria
90mg/dl, HDL 80mg/dl, LDL 190 mg/dl, Serum (+) hypoalbuminemia
creatinine 0.5 mg/dl, Serum albumin 2.1g/dl (+) hypercholesterolemia
(+) edema
What glomerulonephritis syndrome is B.G. most likely
manifesting?

a. Nephrotic syndrome
b. Nephritic syndrome
c. Mixed nephrotic-nephritic syndrome
d. None of the above

75. What is the most likely diagnosis of B.G.? Harrison’s 20th Ed. p.2142

a. Minimal change disease (MCD)


b. Membranous nephropathy
c. Focal-segmental glomerulonephritis
d. membranoproliferative glomerulonephritis

76. What is the expected histopathologic finding in Harrison’s 20th Ed. p.2142
B.G.’s diagnosis?

a. Tram-tracking basement membrane on


electron microscopy
b. Sclerosis less than 50% of glomeruli on light
microscopy
c. Basement membrane thickening on light
microscopy
d. Normal findings on light microscopy

77. B.G,’s condition is highly responsive to which of HPIM 20th Ed Page 2142
the following treatments?

a. ACE inhibitors
b. Corticosteroids
c. Furosemide
d. Albumin infusions

78. Which of the following SECONDARY


glomerulonephritis presents as nephrotic
syndrome?

a. Class II lupus nephritis


b. Hypertensive nephrosclerosis
c. Diabetic nephropathy
d. Post-infectious glomerulonephritis

79. The Kimmelstiel-Wilson nodules in light


microscopy of diabetic nephropathy is described
as?

a. Mesangial sclerosis nodules


b. Nodular glomerulosclerosis
c. Afferent arteriolar hyalinization
d. Collapsing glomerulosclerosis

80. Which of the following is TRUE about the clinical HPIM 20th Ed Page 2137
manifestations of post-streptococcal GN?

a. PSGN develops 1 to 3 weeks after an


impetigo
b. Subepithelial humps is a distinguishing
kidney biopsy feature of PSGN
c. Corticosteroids are required for severe
disease
d. All of the above

81. B.B. is a 9 year old male, came in for consult due HPIM 20th Ed Page 2140
to bipedal edema. 3 days ago, he started having
fever and a productive cough. Yesterday, B.B.’s
urine was noted by his mother to be tea-colored
and noted to have pitting edema of both lower
extremities. He had a history of recurrent
episodes of tea-colored urine that would occur a
few days after a respiratory infection that would
spontaneously resolve. his physical findings are
as follows: BP: 140/90, grade 2 bipedal edema.

Which of the following is the most likely diagnosis of


B.B.?

a. Thin basement membrane


b. IgA nephropathy
c. Post-streptococcal glomerulonephritis
d. Membranoproliferative glomerulonephritis

82. What is the pathologic finding in B.B.’s HPIM 20th Ed Page 2139
diagnosis?

a. Normal glomerulus in light microscopy


b. IgA deposits in the mesangium
c. Subendothelial deposits in electron
microscopy
d. Glomerular basement membrane thinning in
electron microscopy

83. What is the appropriate treatment for B.B.s HPIM 20th Ed Page 2140
hypertension given his pathology and is a proven
supportive therapy for his condition?

a. ACE Inhibitors
b. Diuretics
c. Calcium channel blockers
d. Beta blockers

84. Corticosteroids have a role in the treatment of the HPIM 20th Ed Page 2137
following, EXCEPT:

a. Poststreptococcal glomerulonephritis
b. IgA nephropathy
c. Goodpasture Syndrome
d. Rapidly Progressive Glomerulonephritis

85. Which of the following glomerulonephritis can be HPIM 20th Ed Page 2144
associated with solid malignancies (Colon CA,
Breast CA)?

a. Minimal change disease


b. IgA Nephropathy
c. Membranous Nephropathy
d. Membranoproliferative Glomerulonephritis
86. Deaths in dialysis are usually due to:

a. Renal failure
b. Bone-mineral disorders
c. Cardiovascular diseases
d. Malnutrition

87. Dialysis functions under the following principles,


EXCEPT:

a. Diffusion
b. Osmosis
c. Convection
d. Ultrafiltration

88. Which of the following is TRUE regarding HPIM 20th Ed Page 2123
hemodialysis access?

a. Arteriovenous grafts have the highest long-


term patency rates
b. Tunneled venous catheters are the preferred
access for long term hemodialysis
c. Most tunneled catheters are placed in the
external jugular vein
d. A native fistula created by anastomosis of
an artery to a vein leads arterialization of
the vein

89. The most common acute complication in HPIM 20th Ed Page 2124
hemodialysis, specially among patients with
diabetes mellitus is:

a. Hypertension
b. Hypotension
c. Muscle cramps
d. Anaphylactic reactions

90. The following are true about peritonitis as a HPIM 20th Ed Page 2125
complication of peritoneal dialysis, EXCEPT:

a. Gram-negative rods are the most common


etiologic agent
b. Peritonitis typically develops when there has
been a break in sterile technique during one
or more of exchange procedures
c. Peritonitis in peritoneal dialysis is usually by
at least 100/mm3 leukocyte count, of which at
least 50% are polymorphonuclears
d. Patients usually present with abdominal pain
and cloudy dialysate

91. K.L. is a 24 year-old pregnant woman at 16


weeks AOG consulting you for a routine prenatal
exam, with pyuria on urinalysis and bacteriuria of
> 105 CFU/mL. She does not complain of any
fever, dysuria, flank pain, or vomiting. What is the
next best course of management?

a. Perform repeat urine culture after 48hrs


b. Closely monitor and treat once with
symptoms
c. Treat with antibiotics even if without
symptoms
d. Request for a CT sonogram to rule out
obstruction

92. P.S. is a 64-year old veteran who complains of Source: Harrison’s Nephro 2nd ed, page 257, 254
fever with chills, nausea, left flank pain. PE
reveals left costophrenic angle tenderness and
urinalysis reveals abundant pyuria. What is the
most likely contributing factor for his condition?

a. Maternal history of UTI


b. Prostatic hypertrophy
c. Congenital urinary tract abnormalities
d. Use of spermicide

The symptoms of this conditions is more of


pyelonephritis.

a. Insignificant
b. After 50, years of age, obstruction from
prostatic hypertrophy become common in
men
c. Congenital urinary tract anomalies commonly
causes UTI in neonatal period or infanthood
d. Use of spermicide is associated with cystitis

93. E.C. is a 38-year old female accountant with no


comorbidities who consults your for dysuria,
urinary frequency, and urgency. She does not
have a fever, flank pain, nor vomiting. urinalysis is
positive for leukocyte esterase. What is the next
course of management?

a. Start empiric antibiotics


b. Request for urine cultures
c. Request for abdominal ultrasound
d. Water therapy and repeat urinalysis after 1
week
^A positive leukocyte esterase is interpreted as
positive for UTI. Hence, no need for urine cultures,
abdominal ultrasound and repeat urinalysis.
Pharmacologic na diretso.

(mao ako understanding?? Walay nakastate sa book


na word for word ani.. Or wala ko kakita)

94. Which of the following statements is TRUE Harrison’s Nephro 2nd ed, page 263
among patients with complicated UTI?

a. Empiric antibiotic therapy is first-line


management
b. You should investigate for
structural/functional urinary tract
abnormalities
c. Cultures are not routinely ordered unless with
recurrent infection
d. Associated bacterial species are almost
always multidrug resistant
95. B.L. is a 68-year old bedridden female with spinal Harrison 20th ed.
cord compression and on chronic catheterization, Page 975:
presenting with fever and suprapubic pain. What
is the diagnostic threshold for UTI in her case?
Page 973
a. ≥ 102 CFU/mL
b. ≥ 103 CFU/mL
c. ≥ 104 CFU/mL
d. ≥ 105 CFU/mL

96. J.F. is a 49-year old with flank pain, fever, and Harrisons 20th ed page 975
vomiting with pyuria on urinalysis. What is the
recommended antibiotic therapy in her case?

a. Fluconazole
b. Amoxicillin
c. Ciprofloxacin (empirical treatment for acute
pyelonephritis)
d. Nitrofurantoin

97. What is the major difference of treating UTI HPIM 20th ed. Page 973
among men compared to women?

a. Urologic imaging tests are not routinely


requested
b. Candiduria is more prevalent compared to
Harrisons 20th ed. Page 975
women
c. Cultures are routinely recommended
d. Antibiotic therapy should be started even if
asymptomatic
98. What is the most common pathway of pathogenic Harrison's 20th ed. Chap. 130 page 969-970
bacteria in renal parenchymal infections?

a. Translocation through the colonic wall to the


urinary bladder
b. Direct inoculation during urologic procedures
c. Hematogenous spread from distant focus of
severe infection
d. Ascending from the urethra to the bladder
and ureter

99. What is considered to be the primary anatomic Harrison's 20th ed. Chap. 130 page 970
factor why UTI is predominant among young
women?

a. Vesicoureteral reflux
b. Foreign body obstruction
c. Distance from female urethra to anus
d. Increased prevalence of cystoceles in women

100. The following are associated risk factors for Harrison's 20th ed. Chap. 130 page 969
UTI among women with diabetes, EXCEPT:

a. Increased duration of diabetes


b. Use of sulfonylureas for diabetes
c. Impaired cytokine secretion
d. Bladder dysfunction or DM cystopathy

101. Which of the following statements regarding the Nelson Chapter 538 page 2556
epidemiology of UTI in pediatric patients is
TRUE?

a. Most UTIs in boys occur in the 1st year of


life
b. Lack of circumcision among boys does not
increase the risk of UTI
c. The first UTI in girls usually occurs around
puberty
d. Increased female preponderance to UTI starts
at around 5 years old onwards

102. The following are symptoms of clinical Nelson Chapter 538 page 2557
pyelonephritis among pediatric patients,
EXCEPT:

a. Flank pain
b. Dysuria
c. Diarrhea
d. Fever alone

103. One of your cousins had her 5-year old,


asymptomatic daughter with no comorbidities
undergo urine culture for pyuria on urinalysis,
with finding of positive growth. What should you
advise her?

a. Antibiotic therapy according to culture


sensitivity results for 1 week
b. No treatment is necessary for
asymptomatic pediatric patients
c. repeat urine culture from a suprapubic
aspiration specimen
d. Proceed to voiding cystourethrogram then
treat if with significant reflux

104. What is the recommended role of urine culture


among pediatric patients suspected to have
UTI?

a. Should be performed for all pediatric


patients suspected for UTI
b. Should be reserved for clinical pyelonephritis
cases only
c. Should be ordered only if with no response to
antibiotics after 48hrs
d. Should be ordered only for toilet-trained
pediatric patients

105. In a 10-month old patient suspected of clinical Nelson Textbook of Pediatrics 21st Ed. Chapter 553
pyelonephritis, which of the following results Page 2792
from a sterile collection bag urine sample need
to be confirmed with a catheterized specimen?

a. Single pathogen growth of > 100,000 CFU/mL


b. Abundant pyuria
c. Multiple pathogens on culture growth
d. Positive leukocyte esterase test

106. The following are options for empiric therapy for Nelson Textbook of Pediatrics 21st Ed. Chapter 553
pediatric patients with cystitis, EXCEPT: Page 2793

a. Trimethoprim-sulfamethoxazole
b. Nitrofurantoin
c. Ciprofloxacin
d. Amoxicillin

107. In pediatric patients with atypical or recurrent Nelson Textbook of Pediatrics 21st Ed. Chapter 553
UTIs, the following imaging modalities have Page 2794-2795
been recommended by various medical
organizations, EXCEPT:

a. Renal ultrasonography
b. Ureterocystoscopy
c. Voiding cystourethrogram
d. Dimercaptosuccinic acid scan

108. An 8-year old female child presents at the ER


with fever and chills, nausea, vomiting, and
back pain. CBC shows significant leukocytosis
with abundant pyuria on urinalysis. Which of the
following empiric antibiotics is NOT
recommended for her condition due to low renal
tissue concentrations?

a. Ceftriaxone
b. Nitrofurantoin
c. Ampicillin + Gentamicin
d. Cefixime

109. The following are identified risk factors for UTI Nelson Textbook of Pediatrics 21st Ed. Chapter 553
among pediatric patients, EXCEPT: Page 2791

a. Toilet training
b. Vesicoureteral reflux
c. Sexual activity
d. High-salt diet
Neuro 1
Clinical

1.Which of the following is not observed in a patient with Adams and Victor’s Principles of Neurology 11th Ed p. 493
lesion involving the supramarginal and angular gyri of
the left inferior parietal lobe?
a. Inability to calculate
b. Inability to identify left from right
c. Inability to focus
d. Inability to write

2.Which of the following is not a symptom of increased


intracranial pressure?
a. Double vision
b. Papilledema
c. Headache
d. Vomiting

3.A 41- year-old woman was given acetazolamide for


benign intracranial hypertension. Which of the following
is a possible neuroopthalmologic
side effect?
a. Closed-angle glaucoma
b. Open-angle glaucoma
c. Macular degeneration
d. Retinitis pigmentosa

4.Which of the following statements concerning the


Arnold-Chiari phenomenon is true?
a. The cerebellum never herniates through the foramen
magnum.
b. Sometimes, the condition is congenital.
c. The exit in the roof of the fourth ventricle maybe
blocked.
d. Spinal tap is required to confirm the diagnosis.

The roof of the 4th ventricle is the ventral surface of the


cerebellum. If the cerebellum is displaced/ herniates, the exit
in the roof of the 4th ventricle ( foramen of Lushka and
Magendie) can be blocked.

Adams and Victor’s Neuro, 10th ed, page 1029

5.Which of the following statements are correct


regarding brain injury?
a. If intracranial pressure is more than 20 mmHg,
vasopressor agents should
be used regardless the level of BP
b. Burr hole operation is appropriate management for the
patients with acute
subdural hematoma
c. Post-traumatic seizure is more common in children
than in adult
d. Diffuse axonal injury is the single most important
cause of persistent
disability after traumatic brain injury

6.Which of the following is not considered as a migraine


trigger?
a. Low altitude
b. Chocolates
c. Bright lights
d. Menstruation

7. Which of the following is not a manifestation if the Adams Neurology 10th ed, p797
artery pointed is occluded?

NOTE:
Cranial nerves affected – IPSILATERAL
Tracts - CONTRALATERAL

a. Left hemiparesis
b. Dysarthria
c. Dysphagia
d. Right hemianopsia

8.Which of the following will not occur if there is a lesion


in the prefrontal cortex?
a. Uninhibited and highly distractible
b. Perseveration
c. Neglect part of the body
d. Inability to decide

9. 48-year-old man has had a painful paresthesia in the


left side of his body for 3 years following a stroke.
Infarction in what vascular
territory will most likely produce his symptoms?
a. Interpeduncular thalamoperforators of the posterior
cerebral artery
b. Mesencephalic perforators of the posterior cerebral
arteries.
c. Thalamogeniculate thalamoperforators of the
posterior cerebral arteries.
d. Quadrigeminal branches of the posterior cerebral
artery.
Adams & Victor’s 11th ed. Ch 33, pp 823-824

10. Which of the following statements is not correct


concerning the medical management for acute ischemic
stroke?
a. Antithrombotic treatment within 24 hrs
b. Frequent cuff blood pressure monitoring
c. For decline in neurologic status or uncontrolled blood
pressure, stop infusion, give cryoprecipitate, and
reimage brain emergently
d. Administer rtPA at 0.9 mg/kg IV (maximum 90 mg) IV
as 10% of total dose by bolus, followed by remainder of
total dose over 1 h

11. What is the mean arterial pressure in a patient with


blood pressure of 140/100?
a. 93 mmHg
b. 103 mmHg
c. 113 mmHg
d. 123 mmHg

12. Which of the following organisms causing CNS


infection will respond strongly with Ivermectin?
a. Streptococcus agalactiae
b. All of the above
c. L. monocytogenes
d. Legionella pneumophila

13. Which of the following findings is observed after a Ablation of the frontal eye field will result in deviation
lesion involves the left frontal eye field? of the eyes to the side of ablation.
A. Deviation of the eyes upward Stimulation of the frontal eye field will deviate both
B. Deviation of the eyes to the left eyes in a direction contralateral to the side of
C. Deviation of the eyes downward stimulation.
D. Deviation of the eyes to the right
From Functional Neuroanatomy by Adel K. Fifi, pp. 248

14. Which of the following conditions will not lead to Adam’s and Victor’s Principles of Neurology 11th, Ed
chronic and recurrent aseptic meningitis? Chapter 32, Page 766
a. Premature infants
b. Patients with known carcinoma
c. Patients on NSAIDs
d. HIV patients

15. What primary sensation bypasses the thalamus? Snell Neuroanatomy, 8th ed, pg 367
a. Taste
b. Smell
c. Hearing
d. Sight

Harrison, IM 20th ed, pg 195, pdf pg, 241


16. Abdul is an 8-year old boy admitted due to severe
headache with fever, nausea and vomiting and nuchal
rigidity. He was active few days
ago and even went to Lake Lanao with his cousins for a
picnic. You are suspecting a possible CNS infection.
What is the most likely
causative agent?
a. E. coli
b. Leptospira
c. Naegleria
d. Acanthamoeba

17. Which of the following best describes the Harrison 20th edition page
management of brain abscess? A is wrong because is should be 6-8 weeks parenteral
a. For bacterial brain abscess, 6-8 weeks IV antibiotic antibiotics without follow up.
followed with up to 4-6 months oral antimicrobial therapy C is wrong 1x/2x monthly until resolution of abscess
b. If surgically managed, IV antibiotics for at least 1 (not 5 months)
month. D is wrong because medical therapy alone is not an
c. Monthly neuroimaging up to 5 months if medical option unless neurosurgically inaccessible
management only
d. Medical therapy alone- up to 16 weeks IV

18. Which of these drugs functions as an inhibitor of Goodman 13th ed, p588
antithrombin III?
a. Enoxaparin Heparin, LMWHs (Enoxaparin), and fondaparinux
b. Alteplase have no intrinsic anticoagulant activity; rather, these
c. Heparin agents bind to antithrombin and accelerate the rate
d. Warfarin at which it inhibits various coagulation proteases.

Katzung 14th ed, p610, 612, 614, 619

Heparin directly activates anti-clotting factors,


specifically antithrombin, which inactivates the factors
XIa, IXa, and Xa. Antithrombin inhibits clotting factor
proteases, especially thrombin (IIa), IXa, and Xa.

LMWHs (enoxaparin, dalteparin, tinzaparin) inhibit


activated factor X and have less effect on thrombin.

Warfarin inhibits synthesis of the factors Va and VIIIa


by blocking the y-carboxylation of several glutamate
residues in prothrombin and factors VII, IX, and X as
well as endogenous anticoagulant proteins C and S.

Alteplase is a recombinant human t-PA which


activates plasminogen that is bound to fibrin, thereby
confining fibrinolysis to the formed thrombus and
avoids systemic activation.

19. Which of the following statement is true with respect Answer: A


to CT image shown?
“Although anyone can get a subdural hematoma from
a. Common among alcoholics and infants. an accidental head injury,certain groups of people are
b. “Neuromembranes” develop from periphery, always at risk.. More common in: older adults, athletes,
seen by 6 weeks. medication with anticoagulants, hemophiliacs,
c. Hematoma progressively enlarges because of alcoholics, and babies (shaken baby syndrome).”
recurrent bleeding from Source (internet):
shearing of veins. https://my.clevelandclinic.org/health/diseases/21183-
d. It is radiologically characterized by a more subdural-hematoma
homogenous appearance
compared to an epidural hematoma. “Elderly and alcoholic patients are at higher risk for
acute SDH formation after head trauma due to brain
atrophy.”
Source: Schwartz’s, 11th Ed., p. 1838
20. Which of the following is an exclusion criterion for
thrombolysis?
a. Blood pressure of 160/100 mmHg
b. Hyperdensity in the CT image
c. Patient with right hemiplegia
d. INR of 1.5

Adams and Victor’s Neuro, 10th ed, page 827

21. A 27- year-old man was overthrown forward in a


vehicular accident. On examination, there was complete
motor and sensory loss of both
legs below the inguinal ligament and absence of all deep
tendon reflexes of both legs. Twelve hours later, he
could move the toes and
ankle of his left lower limb and he had return sensation
to his right leg except for loss of tactile discrimination,
vibratory sense, and
proprioceptive sense. He had a band of complete
anesthesia over the right inguinal ligament. His left leg
showed a total analgesia,
thermoanesthesia, and partial loss of tactile sense. His
right leg is totally paralyzed and the muscles were
spastic. Babinski sign was noted in
the right. Which vertebra was damaged?
a. T10
b. T11
c. L1
d. T12

22. Grace is a 36-year-old female who sustained trauma


on the vertebral column after a vehicular accident.
According to his orthopedic
surgeon, she has paraplegia in extension. Which of the
following statements is true?
a. The condition is secondary to the damage of all
descending tracts
immediately after the trauma
b. Overactivity of the gamma efferent nerve fibers
c. Result from damage of the vestibulospinal tract
d. All of the above

Snell’s Neuroana. 8th Ed. Page 178-179.


23. Which of the following structures is pain-insensitive?

a. Proximal 50% of the larger arteries of the circle of


Willis
b. Venous sinuses
c. Dural arteries
d. Choroid plexus

Headache. Adams pg 182

24. Which of the following neuropathologic changes is Adam’s 11th ed Chapter 41 Page 1214
observed in delayed death secondary to methanol
intoxication?
a. Necrosis of the lateral aspect of putamen and
claustrum
b. Atrophy of cerebellar vermis
c. Atrophy and gliosis of the hippocampus
d. Laminar cortical necrosis
25. Which of the following is not an indication for
preventive treatment of migraine?
a. Acute therapy is ineffective, intolerable, and
contraindicated
b. Attacks occur unpredictably
c. Acute therapy is needed more than 2x per week
d. Two or more attacks/month that produce disability for
> 3 days

26. Rosana is a 79-year-old female admitted due to left


sided weakness. On examination, she was noted to
have irregularly irregular rate and
rhythm of 96/min. She is diabetic for 3 years with poor
compliance to medications. Her blood pressure at the
ER was 180/90mmHg. You are
now planning to start her with anticoagulant. She has
history of taking Clopidogrel. Her creatinine clearance is
27 mL/min. Her liver
function is normal. What is her HAS-BLeD score?
a. 6
b. 5
c. 7
d. 4

27. A 16-year-old boy was admitted following a severe C man ata ang answer? D ang naa sa moodle?
motor vehicular accident. He was paralyzed from the Sorry wala ko naka-save og copy.
waist downward for the rest of his
life. Which of the following is not a common complication Since the patient is paralyzed from the waist downward
after his injury? for the rest of his life, so most probably he will be
a. Bedsore bedridden. The common complications of bedridden
b. Urinary infection patients include A (skin infections), B (urinary
c. Nutritional deficiency infection) and D (pneumonia).
d. Pneumonia
Ref: Harrison Neurology in Clinical Medicine, 3rd Ed,
Ch 27, p 258

(snell, 177)
Ans: D
Common Complications: Urinary infection, bed sores,
nutritional def, muscular spasms, pain

28. Which of the following is not part of the diagnostic Aura should lasts less than 1 hr
criteria for migraine with aura?
a. One or more fully reversible aura
b. Headache follows aura within 1 hour of the end of
aura
c. No single aura lasts longer than 4 hours
d. At least one aura symptom that develops gradually
over more than 4 minutes
Ref: Doc Ruben’s Headache PPT (Sorry wa ko kabalo
unsay source ni doc :( )

29. Which of the following is likely to be an additional


feature in a patient with vertigo secondary to a brainstem
stroke involving a single
vascular territory?
a. Locked-in syndrome
b. Facial paresis
c. Skew ocular deviation
d. Facial anesthesia

30. Which of the following is not a manifestation of CNS Adams


sarcoidosis?
a. Bitemporal hemianopsia
b. Hearing loss
c. Cauda equina syndrome
d. Diabetes insipidus
Hearing loss is also included due to CN involvement.
(internet)

31. Which of the following occurs if the M2 segment of


the right middle cerebral artery is occluded?
a. Aprosodia
b. Abulia
Harrison’s IM 20ed p. 3069
c. Right hemianopsia
d. Loss of specific taste

Adam’s Neurology 11ed p.510

32. Which of the following is not observed immediately


after complete transection of the spinal cord at T6 level?
a. Loss of voluntary control of the bladder and bowel
b. Paralysis of volitional leg movements Harrison’s 20th, page 142
c. Hyperactive muscle stretch reflexes
d. Loss of vasomotor reflexes in the legs

33. Which of the following is least likely associated with


the CT image shown?

a. Dejerine-Roussy syndrome
b. Hydrocephalus
c. Left hemiparesis
d. Seizure

34. Which of the following is not seen in spinal shock?


a. Loss of bladder and bowel function
b. Absent long tract signs
c. Increased reflexes
d. Extreme back pain
35. Which of the following mechanisms does
hyperventilation play in reducing intracranial pressure?
a. Arterial vasodilation thereby reducing cerebral
perfusion
b. Vasoconstriction thereby reducing cerebral blood
volume
c. Increase O2 level thereby increasing metabolic activity
d. All of the above

36. Which of the following areas when lesioned causes Anosognosia- lack of recognition of a part of a body
anosognosia? Syndromes caused by lesions of parietal Lobe
a. Left temporal lobe lesions
b. Left posterior parietal lesions Lesions in the occipital lobe may cause Visual
c. Diffuse cerebral disease anosognosia or Anton syndrome
d. Right posterior parietal lesions

Addams 10th ed p 473

37. Which of these drugs in the treatment of stroke can Sorry I can’t find it sa GG, but in MIMS it was stated:
cause life-threatening agranulocytosis and neutropenia? “Adverse Reactions
a. Cilostazol Potentially Fatal: Neutropenia, agranulocytosis,
b. Clopidogrel thrombotic thrombocytopenic purpura and aplastic
c. Dipyridamole anaemia.”
d. Ticlopidine
Harrison’s IM 20th ed. Page 3088

38. area was noted over the left basal ganglia. What is
your most likely diagnosis?
a. CADASIL
b. AV malformation
c. Moyamoya disease
d. Marfan syndrome
39. Where is the lesion in a patient with unilateral miosis,
ptosis, and facial anhidrosis?
a. CN VII
b. CN III
c. The ciliary ganglion
d. The carotid artery

40. Rosana is a 79-year-old female admitted due to left


sided weakness. On examination, she was noted to
have irregularly irregular rate and
rhythm of 96/min. She is diabetic for 3 years with poor
compliance to medications. Her blood pressure at the
ER was 140/90mmHg. You are
now planning to start her with anticoagulant. What is her
CHADs-VASc score?
a. 7
b. 4
c. 5
d. 6

41. In general, what is the most common cause in this


condition?

a. AV malformation
b. Hypertension
c. Ruptured aneurysm
d. Trauma
42. What is the first layer in the retina to receive light?
a. Rods and cones
b. Inner plexiform
c. Outer plexiform
d. Ganglion cells

Guyton and Hall Textbook of Medical Physiology 13th,


p647

43. Which of these areas will cause damage to CN III,


CN IV, CN VI and the ophthalmic division of CN V if an
injury will happen?
a. Pontine tegmentum
b. Midbrain tegmentum
c. Cerebellopontine angle
d. Cavernous sinus

44. This is a CT image of a 58-year-old male with left CT scan is more likely from right sided ischemic stroke
sided weakness taken five hours after admission. Which a. Contralateral dapat (HPIM 20e 3093)
of the following is true
with respect to the CT image shown?

b. Right internal carotid dapat


c. ANSWER (Adams 10e 823)

d. Thrombolysis should be done within the 1st


three hours (HPIM 20e 3081)

a. Right hemianopsia.
b. Carotid dissection of the left internal carotid artery.
c. Hemicraniectomy is indicated.
d. Thrombolysis
45. What do you call a fibrin- rich clot?
a. Yellow clot
b. Red clot
c. White clot
d. Grey clot

46. A 56-year-old man is found unresponsive. He was


immediately rushed to the ER. Upon examination, the
only finding is he has difficulty
with saccades. If this is caused by a stroke, where is the
lesion?
a. Interposited nuclei
b. Superior colliculus
c. Fastigial nucleus
d. Inferior colliculus
47. A 70-year-old man complains of difficulty swallowing,
hoarseness of his voice and giddiness. On physical
examination, he was noted to
have absent gag reflex on the left side, loss of pain and
temperature sense on the left face, and left sided
paralysis of the vocal cords. What
is your most likely diagnosis?
a. Medial medullary syndrome, left side
b. Lateral medullary syndrome, left side
c. Medial medullary syndrome, right side
d. Lateral medullary syndrome, right side

This syndrome manifests ipsilateral Horner syndrome and paralysis


of the palate and vocal cord, hence it is left side.

Adams and Victor’s Neuro, 10th ed, page 820

48. Dario, a 68-year-old male admitted due to severe


headache associated with elevated blood pressure. CT
scan finding as shown. Which of
the following statements is correct?

a. The patient will develop hydrocephalus.


b. Vasospasm peaks on the 14th day if no treatment is
initiated.
c. Maintain blood pressure at 110-130mmHg systolic.
d. Cranial MRI with MRA is better than Transcranial
doppler in determining the
severity of vasospasm
49. Which of the following statement is not true with
respect to the CT image shown?

a. Methylprednisolone
b. Mannitol
c. Trauma
d. VP shunting

50. A 38-year-old female was complaining of severe Ptosis signifies either a CN III palsy or a sympathetic lesion
persistent headache associated with nausea and Clinical effects of CN III palsy also include diplopia, pupils dilated
and fixed to light & loss of lens thickening (see table below)
vomiting. Upon looking at the mirror, her
right pupil looked much larger than the left and her right But superior oblique muscle is innervated by CN IV
upper eyelid appeared to droop.
Which of the following findings is least likely revealed DeMyer’s Neurologic Examination 7th ed p220
during examination?
a. Severe ptosis in the right eye
b. Ataxia of right upper limb
c. Bilateral papilledema
d. Paralysis of superior oblique muscle on the right

51. What is the primary receptor for detecting an edge?


a. Merkel disk
b. Pacinian corpuscle
c. Ruffini ending
d. Meissner corpuscle
52. What cell group in the primary motor cortex will react
to points of light?
a. Blob cells
b. Complex cells
c. Simple cells
d. Stellate cells

a. Color blobs
- areas of the visual cortex which are
necessary for detection of color (I can’t
find a source directly stating that they react to
points of light ;( )
b. Complex cells

c. Simple cells

Guyton 13th ed Ch. 52, p.663-664


d.

Snell 8th ed. Ch. 8, p.280


53. Which of the following predisposing factors for brain
abscess development will respond best to Vancomycin +
Gentamicin treatment?
a. Penetrating head trauma
b. Bacterial endocarditis
c. Lung abscess
d. Congenital heart disease

54. Which of the following cranial nerves do not form


part in the control of autonomic effectors?
a. III
b. VI
c. VII
d. IX

55. A 58-year-old diabetic female who presented with


slurring of speech lasting for 2 hours and a blood
pressure of 130/100mmHg. You
computed the ABCD2 score. What is the risk for
developing stroke in the next seven days?
a. 8.1%
b. 4.1%
c. 1.2%
d. 5.9%

Total score: 5
7-day risk: 5.9%

Harrison’s 20ed p. 3087


2minutemedicine.com

56. A 28-year-old drug addict was admitted due to fever, ● Fever = infectious
weakness of the lower extremities, and urinary ● Weakness of the lower extremities but normal
incontinence. Strength in the upper extremities is strength of the upper extremities = definitely below
normal. MRI of the brain and spine was done and a cervical level
diagnosis of epidural abscess is made. What is the ● Urinary incontinence = thoracic to lower levels
most likely location of the abscess? ○ Epidural abscess = compression; highly unlikely
to be at the level of cauda equina or conus
A. Frontal convexities medullaris since at these levels spinal tapering
B. Cervical spine is evident.
C. Thoracic spine ● Possible level of lesion = thoracic to lumbar
D. Cauda equina spinal cord segments

57. What is the best approach to treating cluster


headaches? A. Sumatriptan is used as treatment for Acute
a. Sumatriptan, 50mg orally at the onset of attacks Cluster Headache (Harrison’s, Chapter 422,
b. Oxygen at 10–12 L/min by nasal cannula for 30 page 3105)
minutes daily for 2 weeks B. Oxygen 100% at 10- 12 L/ min for 15- 20 mins
c. Indomethacin, 25 mg three times daily only (Harrison’s) 10- 15 mins (Adam’s)
d. Surgical consultation for microvascular C. Cluster headache is not an indomethacin-
decompression of the trigeminal nerve responsive headache (Harrison’s and Adam’s)
D. Not a treatment for Cluster headache.

58. Which of these drugs inhibits uptake of adenosine Ticlopidine


into platelets and endothelial cells? ➔ inhibits adenosine diphosphate-mediated
a. Ticlopidine platelet aggregation.
b. Clopidogrel ➔ prevents fibrinogen binding to platelets w/o
c. Dipyridamole modifying GPIIb/IIIa receptor.
d. Cilostazol Clopidogrel
➔ is a thienopyridine prodrug that inhibits the
P2Y12 receptor. (P2Y12 receptor Antagonist)
◆ gg. 13th ed. pg 597
Dipyramidole
➔ interferes with platelet function by increasing
the intracellular concentration of cyclic AMP.
This effect is mediated by inhibition of
phosphodiesterase or by blockade of uptake of
adenosine, thereby increasing the dwell time of
adenosine at cell surface adenosine A2 recep-
tors that link to the stimulation of platelet
adenylyl cyclase.
◆ gg, 13th ed. pg 596
Cilostazol
➔ is an inhibitor of PDE3 and promotes
accumulation of intracellular cAMP in many
cells, including blood platelets. Cilostazol
mediated increases in cAMP inhibit platelet
aggregation and promote vasodilation.
◆ gg. 13th ed. pg 502
59. Which of the following type of cerebral edema is
responsive to steroids?
a. Cytotoxic edema
b. Vasogenic edema
c. Interstitial edema
d. All of the above

60. Which of the following statements characterizes the Doc Flores lecture on CNS infection
early capsule formation in brain abscess? Early capsule formation
a. Prominent cerebral edema ● Decrease in necrotic center
b. Presence of numerous macrophages ● Increase number of fibroblast and macrophage
c. Increase necrotic center ● Maximal degree of neovascularity
d. Appearance of fibroblast and formation of reticulin ● Evolution of mature collagen
● Regression of cerebral edema; increase in
reactive astrocytes

61. Which of the following phases in migraine is Harrison 20th ed, p3096
characterized by the presence of psychological,
neurological and autonomic disturbances? A migraine attack has 3 phases:
a. Premonitory phase 1. Premonitory or prodrome - consists of
b. Resolution phase yawning, tiredness, cognitive dysfunction,
c. Aura phase mood changes, neck discomfort, polyuria, and
d. Headache phase food cravings. It lasts for a few hours to days.
2. Headache - follows with its associated features
such as nausea, photophobia, phonophobia,
and allodynia. These typical migraine
symptoms also emerge in the premonitory
phase.
3. Postdrome (resolution) - includes feeling tired
or weary, problems with concentration, and mild
neck discomfort lasting for hours and
sometimes up to days.
4. Aura - consists of visual disturbances with
flashing lights or zigzag lines moving across the
visual field or of other neurologic symptoms. It
is reported only in 20-25% of patients.

62. Which of the following antimicrobials in the treatment


of CNS infection will Listeria monocytogenes and
Nocardia asteroides responds
best?
a. Metronidazole
b. Cotrimoxazole
c. Ceftriaxone
d. Vancomycin

Harrison’s Neurology, 3rd Ed., p. 502

63. What is the risk of vasospasm after subarachnoid


hemorrhage in a patient with a Hunt and Hess score of
4?
a. 22%
b. 33%
c. 53%
d. 74%

64. Antonio is a 46-year-old man complaining of


symptoms of analgesia and thermoanesthesia on the
medial side of the left hand for 6
months duration. Few days later, he started complaining
of severe numbness of the little finger and even
sustained burn from not feeling
the hot stove. On examination, he has severe loss of
pain and temperature sense involving the eight cervical
and first thoracic dermatomes
of the left hand while tactile discrimination is perfectly
normal in this area. Examination on the right side
showed the same findings of less Severe loss of pain and temperature sense - Lateral
severity. Which tract or tracts is most likely involved? spinothalamic tract is affected
a. Lateral spinothalamic tract
b. Fasciculus cuneatus and anterior spinothalamic tracts Severe numbness - Anterior spinothalamic tract
c. Lateral and anterior spinothalamic tracts
d. Fasciculus cuneatus and lateral spinothalamic tracts Snell 8th ed. p. 143

65. Which of the following risk factors predisposes one


from developing primary noninfectious thrombosis of
dural sinuses?
a. Dehydrated infants and postpartum young
females
b. Heavy male smokers with known cardiac disease
c. Neonates in respiratory failure
d. School age children after head trauma

Harrison’s PIM. 20th Ed. Page 3085.

66. Which of the following conditions is characterized by Malaria


severe headache, seizure, hiccups and bruxism?
a. CNS schistosomiasis
b. CNS malaria
c. CNS cysticercosis
d. CNS toxoplasmosis
Adams ch 31 pg754-760

67. Which of the following statements is not true about Harrison’s Neurologyin Clinical Medicine 3rd ed Chapter 36 page 415
cerebral concussion?
a. Reversible traumatic paralysis of the nervous system.
b. Duration of anterograde amnesia is most reliable
predictor of severity.
c. Tachycardia, hypotension and apnea may be
noted.
d. Loss of consciousness lasting from seconds to hours.

68. A 3-year-old native girl was brought in to the ER with


profound weakness of the left arm with absent reflexes.
The rest of the examination is
normal. What is the most likely causative viral agent?
a. Enterovirus
b. Herpesvirus
c. Rabies
d. Measles

69. Which of the following is not a manifestation if the


artery pointed is occluded?
a. Ptosis of the left eye
b. Dysarthria
c. Numbness of the left arm and leg
d. Dysdiadochokinesia of the left extremities

70. Which of the following can help improve the FOR A: Ref: Harrison Neurology in Clinical Medicine
diagnostic yield in CSF analysis? 3rd Ed, Ch 6, p 37
a. Submit large volume of CSF (>25 ml) for microbiology
b. Centrifuge at high force (3000g) for 30 minutes and
stain and culture the deposit
c. Incubate at 37 C if available or store at 4 C if for
transport.
d. Subculture after 24-hour inoculation in brain heart
infusion broth at 37 C with 5% CO2 Sorry di ko talaga mahanap iyong rationale ng B, C,
at D sa books. Huhu.

Ref: https://www.cdc.gov/meningitis/lab-
manual/chpt06-culture-id.html
FOR B: Centrifuge at 1000 x g for 10-15 minutes

FOR C: Incubation at 37°C is not recommended

FOR D:
71. Which of the following statements concerning head
trauma is true?
a. Contrecoup contusions are produced by
deceleration of head in backward fall that hit the
occiput.
b. Contrecoup and coup fractures are linked with each
other.
c. Contrecoup fractures frequently occur at the floor of
the orbit and ethmoidal plate in occipital injuries.
d. Coup contusion results from a blow to a moving head.

72. Which of the following statements concerning the


lateral medullary syndrome is correct?
a. Contralateral trunk and extremity hypalgesia and
thermoanesthesia may occur.
b. Damage of the contralateral nucleus ambiguus is
notable.
c. There may be ipsilateral analgesia and
thermoanesthesia of the face
d. Common cause is thrombosis of the anterior inferior
cerebellar artery.

73. Which of the following statements is not true Harrison’s 20th ed p 3080
concerning the medical management in patients with
acute ischemic stroke?
a. Blood pressure should be lowered if there is malignant
hypertension or concomitant myocardial ischemia, or if
blood pressure is
>185/110 mmHg and thrombolytic therapy is anticipated.
b. When faced with the competing demands of
myocardium and brain, lowering the heart rate with β1-
adrenergic blocker (such as
Esmolol) can be a first step to decrease cardiac work
and maintain blood pressure. Routine lowering of blood
pressure has been
found to worsen outcomes.
c. Edema peaks on the second or third day but can
cause mass effect for ~5 days. The larger the infarct,
the greater the likelihood
that clinically significant edema will develop.
d. Water restriction and IV Mannitol may be used to raise
the serum osmolarity, but hypovolemia should be
avoided because this may
contribute to hypotension and worsening infarction

74. Which of the following CNS parasitic infections will


have a finding of multiple enhancing lesions in the brain
in neuroimaging?
a. Amebiasis
b. Visceral larva migrans
c. Toxoplasmosis
d. Trichinosis

Adam’s Neurology 11ed. Table 31-5 p. 757


75. Which of the following is not part of the diagnostic
criteria for migraine without aura?
a. Associated with nausea, vomiting, photophobia and
phonophobia
b. Headache lasting 4-72 hours
c. Unilateral in location and pulsating in quality
d. At least 3 attacks 5 previous attacks

ICHD (International Classification of Headache


Disorders)

76. Which of the following characterizes neurosyphilis?


a. All forms of neurosyphilis begin as meningitis.
b. 40-50% will develop stroke secondary to meningitis.
c. Invasion of treponemes takes around 3-36 months.
d. Asymptomatic patients may present with relative
afferent pupillary defect

77. A 58-year-old diabetic female who presented with


slurring of speech lasting for 2 hours and a blood
pressure of 130/100mmHg will have
an ABCD2 score of?
a. 6
b. 7
c. 5
d. 4

78. Which of the following phase in migraine correlates


to cortical spreading depression?
a. Premonitory phase
b. Resolution phase
c. Headache phase
d. Aura phase

79. Which of the following conditions presents with


dementia, gait imbalance and pseudobulbar state
secondary to multiple white matter
infarctions?
a. Pelizaeus-Merzbacher Disease
b. CADASIL
c. Binswanger’s disease
d. Canavan’s disease
Addams p 825
CADASIL- cerebral autosomal dominant arteriopathy
with subcortical infarct and leukoencephalopathy;
Large patches of subcortical myelin loss and
gliosis, in combination with small cortical and
subcortical infarcts
Addams p 825

Pelizaeus-Merzbacher Disease
- X-linked disease of infancy; first signs are
abnormal movements of the eyes (nystagmus);
caused by PLP1 Mutation that codes for myelin
basic protein
Addams p 960

Canavan’s disease (Spongy Degeneration of


Infancy)- autosomal recessive disease
lack of development or rapid regression of
psychomotor function, loss of sight and
optic atrophy, lethargy, difficulty in sucking, irritability,
reduced motor activity, hypotonia followed by spasticity
of the limbs with corticospinal signs, and an enlarged
head (macrocephaly)

Addams p 961-962

80. Which stroke symptom is unlikely to be related to the


others?
a. Geographical disorientation
b. Mild sensory difficulty in the face
c. Flaccid plegia of arm and face
d. Dressing apraxia

81. Which of the following unilateral visual symptom is


observed if an aneurysm develops in the vessel pointed?
a. Absent direct, but intact consensual light reflex
b. Absent direct and consensual light reflex
c. Dilated pupil
d. Hemianopsia
82. A 70-year-old man was rushed to the ER with right
sided weakness. His blood pressure was 160/100mmHg.
Capillary glucose test was
noted at 278mg/dl. After about an hour, the weakness
resolved. You computed his ABCD2 score. What is his
risk of developing stroke 2
days and in 90 days?
a. 5.9% and 17.8%
b. 11.7% and 17.8%
c. 4.1% and 9.8%
d. 8.1% and 17.8 %

70 years old = 1
BP= 1
DM= 1
Unilateral weakness= 2
About an hour= 2
Total: 7 (8.1% risk)
83. Which of the following will be observed if there is a
lesion affecting the corticopontine pathway at its origin in
the cortex or along its
course?
a. Ataxia
b. Difficulty breathing
c. Coma
d. Pinpoint pupils

84. Which of the following trematodes tend to cause Harrison Specialty, Neurology (2016) pg 499
localized spinal cord injury leading to myelitis?
a. Schistosoma japonicum
b. Schistosoma mansoni
c. Schistosoma haematobium
d. Paragonimus

Harrisons IM, 20th ed. pg 3179

B (old 2016)
B and C? (recent 2018)

85. Which of the following statements concerning the


medial medullary syndrome is correct?
a. Contralateral impaired sensations of position and
movement.
b. Contralateral facial paralysis
c. Common cause is thrombosis of posterior
communicating artery.
d. Ipsilateral tongue and contralateral extremity
paralysis

Adam and Victor’s Principle of Neurology, p826

86. Which of the following is not considered part of the


general mechanism of headache?
a. Distention and dilation of intracranial arteries
b. Traction on major intracranial vessels
c. Inflammation of the brain parenchyma
d. Direct pressure on cranial or cervical nerves
87. Which of the following happens if there is a lesion in Harrison’s Principles of Internal Medicine 20th Ed p.
the inferior calcarine cortex? 3072
a. Contralateral upper quadrantanopsia
b. Contralateral lower quadrantanopsia with macular
sparing
c. Contralateral upper quadrantanopsia with macular
sparing
d. Contralateral lower quadrantanopsia

88. Which of the following statements is considered part


of migraine management?
a. Regularization of meals, sleep, and exercise
b. Avoidance of migraine triggers
c. Avoidance of overuse of analgesics
d. All of the above

89. Which of the following is defined as a mental and


behavioral state of reduced comprehension, coherence,
and capacity to reason?
a. Malingering
b. Coma
c. Confusion
d. Delirium

90. In the management of a patient with subarachnoid


hemorrhage and a GCS score of 4. Which of the
following statements is true?
a. With GCS 4, surgery is never warranted.
b. With GCS 4, surgery is always warranted.
c. Surgery is always warranted for Hunt and Hess grade
5.
d. Surgery is never warranted for Hunt and Hess grade
5.

Adams and Victor’s Neuro, 10th ed, page 863


91. Which of the following cranial nerves is not involved
in the control of somatic skeletal muscles?
a. III
b. IV
c. X
d. XII

92. A 70-year-old man was rushed to the ER with right


sided weakness. His blood pressure was 160/100mmHg.
Capillary glucose test was
noted at 278mg/dl. After about an hour, the weakness
resolved. What is his ABCD2 score?
a. 5
b. 6
c. 4
d. 7

93. What sensation is affected in a lesion involving the Adams Neurology 10th ed, p165
ventroposterolateral nucleus of the thalamus?
a. Contralateral proprioception and ipsilateral
temperature sensation
b. Ipsilateral proprioception and contralateral
temperature sensation
c. Contralateral temperature sensation and
contralateral proprioception
d. Ipsilateral proprioception and ipsilateral temperature
sense
Ryan Splittgerber (Snell’s Clinical Neuroanatomy) 8th ed, Chapter
12, p369
94. Which of the following organisms causing CNS
infection will respond strongly with fluoroquinolones?
a. M. tuberculosis
b. Streptococcus pneumoniae
c. L. monocytogenes
d. N. Meningitidis

95. A 26-year-old boy was referred to you complaining of A and C (Snell 8th ed, Ch. 11, p. 351)
double vision. Upon examination, the double vision
became worse in looking to the
right. You also noted mild motor paralysis of the left
extremities and a mild peripheral facial paralysis on the
right.
Which of the following statements is least likely to
cause the symptoms?
a. Imaging may reveal the presence of a small tumor of
the lower part of the right side of pons.
b. The mild left hemiparesis is produced by the
damage of the left corticospinal tract.
c. There was damage to the right CN VI nucleus.
B.
d. The complete right-sided facial paralysis caused by
Because most of the corticospinal fibers crossed to the
involvement of the right CN VII nucleus or its nerve.
opposite side at the decussation of the pyramids or
lower down at the segmental level of the spinal cord,
the muscles of the opposite side would have been
affected.
Interruption of these corticospinal fibers would have
produced the following clinical signs:
(a) a positive Babinski sign;
(b) loss of superficial abdominal and cremasteric
reflexes;
(c) loss of performance of fine, skilled voluntary
movements, especially at the distal ends of the limbs
(Snell 8th ed, Ch. 4, p. 166)

D.

(Snell 8th ed, Ch. 11, p. 352)


96. Which of the following patients should be managed
with antiplatelet agent for stroke prevention?
a. A 70-year old man with atrial fibrillation, diabetes,
hypertension, and congestive heart failure.
b. A 48-year-old man with hyperthyroidism and
intracranial atherosclerotic stenosis.
c. A 45-year old woman with mechanical heart valve.
d. A 55-year old man who suffered an ST –elevation
myocardial infarction last week and has ejection fraction
of 35% with anterior wall
akinesis and left ventricular thrombus.

97. In women, the amygdala is larger than men. Which


of the following stereotypes might be explained by this
anatomic difference?
a. Women are more thoughtful about emotions than
men.
b. Women live longer than men.
c. Women worry more than men.
d. Women seek more college degrees than men.

98. Which of these muscles is involved in patient with Muscles innervated by facial nerve:
Bell’s palsy?
a. Orbicularis oculi
b. Levator palpebrae
c. Superior tarsal
d. Pupillodilator
Levator palpebrae, Superior tarsal, and Pupillodilator
muscles are not innervated by facial nerve, hence are
not involved in bell’s palsy.

DeJong’s the neurologic examination 8ed, p.509

99. Richard is a 7-year-old boy admitted due to TB The WHO revised dosages for the anti-TB meds in
meningitis. He weighs 35kg. What is his daily children (2014)
requirement for the following anti-TB medications? Children up to 25 kilograms
Isoniazid, Ethambutol, Rifampicin and Pyrazinamide. Rifampicin 15 (10-20) mkday (max: 600mg/day)
Isoniazid 10 (10-15) mkday (max: 300mg/day)
A. 525mg- 1, 225mg- 175mg- 350mg Pyrazinamide 35 (30-40) mkday
B. 175mg- 350mg- 525mg- 1, 225mg Ethambutol 20 (15-25) mkday
C. 350mg- 175mg- 525mg – 1, 225mg Children above 25 kg
D. 175mg- 525mg- 350mg- 1,225mg may use the adult dosages and preparation.

DOH National Tuberculosis Program 2014 Drug


Dosage for Adult
Rifampicin 10 (8-12) mg/kg, max 600 mg daily
Isoniazid 5 (4-6) mg/kg, max 400 mg daily
Pyrazinamide 25 (20-30) mg/kg, max 2 g daily
Ethambutol 15 (15-20) mg/kg, max 1.2 g daily

100. Which of the following characterizes the ptosis in


Horner’s syndrome? Di ko makita ang answer sa Books, pati sa net. Huhu
a. Being more pronounced when the patient looks up.
b. Not changing when the patients look up
c. Disappearing or improving when the patients look
up.
d. Being better at rest
Neurology 1 11-What is the mean arterial pressure in a patient with blood pressure of
Clinical –Retake 140/100? 113 mmHg

12-Which of the following organisms causing CNS infection will respond


1-Which of the following is not observe in a patient with lesion involving the
strongly with Ivermectin?
supramarginal and angular gyri of the left inferior parietal lobe? a. Streptococcus agalactiae
a. Inability to calculate b. All of the above
b. Inability to identify left from right c. L. monocytogenes
c. Inability to focus d. Legionella pneumophila
d. Inability to write
13-Which of the following findings is observed after a lesion involves the left
2-Which of the following is not a symptom of increased intracranial frontal eye field?
pressure? a. Deviation of the eyes upward
b. Deviation of the eyes to the left
a. Double vision
c. Deviation of the eyes downward
b. Papilledema d. Deviation of the eyes to the right
c. Headache
d. Vomiting 14-Which of the following conditions will NOT lead to chronic and recurrent
aseptic meningitis?
3-A 41- year-old woman was given acetazolamide for benign intracranial a. Premature infants
hypertension. Which of the following is a possible neuroopthalmologic side b. Patients with known carcinoma
effect? c. Patients on NSAIDs
a. Closed-angle glaucoma d. HIV patients
b. Open-angle glaucoma
c. Macular degeneration 15-What primary sensation bypasses the thalamus?
a. Taste
d. Retinitis pigmentosa
b. Smell
c. Hearing
4-Which of the following statements concerning the Arnold-Chiari d. Sight
phenomenon is true?
a. The cerebellum never herniates through the foramen magnum. 16-Abdul is an 8-year old boy admitted due to severe headache with fever,
b. Sometimes, the condition is congenital. nausea and vomiting and nuchal rigidity. He was active few days ago and
c. The exit in the roof of the fourth ventricle maybe blocked. even went to Lake Lanao with his cousins for a picnic. You are suspecting a
d. Spinal tap is required to confirm the diagnosis. possible CNS infection. What is the most likely causative agent?
a. E. coli
b. Leptospira
5-Which of the following statements are correct regarding brain injury? c. Naegleria
a. If intracranial pressure is more than 20 mmHg, vasopressor agents should d. Acanthamoeba
be used regardless the level of BP
b. Burr hole operation is appropriate management for the patients with 17-Which of the following best describes the management of brain abscess?
acute subdural hematoma a. For bacterial brain abscess, 6-8 weeks IV antibiotic followed with up to 4-6
c. Post-traumatic seizure is more common in children than in adult months oral antimicrobial therapy
d. Diffuse axonal injury is the single most important cause of persistent b. If surgically managed, IV antibiotics for at least 1 month.
disability after traumatic brain injury c. Monthly neuroimaging up to 5 months if medical management only
d. Medical therapy alone- up to 16 weeks IV

6-Which of the following is not considered as a migraine trigger?


18-Antithrombin III inhibitor – Heparin
a. Low altitude
b. Chocolates
19-Which of the following statement is true with respect to CT image shown?
c. Bright lights
d. Menstruation

7-Not a manifestation of middle cerebral artery occlusion


a. Left hemiparesis
b. Dysarthria
c. Dysphagia
d. Right hemianopsia

8-Which of the following will not occur if there is a lesion in the prefrontal
cortex?
a. Uninhibited and highly distractible a. Common among alcoholics and infants.
b. Perseveration b. “Neuromembranes” develop from periphery, always seen by 6 weeks.
c. Neglect part of the body c. Hematoma progressively enlarges because of recurrent bleeding from
d. Inability to decide shearing of veins.
d. It is radiologically characterized by a more homogenous appearance
9-A 48-year-old man has had a painful paresthesia in the left side of his body compared to an epidural hematoma.
for 3 years following a stroke. Infarction in what vascular territory will most
likely produce his symptoms? 20-Which of the following is an exclusion criterion for thrombolysis?
a. Interpeduncular thalamoperforators of the posterior cerebral artery a. Blood pressure of 160/100 mmHg
b. Mesencephalic perforators of the posterior cerebral arteries. b. Hyperdensity in the CT image
c. Thalamogeniculate thalamoperforators of the posterior cerebral arteries. c. Patient with right hemiplegia
d. Quadrigeminal branches of the posterior cerebral artery. d. INR of 1.5

10-Which of the following statements is not correct concerning the medical 21- 27- year-old man was overthrown forward in a vehicular accident. On
management for acute ischemic stroke? examination, there was complete motor and sensory loss of both legs below
a. Antithrombotic treatment within 24 hrs the inguinal ligament and absence of all deep tendon reflexes of both legs.
b. Frequent cuff blood pressure monitoring Twelve hours later, he could move the toes and ankle of his left lower limb
c. For decline in neurologic status or uncontrolled blood pressure, stop and he had return sensation to his right leg except for loss of tactile
infusion, give cryoprecipitate, and reimage brain emergently discrimination, vibratory sense, and proprioceptive sense. He had a band of
d. Administer rtPA at 0.9 mg/kg IV (maximum 90 mg) IV as 10% of total dose complete anesthesia over the right inguinal ligament. His left leg showed a
by bolus, followed by remainder of total dose over 1 h total analgesia, thermoanesthesia, and partial loss of tactile sense. His right
leg is totally paralyzed and the muscles were spastic. Babinski sign was noted
in the right. Which vertebra was damaged?
a. T10
22-Grace is a 36-year-old female who sustained trauma on the vertebral 33-Which of the ff. is least likely associted with the CT image shown?
column after a vehicular accident. According to his orthopedic surgeon, she
has paraplegia in extension. Which of the following statements is true? a. Dejerine-Roussy syndrome
a. The condition is secondary to the damage of all descending tracts b. Hydrocephalus
immediately after the trauma c. Left hemiparesis
b. Overactivity of the gamma efferent nerve fibers d. Seizure
c. Result from damage of the vestibulospinal tract
d. All of the above

23-Which of the following structures is pain-insensitive?


a. Proximal 50% of the larger arteries of the circle of Willis
b. Venous sinuses
c. Dural arteries
d. Choroid plexus
34-Which of the following is not seen in spinal shock?
a. Loss of bladder and bowel function
24-Which of the following neuropathologic changes is observed in delayed b. Absent long tract signs
death secondary to methanol intoxication? c. Increased reflexes
a. Necrosis of the lateral aspect of putamen and claustrum d. Extreme back pain
b. Atrophy of cerebellar vermis
c. Atrophy and gliosis of the hippocampus 35-Which of the following mechanisms does hyperventilation play in
d. Laminar cortical necrosis reducing intracranial pressure?
a. Arterial vasodilation thereby reducing cerebral perfusion
25-Which of the following is NOT an indication for preventive treatment of b. Vasoconstriction thereby reducing cerebral blood volume
migraine? c. Increase O2 level thereby increasing metabolic activity
a. Acute therapy is ineffective, intolerable, and contraindicated d. All of the above
b. Attacks occur unpredictably
c. Acute therapy is needed more than 2x per week 36-Which of the following areas when lesioned causes anosognosia?
d. Two or more attacks/month that produce disability for > 3 days a. Left temporal lobe lesions
b. Left posterior parietal lesions
26-Rosana is a 79-year-old female admitted due to left sided weakness. On c. Diffuse cerebral disease
examination, she was noted to have irregularly irregular rate and rhythm of d. Right posterior parietal lesions
96/min. She is diabetic for 3 years with poor compliance to medications. Her
blood pressure at the ER was 180/90mmHg. You are now planning to start 37-Which of these drugs in the treatment of stroke can cause life-threatening
her with anticoagulant. She has history of taking Clopidogrel. Her creatinine agranulocytosis and neutropenia?
clearance is 27 mL/min. Her liver function is normal. What is her HAS-BLeD a. Cilostazol
score? – 5 b. Clopidogrel
c. Dipyridamole
27-A 16-year-old boy was admitted following a severe motor vehicular d. Ticlopidine
accident. He was paralyzed from the waist downward for the rest of his
life. Which of the following is not a common complication after his injury? 38-Jason is a 5-year-old boy admitted due to sudden onset of right sided
a. Bedsore body weakness. On digital subtraction angiography, a distinct hazy
b. Urinary infection area was noted over the left basal ganglia. What is your most likely
c. Nutritional deficiency diagnosis?
d. Pneumonia a. CADASIL
b. AV malformation
28-Which of the following is not part of the diagnostic criteria for migraine c. Moyamoya disease
with aura? d. Marfan syndrome
a. One or more fully reversible aura
b. Headache follows aura within 1 hour of the end of aura 39-Where is the lesion in a patient with unilateral miosis, ptosis, and facial
c. No single aura lasts longer than 4 hours anhidrosis?
d. At least one aura symptom that develops gradually over more than 4 a. CN VII
minutes b. CN III
c. The ciliary ganglion
29-Which of the following is likely to be an additional feature in a patient d. The carotid artery
with vertigo secondary to a brainstem stroke involving a single
vascular territory? 40-Rosana is a 79-year-old female admitted due to left sided weakness. On
a. Locked-in syndrome examination, she was noted to have irregularly irregular rate and rhythm of
b. Facial paresis 96/min. She is diabetic for 3 years with poor compliance to medications. Her
c. Skew ocular deviation blood pressure at the ER was 140/90mmHg. You are now planning to start
d. Facial anesthesia her with anticoagulant. What is her CHADs-VASc score? – 7

30-Which of the following is not a manifestation of CNS sarcoidosis? 41-In general, what is the most common cause in this condition?
a. Bitemporal hemianopsia
b. Hearing loss
c. Cauda equina syndrome
d. Diabetes insipidus

31-Which of the following occurs if the M2 segment of the right middle


cerebral artery is occluded?
a. Aprosodia
b. Abulia
c. Right hemianopsia
d. Loss of specific taste a. AV malformation
b. Hypertension
32-Which of the following is not observed immediately after complete c. Ruptured aneurysm
transection of the spinal cord at T6 level? d. Trauma
a. Loss of voluntary control of the bladder and bowel
b. Paralysis of volitional leg movements 42-What is the first layer in the retina to receive light?
c. Hyperactive muscle stretch reflexes a. Rods and cones
d. Loss of vasomotor reflexes in the legs b. Inner plexiform
c. Outer plexiform
d. Ganglion cells
43-Which of these areas will cause damage to CN III, CN IV, CN VI and the 50- 38-year-old female was complaining of severe persistent headache
ophthalmic division of CN V if an injury will happen? associated with nausea and vomiting. Upon looking at the mirror, her
a. Pontine tegmentum right pupil looked much larger than the left and her right upper eyelid
b. Midbrain tegmentum appeared to droop. Which of the following findings is least likely revealed
c. Cerebellopontine angle during examination?
d. Cavernous sinus a. Severe ptosis in the right eye

44-This is a CT image of a 58-year-old male with left sided weakness taken c. Bilateral papilledema
five hours after admission. Which of the following is true with respect to the d. Paralysis of superior oblique muscle on the right
CT image shown?
51-What is the primary receptor for detecting an edge?
a. Merkel disk
b. Pacinian corpuscle
c. Ruffini ending
d. Meissner corpuscle

52-What cell group in the primary motor cortex will react to points of light?
a. Blob cells
b. Complex cells
c. Simple cells
d. Stellate cells
a. Right hemianopsia.
53-Which of the following predisposing factors for brain abscess
b. Carotid dissection of the left internal carotid artery.
development will respond best to Vancomycin + Gentamicin treatment?
c. Hemicraniectomy is indicated.
a. Penetrating head trauma
d. Thrombolysis
b. Bacterial endocarditis
c. Lung abscess
45-What do you call a fibrin- rich clot?
d. Congenital heart disease
a. Yellow clot
b. Red clot
54-Which of the following cranial nerves do not form part in the control of
c. White clot
autonomic effectors?
d. Grey clot
a. III
b. VI
46-A 56-year-old man is found unresponsive. He was immediately rushed to
c. VII
the ER. Upon examination, the only finding is he has difficulty with saccades.
d. IX
If this is caused by a stroke, where is the lesion?
a. Interposited nuclei
55-A 58-year-old diabetic female who presented with slurring of speech
b. Superior colliculus
lasting for 2 hours and a blood pressure of 130/100mmHg. You computed
c. Fastigial nucleus
the ABCD2 score. What is the risk for developing stroke in the next seven
d. Inferior colliculus
days?
a. 8.1%
47-A 70-year-old man complains of difficulty swallowing, hoarseness of his
b. 4.1%
voice and giddiness. On physical examination, he was noted to have absent
c. 1.2%
gag reflex on the left side, loss of pain and temperature sense on the left
d. 5.9%
face, and left sided paralysis of the vocal cords. What is your most likely
diagnosis?
56-A 28-year-old drug addict was admitted due to fever, weakness of the
a. Medial medullary syndrome, left side
lower extremities, and urinary incontinence. Strength in the upper
b. Lateral medullary syndrome, left side
extremities is normal. MRI of the brain and spine was done and a diagnosis of
c. Medial medullary syndrome, right side
epidural abscess is made. What is the most likely location of
d. Lateral medullary syndrome, right side
the abscess?
a. Frontal convexities
48-Dario, a 68-year-old male admitted due to severe headache associated
b. Cervical spine
with elevated blood pressure. CT scan finding as shown. Which of the
c. Thoracic spine
following statements is correct?
d. Cauda equina

57-What is the best approach to treating cluster headaches?


a. Sumatriptan, 50mg orally at the onset of attacks
b. Oxygen at 10–12 L/min by nasal cannula for 30 minutes daily for 2 weeks
c. Indomethacin, 25 mg three times daily
d. Surgical consultation for microvascular decompression of the trigeminal
nerve

58-Which of these drugs inhibits uptake of adenosine into platelets and


endothelial cells?
a. Ticlopidine
b. Clopidogrel
c. Dipyridamole
a. The patient will develop hydrocephalus.
d. Cilostazol
b. Vasospasm pea
or not Nimodipine is started, Vasospasm
59-Which of the following type of cerebral edema is responsive to steroids?
will take place starting at 3 days- 14 th day
a. Cytotoxic edema
c. Maintain blood pressure at 110-130mmHg systolic.
b. Vasogenic edema
d. Cranial MRI with MRA is better than Transcranial doppler in determining
c. Interstitial edema
the severity of vasospasm
d. All of the above
49-Which of the following statement is not true with respect to the CT image
60-Which of the following statements characterizes the early capsule
shown?
formation in brain abscess?
a. Methylprednisolone
a. Prominent cerebral edema
b. Mannitol
b. Presence of numerous macrophages
c. Trauma
c. Increase necrotic center
d. VP shunting
d. Appearance of fibroblast and formation of reticulin

61-Which of the following phase in migraine is characterized by the presence


of psychological, neurological and autonomic disturbances?
a. Premonitory phase
b. Resolution phase
c. Aura phase
d. Headache phase
62-Which of the following antimicrobials in the treatment of CNS infection 73-Which of the following statements is not true concerning the medical
will Listeria monocytogenes and Nocardia asteroides responds management in patients with acute ischemic stroke?
best? a. Blood pressure should be lowered if there is malignant hypertension or
a. Metronidazole concomitant myocardial ischemia, or if blood pressure is >185/110 mmHg
b. Cotrimoxazole and thrombolytic therapy is anticipated.
c. Ceftriaxone b. When faced with the competing demands of myocardium and brain,
d. Vancomycin lowering the heart rate with β1-adrenergic blocker (such as Esmolol) can be a
first step to decrease cardiac work and maintain blood pressure. Routine
63-What is the risk of vasospasm after subarachnoid hemorrhage in a patient lowering of blood pressure has been
with a Hunt and Hess score of 4? found to worsen outcomes.
a. 22% c. Edema peaks on the second or third day but can cause mass effect for ~5
b. 33% days. The larger the infarct, the greater the likelihood
c. 53% that clinically significant edema will develop.
d. 74% d. Water restriction and IV Mannitol may be used to raise the serum
osmolarity, but hypovolemia should be avoided because this may
64-Antonio is a 46-year-old man complaining of symptoms of analgesia and contribute to hypotension and worsening infarction
thermoanesthesia on the medial side of the left hand for 6 months duration.
Few days later, he started complaining of severe numbness of the little finger 74-Which of the following CNS parasitic infections will have a finding of
and even sustained burn from not feeling the hot stove. On examination, he multiple enhancing lesions in the brain in neuroimaging?
has severe loss of pain and temperature sense involving the eight cervical a. Amebiasis
and first thoracic dermatomes of the left hand while tactile discrimination is b. Visceral larva migrans
perfectly normal in this area. Examination on the right side showed the same c. Toxoplasmosis
findings of less severity. Which tract or tracts is most likely involved? d. Trichinosis
a. Lateral spinothalamic tract
b. Fasciculus cuneatus and anterior spinothalamic tracts 75-Which of the following is not part of the diagnostic criteria for migraine
c. Lateral and anterior spinothalamic tracts without aura?
d. Fasciculus cuneatus and lateral spinothalamic tracts a. Associated with nausea, vomiting, photophobia and phonophobia
b. Headache lasting 4-72 hours
65-Which of the following risk factors predisposes one from developing c. Unilateral in location and pulsating in quality
primary noninfectious thrombosis of dural sinuses? d. At least 3 attacks
a. Dehydrated infants and postpartum young females
b. Heavy male smokers with known cardiac disease 76-Which of the following characterizes neurosyphilis?
c. Neonates in respiratory failure a. All forms of neurosyphilis begin as meningitis.
d. School age children after head trauma b. 40-50% will develop stroke secondary to meningitis.
c. Invasion of treponemes takes around 3-36 months.
66-Which of the following conditions is characterized by severe headache, d. Asymptomatic patients may present with relative afferent pupillary defect
seizure, hiccups and bruxism?
a. CNS schistosomiasis 77-A 58-year-old diabetic female who presented with slurring of speech
b. CNS malaria lasting for 2 hours and a blood pressure of 130/100mmHg will have an
c. CNS cysticercosis ABCD2 score of? – 5
d. CNS toxoplasmosis
78-Which of the following phase in migraine correlates to cortical spreading
67-Which of the following statements is not true about cerebral concussion? depression?
a. Reversible traumatic paralysis of the nervous system. a. Premonitory phase
b. Duration of anterograde amnesia is most reliable predictor of severity. b. Resolution phase
c. Tachycardia, hypotension and apnea may be noted. c. Headache phase
d. Loss of consciousness lasting from seconds to hours. d. Aura phase

68-A 3-year-old native girl was brought in to the ER with profound weakness 79-Which of the following conditions presents with dementia, gait imbalance
of the left arm with absent reflexes. The rest of the examination is normal. and pseudobulbar state secondary to multiple white matter infarctions?
What is the most likely causative viral agent? a. Pelizaeus-Merzbacher Disease
a. Enterovirus b. CADASIL
b. Herpesvirus c. Binswanger’s disease
c. Rabies d. Canavan’s disease
d. Measles
80-Which stroke symptom is unlikely to be related to the others?
69-Which is NOT a manifestation of occlusion of superior cerebellar artery a. Geographical disorientation
a. Ptosis of the left eye b. Mild sensory difficulty in the face
b. Dysarthria c. Flaccid plegia of arm and face
c. Numbness of the left arm and leg d. Dressing apraxia
d. Dysdiadochokinesia of the left extremities
81- Which of the following unilateral visual symptom is observed if an
70-Which of the following can help improve the diagnostic yield in CSF aneurysm develops in the anterior communicating artery?
analysis? a. Absent direct, but intact consensual light reflex
a. Submit large volume of CSF (>25 ml) for microbiology b. Absent direct and consensual light reflex
b. Centrifuge at high force (3000g) for 30 minutes and stain and culture the c. Dilated pupil
deposit d. Hemianopsia
c. Incubate at 37 C if available or store at 4 C if for transport.
d. Subculture after 24-hour inoculation in brain heart infusion broth at 37 C 82-A 70-year-old man was rushed to the ER with right sided weakness. His
with 5% CO2 blood pressure was 160/100mmHg. Capillary glucose test was noted at
278mg/dl. After about an hour, the weakness resolved. You computed his
71-Which of the following statements concerning head trauma is true? ABCD2 score. What is his risk of developing stroke 2
a. Contrecoup contusions are produced by deceleration of head in days and in 90 days?
backward fall that hit the occiput. a. 5.9% and 17.8%
b. Contrecoup and coup fractures are linked with each other. b. 11.7% and 17.8%
c. Contrecoup fractures frequently occur at the floor of the orbit and c. 4.1% and 9.8%
ethmoidal plate in occipital injuries. d. 8.1% and 17.8 %
d. Coup contusion results from a blow to a moving head.
83-Which of the following will be observed if there is a lesion affecting the
72-Which of the following statements concerning the lateral medullary corticopontine pathway at its origin in the cortex or along its course?
syndrome is correct? a. Ataxia
a. Contralateral trunk and extremity hypalgesia and thermoanesthesia may b. Difficulty breathing
occur. c. Coma
b. Damage of the contralateral nucleus ambiguus is notable. d. Pinpoint pupils
c. There may be ipsilateral analgesia and thermoanesthesia of the face
d. Common cause is thrombosis of the anterior inferior cerebellar artery.
84-Which of the following trematodes tend to cause localized spinal cord 96-Which of the following patients should be managed with antiplatelet
injury leading to myelitis? agent for stroke prevention?
a. Schistosoma japonicum a. A 70-year old man with atrial fibrillation, diabetes, hypertension, and
b. Schistosoma mansoni congestive heart failure.
c. Schistosoma haematobium b. A 48-year-old man with hyperthyroidism and intracranial atherosclerotic
d. Paragonimus stenosis.
c. A 45-year old woman with mechanical heart valve.
85-Which of the following statements concerning the medial medullary d. A 55-year old man who suffered an ST –elevation myocardial infarction last
syndrome is correct? week and has ejection fraction of 35% with anterior wall
a. Contralateral impaired sensations of position and movement. akinesis and left ventricular thrombus.
b. Contralateral facial paralysis
c. Common cause is thrombosis of posterior communicating artery. 97-In women, the amygdala is larger than men. Which of the following
d. Ipsilateral tongue and contralateral extremity paralysis stereotypes might be explained by this anatomic difference?
a. Women are more thoughtful about emotions than men.
86-Which of the following is not considered part of the general mechanism b. Women live longer than men.
of headache? c. Women worry more than men.
a. Distention and dilation of intracranial arteries d. Women seek more college degrees than men.
b. Traction on major intracranial vessels
c. Inflammation of the brain parenchyma 98-Which of these muscles is involved in patient with Bell’s palsy?
d. Direct pressure on cranial or cervical nerves a. Orbicularis oculi
b. Levator palpebrae
87-Which of the following happens if there is a lesion in the inferior calcarine c. Superior tarsal
cortex? d. Pupillodilator
a. Contralateral upper quadrantanopsia
b. Contralateral lower quadrantanopsia with macular sparing 99-Richard is a 7-year-old boy admitted due to TB meningitis. He weighs
c. Contralateral upper quadrantanopsia with macular sparing 35kg. What is his daily requirement for the following anti-TB
d. Contralateral lower quadrantanopsia medications? Isoniazid, Ethambutol, Rifampicin and Pyrazinamide.
a. 525mg- 1, 225mg- 175mg- 350mg
88-Which of the following statements is considered part of migraine b. 175mg- 350mg- 525mg-
management? c. 350mg- 175mg- 525mg – 1, 225mg
a. Regularization of meals, sleep, and exercise d. 175mg- 525mg- 350mg- 1,225mg
b. Avoidance of migraine triggers
c. Avoidance of overuse of analgesics 100-Which of the following characterizes the ptosis in Horner’s syndrome?
d. All of the above a. Being more pronounced when the patient looks up.
b. Not changing when the patients look up
89-Which of the following is defined as a mental and behavioral state of c. Disappearing or improving when the patients look up.
reduced comprehension, coherence, and capacity to reason? d. Being better at rest
a. Malingering
b. Coma
c. Confusion
d. Delirium

90-In the management of a patient with subarachnoid hemorrhage and a


GCS score of 4. Which of the following statements is true?
a. With GCS 4, surgery is never warranted.
b. With GCS 4, surgery is always warranted.
c. Surgery is always warranted for Hunt and Hess grade 5.
d. Surgery is never warranted for Hunt and Hess grade 5.

91-Which of the following cranial nerves is not involved in the control of


somatic skeletal muscles?
a. III
b. IV
c. X
d. XII

92-A 70-year-old man was rushed to the ER with right sided weakness. His
blood pressure was 160/100mmHg. Capillary glucose test was noted at
278mg/dl. After about an hour, the weakness resolved. What is his ABCD2
score? – 7

93-What sensation is affected in a lesion involving the ventroposterolateral


nucleus of the thalamus?
a. Contralateral proprioception and ipsilateral temperature sensation
b. Ipsilateral proprioception and contralateral temperature sensation
c. Contralateral temperature sensation and contralateral proprioception
d. Ipsilateral proprioception and ipsilateral temperature sense

94-Which of the following organisms causing CNS infection will respond


strongly with fluoroquinolones?
a. M. tuberculosis
b. Streptococcus pneumoniae
c. L. monocytogenes
d. N. Meningitidis

95-A 26-year-old boy was referred to you complaining of double vision. Upon
examination, the double vision became worse in looking to the
right. You also noted mild motor paralysis of the left extremities and a mild
peripheral facial paralysis on the right. Which of the following statements is
least likely to cause the symptoms?
a. Imaging may reveal the presence of a small tumor of the lower part of the
right side of pons.
b. The mild left hemiparesis is produced by the damage of the left
corticospinal tract.
c. There was damage to the right CN VI nucleus.
d. The complete right-sided facial paralysis caused by involvement of the
right CN VII nucleus or its nerve
QUESTION RATIONALE
1) This drug is utilized to prevent vasospasm in Harrison’s neurology, chap 28
patients with subarachnoid hemorrhage
 NIMODIPINE

2) Amyloid angiopathy in the elderly is a risk Harrison’s neurology, chap 27


factor for developing
 INTRACEREBRAL HEMORRHAGE

3) Which of the ff. statement is true


regarding the CT image shown?
 It is
usually caused
by rupture of
dural arteries
4) A 19yo man has sustained a head injury in
a motorcycle accident. His initial GCS score
was 3 and he is intubated and sedated. An
ICP monitor is placed and the initial reading is
elevated. Which of the ff. interventions is
most likely to be beneficial?
a. Flattening the head of his bed
b. INCREASING HIS SEDATION
c. Hyperventilation to a pCO2 25
mmHg
d. Corticosteroid

Harrison’s neurology, chap 28


5) Subpial encephalopathy in acute bacterial Doc Ruben’s CNS infection ppt
meningitis is manifested with the ff. EXCEPT
a. SEIZURE
b. Confusion
c. Stupor
d. Vasculitic infarctions
6) A lesion that would affect CN III, CN IV, CN Harrison’s neurology, chap 34
VI and the ophthalmic division of CN V would
most likely be in the?
a. CAVERNOUS SINUS
b. Midbrain tegmentum
c. Pontine tegmentum
d. Cerebellopontine angle

7) A patient with dyscalculia, dysphragia, Adams: answer should be LEFT ANGULAR GYRUS
right-left disorientation, and finger agnosia
will have a lesion in this area
a. RIGHT PARASYLVIAN AREA
b. Left angular gyrus
c. Medial inferior temporo-occipital
region
d. Inferior medial quadrants of
temporal lobe

8) A patient with Bell’s palsy would have Bell’s palsy – paralysis of CN 7


paralysis of which of these ocular related
Unilateral, swelling of CN VII on bony canal  pressure on nerve fibers  lower motor
muscles? neuron-type of paralysis
a. ORBICULARIS OCULI
b. Levator palpebrae
c. Superior tarsal
d. Pupillodilator

9) Which of the following statements are Adams chap 35


correct regarding brain injury?
a. If intracranial pressure is more than
20 mmHg, vasopressor agents
should be used regardless the level
of BP
b. Post-traumatic seizure is more
common in children than in adult
c. Burr hole operation is appropriate
management for patients with acute
subdural hematoma
d. Diffuse axonal injury is the single
most important cause of persistent
disability after traumatic brain injury

10) Which of the ff. inflammatory reactions Harrisons neuro chap 43


occur with Creutzfeldt-Jacob disease
a. NO INFLAMMATION
b. Granulomatous
c. Eosinophilic
d. Lymphocytic

CJD is degenerative
11) Fracture of what structure has the
highest risk of potentially impairing vision?
 CRIBRIFORM PLATE

12) A 56yo male is admitted to the ICU with


hypertensive crisis after cocaine use. Initial
blood pressure is 220/130. On PE the patient
is unresponsive except for extension of upper
extremities to painful stimuli. He has been
intubated for airway protection and is being
mechanically ventilated, with a RR of 14. His
pupils are reactive to light, and there are
normal corneal, cough, and gag reflexes. The
patient has a dense left hemiparesis. CT
reveals a large intracranial bleeding in the
right frontoparietal area. Over the next
several hours the patient deteriorates. The
most recent examination reveals BP of
180/100. The patient now has a dilated pupil
on the right side. The patient continues to
have corneal reflex. You suspect rising ICP
related to the intracranial bleed. What is the
GCS of the patient?
 4

13) All the ff. conditions are prediposing Adams chap 32


factors for meningitis with Streptococcus
pneumoniae EXCEPT
a. CEREBRAL PALSY
b. Sickle cell disease
c. Alcoholism
d. Asplenism

14) A patient shows paralysis of palatal Harrisons neuro chap 34


elevation, the vocal cords, and the trapezius Table 34-4: Jugular Foramen – CN IX, X, XI

and SCM on the right. The gag reflex is also


absent on the right side and he has moderate
difficulty swallowing. The lesion that can
possibly expain the above findings is most
likely located in?
a. RIGHT JUGULAR FORAMEN
b. Right cavernous sinus
c. Right cerebral peduncle and
interpendicular fossa
d. Left cerebellopontine angle

15) A large destructive lesion of the left


posterior frontal region would result in?
 DEVIATION OF EYES TO THE LEFT

16) In the management of brain abscess, Metronidazole interacts with the microbial DNA to break its strand and
metronidazole is considered part of the helical structure leading to inhibition of protein synthesis, degradation, and
empiric treatment. What is the mechanism of cell death.
action of metronidazole? -MIMS
 DNA strand breakage

17) A patient, after a head injury that Adams chap35


contused the right orbit, complains of
diplopia when looking down and to the left.
The diplopia most likely represents weakness
of?
 Right superior oblique or left
inferior rectus muscle

Trochlear nerve – superior oblique


Oculomotor nerve – inferior rectus
18) All of the ff. statements are true of Adams
chronic subdural hematoma, EXCEPT?
a. It is radiologically characterized by
a more homogenous appearance
compared to an epidural
hematoma
b. Hematoma progressively enlarges
because of recurrent bleeding from
shearing of capillaries
c. Classically, this results from mild
head trauma in the elderly,
alcoholics and infants
d. “Neuromembranes” develop from
periphery, always seen by 8 weeks

19) Tesitng the sense of taste would be most


important in a patient presenting with?
 Hemifacial paralysis

20) A platelet-fibrin rich clot is also known


as?
 White clot

21) In a patient with increase ICP, the ff. can MAP goal in IM plat is 110-130 mmHg
be done EXCEPT? Nitroprusside is NOT used as a BP lowering agent

a. Initiate intravenous Nitroprusside


to decrease the mean arterial
pressure to a goal of 100 mmHg
b. Administer IV mannitol at a dose of
1g/kg body weight
c. Administer hypertonic fluids to
achieve a goal sodium level of 155
to 160 mEq/L
d. Consult neurosurgery for an urgent
ventriculostomy

22) A 72/M with DM presents with fever, ppt ni doc flores


headache and altered sensorium. On PE his
temperature is 40.2 C, HR is 103 beats/min.,
BP is 80/50 mmHg. His neck is stiff and he
has photophobia. CSF gram stain is negative.
What is your most likely diagnosis?
Appearance: clear
Opening pressure: 230 mmHg
WBC: 2100 (neutrophils)
Protein: 78 mg/dl
Sugar: 10 mg/dl
Gram stain: negative
 BACTERIAL MENINGITIS

23) An operation that would abolish


automatic breathing is?
 A sagittal cut limited to the dorsal
part of the cervicomedullary
junction

24) Meningitis after cranial surgeries is ppt ni doc


usually due to the ff. organisms, EXCEPT?
a. Group B streptococci
b. S. areus
c. Group A (Streptococcus pyogenes)
d. S. aureus

25) Which of the ff. antimicrobials has the vancomycin has limited ability to cross the blood-brain barrier, and
ability to penetrate the blood brain barrier is concern has been raised that steroid use might further reduce its
decreased by dexamethasone? penetration into the cerebrospinal fluid (CSF) by decreasing
 Vancomycin meningeal inflammation
- Journal
26) Aseptic meningitis can be due to the ff.
EXCEPT?
a. ALCOHOLISM
b. Systemic lupus erythematosus
c. Vasculitides
d. Neoplasm
27) Brain herniations secondary to mass Harrisons neuro chap 17
effect can lead to coma and death due to
brainstem compression. Which of the ff.
types is the LEAST likely to cause brainstem
compression?
a. CINGULATE HERNIATION
b. Transtentorial herniation
c. Cerebellar tonsillar herniation
d. Uncal herniation

28) Which of the ff. is an inherited ischemic Adams CVD


white cerebral white matter disorder
characterized by concentric arteriolar fibrosis
and hyaline thickening?
a. CADASIL
b. Pelizaeus-Merzbacher Disease
c. Canavan’s disease
d. Binswanger’s disease

29) Which of the ff. mechanism does Adams


hyperventilation play in reducing intracranial
pressure?
a. Vasoconstriction thereby reducing
cerebral blood volume
b. Increase O2 level thereby increasing
metabolic activity
c. Arterial vasodilation thereby
reducing cerebral perfusion
d. AOTA

30)Which of the ff. statements best describe


the management of acute ischemic stroke?
a. Aspirin is the only antiplatelet
agent that has been proven
effective for the acute treatment of
ischemic stroke
b. Trials support the use of heparin or
other anticoagulants for patients
with atherothrombotic stroke
c. Hyperthermia is a powerful
neuroprotective treatment in
patients with cardiac arrest in
animal models
d. AOT

31) After interruption of the corpus callosum, Harrisons neuro chap18


the patient would be unable to?
 Execute a verbal command with the
left hand
32) Blood collateral flow within the ischemic Harrison neuro
brain may be BP dependent, there is
controversy about whether BP should be
lowered acutely. Which of the ff. statements
in the management of acute ischemic stroke
is INCORRECT?
 Edema peaks on the 2nd or 3rd day
but can cause mass efect of ~5
days. The larger the infarct, the
greater the likelihood that clinically
significant edema will develop

33) A patient who complained of night


blindness and inability to see objects in the
periphery of the visual fields would have a
lesion affecting which retinal cell type?
 Rod cells

34) If the patient fails to elevate on the right Harrisons neuro chap 34
side when a patient says “Ah”, one would
suspect a lesion of the?
 Right vagus nerve

35) Given these CSF analysis results, what is ppt ni doc


your possible diagnosis?
Appearance: clear
Opening pressure: 150 mmHg
WBC: 200 (lymphocytes)
Protein: 145 mg/dl
Sugar: 70 mg/dl
Gram stain: Negative
 VIRAL

36) Which of the ff. is the earliest


histopathological finding in cerebral
infarction?
 Red Neuron
37) Multiple brain abscesses are most likely
seen in which of the ff. scenario?
a. In a patient with congenital heart
disease
b. In patient with chronic otitis media
c. In patient who underwent
ventriculoperitoneal shunting (VPS)
d. In patient with dental infection
38) Empiric therapy for bacterial meningitis ppt ni doc
in patient age 0-4 weeks includes?
 Cefotaxime plus Ampicillin

39) What blood vessel is most likely involved


in this CT scan image?
 Right
internal carotid
artery

40) Surgical management of brain abscess is The question asks for a CONTRAINDICATION
indicated in the ff. circumstances EXCEPT? o Multiple abscess
o Abscess location is deep and affect dominant locations
a. Multiple abscess
o There is a co-existing meningitis or ependymitis
b. Size less than 3cm
o Improvement or reduction of abscess after antimicrobial therapy
c. There is no co-existing meningitis or o Size less than 3cm
ependymitis
d. Abscess location is in the non-
dominant hemisphere

41) Which of the patients should be managed


with antiplatelet agent for stroke
prevention?
a. A 48yo man with hyperthyroidism
and intracranial atherosclerotic
stenosis
b. A 45yo woman with mechanical
heart valve
c. A 70yo man with Afib, diabetes,
hypertension, and congestive heart
failure
d. A 55yo man who suffered an ST
elevation MI last week and has
ejection fraction of 35% with
anterior wall akinesis and left
ventricular thrombus

42) A patient who lost his recent memory will


have a lesion in this area
a. Inferior medial quadrants of
temporal lobe
b. Medial inferior temporo-occipital
region
c. Right parasylvian area
d. Left angular gyrus

43) The diagnostic criteria for migraine


without aura from the International
Headache Society includes the ff. EXCEPT?
a. At least 3 attacks
b. Associated with nausea, vomiting,
photophobia and phonophobia
c. Unilateral in location and pulsating
in quality
d. Headache lasting 4-72h

44) The risk for stroke in a patient with TIA


and an ABCD2 score of 4 is?
 9.8% in the next 90 days
45) A 68yo man has had painful paresthesia Adams
in the left side of his body for 3 years
following a stroke. His primary care physician
thinks that there must be another
explanation, as a stroke cannot produce pain.
Infarction in what vascular territory will most
likely produce pain?
 Thalamogeniculate
thalamoperforators of the posterior
cerebral artery
46) The ff. are exclusion criteria for
thrombolysis in stroke EXCEPT?
a. BP of 170/100 mmHg
b. Hysterectomy two weeks before
onset of stroke
c. Patient with slurring of speech
d. Meningioma

47) Behavioral modification in migraine ppt ni doc


management includes:
a. Avoidance of migraine triggers
b. Regularization of meals, sleep, and
exercise
c. Avoidance of overuse of analgesics
d. AOTA

48) If the patient’s left eye fails to adduct


when attempting to look to the right, but the
eye adducts during covergence, the lesion is
in the?
 Left medial longitudinal fasciculus

49) A 42yo woman presents with a sudden


onset of right-sided facial weakness,
weakness of the right arm and some
weakness of the right leg. Further
examination revealed that the patient is also
aphasic. If this patient’s symptoms are due to
mass effect, which of the ff. mechanisms
could produce the symptoms?
a. Increase in ICP
b. Compression and destruction of
tissue
c. Displacement of structures and
herniation
d. AOTA
50) A state defined as a mental and
behavioral state of reduced comprehension,
coherence, and capacity to reason
 Confusion

51) Pain sensitive structures include the ff. ppt ni doc


EXCEPT?
a. Dura over convexity
b. Venous sinuses
c. Dural arteries
d. Proximal 50% of the larger arteries
of the circle of Willis

52) What is your diagnosis? Harrisons


 Right superior quadrantanopsia

53) Location of lesion in #52


 Temporal optic radiation

55) Which of the ff. statement is TRUE in the Harrisons


management of acute bacterial meningitis?
a. Empirical therapy should be
administered promptly to reduce
mortality and morbidity
b. An imaging study is necessary at all
cost to avoid herniation
c. High dose steroid is indicated in all
cases of meningitis
d. Lumbar puncture at the ER is
necessary to identify the causative
agent as soon as possible

56) Which of the ff. is LEAST likely to seen in Harrisons


patients with HSV encephalitis?
a. Low CSF glucose concentration
b. Headache, fever, altered level of
sensorium as the presenting
symptoms
c. Lymphocytic pleocytosis in the CSF
analysis
d. Focal or generalized seizure

57) BP of 190/80mmHg will have a MAP of?


 116 mmHg

58) The cortical spreading depression in


classic migraine clinically relates to?
 Aura phase

59) Which of the ff. is a direct thrombin


inhibitor?
a. Dabigatran
b. Rivaroxaban
c. Endoxaban
d. Apixaban

60) Cysticercosis is produced in the larval


stage of?
 Taenia solium

61) The ff. are TRUE about the ppt ni doc


histopathologic changes in the early capsule
formation of the brain abscess EXCEPT?
a. Increase in cerebral edema
b. Evolution of mature collagen along
the inflammatory border
c. Increase in the number of fibroblast
and macrophages in the
inflammatory border
d. The degree of neovascularization is
maximal

62) Treatments designed to reverse or lessen Harrisons


the amount of tissue infarction and improve
clinical outcome
a. Endovascular revascularization
b. IV thrombolysis
c. Medical support
d. AOTA

63) The psychological, neurological and ppt ni doc


autonomic disturbances as a result of
hypothalamic dysfunction is observed
during?
 Premonitory phase

64) A patient with prosopagnosia will have a Prosopagnosia (from Greek prósōpon, meaning "face", and agnōsía, meaning "non-
lesion in this area knowledge"), also called face blindness,[2] is a cognitive disorder of face perception in which
the ability to recognize familiar faces, including one's own face (self-recognition), is impaired,
a. Medial inferior temporo-occipital while other aspects of visual processing (e.g., object discrimination) and intellectual
region functioning (e.g., decision-making) remain intact.
b. Right parasylvian area
c. Inferior medial quadrants of Acquired prosopagnosia results from occipito-temporal lobe damage and
is most often found in adults.
temporal lobe
d. Left angular gyrus

65) General mechanism of headache includes ppt ni doc


the ff. EXCEPT?
a. Inflammation of the brain
parenchyma
b. Traction on major intracranial
vessels
c. Distention and dilation of
intracranial arteries
d. Direct pressure on cranial and
cervical nerves

66) The ff. are possible findings in this patient The CT scan image is of a patient with Dejerine-Roussy syndrome
EXCEPT?
a. Altered
taste sensation
b. Severe
numbness over
the right
extremities
c. Seizure
d. Dejerine
-Roussy
syndrome

67) Ptosis in Horner’s syndrome is


characterized by?
 Disappearing or improving when
the patient looks up

68) The site at which the smallest lesion can


produce complete loss of consciousness with
minimal effects on other functions is the?
 Rostral midbrain tegmentum

69) Rosana is a 72yo female admitted due to


left sided weakness. On examination, whe
was noted to have irregularly irregular rate
and rhythm of 96/min. She is diabetic for 3
years with poor compliance to medications.
Her BP at the ER was 140/90mmHg. You are
now planning to start her with anticoagulant.
What is her CHADs-VASc score?
 6

70) The diagnostic criteria for migraine with IHS guidelines


aura from the IHS includes at least 2 attacks
with at least 3 of the ff. associated signs and
symptoms EXCEPT?
a. No single aura lasts longer than 4
hours
b. One or more fully reversible aura
c. At least one aura symptom that
develops gradually over more than 4
min.
d. Headache follows aura within 1 hr of
the end of aura

71) Which of the ff. is the most common Harrisons


manifestation of small-vessel stroke Lacunar syndrome is a small vessel stroke

syndrome?
a. Dysarthria and clumsy hand or arm
due to infarction in the ventral
pons or in the genu of the internal
capsule
b. Pure motor hemiparesis from an
infarct in the posterior limb of the
internal capsule of the pons; pure
sensory stroke form an infarct in the
somatosensory area
c. Ataxic hemiparesis from an infarct in
the cerebellum
d. AOTA

72) A 42yo woman presents with a sudden


onset of right-sided facial weakness,
weakness of the right arm and some
weakness of the right leg. Further
examination revealed that the patient is also
aphasic. Which artery (or major branch) has
probably been occluded in this patient?
 Left middle cerebral artery
73) In diffuse axonal injury (DAI), small
intracerebral hematoms are often seen in?
 Corpus callosum
74) A 62yo diabetic female who presented
with slurring of speech lasting for 2hrs and a
BP of 150/90mmHg will have an ABCD2 score
of?
 6

75) The examination finding of dilated pupil


on the right side is indicative of?
 Uncal herniation

76) Assuming support of breathing and blood


pressure, if required by the site of the lesion,
which surgical lesion would abolish
consciousness?
 Transection of the midbrain
tegmentum

77)A decrease in cerebral blood flow to zero Harrisons


causes death of brain tissue within 4-10 min.;
which of these values cause infarction within
an hour?
 Values <16-18 ml/100g tissue per
minute

78) A patient with Bell’s palsy complains of


excessive loudness of sounds. This finding
indicates paralysis of which muscle?
 Stapedius muscle

79) Rosana a 72yo female admitted due to


left sided weakness. On examination, her BP
is at 130/90mmHg. She is diabetic for 3 years
with poor compliance to medications. Her
cranial CT scan showed infarction in the right
basal ganglia. What antiplatelet would you
recommend her to start?
 Cilostazol

80) The drug of choice in the treatment of


Cryptococcal meningitis is?
 Amphotericin B

81) Anosognosia is most characteristic of


which of the ff. lesions?
 Right posterior parietal lesions

82) A decerebrate posture generally indicates


a lesion of the?
 Midbrain

83) Indications for preventive therapy in ppt ni doc


migraine includes the ff., EXCEPT?
a. Attack occur unpredictably
b. Acute therapy is ineffective,
intolerable, and contraindicated
c. Two or more attacks/month that
produce disability for >3days
d. Acute therapy is needed more than
2x per week

84) A 69yo man is brought to the ER after


sustaining a head injury in a motor vehicle
accident. On presentation, his GCS is 8.
Which of the ff. would prompt insertion on
an ICP monitor?
a. Evidence of intracranial
hemorrhage
b. Hypotension
c. Decerebrate posturing
d. His age

85) The classic clinical triad of brain abscess harrisons


includes the ff. EXCEPT?
a. Seizure
b. Focal neurologic signs
c. Headache
d. Fever
86) This patient will most likely present with
the ff. findings EXCEPT?
a. Left arm and leg weakness
b. Dysarthria
c. Right homonymous hemianopsia
d. Apraxia

87) The most common cause of CNS fungal


infection is?
 Cryptococcus neoformans
88) Methods to improve diagnostic yield in
CSF analysis includes the ff., EXCEPT?
a. Sumbit large volume of CSF (>20
ml) for microbiology
b. Examine CSF before or shortly after
antibiotics started
c. Gram stain and inoculation for
culture as soon as possible
d. Centridufe at high force (3000g) for
20 min. and stain and culture the
deposit

89)The ff. can trigger migraine attacks


EXCEPT?
a. Low altitude
b. Bright lights
c. Menstruation
d. Chocolate
90) Neuroimaging prior to lumbar puncture
in suspected menningitis is indicated in the
ff., EXCEPT?
a. Positive Kernig’s sign
b. Focal neurologic abnormality
c. Depressed consciousness
d. Known CNS mass lesion

91) The ff. drugs are used in migraine for


preventive pharmacotherapy, EXCEPT?
a. Sumatriptan
b. Metoprolol
c. Methysergide
d. Propanolol

92) Wallenberg syndrome is due to occlusion


of the vertebral artery or PICA. Also called
the lateral medullary syndrome, this consists
of the ff. neurologic deficits EXCEPT?
a. Ipsilateral loss of pain and
temperature
b. Incoordination
c. Ipsilateral facial sensory loss
d. Ipsilateral Horner’s syndrome

93) The ff. are correctly paired EXCEPT?


a. Global aphasia/left inferior parietal
area
b. Motor aphasia/left posterior-
inferior frontal region
c. Auditory agnosia/left superior
temporal gyrus
d. Dyslexia/lateral surface of the left
occipital lobe
94) What is your diagnosis?
 EPIDURAL HEMATOMA

95) A patient with a skull fracture through


the floor of the right middle fossa has no tear
secretion on that side. The most likely cause
is damage to the?
 Greater superficial petrosal nerve

96) In the locked-in syndrome, the lesion


usually involves the?
 Midbrain or pontine basis

Penetrating arteries to the medial and paramedial portions of the pontine


base and tegmentum originate from the basilar artery at a number of
different levels.
97) A patient with unilateral miosis, ptosis,
facial anhidrosis would most likely have a
lesion of?
 Carotid artery
98) Decerebrate rigidity can be abolished by
transection of the?
 Vestibulospinal tract

99) Clopidogrel is prescribed as stroke


prophylaxis for a 79yo man. You check a
complete blood count every 2 weeks during
the first 3 mos. of treatment to look for
which of the ff.?
a. Neutropenia and
thrombocytopenia
b. Anemia and neutropenia
c. Anemia and leukocytosis
d. Neutropenia and thrombocytosis

100) What is the best approach to treating


cluster headaches?
 Sumatriptan, 50mg orally at the
onset of attacks
ByCLINICAL
1. . A 28 year old female was
referred for evaluation for
thrombocytopenia. She denies
symptoms except ​cough and cold 2
weeks​ PTC, ​petechiae on both
legs​. She denies any other illness.
CBC was normal​ but ​platelets is
28,000​. PBS was normal except for
the decreased platelets which are
slightly large. which of the ff. Is ​not
correct​ management?
a. Admit pt for platelet
transfusion
b. Advice patient with activity
modification to avoid trauma
c. Prednisone 1 mg/kg/day
d. Test patient for HIV

2. What disease has Pentad findis


that include microangipathic
hemolytic anemia,
Thrombocytopenia,renal failure,
neurologic findings, and fever?
TTP
HUS

3. Most common inherited bleeding


disorder
a.) Hermansky Pudlak syndrome
b.) Bernard Soulier Syndrome
c.) Hemophilia A
d.) ​Von willebrand Disease
4. True about Hemophilia except
A. X-linked recessive
hemorrhagic disease due to
mutations in the F8 or F9
gene
B. M>F and women are
symptomatic
C. Prolonged aPTT but normal
platelet count and bleeding
time
D. Diagnosis is made by

5.

6.
7. In a non transfusion therapy of
hemophilia, this is a synthetic
vasopressin analogue that increases
factor VIII and VWF but not factor
IX. through mechanism of release of
endothelial cell
a. Recombinant interleukin X1
b. DDAVP
c. Tranexamic
d. Norepinephrine

8.

9.
What are the [​MOST COMMON​]
causes of DIC?
Options:
A.Bacterial infection,
B. Pregnancy Complications
C. malignant disorder,
D. All of the above

10.
11. ​CENTRAL​ mechanism of DIC
a. Consumption of platelets and
coagulation factors
b. RBC damage and hemolysis
c. Uncontrolled thrombin
generation
d. Fibrin deposits in
microcirculation

12. Not a part of the criterion for


Hemolytic Anemia
a. Jaundice
b. Normo Macrocytic anemia
c. Reticulocytosis
d. Hyperbilirubinemia

13.Which of the following is true


about G6PD
a.Males have only one G6PD gene
b.Females have two G6PD gene
c.Most of mutations are single
missense point mutations
d.AOTA
14. Pt came in with body malaise
and weakness, pale. What lab
results indicate acute hemolytic
attack?
A. Normocytic hypochromic,
hemoglobinemia,hemoglobin
uria, low LDH
B. Microcytic hypochromic,
hemoglobinemia,
hemoglobinuria, high LDH
C. Normocytic normochromic,
hemoglobinemia,
hemoglobinuria, high LDH
D. Microcytic hyperchromic,
hemoglobinemia,
hemoglobinuria, high LDH

15. (Something like) Blood smear


description of G6PD deficiency HA
A. Anisocytosis, polychromasia,
spherocytes

16. True about G6PD except NADPH


A. G6PD gene is X-linked
B. increased risk of Neonatal
Jaundice
C. In G6PD-deficient red cells,
NADH production is limited
17.blood transfusion need not be
type specific.
A. AIHA
B. G6PD def
C. hereditary spherocytosis
D. Pyruvate kinase def
Case
Pt wt RUQ pain, splenomegaly,
jaundice, gallstone. PBS normocytic,
Inc MCHC. Grandfather died with
same condition 5yrs ago

19. Initial impression


A. H. St
B. HE
C. HS
D. PKD

Actually, the answer for this could either be HE or


HS given the similarities in presentation (but the
question is initial impression)
20. After the test, it was found out
that there is mutation in EPB41.

Ans. Hereditary elliptocytosis

21.

22.

23. ↓ serum iron, ↓ % saturation with answer B.


normal rbc morphology Figure 93-2 page 685 Harrison's 20th edition

a. normal erythropoiesis Remember that ​IRON-DEFICIENT


b. negative-iron balance ERYTHROPOIESIS is defined by:
c. iron-deficient erythropoiesis ● ↓Serum iron (more than negative iron balance)
d. iron-deficiency anemia ● ↓ %Transferrin sat
● ↑ RBC protoporphyrin
These are not found in Negative-iron balance.
24. The treatment/s for the condition Not sure if IDA ni na case please advise
in #23 is? P. 686 harrisons 20th ed
​A. Oral iron therapy
B. Whole blood transfusion
C. Stem cell transplant
D. IV Dextran
25. 50 yo woman w/ malaise fatigue
weakness
Spastic, loss of vibratory sense... in
the lower Extremities

What deficiency?
A.
B
C. B6
D​. ​B12

26.Vit b12 lab findings


Mao ni ang mga labs. D nko
mremember ang specific choices
pero mao ni ang labs:
Hgb, mcv, leukopenia,
thrombocytopenia

27. What is the management for the


case ?
A. Lifetime vitamin supplement,
SC
B. Lifetime vitamin supplement,
PO
C. Lifetime vitamin supplement,
IM
D. All of the above
28.Definite cause sa hemolysis sa
g6pd patient
a. Quinine
b. dapsone
c. ciprofloxacin

29. Mgt of previous case (DIC)


except
a. tx of underlying dse
b. ffp and platelet
c ​heparin
d none of the above
30.all of the three condition cause
microcytic hypochromic except:
a. B thalassemia
b. Hypoxia erythrocytosis
c. Polycythemia vera
d. Iron deficiency anemia

31.

32. Which of the following is /are not


true of hereditary spherocytosis?
A. The main clinical findings are
jaundice, an enlarged
spleen, and often anemia.
B. Severe cases may present in
infancy with severe anemia
C. Increased MCHC on an
ordinary blood count
D. All of the above
33.Management for spherocytosis Harrison 20th Edition Page 712
anemia
A.splenectomy, moderate anemia
B. ​Delay splenectomy until puberty if
moderate, at 4-6years.old, if severe
C. Antipneumococcal vaccine before
splenectomy is unwarranted
D. Nota

34. Triad in PNH except:


A. Jaundice
B. Pancytopenia
C. Venous thrombosis
D. Intravascular hemolysis

35.

36. Atypical HUS


a. Microangiopathic HA,
thrombocytopenia
37.Used to diagnose March
Hemoglobinuria:

a. Blood culture
b. Peripheral blood smear
c. Targeted history taking
d. Donath-landsteiner antibody

38. Triad of AIHA


a. -----, jaundice, mild anemia
b. hepatosplenomegaly, severe
anemia, thrombocytopenia
c. hemoglobin as low as <4
g/dL, jaundice, and enlarged
spleen
d. jaundice, severe anemia,
thrombocytopenia

39. A 33 y/o chinese woman with


low hematocrit, healthy no other
complaints. The bout during her
childhood but with no bleeding
tendencies. Her brother has the
same case. PE reveal pale
conjunctiva; Hgb 9.4 g/dl, Hct 28.9.
Iron level Normal.

What is the diagnosis?


A. Hemoglobin C
B. Iron deficiency
C. Alpha Thalassemia
D. Anemia of chronic disease

40.what is the treatment for this


patient? (Refer to #39)

A) no treatment indicated
B) Blood transfusion
C) Iron supplement
D) Splenectomy
41. Premature release of
reticulocyte from the bone marrow
a. ​Inc. EPO stimulation
b. Necrosis filtration
c. Fibrosis
d. AOTA

42.true about the test of iron


transport and storage
a. % transferrin saturation is
calculated by dividing TIBC
by pxt iron x 100
b. Serum ferritin is not an
accurate indicator of total
body iron stores
c. Serum ferritin of 10-15%
indicates depleted iron stores
43. Classification of anemia with ret
index of <2.5 with maturation
disorder
Answer: folate deficiency

All other choices are not under


maturation disorders
44. Classification of anemia with ret.
index of <2.5
A. Thalassemia
B Hemoglobinopathies
C Sideroblastic
D Anaplastic

45.

46. Skin and mucous membranes


become pale when the hgb level is
less than
A. 70-80 g/L
B. 75-85 g/L
C. 80-100 g/L
D. 90-100 g/L
47. Which is true about red cell page 393 (sa book mismo) Harrison 19th ed.
indices...
A. Microcytosis:low MCH
B. MCV is sensitive to small
populations of microcytes and
macrocytes
C. Macrocytosis: high MCV
D. MCV: defects in Hb synthesis

Sorry kalimot kos wording sa A pero


murag ang correct answer ani kay
C.

48. Prolonged aPTT and PT


A.
B.
C. Early Vit K deficiency
D. Direct thrombin inhibitors

49. True about platelet function and


thrombosis assays except?

-bleeding time has been found to


predict bleeding risk with surgery

50.In hypoproliferative anemia, for


renal anemia is? (bsta something
in.ana huhu sowee)
a.​normocytic, normal TIBC
b.normocytic high saturation
c.normocytic, high MCV
d.normocytic, low iron
51. Acquired intracorpuscular
hemolytic anemia?
A. Hemoglobinopathies
B. Paroxysmal Nocturnal
Hemoglobinuria​ (Answer)
C. Familial (Atypical) Hemolytic
Uremic Syndrome
D. All of the above

52. risk of clinical hemolysis of


G6PD patients

A.
B.
C. ciprofloxacin
D. ​dapsone

53. Underlying systemic diseases


that cause or exacerbate a bleeding
tendency except

A. Hyperthyroidism
B. Liver disease
C. Severe renal impairment
D. Amyloidosis
54. True about management of DIC

55. True about hemophilia


A. autosomal dominant
hemolytic disease
B. male subjects are clinically
affected while females who
carry 1 mutated gene are
mostly asymptomatic
C. clinically hemophilia a and b
are distinguishable
D. rarely is recurrent
hemarthrosis manifested
56. Correctly matched indications
and/or dosage for medications used
in Hemophilia
A. Tranexamic acid - to control
hematuria
B. Factor VIII - require
injections once a day
C. Cryoprecipitate - contains
FVIII
D. DDAVP - transient increase
in VIII and vWF
57. Fresh whole blood or WB is
maybe considered as indication for:
a. Evident massive blood loss
and coagulopathy
b. Trauma casualties who will
be requiring massive
transfusion but only when
specific blood products are
not available or when blood
products are not enough to
resuscitate the patient
c. Patients with hemorrhagic
shock when optimal specific
blood product therapy is not
available or when blood
products are not enough to
resuscitate the patient
d. All of the above
58.

59. True about premedications in


blood transfusions
A. Diphenhydramine and
Paracetamol is not recommended
B. Risk of paracetamol is related to
hepatotoxicity
C. Risk of diphenhydramine is
related to _____ and antihistamine
effects
D. All of the above

60.Routine for Blood Transf


a. Syphilis
b. Malaria
c. Hiv
d. ​Aota

61. In the emergent need for RBC


transfusion, in which recipient for
ABO and RHD group cannot be
obtained, transfusion of the following
is the safest
A. AB+ red cells
B. AB- red cells
C. O+ red cells
D. O- red cells
62.1 unit of packed RBC will
increase the hematocrit by
ANS: 3%

63.Surgical indications of PRBC


includes
a. Acute blood loss >2000 ml
b. Normovolemic px with Hg
7g/dl with cardiac disease
c. Blood loss >40% blood
volume
d. AOTA

64.
65.50 year old vegan, pallor, fatigue, Vitamin B 12
spastic weakness both legs and
anesthesia progress proximally.
P.E.: loss of balance, vibration and
position sense in both lower
extremities, spastic weakness with
absent DTRs, hepatosplenomegaly

a. Vitamin B1
b. Vitamin B2
c. Vitamin B6
d. Vitamin B12

66.44 year old, male,in the ER was


in for low grade fever,recently visited
a Malaria endemic area, a day prior,
he was diagnosed as having
Malaria, unrecalled IV and tablet
malaria medications were given for 2
days, no abdominal tenderness
noted. Which dengue category does
the patient belong in the CDC
category?

A. Probable without warning


signs
B. Dengue with warning signs
C. Severe dengue
D. Severe Hemorrhagic dengue

67.
68. Management for #66?

A. Admit patient and administer


fluids
B. Outpatient treatment and
order for CBC and platelets
C. -
D. -

69. Characteristics of significant


parasitization in P. falciparum
include the following except
a.)​ vasoconstriction
b.) hemoglobinuria
c.) microthrombi formation
d.) greater than 20% of RBC
parasitized= 50% mortality

70. Which of the following is true


about Chloroquine
A. Effective against
gametocytes of ​P. falciparum
B. Used to treat ​P. vivax, P.
ovale, and P. malariae
C. Both a and b
D. None of the above
71.

72. quinine side effects, except


A. Cinchonism
B. Hypoglycemia
C. Paresthesia
D. Epigastric pain

73. How many serotypes are there


in Dengue?

A. I
B. II
C. III
D. IV

74. Dengue virus genotypes that are


frequently associated with severe
disease?

ANS: ​Genotypes DEN-2 and DEN-3


75.

76. vector of dengue, except


a. albopictus
b. polynesiensis
c. aegypti
d. philippinenesis

77. The following are true about MIDGUT NOT HINDGUT


Dengue life cycle, EXCEPT:
A. Two phases of viremia
B. Intrinsic and extrinsic
incubation period
C. White blood cells and
lymphatics are affected
D. Virus replication in mosquito
hindgut and other organs
especially salivary glands
78. How would you treat the patient?
a. Admit and give supportive
management
b. Repeat cbc with Igm and IgG
c. Splenomegaly
d. Outpatient mngt

*Scenario(sa maremember ko lang


huhu)
Patient was Male, adult(kalimot ko
sa exact age)
-Fever(+)
-NS1 test (+)
-No other complain(s)

79Which of the following is true rash


of dengue
a.Maculopapular rash
b.appears on day 3-5
c.appears near defervescence
d.AOTA
80. Dengue fever lab results, except SGPT (increased in dengue =/> 1000U/L)
or least?? WBC 2×10^9/L (decreased in dengue)
A. SGPT 100U/L Platelet 20,000 (decreased in dengue)
B. WBC 2×10^9/L (decreased Hct 0.5 (increased in dengue)
in dengue)
C. Platelet 20,000 (decreased in
dengue)
D. Hct 0.5 (increased in
dengue)

81. 9 year old patient came in with


her mother
CC:
● nausea
● Rash
● persistent vomiting
● (+) tourniquet test
● Abdominal pain

What is the dengue classification?


A. Probable dengue
B. Probable dengue w/ warning
signs
C. Severe dengue

82.Severe Dengue
A. Fluid accumulation w/out
respiratory disease
B. AST and ALT >= 500 U/ L
C. ​Impaired consciousness
D. AOTA
83. ……………..She had a postural
drop in BP
A. >10% blood loss
B. >20%
C. ​>30%
D. >40%

84.the presence of increased


hemoglobin and hct, elevated red
cell mass, low EPO levels with
positive JAK2 mutations is a
confirmation test for?
A. erythrocytosis
B.​ polycythemia vera
C. secondary polycythemia
D. AOTA
85. Case 25 Female pt,
preemployment, with anemia. First
initial classification to consider for
anemia
A. MCV
B. ​RC
C. RDW
D. CBC??

86. What systemic disease that Naa ni sa wendy na first


cause or exacerbate a bleeding
tendency.

Ans. Hyperthyroidism

87.

88. A cardinal rule is that the


appearance of iron deficiency in an
adult male means?
a.
b. GI blood loss
c. Inadequate diet
d. Malabsorption
89. patient's results showed
hypochromic microcytic rbc, low hb
hc, low serum ferritin, inc rdw

a. alpha thalassemia
b. sideroblastic anemia
c. anemia of chronic dse
d. iron def anemia

Table 93-4 from Harrison 20th Ed., page 687


Regarding the RDW from Harrison 20th Ed., page 379

90. A 37-year old female presented P. 687 Harrison 20th ed


with normocytic hypochromic
anemia who came to you for
evaluation. Lab results showed
normal serum iron, normal TIBC,
elevated transferrin saturation,
elevated ferritin, and normal
hemoglobin electrophoresis and
normal RDW is MOST
CONSISTENT with which of the ff?

​A. Alpha thalassemia


B. Anemia of chronic inflammation
C. Anemia of chronic disease
D. Iron deficiency anemia
91. . True regarding the
management of iron deficiency
anemia
A. EPO at 50-150 U/kg 3x a
week is useful in augmenting
iron stores
B. IV iron can be given to
patients who cannot tolerate
oral iron
C. Red cell transfusion is
recommended to patients
with hemoglobin less than
90g/dl
D. Oral iron should be taken
with food
92.A percent transferrin saturation
>50%, If persistent, what is the likely
resulting condition?
Iron overload

93.
94.

95. Cause of relative erythrocytosis


a. ​hemoconcentration secondary to
tobacco
b. carbon monoxide poisoning
c. renal artery stenosis
d. sleep apnea syndrome

96. DIC parameters

Answer:
decreased platelet count,
increased PT
increased FDP
97.

98.

99.

100. Thrombocytopenia caused by


platelet GPIIb/IIIa in drugs such as
abciximab may occur after exposure
of the drug in how many days?
a. 1 day
b. 1 week
c. 2 weeks
d. 3 weeks
Unknown #
Its regulation is important a critical
component of of O2 affinity of Hgb
Ans: 2,3-BPG

Unknown # See fig. above (96-1)


Key molecule that is used to reduce
glutathione, which protects the red
cell against oxidative stress
Ans: NADPH
Unknown #
Most common infectious cause of
HA in endemic areas
Ans: Malaria

Unknown #
Main etiologic agent of HUS
ANS: E. coli O157:H7

Unknown #
Appropriate diagnostic procedure for
PNH
Ans: Flow cytometry to display a
CD59(−) red ce

Unknown #
Mechanism of HA in mismatched
blood transfusion
Ans: ABO (in)compatibility
Unknown #
Which of the ff is correctly matched:
Ans: X-ray damages DNA: Aplastic
anemia

Unknown #, 99 ata
A case given about PV
Asymptomatic ang patient, with uric
acid level of 9mg/dL (if tama
pagkakarember ko)
Ans: No treatment indicated

Other choices:
● Follow up after xx weeks
● Give allopurinol
● Phlebotomy..
2021 CLINICAL

QUESTION RATIONALE

1.A 30-year old female was brought to the ER after 2020 Clinical Exam
sustaining a stab wound in her left upper back. The patient
was noted to be normotensive but tachycardic. She,
however, displayed a postural drop in blood pressure. The
patient most probably has lost blood volume as much as

a. >30%
b. >40%
c. >10%
d. >20%

2.The presence of increased hemoglobin and hematocrit,


elevated red cell mass, low erythropoietin levels with
positive JAK2 mutation is confirmatory for:

a. All of the above


b. Polycythemia vera
c. Secondary polycxythemia
d. Relative erythrocytosis

Harrison’s 20th Ed page 734

3. A 20-year-old female was referred to your clinic for


workup of anemia. She was noted to be anemic after
routine preemployment checkup. Which of the laboratory
parameters is considered as key in the classification of the
patient’s anemia?

a. mean cell volume


b. red cell distribution width
c. reticulocyte count
d. serum ferritin levels

Reticulocyte count daw to sa ibang R

4.Underlying systemic diseases that cause or exacerbate Should be “hypothyroidism”


a bleeding tendency EXCEPT
a. severe renal impairment
b. hyperthyroidism
c. amyloidosis
d. liver disease

Harrison’s 19E p.404

5. A 21-year-old male was referred to you for workup of Hemophilia A = mutation in Factor VIII gene
bleeding. CBC and protime were normal. Activated partial
thromboplastin time was prolonged. Which of the following
diagnosis is most compatible with the findings?

a. hemophilia A
b. warfarin anticoagulation
c. disseminated intravascular coagulation
d. factor VII deficiency Harissons 20th ed p 830-831

6.The appearance of iron deficiency anemia in an adult


male suggest which condition until proven otherwise.

a. acute or chronic inflammation


b. inadequate diet
c. malabsorption
d. gastrointestinal blood loss

20th Ed. Harrison’s Principles of Internal Medicine 2018


7.The findings of low serum ferritin, elevated total
iron-binding capacity, low transferrin saturation, low serum
iron and normal RBCmorphology is MOST consistent with
what stage of iron deficiency?

a. negative iron balance


b. iron deficiency anemia
c. normal iron hemostasis
d. iron-deficient erythropoiesis

Harrison’s 20th Ed P685

8. A 37-year old female noted to have microcytic Diko mahanap…. Help


hypochromic anemia on routine testing was referred to
you for evaluation. Laboratory revealed Normal serum
iron and total iron-binding capacity, slightly elevated
transferrin saturation, slightly elevated serum ferritin,
normal hemoglobin electrophoresis and normal RDW is
MOST CONSISTENT with which of the following
conditions?

a. iron deficiency anemia


b. alpha thalassemia
c. anemia of chronic disease
d. anemia of chronic inflammation

9.Which of the following is TRUE regarding the HPIM 20th Ed. p. 686
management of iron deficiency anemia?

a. Intravenous iron can be given to patients who are


unable to tolerate oral iron.
b. Oral iron preparations should be taken with food.
c. Erythropoietin at 50-150 U/kg three times a week is
useful in augmenting iron stores.
d. Red cell transfusion is recommended for patients with
hemoglobin <90 g/dL
10. A transferrin saturation >50% indicates that a
disproportionate amount of iron bound to transferrin is
being delivered to nonerythroid tissues. If this persists,
what condition may occur?

a. hemolytic anemia
b. negative iron balance
c. iron deficiency anemia
d. iron overload

11.Which among the following etiologies of HPIM 20th ed p.688


hypoproliferative anemia is INCORRECTLY matched with
their main pathophysiology?

a. Anemia of chronic kidney disease: failure of EPO


production by diseased kidney
b. Anemia of endocrine disorders: testosterone mediated
decrease in erythropoeisis
c. Anemia of inflammation: TNF and IFN-y- mediated
suppression of EPO production
d. Anemia of protein starvation: impaired EPO release in
proportion to reduction in metabolic rate

12. A useful test in the clinic to determine patient’s ability


to absorb iron is called

a. iron tolerance test


b. transferrin saturation
c. serum ferritin test
d. iron dextran test
HPIM 20th ed. Page 687

13.Which of the following statements is TRUE for the HPI 20th Chapter 93 page 689
treatment in hypoproliferative anemia? A.
a. all of the above B. If iron levels are ADEQUATE
b. Erythropoeitin administration raises hemoglobin levels C. Above 11g/dL
to 10-12g/dl within 4-6 weeks even if iron levels are D. Patient WITHOUT
inadequate
c. Patient’s with more physiologic compromise may need
to have their hemoglobin levels above 9g/dL.
d. Patients with cardiovascular disease can tolerate
hemoglobin above 7-8g/dL and do not require
intervention.

14. The use of fresh whole blood is generally discouraged. PCPG for the Rational Use of Blood and Blood
However, its use may be considered in which of the Products and Strategies for Implementation
following situations? pp 24 and 33

a. trauma casualties
b. massive blood loss 1-1.5 times of blood volume in 24
hours
c. patients with hemorrhagic shock
d. Exchange transfusion in TTP/HUS

I think this should be an EXCEPTION question. A-C are all


TRUE

15.In the event of an emergent need for red blood cell Doc Lluisma-Limpahan Lec: Blood Transfusion based on
transfusion and the recipient’s ABO and Rh(D) group CPG slide 20
cannot be obtained, transfusion of which of the following
blood products is the safest?

a. AB-positive red cells


b. AB-negative red cells
c. O-positive red cells
d. O-negative red cells

16. Infusion of 1 unit packed red blood cells increases Doc Lluisma-Limpahan Lec: Blood Transfusion based on
hematocrit by how much CPG; Slide 30

a. 2%
b. 4%
c. 3%
d. 1%
17.Which of the following surgical indication/s can packed
red blood cell be transfused?

a. all of the above


b. acute blood loss 40% of blood volume
c. acute blood loss >2000mL
d. normovolemic patients with acute anemia with
Hgb<7g/dL who have cardiac disease

18. Packed RBC may be given in patients with hemoglobin Doc Lluisma-Limpahan Lec: Blood Transfusion based on
concentration of <10g/dL in such cases when there is? CPG

a. disabling angina pectoris


b. pulmonary embolism
c. during chemotherapy of cancer patients
d. all of the above

19.What is the best blood product to be given for those CPG Guidelines
immunocompromised patients to reduce the risk of
Cytomegalovirus infection and transfusion associated with
graft versus host disease?

a. washed RBCs
b. irradiated blood components
c. leukocyte-reduced red cells
d. fresh whole blood

20. Which of the following is an accepted indication for


random donor platelet transfusion?

a. 27-year old patient receiving therapy for acute leukemia


at a threshold of 15,000 u/L
b. 40-year old with acute subdural hematoma with a
platelet count 90 x109
c. 30- year old patient with ongoing massive bleeding with
platelet count 60 x109
d. 50-year old patient receiving massive blood transfusion
with a platelet count 25 x109

21.Serum homocysteine is raised in both early cobalamin Harrison, 20th ed. Chapter 95: Megaloblastic Anemia,
and folate deficiency, but may also be raised in other page 707
conditions EXCEPT:

a. hyperthyroidism
b. pyridoxine deficiency
c. chronic renal disease
d. alcoholism
A. Should be HYPOthyroidism

22.Which of the following drugs shows definite risk of Harrisson’s 20th ed pg 716
causing clinical hemolysis?

a. dapsone
b. primaquine
c. all of the above
d. nitrofurantoin

23.Which among the following causes of intravascular A → Correct


hemolysis is INCORRECTLY matched with their main
pathophysiologic mechanisms?

a. Mismatched blood transfusion: ABO incompatibility


b. Paroxysmal nocturnal hemoglobinuria: B → Correct
complement-mediated destruction of CD55- and
CD59-deficient cells
c. Microangiopathic hemolytic anemia: immune lysis of
normal red cells
d. Septicemia: exotoxins produced by Clostridium
perfrigens C → Wrong (dili siya immune mediated lysis but instead
mechanical)
D → Correct

24.Which of the following is considered as first- line HPIM 20th Ed. p. 719
treatment for autoimmune hemolytic anemia?

a. Splenectomy
b. Intravenous Immunoglobulin
c. Rituximab
d. Glucocorticoids

25.A 50-year old male with a long-standing history of


paroxysmal nocturnal hemoglobinuria was admitted at the
wards for anemia, teacolored urine and moderate acute
back pain. Which is the LEAST likely laboratory result in
this patient?

a. elevated unconjugated bilirubin


b. elevated haptoglobin
c. elevated LDH
d. increased reticulocytes HPIM 20th Ed. p. 721

26.What is the MOST common early symptom of aplastic


anemia?

a. weight loss
b. bleeding
c. easy fatigability
d. dyspnea
Harrisons 20th ed chapter 98 page 726
27.The diagnosis of aplastic anemia is usually Harrison’s 20th ed chapter 98 page 727
straightforward, based on the combination of which of the
following:

a. Pancytopenia with presence of immature myeloid forms


b. Pancytopenia with a dry tap during bone marrow
aspiration
c. Pancytopenia with dysmorphic but normocellular bone
marrow
d. Pancytopenia and fatty bone marrow

28. Which is the most common preceding infection in


aplastic anemia?

a. HIV-1
b. Parvovirus B19
c. Epstein Barr Virus
d. Hepatitis Virus

Harrisons 20th ed Ch98 p725

29.This is a disorder characterized by a triad of mucous Harrisons 20th ed ch98 p725


membrane leukoplasia, dystrophic nails, reticular
hyperpigmentation and with thedevelopment of aplastic
anemia in childhood.

a. Dyskeratosis Congenita
b. Shwachman-Diamond syndrome
c. Fanconi Anemia
d. Paroxysmal Nocturnal Hemoglobinuria

30.In the era of effective immunosuppressive therapies, HPoIM 20e p. 727


which of the following may be better predictors of In the era of effective immunosuppressive therapies,
response to treatment and long-term outcome in patients absolute numbers of reticulocytes (>25,000/ μL) and
with aplastic anemia? lymphocytes (>1000/μL) may be better predictors of
response to treatment and log-term outcome.
a. corrected reticulocyte count <1%
b. platelet count <20,000/uL
c. absolute neutrophil count <500/uL
d. lymphocytes >1000/uL

31.Mutation in which domain that appears to have a HPIM 20th Edition, Chapter 59, page 393
central role in the pathogenesis of polycythemia vera

a. BCR-ABL1 fusion
b. JAK2 mutation
c. TP53 mutation
d. PML-RARA fusion
32. Which of the following statements is true of HPIM 20th p734
polycythemia vera?

a. aquagenic pruritus may distinguish polycythemia vera


from other causes of erythrocytosis
b. intraabdominal venous thrombosis is particularly
common in young men
c. isolated thrombocytosis, leukocytosis or splenomegaly
may be the late sign
d. diastolic hypertension is also a feature of red cell mass
elevation

33.Which of the following causes of Relative


Erythrocytosis?

a. carbon monoxide intoxication


b. sleep apnea syndrome
c. renal artery stenosis
d. hemoconcentration secondary to tobacco
HPoIM 20e p. 734

34. Only these three situations cause microcytic


erythrocytosis EXCEPT

a. iron deficiency anemia


b. hypoxic erythrocytosis
c. Beta thalassemia trait
d. polycythemia vera

35.Which of the following is a complication of D. pero possible and C.


Polycythemia Vera?

a. aortic aneurysm
b. chronic obstructive pulmonary disease
c. all of the above
d. peptic ulcer due to H. pylori

??

Harrison’s 20th ed chapter 99 page 735

36. A 34- year old female with polycythemia vera had her Di ko mahanap. Sorry
routine checkup after 3 months with blood tests showing
Hgb of 140g/L, Hematocrit 45% and serum uric acid
9mg/dL. Patient is asymptomatic. What is your next step
in managing this patient?

a. Observe and repeat blood tests after 1 week.


b. Start with Anticoagulant to prevent thrombosis.
c. Administer allopurinol to avoid further elevation of uric
acid.
d. Advise Phlebotomy to reduce hyperviscosity
37.Which of the following causes of erythrocytosis should
be suspected in any patient who develop hepatic vein
thrombosis?

a. Gaisbock’s syndrome
b. Renal Artery Stenosis
c. Hepatopulmonary syndrome
d. Polycythemia vera

Harrisons 20th ed page 734

38. A 50-year old female was referred to you for ??


evaluation of isolated thrombocytopenia on CBC. What is
the key step in the evaluation of the patient’s problem?

a. peripheral blood smear


b. bone marrow examination
c. platelet aggregometry
d. bleeding time assessment

39.After how many days or weeks of initial exposure to


Abciximab will thrombocytopenia be manifested?

a. 2 weeks
b. 1 day
c. 1 week
d. 3 weeks

Harrison’s 19E p.727


40. Which of the following is true regarding
Heparin-induced thrombocytopenia?

a. it is associated with bleeding and decreases the risk for


thrombosis
b. results from antibody formation to a complex of platelet
specific PF4 and heparin
c. most patient develop thrombocytopenia after exposure
to heparin for 21 days
d. thrombocytopenia is usually severe with nadir counts Pa double-chick rako guys. #LetLeniAnswer
rarely <20,000u/L

41.A 28/F was referred to you for evaluation of


thrombocytopenia. She denies symptoms save for a bout
of cough and colds 2 weeks prior to consult. PE shows
some petechiae on both lower extremities. She denies any
other illnesses. Complete blood count revealed normal
hemoglobin, WBC and differential but with platelet count
of 28,000/uL. Peripheral blood smear was normal save for
decreased platelets which are slightly enlarged. Which of
the following DOES NOT constitute appropriate
management for the patient?

a. Admit the patient for transfusion of platelet products


b. test for HIV infection and Hepatitis C
c. advise the patient regarding modification of activities to
avoid trauma
d. start prednisone at 1mg/kg/day Doc Limpahan’s PPT based on CPG - slide 35

42.Which condition is historically characterized by a


pentad of findings that include microangiopathic hemolytic
anemia, thrombocytopenia, renal failure, neurologic
findings and fever?

a. Hemolytic uremic syndrome


b. Thrombotic thrombocytopenic purpura
c. Henoch-Schonlein purpura
d. Immune thrombocytopenic purpura

HPIM 20th Ed P826

43. What is the most common inherited bleeding disorder? HPIM 20th Ed Page 828

a. von Willebrand disease


b. Bernard-Soulier syndrome
c. Hermansky-Pudlak syndrome
d. Hemophilia A

44. The following statements are true of hemophilia HPIM 20th ed. P. 830
EXCEPT:

a. Diagnosis is made after specific determination of FVIII


or FIX clotting activity
b. X-linked recessive hemorrhagic disease due to
mutations in the F8gene or F9 gene
c. M>F affected but women are symptomatic
d. Prolonged aPTT but normal bleeding time and platelet
counts

Only males are affected; women are asymptomatic

45.The most common inherited factor deficiencies are the


hemophilias caused by deficiencies of which factors?

a. vWF
b. Fibrinogen
c. Factor VIII and IX
d. Factor II, V, VII, X, XI and XIII

46.An isolated prolonged aPTT suggests which HPIM 20th ed. P.830
coagulation disorder?

a. all of the above


b. FVII deficiency
c. FV, FX, FII deficiencies or fibrinogen abnormalities
d. Hemophilia or FXI deficiency

47.In non-transfusion therapy in hemophilia, this is a


synthetic vasopressin analog that causes a transient rise
in FVIII and von Willebrandfactor (VWF) but not FIX
through a mechanism involving release from endothelial
cells:

a. DDAVP
b. Norepinephrine
c. Tranexamic acid HPIM 20th ed. Page 832
d. Recombinant IL11
48. This is the major cause of morbidity and second
leading cause of death in hemophilia patients exposed to
older clotting factor concentrates:

a. Hepatitis C infection
b. Cholangiocarcinoma
c. Coronary artery disease
d. Hepatocellular carcinoma

49.Which of the following is a common cause in DIC? Harrison pp 834

a. Malignant disorders: solid tumors or acute


promyelocytic leukemia
b. Bacterial sepsis
c. Obstetric causes
d. all of the above

50.What is the most sensitive test for DIC? Harri pp 835-6

a. prolongation of PT and/or aPTT


b. elevated levels of FDP
c. presence of schistocytes in the blood smear
d. platelet counts 100,000/uL or a rapid decline in platelet
numbers

51.The central mechanism of DIC is: Harrison’s 20th Ed. page 835

a. RBC damage and hemolysis


b. Fibrin deposits in the microcirculation
c. Consumption of platelets and coagulation factors
d. Uncontrolled thrombin generation
52. In diagnosis of hemolytic anemia, the following are
seen, EXCEPT:

a. Normomacrocytic anemia
b. Jaundice
c. Reticulocytosis
d. Hyperbilirubinemia

* macrocytic dapat

53.Which of the following is NOT TRUE of G6PD


Deficiency?

a. Males have only one G6PD gene, they are hemizygous


for this gene.
b. G6PD Deficiency is of single missense point mutations,
entailing single amino acid replacements in G6PD protein.
c. G6PD in red cells functions as the only source of NADH
and that its deficiency leads to oxidative stress via
glutathione (GSH).
d. All of G6PD deficient patients have an increased risk of
developing neonatal jaundice (NNJ).

C. NOT SURE PERO DAPAT ATA NADPH


54. A 10- year old boy was brought into the ER with chief Harrison Chapter 96 page 715
complaint of body malaise and weakness. He was
admitted and laboratory were requested. After 2 hours,
results of blood and urine work up came in. Which of the
following will confer your impression of acute hemolytic
attack?

a. Normocytic-normochromic anemia, hemoglobinemia,


hemoglobinuria, high LDH.
b. Microcytic- hypochromic anemia, hemoglobinemia,
hemoglobinuria, high LDH.
c. Macrocytic-hyperchromic anemia, hemoglobinemia,
hemoglobinuria, high LDH.
d. Normocytic-hypochromic anemia, hemoglobinemia,
hemoglobinuria, low LDH.

55.After thorough history taking of the above case, G6PD Harrison, 20th Ed. Chapter 96. Hemolytic Anemia,
Deficiency is considered. Blood film will show which of the page 715
following that will support the new diagnosis?

a. Red cells that are of unequal size, polychromatic, and


some are spherical in shape.
b. Red cells that are abnormally shaped, spherical in
shape, and with abnormally high number of immature red
blood cells.
c. Red cells that appear to have unevenly distributed
hemoglobin and red cells to have had parts of them bitten
away.
d. Anisocytosis, Polychromasia, and spherocytes

56. Ten years later, the above patient is now a Med 20th Ed. Harrison, chapter 96, page 717
student in a certain province in Mindanao. However, he
was brought in the emergency room because of difficulty The AHA of G6PD deficiency is largely preventable by
of breathing. Upon physical examination, he is generally avoiding exposure to triggering factors of previously
pale with pale conjunctivae. STAT CBC work up shows screened subjects. Of course, the practicability and
low hemoglobin level and leukocytosis. What will be your cost-effectiveness of screening depend on the
first treatment? prevalence of G6PD deficiency in each individual
community. Favism is entirely preventable in
a. Blood transfusion of patient’s blood type, properly G6PD-deficient subjects by not eating fava beans.
crossmatched and screened. Drug-induced hemolysis can be prevented by testing for
b. Medical treatment with Prednisone immediately G6PD deficiency before prescribing; in many cases one
together with Rituximab, and IVIG. can use alternative drugs. When AHA develops and once
c. Run various tests to determine the cause of anemia. its cause is recognized, no specific treatment is needed
d. Transfusion of incompatible blood, so that the in most cases. However, if the anemia is severe, it
transfused red cells will be destroyed no less but no more may be a medical emergency, especially in children,
than the patient’s own red cells requiring immediate action, including blood
transfusion.

“Letter C ang answer because there might be other


causes of anemia aside from G6PD. Not B because it is
not immune mediate. Not D because again it's not
immune mediated”

57.Transfusion of incompatible blood (packed RBCs) is


considered treatment for what condition?

a. Hereditary Spherocytosis
b. Pyruvate Kinase deficiency
c. Autoimmune Hemolytic Anemia
d. G6PD Deficiency

Harrison’s 20th pg 719

58. What is the most frequent infectious cause of Harrisons 20th ed. P. 718
Hemolytic Anemia in endemic areas?

a. Malaria
b. Shigella dysenterae type 1
c. Shiga-toxin producing E. coli O157:H7
d. Clostridium perfringens sepsis

59.A 45-year old man was brought to the emergency room HPIM 20th Ed. p. 712
because of right upper quadrant pain after eating an ‘oily
rich meal’. The man was noted to be jaundiced and upon
palpation of the abdomen, spleen was tender and
enlarged. Ultrasound revealed presence of gallstones.
Blood work up revealed normocytic red cells with
increased mean corpuscular hemoglobin concentration.
Family history revealed that his grandfather died of the
same condition 5 years ago. What is your initial
impression?

a. Hereditary Spherocytosis
b. Hereditary elliptocytosis
c. Hereditary stomatocytosis
d. Pyruvate Kinase Deficiency

60. DNA tests were requested and found out that EPB41
gene was defective (with mutation). What will be your
diagnosis? HPIM 20th Ed. p. 711

a. Hereditary elliptocytosis
b. Pyruvate Kinase Deficiency
c. Hereditary stomatocytosis
d. Hereditary Spherocytosis

61.Which of the following treatment will be more beneficial


to the patient?

a. Avoid splenectomy, use therapeutic measures


b. Splenectomy
c. Antipneumococcal vaccination
d. Cholecystectomy

62. A 30-year old female presents with persistent fatigue Harrison’s 20th ed page 387 (Approach to anemia)
and weakness. She has noted heavy menstrual bleeding
and frequently eats ice. Upon P.E., you noted pallor,
atrophic tongue and moist mucous membranes. What lab
tests will you order?

a. Peripheral blood smear


b. Serum ferritin
c. All of the above
d. CBC, platelet count
63.Upon release of the patient’s lab results, there was a
note of hypochromic, microcytic RBCs, low hemoglobin
and hematocrit, elevated RDW, low MCV and MCHC, low
serum ferritin. These findings are consistent with:

a. iron deficiency anemia


b. anemia of chronic disease
c. sideroblastic anemia
d. alpha thalassemia

Harrisons 20th ed ch93 685


Robbins Patho Ch 14 652
Hematology Basic Principles and Practice 7th ed Ch 34
p462

64. Treatment option/s for the condition in #63 is: Harrisons 20th ed ch93 686

a. whole blood transfusion


b. stem cell transplant
c. IV dextran
d. oral iron therapy
65.A 50-year old vegan woman presents with pallor and Harrison’s IM (20th ed) pg 2310
fatigue. She also reports spastic weakness of both legs
and anesthesia progressing proximally from her distal
extremities. Upon P.E., she has loss of balance, vibratory Pg 703
and position sense in both lower extremities, spastic Adults Dietary cobalamin deficiency arises in vegans who
weakness of lower extremities with absent DTRs, omit meat, fish, eggs, cheese, and other animal products
hepatosplenomegaly. What is your most likely diagnosis? from their diet
Pg 701
a. Vitamin B6 deficiency The patient, more frequently male, typically presents with
b. Vitamin B2 deficiency paresthesias, muscle weakness, or difficulty in walking but
c. Vitamin B12 deficiency sometimes with the dementia, psychotic disturbances, or
d. Vitamin B1 deficiency visual impairment. There is usually loss of proprioception
and vibration sensation with positive Romberg and
Lhermitte signs.

66. What would be the expected CBC result for this case? HPIM 20th p702

a. normal hemoglobin, low MCV, normal WBC,


thrombocytopenia
b. decreased hemoglobin, low MCV, normal WBC,
thrombocytopenia
c. decreased hemoglobin, high MCV, mild leukopenia,
thrombocytopenia
d. normal hemoglobin, high MCV, leukopenia,
thrombocytopenia

67.What is the best management for this patient? HPIM 20th p707

a. All of the above


b. Lifelong, regular vitamin supplementation IM
c. Lifelong, regular vitamin supplementation PO
d. Lifelong, regular vitamin supplementation SL —--------------------------------------------------------------------------
68. A 42-year old woman was hospitalized in the ICU for
E. coli sepsis and begins to display extensive skin and
mucous membrane bleeding. She is being treated with
IV antibiotics and is on pressor therapy. She is noted to
be tachycardic and hypotensive. She is hooked to a
mechanical ventilator. There is note of multiple diffuse
petechiae and hematomas, bleeding from IV and
pulmonary artery catheter sites. The most sensitive test
for this case is:

a. FDP test
b. APTT
c. Protime
d. D-dimer test
HPoIM 20e p. 835

69.Management for this case will include, EXCEPT: Harrisons 20th Ed Ch 112 pp 836

a. transfusion of fresh frozen plasma and platelet


b. correcting the underlying disorder
c. administration of heparin
d. none of the above

70. The DIC score is based on the following parameters: Harrisons 20th ed ch112 p835

a. platelet count, decrease in fibrin markers, PT


prolongation, fibrinogen level
b. platelet count, decrease in fibrin markers, APTT
prolongation, fibrinogen level
c. platelet count, increase in fibrin markers, APTT
prolongation, fibrinogen level
d. platelet count, increase in fibrin markers, PT
prolongation, fibrinogen level

71.A 29-year old male came to you for easy fatigability Harrisons 20th ed ch93 686
and lethargy. He was diagnosed with gallstones several
years ago and that his mother had splenectomy while in
her 20s. Upon P.E., you note jaundice and icteric sclerae,
splenomegaly. What is the LEAST LIKELY laboratory test
that you will order for this case?

a. CBC, platelet count


b. Osmotic fragility test
c. Peripheral blood smear
d. Fractionated bilirubin

72. Which of the following is/are not true of hereditary Harrisons 20th Ed Ch 96 page 712
spherocytosis?

a. The main clinical findings are jaundice, enlarged spleen,


and severe anemia.
b. All of the above
c. Severe case may present in infancy with severe
anemia.
d. Anemia is usually normocytic with increase Mean
Corpuscular Hemoglobin Concentration

73.What is the triad of paroxysmal nocturnal


hemoglobinuria (PHN)?

a. Severe anemia, hemolysis, venous thrombosis


b. Persistent hemolysis, pancytopenia, jaundice
c. Hemolysis, pancytopenia, venous thrombosis
d. Venous thrombosis, splenomegaly, mild anemia

HPoIM 20e p. 720

74. The following is true regarding the management of HPIM 20th p712
hereditary spherocytosis.

a. Antipneumococcal vaccination before splenectomy is


not warranted.
b. Delay splenectomy until puberty in moderate cases or
until 4-6 years of age in severe cases.
c. None of the above
d. In mild cases, do splenectomy
75.The clinical feature of pyruvate kinase deficiency is that
of a Hemolytic anemia that often presents in newborn with
neonatal jaundice. Pyruvate kinase is an enzyme involved
in the glycolytic pathway that converts what reaction?

a. Pyruvate to lactate
b. Phosphoenolpyruvate to pyruvate
c. Glucose 6-phosphate to 6-phosphogluconate
d. 2-phosphoglycerate to phosphoenolpyruvate

76. Which of the following describes familial (atypical) Harrison’s IM (20th ed) p717
hemolytic-uremic syndrome?
Familial (Atypical) Hemolytic-Uremic Syndrome
a. Characterized by increased intracellular sodium in red (aHUS) This term is used to designate a group of rare
cells, with concomitant loss of potassium, and low MCHC. disorders, mostly affecting children, characterized by
b. Characterized by persistent jaundice and is usually microangiopathic HA with presence of fragmented
associated with very high reticulocytosis and may require erythrocytes in the peripheral blood smear,
regular blood transfusion treatment. thrombocytopenia (usually mild), and acute renal failure.
c. Characterized by morphologic abnormality of the red
cells known as basophilic stippling
d. Characterized by microangiopathic hemolytic anemia
with presence of fragmented erythrocytes in the peripheral
blood smear, thrombocytopenia and acute renal failure.

77.In patients with “march hemoglobinuria”, the Harrisons 20th ed ch96 718
appropriate diagnostic procedure is

a. . Red Cell morphology on blood smear


b. Flow cytometry to display CD59(-) red cell population
c. Test for Donath-Landsteiner antibody
d. Targeted History taking
78. What is the triad of autoimmune hemolytic anemia?

a. Hepatosplenomegaly, severe anemia,


hyperbilirubinemia
b. Passing of blood instead of urine, jaundice, and mild
anemia
c. Hemoglobin level of 4g/dL, Splenomegaly, massive red
cell removal
d. Jaundice, severe anemia, hyperbilirubinemia

Harrison 20th ed Ch96 p718

79.A 33-year old Chinese woman is found to have low Harrison’s 20th ed page 688
hematocrit on routine screening labs. She is otherwise ???
healthy with no complaints. She recalls having a previous
bout with anemia during childhood but denies bleeding
tendencies. Her brother had a similar anemia. Pertinent
P.E. revealed pale conjunctivae. Lab results revealed Hgb
9.4 gm/dL and Hct 28.9%. Iron studies normal. Stool
occult blood negative. What is the most likely diagnosis?

a. Alpha thalassemia
b. Hemoglobin C disease
c. Iron-deficiency anemia
d. Anemia of chronic disease

80. Management for this case would include: Harrisons 20th ed ch.94 pg 697

a. Iron supplementation
b. No treatment indicated.
c. Blood transfusion
d. Splenectomy

If base dito sa yellow highlighted text, puedeng no


treatment indicated since genetic counseling lang
required.

Even if HBH pa type ng kanyang alpha thalassemia eh as


per the greenhighlighted text kay excessive anemiai or
transfusion requirement develop lang kelangan ng
splenectomy:
As per Hgb ng px kay moderate pa yung anemia-so
not excessive - pero di ko alam sa hematocrit values
As per transfusion - not included siya sa indications ng
FWB or WB then kung PRBC naman if hgb is between
6-10 g/dL kay ang transfusion i base sa px risk of
complication or inadequate oxygenation but wala nito both
ang px

For iron supplementation as per the green highlighted text


kay iron overload can occur sa severely affected px so i
think to avoid iron overload no need na mag iron
supplementation

81.A newborn girl born at term is noted to have a large Disorder of Platelet Questions from internet
dark-red cutaneous lesion over most of the buttock area.
Petechiae were present over her back. A CBC showed a
Hb of 150 g/L, a normal WBC and differential count and a
platelet count of 15 x 109/L. The fibrinogen level was 0.5
g/L. Cryoprecipitate and platelets were administered.
What is the recommended therapy in this case?

a. corticosteroids
b. amicar
c. IV vincristine weekly
d. interferon-alpha

82. A 10- month old boy is admitted to hospital with Di ko sure kay wala nako nakita :(, pero akong PI kay
pneumonia. Physical examination reveals some patches Aplastic Anemia kay recurrent infections, hence Bone
of eczema on his limbs. A CBC shows a platelet count of Marrow aspirate/biopsy. Please change nalang if nakita
15 x 109/L with small platelets on the peripheral smear. A ninyo ang answer :)
history of recurrent ear infections is elicited. The family
history is negative for individuals with platelet disorders. SAME RATIO SA NUMBER 84
The key diagnostic investigation is:

a. measurement of platelet autoantibodies


b. an immunological work-up
c. gene/gene product studies
d. bone marrow aspirate/biopsy

83. You are asked to see a 6-year old girl who underwent Possible Heparin-Induced Thrombocytopenia ni since si
open heart surgery 7 days previously and who developed patient kay nag undergo siya ug major surgery
moderate thrombocytopenia (platelet count 50 x 109/L)
over a period of 3 days. She is clinically stable with no
evidence of infection. Her platelet count before surgery
was normal. Fibrinogen and D-dimer tests are normal.
The most appropriate immediate next step is:

a. request heparin-associated antibody testing


b. bone marrow aspirate
c. stop all heparin in IV lines, flushes
d. measure a reticulated platelet count

So…. According to Harrison's 20th ed. P. 850 nga stop jud


daw si Heparin

INTERNET

84. A 10- month old boy is admitted to hospital with ??anemia


pneumonia. Physical examination reveals some patches NAA SA INTERNET NI NGA QUESTION
of eczema on his limbs. A CBC shows a platelet count of
15 x 109/L with small platelets on the peripheral smear. A
history of recurrent ear infections is elicited. The family
history is negative for individuals with platelet disorders.
The key diagnostic investigation is:

a. measurement of platelet autoantibodies


b. an immunological work-up
c. gene/gene product studies
d. bone marrow aspirate/biopsy
https://studylib.net/doc/5827664/multiple-choice-questions
-for-the-board-review-course

85.A 28-year old female is brought to the emergency room IM platinum 3rd Edition
after having seizure at work. Patient also complains of
increased fatigue which she noted several weeks ago. Thrombotic thrombocytopenic purpura (TTP): Classic
P.E. revealed HR 126 bpm, temp 39.3, pallor, multiple Pentad but not often seen. Mnemonics (FAT RN)
petechiae and normal neurologic exam. Lab results
F: Fever
revealed plt ct 53,000, elevated retic count, altered platelet
morphology, elevated BUN and creatinine, increased LDH, A: microangiopathic hemolytic anemia
normal PT and APTT. T: thrombocytopenia
What is the best treatment option for this case? R: renal failure
N: neurologic decline
a. Administration of glucocorticoids
b. Plasma exchange Treatment: plasma exchange (Mainstay tx);
c. Platelet transfusion corticosteroids; Immunomodulatory therapies
d. Splenectomy
(rituximab, vincristine, cyclosphosphamide);
splenectomy

86. For items 86-90. A 44-year old man came in for


low-grade fever with no other symptoms. Five weeks ago,
he was diagnosed with malaria and treated at a local
hospital. The blood smear at that time was reported by the
hospital as positive for malaria, species undetermined. He
was then treated with 2 days of IV fluids (nature unknown)
and tablets (nature unknown) and recovered. Pathology of
malaria may include

a. anemia due to red cell destruction


b. all of the above
c. red blood cell sequestration
d. recurrent disease due to cryptic infections in the liver

87.Characteristics of significant parasitization of P. Harrison Chapter 219 page 1578-1579;1583


falciparum include the following except:

a. vasoconstriction
b. microthrombi formation
c. if > 20% of erythrocytes are parasitized, mortality = 50%
(P falciparum)
d. hemoglobinuria
88. Which of the following is true of Chloroquine (Ch 219 Malaria Harrisons p1584)
pharmacology?

a. effective against gametocytes of P falciparum


b. used to treat P. vivax, P. ovale, P. malariae attacks
c. both a and b
d. none of the above

89.Which of the following is a characteristic of quinine’s


(Quinamm) antimalarial activity?

a. gametocidal for P. malariae


b. gametocidal for P. ovale
c. slow onset
d. poorly effective blood schizonticide against P vivax * Hindi ko sya mahanao sa Harri at GG

90.Adverse effects of quinine EXCEPT Goodman and Gilman 13th Ed page 979

a. hypoglycemia
b. epigastric pain
c. paresthesias (Not sure)
d. cinchonism

91.For items 91-100. A 60-year old lawyer came in for Dengue WHO p 14
fever and joint pains. noted 3 days PTA. Patient is
non-hypertensive, non-diabetic. He is a smoker and
alcoholic beverage drinker. Pertinent P.E. revealed temp.
39 C with note of abdominal tenderness at the
epigastrium. Lab Results revealed a positive Dengue NS1
test, WBC 4 x 10
/L, platelet count 50,000.
How many serotypes does the Dengue virus have?

a. 1
b. 2
c. 3
d. 4

92. Which of the following genotypes are associated with Dengue WHO p 14
severe disease?

a. DEN-2 AND DEN-4


b. DEN-1 AND DEN-2
c. DEN-2 AND DEN-3
d. DEN-1 AND DEN-3

93.How do you classify the patient mentioned above ??


according to the CPG on Dengue fever?

a. Dengue fever with warning signs


b. Severe dengue
c. Dengue shock syndrome
d. Dengue fever without warning signs

94. The following are possible vectors of the dengue virus


EXCEPT:

a. Aedes polynesienses
b. Aedes aegypti
c. Aedes albopticus
d. Aedes philippinensis HPIM 20th ed page 3403
Journal

95.Which of the following statements is NOT TRUE Dengue WHO (2009) p.16
regarding dengue life cycle?

a. There is an extrinsic, as well as intrinsic, life cycle.


b. There are 2 phases of viremia.
c. White blood cells and lymphatics are affected.
d. Virus replication occurs in the mosquito hindgut and Occurs in the *midgut
other organs especially in the salivary glands.

96.What is the best treatment option for the patient


mentioned above?

a. Admit patient and give supportive therapy. ???


b. Monitor CBC, platelet count as outpatient.
c. Do platelet transfusion.
d. Repeat Dengue NS1 test, including Dengue IgG and
IgM.

97.Which of the following statements is true regarding the HPIM 20th ed. P. 1504
appearance of rash in dengue fever?

a. All of the above


b. It begins on the trunk and spreads to the extremities
and face.
c. The rashes appear nearing defervescence.
d. It occurs on the day 3-5 of illness.

98. What is the LEAST LIKELY laboratory finding in Not sure but I think ang SGPT is only ordered if the patient
dengue fever? is under Group C? So least likely siya?

a. Hematocrit: 0.50
b. Platelet count: 20,000/L
c. WBC: 2 X 10 9/L
d. SGPT: 100 U
99.A 9-year old female came in with her mother in the
emergency room for complaints of fever for 4 days. She
also noted nausea, body pains and rashes. Tourniquet
test was positive and had RUQ tenderness and
hepatomegaly on P.E. After you examine her, the patient
vomited 5 times. What is the dengue classification of this
patient?

a. Dengue fever without warning signs


b. Probable dengue
c. Severe dengue
d. Dengue with warning signs

DENGUE Guidelines for Diagnosis, Treatment,


Prevention and Control 2009.pdf (WHO)

100.Which is a criterion for severe dengue?

a. Fluid accumulation without respiratory distress a. Fluid accumulation without respiratory distress
b. Impaired consciousness b. Impaired consciousness - correct
c. All of the above c. All of the above
d. AST and ALT more than or equal to 500 U/L d. AST and ALT more than or equal to 500 U/L

From CDC ito siya, walang specific sa CFG.


https://docs.google.com/document/d/1D-iscoku8EqNeVYO
s1WhXJxsLCnNaUIYugxP9uU9nA0/edit
IM Platinum pg.264- 3rd Edition 2018

You might also like